You are on page 1of 421

Item: lof73 ~. , .

M k <:] t> al ~· ~
QIO: 1462 .l. ar Previous Next lab 'lifllues Notes Calculator

•1 •
A 55-yea r-old woman, found in a confused and obtunded state, is rushed to a near by hospita l. The daughte r
•2
info rms the eme rgency department physician that her mother is a gene ral ly hea lthy woman who sees a
•3 psychiatrist every 2 weeks fo r long -stand ing depression . She has been on the same antidepressant for many
•4 years. On physica l examination, the patient responds incoherently to painful stimu li. Pu lse is 124/min, blood
•5
pressu re is 72/44 mm Hg, and temperature is 38 .9°C ( 102° F) . Skin is wa r m and f lushed, pupils are dilated, and
no bowe l sounds can be heard. The patient's ECG is shown in the image . The physician suspects an overdose of
•6 her antidep ressant.
~
"·· -~
0

•7 HR: 107 "< LJU/'

•8
•9
• 10 v

· 11 . ,_
I mage court esy of Wikipedia
• 12
• 13
What is the most like ly mechan ism of action of th is patient's antidepressant medication?
• 14
:
• 15
• 16
• 17 A. Blockade of dopam ine receptors with in the central ne rvous system
• 18 B. Blockade of reuptake of dopamine with in the central ne rvous system
• 19
C. Blockade of reuptake of norepineph r ine and seroton in with in the central nervous system
• 20
• 21
D . Enhancement of the release of norepinephrine and serotonin

6
lock
s
Suspend
0
End Block
~. , . M t> al ~· ~

-
Item: lof73 k <:]
.l. ar

.
QIO: 1462 Previous Next lab 'lifllues Notes Calculator

• 1 g Y P PY g Y Y
psychiatrist every 2 weeks for long -stand ing depression . She has been on the same antidepressant for many
•2
years. On physica l examination, the patient responds incoherently to painful stimu li. Pu lse is 124/min, blood
•3 pressu re is 72/44 mm Hg, and temperature is 38 .9°C ( 102° F) . Skin is wa r m and f lushed, pupils are dilated, and
•4 no bowe l sounds can be heard. The patient's ECG is shown in the image . The physician suspects an overdose of
•5
her antidep ressant.
HR:107 ~ ~ " ' .,,;
•6 - ..-1
•7
•8
v
•9
• 10 I mage courtesy of Wtktpedta

· 11
• 12 What is the most like ly mechan ism of action of th is patient's antidepressant medication?
• 13 :
• 14
• 15
A. Blockade of dopam ine receptors with in the central ne rvous system
• 16
B. Blockade of reuptake of dopamine with in the central ne rvous system
• 17

• 18 C. Blockade of reuptake of norepineph r ine and seroton in with in the central nervous system
• 19 D . Enhancement of the release of norepinephrine and serotonin
• 20
E. Stimulation of serotonin receptors
• 21

6
lock
s
Suspend
0
End Block
Item: lof73 ~. , . M k <:] t> al ~· ~
QIO: 1462 .l. ar Previous Next lab 'lifllues Notes Calculator

1 •

•2 The correct answer is C. 72°/o chose this.


•3 This patient is exhibiting symptoms of tricyclic antidepressant (TCA) ove rdose. Mental status changes,
hypotension, and anticholinergic effects (flushing, hyperthe rmia, di lated pupils, intestinal ileus, and urinary
•4
retention) are the classic symptoms seen in TCA overdose. Other adverse effects include ventricu lar
•5 ar rhythmias, as shown in the ECG, which can be fata l. TCAs such as amitr iptyl ine and imipramine inhibit
•6 neuronal reupta ke of norepineph r ine and seroton in in the central ne rvous system . This increases the synaptic
•7 concentration of these neurotransmitters and potentiates their actions. Sod ium bica rbonate is used to t reat TCA
overdose .
•8
Tricyclic antidepressant Anticholinergic Amitriptyline Imipramine Sodium bicarbonate Urinary retention Antidepressant Central nervous system Serotonin
•9
Hyperthermia Norepinephrine Hypotension Ileus Neurotransmitter Mydriasis Cardiac arrhythmia Electrocardiography Reuptake Tricyclic Sodium Nervous system
• 10
Drug overdose Flushing (physiology) Altered state of consciousness Mental status examination
· 11
• 12
A is not correct. 7°/o chose this .
Typica l (halope ridol, fluphenazine) and atypica l (risper idone, clozapine) antipsychotics block dopamine receptors
• 13
with in the central nervous system. The atypical antipsychotics are un ique in that they also block se roton in
• 14 receptors. In contrast to antipsychotics, tricyclic antidepressants, which this patient was given, do not affect
• 15 dopamine receptors .
Fluphenazine Risperidone Haloperidol Clozapine Central nervous system Serotonin Dopamine Tricyclic antidepressant Antipsychotic Atypical antipsychotic
• 16
Antidepressant Tricyclic Nervous system Receptor (biochemistry)
• 17

• 18 B is not correct. 6°/o chose this .


• 19 Dopamine reupta ke inhibito rs serve to increase the extrace llular concentration of dopamine. Examples of drugs
• 20
that wo rk, at least in part, by this mechanism include buproprion and methylphenidate. Overdose of th is type of
medication may cause psychosis (a mechanism opposite that of antipsychotics) .
• 21 Dopamine Methylphenidate Psychosis Antipsychotic Reuptake Dopamine reuptake inhibitor Extracellular Pharmaceutical drug

6
lock
s
Suspend
0
End Block
Item: lof73 ~. , . M k <:] t> al ~· ~
QIO: 1462 .l. ar Previous Next lab 'lifllues Notes Calculator

1 Dopamine reuptake inhibitors serve to increase the extrace llular concentration of dopamine. Examples of drugs
. 2 that work, at least in part, by this mechanism include buproprion and methylphenidate. Overdose of th is type of
•3
medication may cause psychosis (a mechanism opposite that of antipsychotics) .
Dopamine Methylphenidate Psychosis Antipsychotic Reuptake Dopamine reuptake inhibitor Extracellular Pharmaceutical drug
.4
•5 D is not correct. 10% chose this.
•6 Mirtazapine is an antidepressant that acts as an a 2 -antagon ist to increase release of norepineph r ine and
seroton in. The sedating effect of mirtazapine decreases as the dosage increases. Tricyclic antidep ressants do
.7
not enhance the release of these a mines but prevent them from being removed f rom the synaptic cleft.
•8 Mirtazapine Serotonin Antidepressant Norepinephrine Synaptic cleft Chemical synapse Tricyclic antidepressant Tricyclic Sedative Synapse Sedation Amine

•9
E is not correct. 5°/o chose this .
• 10 Buspi rone is a stimulant of serotonin (5-hydoxytryptamine 1A) recepto rs and is used primarily in the treatment of
· 11 general ized anxiety disorder.
Buspirone Generalized anxiety disorder Serotonin Anxiety disorder Stimulant Anxiety
• 12
• 13
• 14 Bottom Line:
• 15
Tricyclic antidepressants (TCAs) inhibit norepineph r ine and seroton in uptake from the synaptic cleft. TCA
• 16 toxicity is characterized by mental status changes, hypotension, and anticholinergic effects (flushing,
• 17 hyperther mia, di lated pupils, intestina l ileus, and urinary retention) . Ove rdose of a TCA can be fata l because of
ar rhythmias; TCA ove rdose is treated by the administration of sodium bicarbonate .
• 18
Anticholinergic Sodium bicarbonate Serotonin Norepinephrine Urinary retention Hyperthermia Synaptic cleft Hypotension Ileus Tricyclic antidepressant
• 19 Chemical synapse Antidepressant Mydriasis Cardiac arrhythmia Sodium Tricyclic Toxicity Synapse Drug overdose Flushing (physiology)
• 20 Altered state of consciousness

• 21

6
lock
s
Suspend
0
End Block
Item: lof73 ~. , . M k <:] t> al ~· ~
QIO: 1462 .l. ar Previous Next lab 'lifllues Notes Calculator

1
. 2 FA17 p283.1

•3 Torsades de pointes Polymorphic ventricular tachycardia, Drug-induced long QT (ABCDE):


.4 characterized by shifting sinusoidal waveforms AntiArrhythmics (class IA, III)
on ECC ; can progress to ventricular AntiBiotics (eg, macrolides)
•5
fibrillation (VF). Long QT interval Anti"C "ychotics (eg, haloperidol)
•6 predisposes to torsades de pointes. Caused by AntiDepressants (eg, TCAs)
.7 drugs, l K+, l .\llg2+, congenital abnormalit·ies. AntiEmetics (eg, ondansetron)
•8 Treatment includes magnesium sulfate . Torsades de pointes = twisting of the points
•9 Congenital long QT inherited disorder of myocardial repolarization,
• 10 syndrome typically due to ion channel defects; t risk of
sudden cardiac death (SC D) clue to torsacles de
· 11
pointcs. Includes:
• 12
• Romano-Ward syndrome- autosomal
• 13 dominant, pure ca rdiac phenotype {no
• 14 deafness).
• 15 • Jervell and Lange-Nielsen syndrome-
• 16
autosomal recessive, sensorineural deafness.
• 17

• 18 FA17 p 239.2
Drug reactions- cardiovascular
• 19
DRUG REACTION CAUSAL AGENTS
• 20
Coronary vasospasm Amphetamines, cocaine, ergot alkaloids, sumatriptan
• 21

6
lock
s
Suspend
0
End Block
Item: lof73 ~. , . M k <:] t> al ~· ~
QIO: 1462 .l. ar Previous Next lab 'lifllues Notes Calculator

1
. 2 FA17 p 239.2
Drug reactions-cardiovascular
•3
DRUG REACTION CAUSAL AGENTS
.4
Coronary vasospasm Amphetamines, cocaine, ergot alkaloids, sumatriptan
•5
Cutaneous flushing Va ncomycin, Adenosine, N iacin, N itrates, C a 2+ channel blockers, Echinocandins (VAl "'CE)
•6
Dilated Anthracyclines (eg, doxorubicin, daunorubicin); prevent with dexrazoxane
.7
cardiomyopathy
•8
Torsades de pointes AntiArrhythmics (class lA, Il l), antiBiotics (eg, macrolides), anti"C "ychotics (eg, halope ridol),
•9
antiD epressants (eg, TCAs), anti Emetics (eg, ondansetron) (ABCDE)
• 10

· 11
FA17 p 545.4
• 12
Tricyclic Amitriptyline, nortriptyline, imipramine, desipram ine, clomipramine, doxcpi n, amoxapine.
• 13
antidepressants
• 14
MECHANISM Inh ibit 1 E and 5-HT reuptake.
• 15
CLINICAL USE Major depression, OCD (clomipramine), periphe ral neuropathy, chronic pain, migraine
• 16
prophylaxis. locturnal e nuresis (imipramine, although adverse effects may limit use).
• 17
ADVERSE EFFECTS Sedation, a 1-blocking effects including postural hypote nsion, and atropine-like (anticholinergic)
• 18
side effects (tachycardia, urinary retention, dry mouth). 3° TCAs (amitriptyline) have more
• 19 zo
antichol inergic effects than TCAs (nortriptyline). Can prolong QT interval.
• 20 Tri-C's: C onvulsions, C oma, C ardiotoxicity (arrhythmia due to 1 a+ channel inhibition);
• 21 also respiratory depression, hype rpyrexia. Confusion and hallucinations in elderly due to
- - •-! - L - 1 ! • • • - · • • · .1 . . C C •• o . l • • • ._ .• _ , J• • • L . •- • - · •- - l • . • o • •l • . oL • • 1 .1 . .1 \ "'1-. ••• o • • • • o 1\. T. II r"tl""'\.

6
lock
s
Suspend
0
End Block
Item: 2 of 73 ~ 1 • M k -<:J 1>- Jil ~· !:';-~
QIO: 1490 ..L ar Pre v ious Next Lab fli!ltues Not es Calcula t o r

IAA]
& &
1
A 35-year-old female presents to her primary care doctor with a ch ief complaint of a 4.5-kg {10-lb)
•2
unintentional weight loss over the past 2 months. During th is time she has also been experiencing feelings of
•3 worthlessness and is having trouble sleeping at night. She is no longer interested in activities that she used
·4 to enjoy, and she finds herself crying more often than usual. The physician prescribes a medication to help with
her symptoms.
•5
•6
What is a commonly reported adverse effect of the first-line medication for this patient's illness?
•7

·8 :
A. Dry mouth
.9
• 10 B. Neuromuscular dysfunction
• 11 C. Orthostatic hypotension
• 12
D. QT prolongation
• 13
• 14
E. Sexua l dysfunction
• 15
• 16
• 17
• 18
• 19
• 20
• 21

a
Lock
s
Suspend
8
End Bl ock
Item: 2 of 73 ~. , . M k <:] t> al ~· ~
QIO: 1490 .l. ar Previous Next lab 'lifllues Notes Calculator

1 •

2
The correct answer is E. 79°/o chose this.
The patient exh ibits symptoms of majo r dep ressive disorder, wh ich is thought to be caused primarily by a
•3
deficiency in serotonin levels within the brain . Selective se roton in reuptake inhibitors (SSRis) are the f irst- line
•4 treatment for depression; these drugs (eg, f luoxetine and paroxetine) act to increase synaptic levels of
•5 seroton in by inhibiting the reuptake of this key neu rotransmitter. Many patients who take SSRis report sexua l
•6 dysfunction as a side effect. It is important to note that mirtazapine cou ld also be considered for this patient; it
has antidepressive properties and can help with insomnia, since it also acts as a sedative. Furthermo re,
•7
mirtazepine can increase appetite, which might he lp counter the weight loss expe rienced by this patient .
•8 Mirtazapine Major depressive disorder Paroxetine Fluoxetine Serotonin Neurotransmitter Insomnia Sedative Selective serotonin reuptake inhibitor

•9 Sexual dysfunction Adverse effect Side effect Reuptake Weight loss Depression (mood) Brain Human brain

• 10
A is not correct. 8°/o chose this.
· 11 Symptoms such as dry mouth, constipation, and urina ry retention are common with med ications with
• 12 antichol inergic properties such as tricycl ic antidepressants (TCAs). Although TCAs are used for obsessive-
• 13 compulsive diso rder (ie, clomipramine), per iphera l neu ropathy, ch ron ic pain, and migraine prophylaxis, they are
not a first-line t reatment fo r dep ression due to their many side effects. Other medications with anticho linergic
• 14
properties are fi rst-gene ration antihistamines (eg, diphenhyd ramine) and musca rinic antagonists (eg,
• 15 glycopyrrolate).
• 16 Anticholinergic Diphenhydramine Clomipramine Obsessive-compulsive disorder Urinary retention Migraine Constipation Glycopyrronium bromide

• 17 Peripheral neuropathy Tricyclic antidepressant Muscarinic antagonist Muscarinic acetylcholine receptor Antihistamine Xerostomia Antidepressant Chronic pain

• 18 Major depressive disorder Depression (mood) Tricyclic Preventive healthcare Therapy Side effect

• 19
B is not correct. 3°/o chose this .
• 20 Antipsychotics can cause neuromuscu lar adverse effects such as extrapyramidal symptoms (eg, dystonias,
• 21 pa rkinsonism, akathisia) or neuroleptic ma lignant synd rome. The risk is highest for typical, high potency

6
lock
s
Suspend
0
End Block
Item: 2 of 73 ~. , . M k <:] t> al ~· ~
QIO: 1490 .l. ar Previous Next lab 'lifllues Notes Calculator


1 B is not correct. 3°/o chose this .
2 Antipsychotics can cause neuromuscular adverse effects such as extrapyramidal symptoms (eg, dystonias,
•3 parkinsonism, akathisia) or neuroleptic malignant synd rome. The risk is highest for typical, high potency
anti psychotics ( eg, haloper idol, trifluoperazine, fluphenazine) . There is no indication in this patient's history that
•4
war rants the administration of an antipsychotic medication.
•5 Neuroleptic malignant syndrome Fluphenazine Haloperidol Trifluoperazine Akathisia Antipsychotic Extrapyramidal symptoms Parkinsonism Extrapyramidal system
•6 Neuromuscular junction Pharmaceutical drug
•7
C is not correct. 5°/o chose this .
•8
Selective serotonin reuptake inhibitors (SSRI) are not commonly associated with orthostatic hypotension. I n
•9 contrast, due to the a 1-blocking effects, orthostatic hypotension may be seen with tricyclic antidepressants
• 10 (TCA) . Due to their side effects, TCAs are not recommended as first-line phar macologic treatment for
· 11 depression, and therefore th is patient is un likely to have been prescribed a TCA. Other drugs that can cause
orthostatic hypotension include a-adrenergic receptor blockers (eg, prazosin, terazosin), particu larly in patients
• 12
taking them for the fi rst time .
• 13 Prazosin Orthostatic hypotension Serotonin Selective serotonin reuptake inhibitor Hypotension Tricyclic antidepressant Antidepressant Reuptake

• 14 Major depressive disorder Terazosin Depression (mood) Tricyclic Side effect Pharmacology Receptor (biochemistry)

• 15
D is not correct. 5°/o chose this .
• 16
Selective serotonin reuptake inhibitors are associated with the risk of QT prolongation . However, th is side effect
• 17 is much less common than the side effect of sexua l dysfunction. QT prolongation is more common ly seen in
• 18 patients treated with tricycl ic antidepressants. Other med ications that may prolong the QT interval are
• 19
antiarryhthmics (eg, sotalol), antibiotics (macrolides), and antipsychotics (eg, haloper idol) .
Haloperidol Sotalol Serotonin QT interval long QT syndrome Selective serotonin reuptake inhibitor Tricyclic antidepressant Antipsychotic Sexual dysfunction
• 20
Antidepressant Macrolide Adverse effect Reuptake Antibiotics Side effect
• 21

6
lock
s
Suspend
0
End Block
Item: 2 of 73 ~. , . M k <:] t> al ~· ~
QIO: 1490 .l. ar Previous Next lab 'lifllues Notes Calculator
J
1

2 C is not correct. 5°/o chose this.


•3 Selective serotonin reuptake inhibitors (SSRI) are not commonly associated with orthostatic hypotension. In
.4
contrast, due to the a 1 -blocking effects, orthostatic hypotension may be seen with tricyclic antidepressants
(TCA) . Due to their side effects, TCAs are not recommended as first-line phar macologic treatment for
•5 depression, and therefore th is patient is un likely to have been prescribed a TCA. Other drugs that can cause
•6 orthostatic hypotension include a-adrenergic receptor blockers (eg, prazosin, terazosin), particu larly in patients
.7 taking them for the fi rst time.
Prazosin Orthostatic hypotension Serotonin Selective serotonin reuptake inhibitor Hypotension Tricyclic antidepressant Antidepressant Reuptake
•8
Major depressive disorder Terazosin Depression (mood) Tricyclic Side effect Pharmacology Receptor (biochemistry)
•9
• 10 D is not correct. 5°/o chose this .
· 11 Selective serotonin reuptake inhibitors are associated with the risk of QT prolongation . However, th is side effect
is much less common than the side effect of sexua l dysfunction. QT prolongation is more common ly seen in
• 12
patients treated with tricycl ic antidepressants. Other med ications that may prolong the QT interval are
• 13 antiarryhthmics (eg, sotalol), antibiotics (macrolides), and antipsychotics (eg, haloper idol) .
• 14 Haloperidol Sotalol Serotonin QT interval long QT syndrome Selective serotonin reuptake inhibitor Tricyclic antidepressant Antipsychotic Sexual dysfunction

• 15 Antidepressant Macrolide Adverse effect Reuptake Antibiotics Side effect

• 16
• 17
Bottom Line:
• 18
Selective serotonin reuptake inhibitors (SSRis) are fi rst-line treatment for depression. Sexual dysfunction is a
• 19 commonly reported side effect of SSRis.
• 20 Serotonin Selective serotonin reuptake inhibitor Sexual dysfunction Side effect Reuptake Adverse effect Major depressive disorder Depression (mood) Therapy
First-line treatment
• 21

6
lock
s
Suspend
0
End Block
Item: 2 of 73 ~ 1 • M k -<:J 1>- Jil ~· !:';-~
QIO: 1490 ..L ar Pre v ious Next Lab fli!ltues Not es Calcula t o r
• p
1
Repeated nighttime awakenings
2
Early-morning awakening (terminal
•3
insomnia)
·4
•5
FA17 p 545.1
•6
Selective serotonin Fluoxetine, Au\'oxamine, paroxetine, sertraline, escitalopram, citalopram.
•7
reuptake inhibitors
·8 MECHANISM Inhibit 5-HT reuptake. It normally takes 4-8 weeks for antidepressants
.9 to have an effect.
CliNICAl USE Depression, generalized anxiety disorder,
• 10
panic disorder, OCD, bulimia, social anxiety
• 11 disorder, PT SD, premature ejaculation,
• 12 premenstrual dysphoric disorder.
• 13 ADVERSE EFFECTS Fewer than TCAs. C l distress, Sl 0 11 , sexual
• 14 dysfunction (anorgasmia, l Iibido).
• 15
• 16 FA17 p 530.3
• 17 Mood disorder Characterized by an abnormal range of moods O J' internal emotional states and loss of control o' er
• 18 them. Se,·erity of moods causes distress and impairment in social and occupational functioning.
• 19 Includes major depressi,·e disorder, bipolar disorder, dysthymic disorder, and cyclothymic
• 20 disorder. Episodic superimposed psychotic features (delusions or hallucinations) may be present.
• 21

a
Lock
s
Suspend
8
End Bl ock
Item:3of73 ~. , . M k <:] t> al ~· ~
QIO: 5144 .l. ar Previous Next lab 'lifllues Notes Calculator


1
2
A 29-year-old physician presents to the cl inic with her husband because she has been feeling sad for the past I"' Aj
2 years . She is constantly fatigued and having difficu lty concentrating at work, though she sti ll enjoys her A
•3 job. She has no history of major depression . Last month, she was ful l of energy, qu ite happy, and able to
•4 work overnight cal ls every 3 nights without fee ling t ired . Her husband den ies other behavior changes or changes in
her speech du r ing th is period of elevated mood . The patient reports that her mood has fluctuated like this for
•5
several years. Although she is always able to manage her daily activit ies, her husband is becoming frustrated by
•6 her constant "ups and downs."
•7
•8 Wh ich of the fo llowing is the most like ly diagnosis?
•9
:
• 10 A. Bipo lar I disorder
· 11
B. Bipo lar II disorder
• 12
• 13
C. Borderl ine personality disorder
• 14 D . Cyclothymic disorder
• 15 E. Dysthymic disorder
• 16
• 17

• 18
• 19
• 20
• 21

6
lock
s
Suspend
0
End Block
Item:3of73 ~. , . M k <:] t> al ~· ~
QIO: 5144 .l. ar Previous Next lab 'lifllues Notes Calculator

1 •

2 The correct answer is D. 51°/o chose this.


3 Cyclothymia is characterized by periods of hypomanic symptoms that do not meet cr iteria for full hypomania
•4 and periods of depression that do not meet criteria for a major depressive episode. Diagnostic and Statistical
Manual, 5e (DSM 5) criteria for diagnosis of hypomania are elevated mood and at least three of the fol lowing:
•5
inflated self-esteem, decreased need for sleep, increased need for tal king or fast speech, racing thoughts,
•6 distractibility, increase in goal-di rected activity, and excessive pursuit of activities with negative consequence,
•7 such as gambl ing or promiscuity. (Note: Although manic and hypoman ic episodes have the same key diagnostic
criteria, the DSM-5 states that manic episodes are clinically more severe and have more functional impairment
•8
than occu rs with hypomanic episodes). Though this patient has some hypoman ic symptoms, the stem does not
•9 describe enough symptoms to meet the criteria for a hypoman ic episode. Add itionally, she does not cu r rently
• 10 meet the cr iteria for a major depressive episode (5/9 SIGECAPS symptoms). As a result, cyclothymic disorder is
· 11
the best diagnosis. Symptoms must be present for at least 2 years to diagnose this disorder, and the patient
must never be symptom free for more than 2 months during those 2 years . The disease course is chronic, and
• 12
many patients eventually develop bipolar disorder. Treatment approaches vary, but lithium and psychotherapy
• 13 are sometimes successful.
• 14 Cyclothymia Hypomania Bipolar disorder DSM-5 Major depressive episode Euphoria Depression (mood) Diagnostic and Statistical Manual of Mental Disorders

• 15 Self-esteem Psychotherapy Racing thoughts Mania Symptom Major depressive disorder lithium Medical diagnosis Promiscuity Distraction

• 16 A is not correct. 15% chose this.


• 17 Patients are diagnosed with bipola r I disorder if they have had at least one manic or mixed episode .
• 18 Additionally, patients may have had major depressive episodes, dysthymia, hypoman ia, or euthymia; however,
• 19
none of these are required for diagnosis. Manic and hypomanic episodes have similar key diagnostic cr iteria, but
manic episodes are always more extreme and disabling . For example, during a manic episode, thoughts are so
• 20 disorganized that completing goal-di rected activities is usually impossible; in contrast, hypoman ic episodes
• 21 often lead to increased productivity . Manic episodes can cause psychosis, and patients often become dangerous

6
lock
s
Suspend
0
End Block
Item:3of73 ~. , . M k <:] t> al ~· ~
QIO: 5144 .l. ar Previous Next lab 'lifllues Notes Calculator


1 A is not correct. 15% chose this .
2 Patients are diagnosed with bipola r I disorde r if they have had at least one manic or mixed episode.
3 Add itiona lly, patients may have had majo r depressive episodes, dysthymia, hypoman ia, or euthymia; however,
none of these are required fo r diagnosis. Manic and hypoman ic episodes have simila r key diagnostic cr iteria, but
•4
manic episodes are always more extreme and disabling . For example, du ring a manic episode, thoughts are so
•5 disorganized t hat comp leting goal-di rected activities is usua lly impossib le; in contrast, hypoman ic episodes
•6 often lead to increased productivity. Manic episodes can cause psychosis, and patients often become dange rous
•7 to them se lves, requi ring hospita lization. Based on the history provided in the stem, the patient has never
reached the level of dysfunction requi red to diagnose a m anic episode .
•8
Mixed affective state Bipolar I disorder Dysthymia Hypomania Mania Psychosis Manic episode Major depressive episode Euthymia (medicine) Medical diagnosis
•9
Bipolar disorder
• 10
B is not correct. 22% chose this.
· 11
Bipo lar II disorder is defined as having a histo ry of at least one major depressive episode and at least one
• 12
hypomanic episode. Th is patient's relative ly mi ld mood symptoms do not meet the crite ria for major depressive
• 13 disorder or for hypomania, so she cannot be diagnosed with bipolar II diso rder .
• 14 Major depressive disorder Bipolar II disorder Hypomania Major depressive episode

• 15 C is not correct. 4°/o chose this.


• 16 The mood labi lity associated with borderline pe rsonality disorder (BPD) can resemb le bipolar diso rder. BPD is a
• 17 cluste r B pe rsonal ity disorder, character ized by unstable moods, behaviors, and inte rpersona l relationships .
• 18
Patients often have feelings of emptiness, problems with self -image, impulsivity, and a histo ry of repeated
suicida l th reats or attempts or se lf-mutilation. Spl itting is a def ense mechanism common ly observed in
• 19
bo rder line patients. The ups and downs of cyclothymic and bipolar disorde rs last wee ks to months, whe reas
• 20 mood can f luctuate dramatical ly throughout the day in BPD. This patient's mood changes are more consistent
• 21 with cyclothymic disorder. I n addition, we do not know if she has other pe rsona lity traits associated with BPD,
• -- ···- -----~ ....1:------ . . . _....... :~ . . . - -- . . ----1 :..... ...1 :- - .... ...J- ....

6
lock
s
Suspend
0
End Block
Item:3of73 ~. , . M k <:] t> al ~· ~
QIO: 5144 .l. ar Previous Next lab 'lifllues Notes Calculator

1 • Major depressive disorder Bipolar II disorder Hypomania Major depressive episode

2 C is not correct. 4°/o chose this.


3 The mood labi lity associated with borderline personality disorder (BPD) can resemb le bipolar disorder. BPD is a
.4 cluster B personal ity disorder, character ized by unstable moods, behaviors, and interpersona l relationships.
•5
Patients often have feelings of emptiness, problems with self-image, impulsivity, and a history of repeated
suicida l th reats or attempts or se lf-mutilation. Spl itting is a defense mechanism common ly observed in
•6
border line patients. The ups and downs of cyclothymic and bipolar disorders last weeks to months, whereas
.7 mood can f luctuate dramatical ly throughout the day in BPD. This patient's mood changes are more consistent
•8 with cyclothymic disorder. In addition, we do not know if she has other persona lity traits associated with BPD,
so we cannot diagnose her with a personal ity disorder .
•9
Borderline personality disorder Bipolar disorder Cyclothymia Emotional dysregulation Personality disorder Defence mechanisms Self-harm lability Impulsivity
• 10
Self-image Trait theory Interpersonal relationship
· 11
• 12
E is not correct. 8°/o chose this .
Dysthymic disorder is also a mi ld mood syndrome . Patients are chronica lly unhappy, but do not meet cr iteria for
• 13
major depressive disorder. At least 2 depressive symptoms for 2 years are necessary for diagnosis, and the
• 14 patient cannot be symptom free for longer than 2 months. This patient has been having depressive symptoms
• 15 for the past 2 years, but she has hypomanic symptoms as wel l, ma king cyclothymia the more like ly diagnosis .
Cyclothymia Major depressive disorder Dysthymia Hypomania Symptom
• 16
• 17

• 18 Bottom Line:
• 19
Cyclothymia is a mood synd rome characterized by alternating periods of hypomanic and depressive symptoms.
• 20 Cyclothymia Hypomania

• 21

6
lock
s
Suspend
0
End Block
Item:3of73 ~. , . M k <:] t> al ~· ~
QIO: 5144 .l. ar Previous Next lab 'lifllues Notes Calculator
- - -
1 •
Cyclothymia is a mood synd rome characterized by alternating periods of hypomanic and depressive symptoms.
2
Cyclothymia Hypomania
3
•4
•5
lijl;fiiJI•J for year:[ 2017 ..
•6 FI RST AID FAC T S

•7
FA17 p 531 .2
•8
Bipolar disorder Bipolar I defined by presence of at least I man ic episode +1- a hypomanic or depressive episode.
•9
(manic depression) Bipolar II defi ned by presence of a hypomanic and a depressive episode.
• 10
Patient's mood and fun ctioning usually return to normal between episodes. Use of antidepressants
· 11 can precipitate mania. lligh suicide risk. Treatment: mood stabi lizers (eg, lithium, valproic acid,
• 12 carbamazepine, lamotrigine), atypical antipsychotics.
• 13 Cyclothymic disorder- milder form of bipolar disorder lasting at least 2 years, fluctuating
• 14 between mild depressive and hypomanic symptoms.
• 15
• 16 FA17 p 530.3
• 17 Mood disorder Characterized by an abnormal range of moods or internal emotional states and loss of control over
• 18 them. Severity of moods causes distress and impairment in social and occupational functioning.
• 19 Includes major depressive disorder, bipolar disorder, dysthymic disorder, and cyclothymic
• 20
disorder. Episodic superimposed psychotic features (delusions or hallucinations) may be present
• 21

6
lock
s
Suspend
0
End Block
Item: 4 of 73 ~ 1 • M k -<:J 1>- Jil ~· !:';-~
QIO: 460 9 ..L ar Pre v ious Next Lab fli!ltues Not es Calcula t o r

IAA]
& &
1
A 39-year-old woman presents to the emergency department because she "desperately wants to commit
2
suicide." On physical examination she has numerous burns and cut marks along her arms and legs. After
3 completion of the physical examination and history, the patient pra ises the physician for being so caring and
•4 requests a different hospital bed. When told that al l other beds are currently in use, she becomes enraged and
demands a different physician immediately.
•5
•6
Which of the following is the most likely diagnosis?
•7

·8 :
A . Antisocial personality disorder
.9
• 10 B. Borderline personality disorder
• 11 C. Dependent personality disorder
• 12
D. Paranoid persona lity disorder
• 13
• 14
E. Schizotypal disorder
• 15
• 16
• 17
• 18
• 19
• 20
• 21

a
Lock
s
Suspend
8
End Bl ock
Item:4of73 ~. , . M k <:] t> al ~· ~
QIO: 4609 .l. ar Previous Next lab 'lifllues Notes Calculator

1 •
The correct answer is B. 85°/o chose this.
2
Borderl ine personality disorder is cha racterized by violent swings in affect, recurrent suicidal ideation,
3 impulsivity, self-mutilation, and strained interpersona l relationships. This patient demonstrates su icida l
4 tendencies in add it ion to vio lent mood swings when her request for a change of beds cannot be accommodated.
•5 Borderl ine personality disorder occurs three times more often in women than in men.
Borderline personality disorder Suicidal ideation Personality disorder Suicide Self-harm Impulsivity Suicide crisis Ideation (creative process)
•6
Interpersonal relationship Mood swing
•7
•8 A is not correct. 3°/o chose this .
•9 Antisocia l persona lity disorder is a complete disrega rd for the rights of others and is cha racterized by crimina l
behavior .
• 10
Antisocial personality disorder Personality disorder Anti-social behaviour
· 11
C is not correct. 5°/o chose this .
• 12
Those with dependent persona lity disorde r are clingy and constantly need to be taken care of, which is not the
• 13
case in this cl inical picture .
• 14 Dependent personality disorder Personality disorder

• 15
D is not correct. 2°/o chose this .
• 16
Paranoid personal ity diso rder is character ized by the inherent be lief that the wo rld is a dangerous and
• 17 th reatening place. Upon meeting these individua ls they often project strength and capability, and thei r distrust
• 18 and suspiciousness of everyone is evident. These ind ividuals tend to bel ieve in va rious conspiracy theo ries .
Paranoid personality disorder Personality disorder Conspiracy theory Paranoia
• 19
• 20 E is not correct. 5°/o chose this .
• 21 Patients with schizotypa l persona lity diso rder are cha racterized by inte rpersona l awkwardness. They tend not to
• I o II , , I I I I I I , I • • I , I • , , • , I • , • o o •

6
lock
s
Suspend
0
End Block
Item:4of73 ~. , . M k <:] t> al ~· ~
QIO: 4609

1
·- _,.
.l. ar Previous Next lab 'lifllues

Antisocial personality disorder Personality disorder Anti-social behaviour


Notes Calculator

2
3 C is not correct. 5°/o chose this.
4 Those with dependent persona lity disorder are clingy and constantly need to be taken care of, which is not the
case in this cl inical picture.
•5 Dependent personality disorder Personality disorder
•6
D is not correct. 2°/o chose this.
•7
Paranoid personal ity disorder is character ized by the inherent be lief that the world is a dangerous and
•8
threatening place. Upon meeting these individua ls they often project strength and capability, and their distrust
•9 and suspiciousness of everyone is evident. These ind ividuals tend to bel ieve in various conspiracy theories .
Paranoid personality disorder Personality disorder Conspiracy theory Paranoia
• 10
· 11 E is not correct. 5°/o chose this.
• 12 Patients with schizotypa l persona lity disorder are characterized by interpersona l awkwardness. They tend not to
• 13 relate we ll to others and do not understand why something so inherently interesting to them is not interesting
to others. Odd be liefs and eccentric appearance are also typica l.
• 14
Schizotypal personality disorder Personality disorder
• 15
• 16
• 17 Bottom Line:
• 18 Borderl ine personality disorder is cha racterized by violent swings in affect, recurrent suicidal ideation,
• 19
impulsivity, self-mutilation, and strained interpersona l relationships .
Borderline personality disorder Suicidal ideation Personality disorder Self-harm Impulsivity Interpersonal relationship Suicide crisis Borderline {2008 film)
• 20
• 21

6
lock
s
Suspend
0
End Block
Item: 4 of 7 3 ~ 1 • M k -<:J 1>- Jil ~· !:';-~
QIO: 460 9 ..L ar Pre v ious Next Lab fli!ltues Not es Calcula t o r
& &
1
FA17 p 535.2
2
Cluster B personality Dramatic, emotional, or erratic; genetic "\\'ild."
3
disorders association with mood disorders and substance Cluster B: Bad, Borderline, AamBoyant, must
4
abuse. be the Best
•5
Antisocial Disregard for and violation of rights of others Anli~ocial =~oeiopath.
•6
,,·ith lack of remorse, criminality, impulsi,·ity; Bad.
•7 males> females; must be~ 18 years old and
'
·8 ha,·e history of conduct disorder before age 15.
.9 Conduct disorder if< 18 years old.
• 10 Borderline Unstable mood and interpersonal relationships, Treatment: dialectical behavior therapy.
• 11
impulsivity, self-mutilation, suicidality, sense Borderline.
of emptiness; females> males; splitting is a
• 12
major defense mechanism .
• 13
Histrionic Excessive emotionality and excitability, FlamBoyant.
• 14
attention seeking, sexually pro,•ocativc, overly
• 15 concerned with appearance.
• 16 Narcissistic G randiosity, sense of entitlement; lacks emp<1thy Must be the Best.
• 17 and requires excessive admiration; often
• 18 demands the "best" and reacts to criticism
with rage.
• 19
• 20
FA17 p 534.3
• 21
• PArcnn:olitv

a
Lock
s
Suspend
8
End Bl ock
Item: 4 of 7 3 ~ 1 • M k -<:J 1>- Jil ~· !:';-~
QIO: 460 9 ..L ar Pre v ious Next Lab fli!ltues Not es Calcula t o r

:t . II~···· I
1
Conduct disorder if< 18 years old.
2
Borderline Unstable mood and interpersonal relat ionships, Treatment: dialectica l behavior therapy.
3
impulsivity, self-mutilation, suicida lity, sense Borderline.
4
of emptiness; females> males; splilling is a
•5 major defense mechanism.
•6 Histrionic Excessi,·e emotionalitr and excitability, FlamBorant.
.7 attention seeking, sexuallr pro,·oe<~ ti,·e, O\'Crly
·8 concemed with appearance.
.9 Narcissistic Grandiosity, sense of entitlement; lacks empathy \lust be the Best.
• 10 and requires excessive admiration; often
demands the ''best" and reacts to criticism
• 11
with rage.
• 12
• 13
FA17 p 534.3
• 14 Personality
• 15
Personality trait An enduring, repetitive pattern of perceiving, relating to, and thinking about the environment and
• 16 oneself.
• 17 Personality disorder InAexible, maladaptive, and rigidly pervasive pat tern of behavior causing subjective distress
. 18 and/or impaired functioning; person is usuallr not aware of problem. Usually presents by early
• 19 adulthood .
• 20
Three clusters, A, B, and C; remember as Weird, \\ ild, and Worried based on symptoms.
• 21

a
Lock
s
Suspend
8
End Bl ock
Item: 5 of 73 ~ 1 • M k -<:J 1>- Jil ~· !:';-~
QIO: 1485 ..L ar Pre v ious Next Lab fli!ltues Not es Calcula t o r

IAA]
& &
1
A 70-year-old widowed man who lives alone visits his physician with complaints of headaches, decreased
2
appetite, weight loss, insomn ia, and anhedon ia. Medical history is significant for symptomatic benign
3 prostatic hyperplasia. An electrocardiogram taken prior to starting med ication shows first- degree heart block.
4
•5 Which of the following would be the best option for treating this patient's symptoms?
•6
:
•7 A. Amitriptyline
·8 B. Chlorpromazine
.9
C. Doxepin
• 10
• 11 D. Lithium
• 12 E. Paroxetine
• 13
• 14
• 15
• 16
• 17
• 18
• 19
• 20
• 21

a
Lock
s
Suspend
8
End Bl ock
Item:5of73 ~. , . M k <:] t> al ~· ~
QIO: 1485 .l. ar Previous Next lab 'lifllues Notes Calculator

1 •
The correct answer is E. 61 °/o chose this.
2
The patient described in the question is suffer ing from depression. Se lective serotonin reupta ke inhibito rs
3
( SSRis, eg, pa roxetine, fluoxetine) are first-line treatments for depression and have a relatively safe ca rdiac
4 profi le. As a resu lt, pa roxetine should not lead to any adverse cardiac effects in a patient with f irst-degree heart
5 block . Also, given its lack of antichol inergic activity, it is less likely to exacer bate any urina ry retention
•6
associated with this patient's ben ign prostatic hyperplasia. Side effects of SSRis genera lly include decreased
libido, akathesia, and increased risk of fractu res.
•7 Akathisia Benign prostatic hyperplasia Anticholinergic Paroxetine Fluoxetine Serotonin libido Urinary retention Selective serotonin reuptake inhibitor
•8 Major depressive disorder Depression (mood) Reuptake Hyperplasia Heart block Adverse drug reaction Side effect
•9
A is not correct. 13% chose this .
• 10
Amitr iptyl ine, a tricyclic antidepressant (TCA), often causes antichol inergic adverse effects and wou ld be
· 11 relative ly contraindicated in a patient suffering from ur inary retention due to benign prostatic hyperplasia. TCAs
• 12 also may cause tachycardia, and therefore wou ld be particula r ly dangerous in a patient w ho suffers from fi rst-
• 13 degree heart block .
Tricyclic antidepressant Benign prostatic hyperplasia Anticholinergic Amitriptyline Urinary retention Antidepressant Tachycardia Heart block Contraindication
• 14
Tricyclic Hyperplasia
• 15
• 16 B is not correct. 11% chose this .
• 17 Ch lorpromazine is an antipsychotic agent. Because this patient is not experiencing psychotic symptoms,
chlorpromazine is not ind icated. Adverse effects of chlor promazine incl ude extrapyramidal effects, sedation,
• 18
decreased blood pressure, and prolonged QT intervals .
• 19 Chlorpromazine Antipsychotic Extrapyramidal symptoms Sedation Psychosis Extrapyramidal system Blood pressure long QT syndrome

• 20
C is not correct. 10% chose this .
• 21
• f)oxP.n in ic; rt trir:vd ir: rtntirlP.n rP.c;c;;=tnt thrtt r:rtn hP. LJ<;P.rl for rlP.nrP.c;c;ion rtnrl rtnXiP.tV . h tJt it hr~c; thP. notP.ntirt l to
6
lock
s
Suspend
0
End Block
Item:5of73 ~. , . M k <:] t> al ~· ~
QIO: 1485

1
.-
.l.
-- -
ar
...... - . - ..... -..
Previous
-
Next lab 'lifllues Notes
- ..:_
Calculator

Chlorpromazine Antipsychotic Extrapyramidal symptoms Sedation Psychosis Extrapyramidal system Blood pressure long QT syndrome
2

3
C is not correct. 10% chose this.
4
Doxepin is a tricycl ic antidepressant that can be used for depression and anxiety, but it has t he potential t o
cause anticho linergic adverse effects, includ ing tachyca rdia and decreased blood pressu re. The refore it would be
5 re lative ly contraind icated in this patient, who has urina ry retent ion due to benign prostatic hyperplasia and first-
•6 degree heart block .
.7 Tricyclic antidepressant Benign prostatic hyperplasia Anticholinergic Doxepin Antidepressant Urinary retention Tachycardia Heart block Major depressive disorder

•8 Depression (mood) Anxiety Blood pressure Tricyclic Contraindication Benignity Hyperplasia

•9 D is not correct. 5°/o chose this .


• 10 Lith ium is prescribed f requently for patient s with bipolar disorde r. It has a narrow the rapeutic index, which
· 11 wou ld make it a poor choice in an elderly pat ient wit h comorbid ities . Adverse effect s of lit hium include
gastroint est ina l distress, polyu ria, dry mouth, t remor, rashes, and periphe ral edema .
• 12
Therapeutic index Bipolar disorder Polyuria Peripheral edema Tremor Xerostomia lithium Edema Comorbidity Human gastrointestinal tract Gastrointestinal tract
• 13
Gastrointestinal disease
• 14
• 15
• 16 Bottom Line:
• 17 I n patients with under lying arrhyt hmias, urinary retention, or g laucoma, t ricycl ic antidepressants wou ld be a
• 18
poo r choice fo r the t reatment of depression, because antichol inergic adverse effects can exacerbate all these
cond it ions. SSRis can be used to treat depression and have a relatively safe ca rdiac profile .
• 19 Anticholinergic Glaucoma Urinary retention Tricyclic antidepressant Selective serotonin reuptake inhibitor Tricyclic Antidepressant Cardiac arrhythmia
• 20 Major depressive disorder Depression (mood)

• 21

6
lock
s
Suspend
0
End Block
Item: 5 of 7 3 ~ 1 • M k -<:J 1>- Jil ~· !:';-~
QIO: 1485 ..L ar Pre v ious Next Lab fli!ltues Not es Calcula t o r
& &
1
FA17 p546.1
2
Monoamine oxidase Tranylcypromine, P henelzine, Isocarboxazid, Selegil inc (selecti,·e ~.\0-B inhibitor).
3
inhibitors (~lAO Takes Pride In Shanghai).
4
MECHANISM 'onselecti,·e .\llr\0 inhibition t b·cl~ of amine nemotransmitters (norepinephrine, 5-IIT,
5
dopamine).
•6
CliNICAL USE Atypical depression, an-:iet}. Parl.inson disease (selcgiline).
•7
ADVERSE EFFECTS C l$ stimulation; hrpertensive crisis, most notably ''ith ingestion of tyramine, which is found in
·8
many foods such as aged cheese and wine. Tyramine displaces other neurotransmitters (eg, NE)
.9
into the S}11aptic cleft - t sympathetic stimulation. Contraindicated with SSRls, TCAs, St. John's
• 10 wort, meperidine, dexiromethorphan (to prc,·ent serotonin syndrome).
• 11 Wait 2 \\·eeks after stopping MAO inhibitors before starting serotonergic drugs or stopping dietary
• 12 restrictions.
• 13
• 14 FA17 p 545.1
• 15
Selective serotonin Fluoxetine, Auvoxamine, paroxetine, scrtral inc, cscita lopmm, citalopram.
• 16 reuptake inhibitors
• 17 MECHANISM Inhibit 5-I IT reuptake. It normally lakes 4-8 weeks for antidepressants
• 18 CLINICAL USE Depression, generalized anxiety disorder, to have an effect.
• 19 panic disorder, OCD, bulimia, social amiety
• 20 disorder, PTSD, premature ejaculation,
• 21 premenstTual dysphoric disorder.

a
Lock
s
Suspend
8
End Bl ock
Item: 5 of 7 3 ~ 1 • M k -<:J 1>- Jil ~· !:';-~
QIO: 1485 ..L ar Pre v ious Next Lab fli!ltues Not es Calcula t o r
& &
1
FA17 p 545.1
2
Selective serotonin F'luoxetine, Am·oxamine, paroxetine, sert raline, escitalopram , citalopram.
3 reuptake inhibitors
4
MECHANISM Inhibit 5-HT reuptake. II normally takes 4-8 weeks for antidepressants
5 to have an effect.
CliNICAL USE De pression, gene ralized anxiety disorder,
•6
panic disorder, O C D , bulimia, social a n,iety
.7 disorde r, PTSD, premature ejaculation,
·8 premenstrual dysphoric disorde r.
.9 ADVERSE EFFECTS Fe\\ e r than T C s. C l d i~ tress, SIAO II, se, ual
• 10 dysfunction (anorgasmia, l Iibido).
• 11

• 12 FA17 p 545.4
• 13 Tricyclic Amitriptyline, nortriptyli ne, im iprami ne, desipra m ine, clomipramine, doxe pin, amoxapine.
• 14 antidepressants
• 15 MECHANISM In hibit E and 5-IIT reuptake.
• 16 CliNICAL USE Major depression, O C D (clomipramine), periphe ral neuropathy, chronic pain, migraine
• 17 prophylaxis. octurn <ll e nuresis {inlipram inc, allhough adve rse effects may limit use).
. 18 ADVERSE EFFECTS Sedation, a 1-blocking effects incl11ding postural hypote nsion, and atropine-like (anticholine rgic)
• 19 side effects (tachyca rdia, minary rete ntion, dry mouth). 3° T C As (amitriptyline) have more
• 20
anticholinergic effects than 2° T CJ\s (nortriptyline). Can prolong QT inte rval.
Tri-C's: C om·ulsions, C oma, C ardiotoxicity (arrhythmia due to ' a+ channel inhibition);
• 21
• a lso resnira ton· cle nression. lw nernvrexia. C:onfmion ancl ha llnc inations in elclerlv cine to

a
Lock
s
Suspend
8
End Bl ock
Item: 6 of 73 ~ 1 • M k -<:J 1>- Jil ~· !:';-~
QIO: 1126 ..L ar Pre v ious Next Labfli!llues Notes Calcula t o r

IAA]
& &
1
A 35-year-old woman was hospitalized for a manic episode several weeks ago . With medication, her mania
2
resolved, she slept through the night and was subsequently discharged. She now presents to her primary
3 care physician wit h complaints of frequent urination, weight gain, and tremor. Physical examination revea ls
4 an en larged thyroid gland.
5
•6 The drug responsible for these adverse reactions also causes which of the following congenital anomalies in a fetus
if it is taken by the pregnant mother?
•7

·8 :
A. Congenital deafness and notching of the teeth
.9
• 10 B. Congenital goiter
• 11 C. Ebstein anomaly
• 12
D. Ei senmenger syndrome
• 13
• 14
E. Neura l tube defects
• 15
• 16
• 17
• 18
• 19
• 20
• 21

a
Lock
s
Suspend
8
End Bl ock
Item: 6 of 73 ~. , . M k <:] t> al ~· ~
QIO: 1126 .l. ar Previous Next Lab 'lifllues Notes Calculator

1 •

2 The correct answer is C. 68°/o chose this.


3 Lithium is a first-l ine drug used in the treatment of bipo lar diso rder. In
4
addition, the patient's report of po lyuria (nephrogen ic diabetes insipidus),
weight gain (hypothyroidism), and tremo r is consistent with lithium toxicity.
5
In gene ral, lithium shou ld not be given to pregnant women because of the
6 r isk of Ebstein anomaly, a congenita l cardiac malformation in which the
•7 tricuspid valve is displaced inferiorly into the right ventricle ( illustrated in the
diagram) . On examination, there are widely split fi rst and second heart
•8
sounds due to right bundle branch block. A systol ic murmur from tricuspid Normal Ebstein
•9 regurg itation is also commonly heard.
Image courtesy of Wikimedia
• 10
Lithium can cause hypothyroidism by inhibiting thyroid hormone secretion, Commons
· 11 which in turn leads to TSH excess and subsequent pro liferation of the thyro id
• 12 gland (goiter). Euthyroid goite r can also occur with lithium usage, and overall, 40% -50 % of patients on lithium
develop goiter. Other adve rse effects of lithium incl ude polyd ipsia, gastric distress, and ataxia .
• 13
Hypothyroidism Bipolar disorder Polyuria Diabetes insipidus Goitre Polydipsia Thyroid hormone Tricuspid valve Diabetes mellitus Ventricle (heart) Thyroid
• 14
Vomiting Ataxia lithium (medication) Bundle branch block Tremor Thyroid-stimulating hormone Hormone Abdominal pain Congenital heart defect Heart sounds
• 15
Congenital disorder lithium Heart murmur Tricuspid insufficiency Systole Euthyroid Weight gain
• 16
• 17 A is not correct. 6°/o chose this .
• 18 The combination of Hutch inson teeth, described as central ly notched upper incisors, and congenita l deafness is
• 19 a key feature of congenital syph ilis syndrome . These findings often are accompanied by hydrocepha lus,
intellectual disability, and facial abnormalities such as frontal bossing, saddle-nose deformity, and maxillary
• 20
hypoplasia .
• 21 Hydrocephalus Syphilis Congenital syphilis Intellectual disability Hearing loss Hutchinson' s teeth Congenital disorder Incisor Hypoplasia Saddle nose

6
lock Suspend
s 0
End Block
Item: 6 of 73 ~. , . M k <:] t> al ~· ~
QIO: 1126 .l. ar Previous Next Lab 'lifllues Notes Calculator

1 •
B is not correct. 9°/o chose this.
2
Thyroid drugs such as rad ioactive iodine, potassium iodide, and propylthiouraci l (PTU) can cause congen ital
3 goiter in fetuses. However, the on ly drug that is absolutely contraind icated during pregnancy is radioactive
4 iodine, as the fetal thyroid is susceptible to ab lation. Radioative iodine treatment can be associated with so re
5 th roat, fatigue, and parotid gland swe lling. PTU is frequently given in the first trimester then switched to
methimazole to avoid liver damage in the pregnant woman . Methimazole is avoided in the fi rst trimester
6
because it has a highe r association with birth defects. PTU can also cause gastrointestinal symptoms, edema,
•7 and agranu locytosis. Potassium iod ide can be associated with gastrointestinal symptoms, acne, and swelling of
•8 the pa rotid glands .
Parotid gland Propylthiouracil Agranulocytosis Potassium iodide Methimazole Goitre Iodine Iodine-131 Thyroid Isotopes of iodine Acne vulgaris liver Edema
•9
Congenital disorder Iodide Potassium Fatigue (medical) Pregnancy Gastrointestinal tract
• 10

· 11 D is not correct. 6°/o chose this.


• 12 Eisenmenge r syndrome occu rs when an uncorrected left-to-right shunt (ventricu lar septal defect, atria l septal
• 13 defect or patent ductus arte riosus) leads to increased pulmonary blood flow, causing pulmonary arterial
hypertension. Subsequently the shunt becomes right-to-left, causing cyanosis. It is not directly associated with
• 14
lithium exposure. Risk factors for septa l defects include maternal alcoho l consumption or a chi ld with Down
• 15 syndrome .
• 16 Atrial septal defect Down syndrome Eisenmenger' s syndrome Cyanosis Patent ductus arteriosus Pulmonary hypertension Hypertension Ductus arteriosus

• 17 Cardiac shunt Atrium (heart) Shunt (medical) lithium

• 18 E is not correct. 11% chose this.


• 19 Valp roic acid is an effective medication for treating mania and absence seizu res, as well as generalized and
• 20 partial epilepsy. Its use is often contraindicated in pregnancy because of the risk of neura l tube defects. The risk
• 21
to the fetus may be reduced by fol ic acid supp lementation started 3 months prior to pregnancy and continued
• thrr>1 1nhr>11t th"' "'ntir"' nr.,nn::>nr\1 ()th"'r ::.rht.,rc::"' .,ff.,rtc:: ,-,f \l::> lnrnir ::>rirl i nrl, ,rJ., rl\lc::n.,nc::i::> "'"'inht n::>in/ lnc::c::

6
lock
s
Suspend
0
End Block
Item: 6 of 73 ~. , . M k <:] t> al ~· ~
QIO: 1126 .l. ar Previous Next Lab 'lifllues Notes Calculator

1 •
Eisenmenger syndrome occurs w hen an uncorrected left -to-right shunt (ventricu lar septal defect, atria l septal
2 defect or patent ductus arteriosus) leads to increased pulmonary blood flo w, causing pulmonary arter ial
3 hypertension. Subsequently the shunt becomes right-to-left, causing cyanosis. It is not di rectly associated with
4
lithium exposu re. Risk factors for septa l defects include maternal alcoho l consumption or a chi ld with Dow n
syndrome .
5 Atrial septal defect Down syndrome Eisenmenger' s syndrome Cyanosis Patent ductus arteriosus Pulmonary hypertension Hypertension Ductus arteriosus
6 Cardiac shunt Atrium (heart) Shunt (medical) lithium
•7
E is not correct. 11% chose this .
•8
Valproic acid is an effective medication for t reating mania and absence seizures, as well as generalized and
•9
partial epilepsy . I ts use is often contraindicated in preg nancy because of the risk of neura l tube defects. The risk
• 10 to the fetus may be reduced by fol ic acid supp lementation started 3 months prior to pregnancy and conti nued
· 11 throughout the entire pregnancy . Other adverse effects of valproic acid include dyspepsia, weight gain/ loss,
f atigue, edema, acne, dizziness, hair loss, and hy peram monemia leading to encephalopathy .
• 12
Valproate Folic acid Indigestion Epilepsy Hyperammonemia Absence seizure Acne vulgaris Epileptic seizure Encephalopathy Neural tube Edema Mania Dizziness
• 13
Fatigue (medical) Fetus Hair loss Neural tube defect Contraindication Pharmaceutical drug Pregnancy Partial seizure
• 14
• 15
• 16 Bottom Line:
• 17 Lithium is a fi rst-l ine dr ug used in t he t reatm ent of bipo lar disorder. I t has a narrow therapeutic window, so
• 18
drug leve ls must be m onitored careful ly. Lith ium is contra indicated in pregnancy as it has been associated with
Ebstein anoma ly in t he fetus .
• 19 Bipolar disorder lithium Fetus Contraindication Pregnancy
• 20
• 21

6
lock
s
Suspend
0
End Block
Item: 6 of 7 3 ~ 1 • M k -<:J 1>- Jil ~· !:';-~
QIO: 1126 ..L ar Pre v ious Next Labfli!llues Notes Calcula t o r
& &
1
2 FA17 p290.1
3 Congenital cardiac DISORDER DEFECT
4 defect associations Alcohol exposure in utero (fetal alcohol VSD, PDA, ASD, tetralogy of Fallot
5 syndrome)
6 Congenital rubella PDA, pulmonary artery stenosis, septal defects
0 7 Down svndrome V septal defect (endocardial cushion defect),

o8 VSD,ASD
.9 Infant of diabetic mother Transposition of great ,·essels
• 10 ~ larfan syndrome I P, thoracic aortic aneurysm and dissection,
• 11 aor tic regurgitation
• 12 Prenatal lithium exposure Ebstein anomalr
• 13 Turner syndrome Bicuspid aortic valve, coarctation of aorta
0 14 vVilliams synd rome Supravalvular ao rtic stenosis
• 15 22qll syndromes Truncus arteriosus, tetralogy of J:<allot
0
16
0
17
FA17 p 288.1
• 18 Congenital heart diseases
• 19
RIGHT-TO-lEfT SHUNTS Early cyanosis-"blue babies." Often diagnosed The 5 Ts:
• 20 prenatally or become e' ·ident immediatelr 1. Truncus arteriosus (1 ,·essel)
• 21 after birth. Usually require urgent surgical 2. Transposition (2 switched vessels)

a
Lock
s
Suspend
8
End Bl ock
Item: 6 of 7 3 ~ 1 • M k -<:J 1>- Jil ~· !:';-~
QIO: 1126 ..L ar Pre v ious Next Labfli!llues Notes Calcula t o r
& &
1
2 FA17 p288.1

3 Congenital heart diseases


4 RIGHI·IO·LEFT SHUNIS Early cyanosis-"blue babies." Often diagnosed The 5 Ts:
5
prenatally or become e\'ident immediately I. Truncus arteriosus (I ,·essel)
after birth. Usually require urgent surgical 2. Transposition (2 switched vessels)
6
treatment and/or maintenance of a PDA. 3. 'I ricuspid atresia (3 = Tri)
7
0
4. 'Jetralogy of Fallot (4 = 1e tra)
o8 5. T. P R (5 letters in the name)
.9 Persistent truncus Truncus arteriosus fails to di,•ide into
• 10 arteriosus pulmonary trunk and aorta due to lack of
• 11 aorticopulmonary septum formation; most
patients have accompanying VSD.
• 12
• 13
D-transposition of Aorta leaves RV (anterior) t~nd pulmont~ry trunk
great vessels leaves LV (posterior) .... separation of systemic
0 14
and pulmonary circulations. I ot compatible
• 15 with li fe unless a shunt is present to allow
0
16 mixing of blood (eg, VSD, PDA, or patent
0
17 foramen ovale). Right
ventricle
• 18
Due to failure of the aorHcopulmonary septum
to spiral.
• 19
Without surgical interYention, most infants die Ventricular
• 20 within the first few months of life. septum
• 21

a
Lock
s
Suspend
8
End Bl ock
Item: 6 of 7 3 ~ 1 • M k -<:J 1>- Jil ~· !:';-~
QIO: 1126 ..L ar Pre v ious Next Labfli!llues Notes Calcula t o r
& &
1
FA17 p268.2
2
Heart morphogenesis F'irst functional organ in vertebrate embryos; beats spontaneously by week 4 of development.
3
Cardiac looping Primary heart tube loops to cstabl ish left-right Defect in left-right dynein (invoked in LIR
4
polarity; begins in ""eek-+ of gestation. asymmetry) can lead to dexirocardia, as seen
5 in Kartagener syndrome (primary ciliary
6 dyskinesia).
0 7 Septation of the chambers
o8 Atria 6. (Not shown) Septum secundum and septum
0 Septum primum gro"s to\\ard endocardial
.9 cushions, narrowing foramen primum. primum fuse to fom1 the atrial septum.
• 10 @ Foramen secundum forms in septum 7. (t ot shown) Foramen ovale usuall) closes
• 11 primum (foramen primum disappears). soon after birth because of t L pressure.
• 12
E) Septum secundum develops as foramen Patent foramen ovale- causcd by f~t ilurc of
secundum maintains right-to-left shunt. septum primum and septum secundum
• 13
0 Septum secundum expands and covers most to fuse after birth; most arc left untreated.
14 of the foramen secundum. T he residual Can lead to paradoxical emboli (venous
0

• 15 foramen is the foramen ovale. thromboemboli that enter systemic arterial


0
16 0 Remaining portion of septum primum forms circulation), simi lar to those resulting from
0
17
\·alve of foramen ovale. an ASD.
. 18
Septum Foramen Developing
• 19 primum secundum
@ sepwm
secundum
• 20 Foramen
primum Dorsal ~ Foramen
• 21 endocardial Septum Septum
pnmum secundum ntimum
• r11<hinn

a
Lock
s
Suspend
8
End Bl ock
Item: 7 of 73 ~ 1 • M k -<:J 1>- Jil ~· !:';-~
QIO: 1482 ..L ar Pre v ious Next Lab fli!ltues Not es Calcula t o r

IAA]
& &
1
A 42-year-old man presented to his primary care physician severa l months ago w ith compla ints of fatigue,
2
loss of appetite, difficulty concentrating, decreased sleep, and feel ings of worthlessness. He has been
3 receiving pharmacologic treatment for these symptoms and has now begun to develop urinary retention,
4 blurred vision, constipation, and aggravation of his glaucoma.
5
6 Which of the following drugs is this patient most likely taking?
0 7 :
o8 A . Amitriptyl ine
.9 B. Bupropion
• 10
C. Fluoxetine
• 11
D. Phenelzine
• 12
• 13 E. Trazodone
0 14
• 15
0
16
0
17
• 18
• 19
• 20
• 21

a
Lock
s
Suspend
8
End Bl ock
Item: 7 of 73 ~. , . M k <:] t> al ~· ~
QIO: 1482 .l. ar Previous Next lab 'lifllues Notes Calculator

1 •

2 The correct answer is A. 62°/o chose this.


3 This patient is experiencing the atropine-l ike anticho linergic adverse events associated with t r icyclic
antidepressants (TCAs). TCAs, which include nortriptyline and amitriptyline, inhibit neu rona l uptake of
4
norepinephrine and serotonin and also block musca rinic, a-adrenerg ic, serotonergic, and histamine receptors.
5 This leads to various other adverse effects, including orthostatic hypotension, sedation, and anticholinerg ic
6 symptoms .
Anticholinergic Orthostatic hypotension Amitriptyline Nortriptyline Norepinephrine Histamine Hypotension Serotonin Serotonergic Tricyclic antidepressant
7
Muscarinic acetylcholine receptor Sedation Antidepressant Histamine receptor Tricyclic
•8
•9 B is not correct. 9°/o chose this .
• 10 Bupropion is a heterocycl ic antidepressant that can also assist in smoking cessation . It has been known to cause
· 11
dry mouth and tachycardia, but it would not be responsib le for the other anticholinerg ic effects that th is patient
is experiencing .
• 12 Anticholinergic Bupropion Antidepressant Smoking cessation Tachycardia Xerostomia
• 13
C is not correct. 11% chose this .
• 14
Fluoxetine, a selective serotonin reuptake inhibitor, does not have anticho linergic properties and wou ld not
• 15
cause such adverse effects. Its most common associated adverse eff ects are t remor, anorexia, and nausea .
• 16 Selective serotonin reuptake inhibitor Anticholinergic Fluoxetine Serotonin Serotonin reuptake inhibitor Tremor Nausea Reuptake Anorexia nervosa

• 17 Anorexia (symptom) Reuptake inhibitor Enzyme inhibitor

• 18
Dis not correct. 11% chose this .
• 19
Phenelzine, a monoamine oxidase inhibitor, wou ld not cause antichol inergic adverse effects such as these
• 20 because it has no activity at cholinergic neurons. Its most dangerous adverse effect is the deve lopment of
• 21 hypertensive crisis when tyramine-containing foods, such as cheese or red wine, are consumed while taking
... ·-·-- ·- - •- :._-
6
lock
s
Suspend
0
End Block
Item: 7 of 73 ~. , . M k <:] t> al ~· ~
QIO: 1482 .l. ar Previous Next lab 'lifllues Notes Calculator

1 •
B is not correct. 9°/o chose this.
2
Bupropion is a heterocycl ic antidepressant that can also assist in smoking cessation. It has been known to cause
3 dry mouth and tachycardia, but it would not be responsib le for the other anticholinerg ic effects that th is patient
4 is experiencing.
Anticholinergic Bupropion Antidepressant Smoking cessation Tachycardia Xerostomia
5
6 C is not correct. 11% chose this.
7 Fluoxetine, a selective serotonin reuptake inhibitor, does not have anticho linerg ic properties and wou ld not
•8 cause such adverse effects. Its most common associated adverse effects are tremor, anorexia, and nausea .
Selective serotonin reuptake inhibitor Anticholinergic Fluoxetine Serotonin Serotonin reuptake inhibitor Tremor Nausea Reuptake Anorexia nervosa
•9
Anorexia (symptom) Reuptake inhibitor Enzyme inhibitor
• 10

· 11 Dis not correct. 11% chose this.


• 12 Phenelzine, a monoamine oxidase inhibitor, wou ld not cause antichol inergic adverse effects such as these
because it has no activity at cholinerg ic neurons. Its most dangerous adverse effect is the deve lopment of
• 13
hypertensive crisis when tyramine-containing foods, such as cheese or red wine, are consumed while taking
• 14 phenelzine.
• 15 Monoamine oxidase inhibitor Phenelzine Anticholinergic Hypertensive crisis Cholinergic Hypertensive emergency Monoamine oxidase Monoamine neurotransmitter

• 16 Neuron Adverse effect Oxidase Enzyme inhibitor

• 17
E is not correct. 7°/o chose this .
• 18 Trazodone inh ibits the reuptake of serotonin and also significantly blocks histamine (H 1 ) and alpha 1-adregnergic
• 19 receptors. I t wou ld not affect the activity of cholinerg ic receptors and thus could not cause such symptoms. I t is
• 20 most commonly associated with adverse effects such as sedation, priapism, and postural hypotension .
Priapism Serotonin Trazodone Histamine Cholinergic Orthostatic hypotension Hypotension Sedation Reuptake Receptor (biochemistry)
• 21

6
lock
s
Suspend
0
End Block
Item: 7 of 73 ~. , . M k <:] t> al ~· ~
QIO: 1482 .l. ar Previous Next lab 'lifllues Notes Calculator

1
• • - - - - - 'f- - • • - •• - . - • • •• - • - 'f- - • - • • •• - - - • lt.. . .
cause such adverse effects. Its most common associated adverse effects are tremor, anorexia, and nausea.
2 Selective serotonin reuptake inhibitor Anticholinergic Fluoxetine Serotonin Serotonin reuptake inhibitor Tremor Nausea Reuptake Anorexia nervosa

3 Anorexia (symptom) Reuptake inhibitor Enzyme inhibitor

4
Dis not correct. 11% chose this.
5
Phenelzine, a monoamine oxidase inhibitor, wou ld not cause antichol inergic adverse effects such as these
6 because it has no activity at cholinerg ic neurons. Its most dangerous adverse effect is the deve lopment of
7 hypertensive crisis when tyramine-containing foods, such as cheese or red wine, are consumed while taking
•8 phenelzine .
Monoamine oxidase inhibitor Phenelzine Anticholinergic Hypertensive crisis Cholinergic Hypertensive emergency Monoamine oxidase Monoamine neurotransmitter
•9
Neuron Adverse effect Oxidase Enzyme inhibitor
• 10

· 11 E is not correct. 7°/o chose this.


• 12 Trazodone inh ibits the reupta ke of serotonin and also significantly blocks histamine (H1 ) and alpha 1-adregnergic
receptors. I t wou ld not affect the activity of cholinerg ic receptors and thus could not cause such symptoms. I t is
• 13
most commonly associated with adverse effects such as sedation, priapism, and postural hypotension .
• 14 Priapism Serotonin Trazodone Histamine Cholinergic Orthostatic hypotension Hypotension Sedation Reuptake Receptor (biochemistry)

• 15
• 16
Bottom Line:
• 17
TCAs are known for their antichol inergic adverse effects ( urinary retention, dry mouth, tachycard ia,
• 18
constipation, and blurry vision ). Tertiary TCAs like amitriptyline tend to have more anticholinerg ic effects than
• 19 secondary TCAs like nortriptyline. Elderly patients are particularly susceptible to these adverse effects.
• 20 Anticholinergic Amitriptyline Nortriptyline Constipation Tachycardia Tricyclic antidepressant Xerostomia

• 21

6
lock
s
Suspend
0
End Block
Item: 7 of 73 ~ 1 • M k -<:J 1>- Jil ~· !:';-~
QIO: 1482 ..L ar Pre v ious Next Lab fli!ltues Not es Calcula t o r
& &
1
2 FA17 p 545.4
3 Tricyclic Amitriptyline, nortriptyline, imipram ine, desipramine, clomipramine, doxepin, amoxapinc.
4 antidepressants
5 MECHANISM Inhibit NE and )-liT re uptake.
6 CLINICAL USE ~lajor depression , OCD (clom ipramine), peripheral neuro pathy, c hronic pain, m igraine
7 prophylaxis. l'\octu rnal e nuresis (imipramine, although ad,·e rse effects may limit use).
·8 ADVERSE EFFECTS Sedation, a 1-blocking effects includi ng postural hypotension, and atropine-like (anticholinergic)
.9 side effects (tachyca rdia, urinary rete ntion, d r~ mouth). 3° T CAs (amitriptyline) have more
• 10 anticholine rgic effects than 2° TCAs (nortriptyline). Can prolong QT inten·al.
Tri-C's: C onvulsions, C om a, C ardiotoxicity (arrhythmia due to 1 a+ channel inhibition);
• 11
also respiratory depression, hype rpyrexia. Confusion and hallucinations in elde rl y due to
• 12
anticholinergic side effects (nortript)·line beller tolerated in the elde rly). Treatment: 'a11C03 to
• 13 preve nt arrhythmia.
• 14
• 15
FA17 p 545.1
• 16
Selective serotonin F'luoxetine, Auvoxamine, paroxetine, sertral ine, escitalo pram, citalopram.
• 17 reuptake inhibitors
• 18
MECHANISM Inhibit 5-HT re uptake. It normally takes 4-8 weeks for antide pressants
• 19 to have an effect.
CLINICAL USE De pression, generalized anxiety disorder,
• 20 panic disorde r, O C D, bulim ia, social am.iety
• 21 disorder, PT SD, prematme ejaculation,

a
Lock
s
Suspend
8
End Bl ock
Item: 7 of 73 ~ 1 • M k -<:J 1>- Jil ~· !:';-~
QIO: 1482 ..L ar Pre v ious Next Lab fli!ltues Not es Calcula t o r
& &
1
FA17 p545.1
2
Selective serotonin F'luoxetine, Atwoxamine, paroxeline, sertraline, escitalopram , citalopram.
3
reuptake inhibitors
4
MECHANISM Inhibit 5-HT reuptake. It no rmally takes 4-8 weeks for antidepressants
5
CLINICAL USE Depression, generalized am.iety disorder, to ha,·e an effect.
6
panic disorder, O C D, bulimia, social anxiety
7
disorder, PTSD, premature ejaculation,
·8 premenstrual dysphoric disorder.
.9 ADVERSE EFFECTS Fe\\ Cr than T CAs. Gl distress, SIJ\011 , se,ual
• 10 dysfunction (anorgasmia, l Iibido).
• 11
• 12
FA17 p 546.1
• 13
Monoamine oxidase Tranylcypromine, P henelzine, lsoc~1rhoxa zid, Sclegilinc (selecti,·e YIAO-B inhibitor).
• 14 inhibitors {MAO Takes P ride In Shanghai).
• 15
MECHANISM Nonselective YfAO inhibition t levels of amine neurotransm itters {norepinephrine, 5-ITT,
• 16 dopami ne).
• 17 CLINICALUSE Atypical depression, anxiety. Parkinson disease (selegiline).
• 18
ADVERSE EFFECTS C S stimulation; hyp ertensive crisis, most notably with ingestion of tyramine, which is found in
• 19 many foods such as aged cheese and wine. Tyrami ne displaces other neurotransmitters (cg, 1 E)
• 20 into the synaptic cleft - t sympathetic stimulation. Contraindicated with SSRls, T CAs, St. John's
• 21 wort, meperidine, dextromethorphan (to pre,·ent serotonin syndrome).
• , ' 1 · • .., r. ' •, ~ · 1 · 1 •• 1 r . .· · 1 1· •

a
Lock
s
Suspend
8
End Bl ock
Item: 7 of 73 ~ 1 • M k -<:J 1>- Jil ~· !:';-~
QIO: 1482 ..L ar Pre v ious Next Lab fli!ltues Not es Calcula t o r
&
.. .. . . &
1
MECHANISM Inhibit 5-HT reuptake. It normally takes 4-8 weeks for antidepressants
2
CLINICAL USE Depression, generalized anxiety disorder, to have an effect.
3
panic disorder, OCD, bulimia, social am.iety
4
disorder, PTSD, premature ejaculation,
5 premenstrual dysphoric disorder.
6 ADVERSE EFFECTS Fe"er than TCAs. Gl distress, SIADII, SC\ual
7 drsfunction (anorgasmia, l libido).
·8
.9
FA17 p 546.1
• 10
Monoamine oxidase Tranylcypromine, Phenelzine, lsocarboxazid, Sclegili nc (selecti,·e ~1AO -B inhibitor).
• 11
inhibitors ( ~ lAO Takes Pride In Shanghai).
• 12
MECHANISM t'-:onselecti,·e .\!JAO inhibition t le\'cls of amine neurotransmitters (norepinephrine, 5-HT,
• 13 dopamine).
• 14
CLINICAL USE Atypical depression, anxiety. Parkinson disease (sclegil ine).
• 15
ADVERSE EFFECTS C1 S stimulation; hypertensive crisis, most notably with ingestion of tyramine, wh ich is found in
• 16 many foods such as aged cheese and wine. Tyramine displaces other neurotransmitters (cg, E)
• 17 into the synaptic cleft - t sympathetic stimulation. Contraindicated with SSRls, TCAs, St. Joh n's
. 18 wort, meperidine, dextromethorphan (to prevent seroton in syndrome) .
• 19 Wait 2 weeks after stopping .\!JAO inhibitors before starting serotonergic drugs or stopping dietary
restrictions.
• 20
• 21

a
Lock
s
Suspend
8
End Bl ock
Item:8of73 ~. , . M k <:] t> al ~· ~
QIO: 1133 .l. ar Previous Next lab 'lifllues Notes Calculator

1 •
A 43-year-old woman in a psychiatry ward is adm itted to a med ical floor after attempting suicide by
2
med ication overdose. She states that she recently broke up (for the fifth t ime) with her boyfr iend, who
3 disapproves of her cocaine and ma r ijuana use. She also states that she has made " hundreds" of su icide
4 attempts and has been hospital ized many t imes . She is very compliant du r ing the interview as she states that "I
on ly like doctors, not nu rses because only doctors understand mel" Physical examination reveals tachycardia,
5
reddened conjunctivae, and an eroded nasa l septum.
6
7
Wh ich of the fol lowing defense mechanisms is most closely associated with this patient's condition?
•8
:
•9
A. Denia l
• 10
B. Humor
· 11
• 12 C. Reaction formation
• 13 D . Splitting
• 14
E. Subl imation
• 15
• 16
• 17

• 18
• 19
• 20
• 21

6
lock
s
Suspend
0
End Block
Item:8of73 ~. , . M k <:] t> al ~· ~
QIO: 1133 .l. ar Previous Next lab 'lifllues Notes Calculator

1 • •
The correct answer is D. 75°/o chose this.
2
Spl itting is a defense mechan ism associated with bo rder line personal ity diso rder (BPD) . Patients with BPD often
3 engage in self-destructive behaviors such as cutting or burning themse lves with cigarettes. They have
4 extremely labile moods, impulsivity, and unstable relationships. Splitting involves the either/or categorization of
5 things as all good or al l bad. The patient might state that the male resident is the only doctor who has eve r
unde rstood her, and that al l the other doctors are ignorant.
6 Borderline personality disorder Defence mechanisms Personality disorder Impulsivity lability
7
A is not correct. 4°/o chose this.
8
Denia l involves the refusa l to accept a real ity that a pe rson deems unsettl ing. For instance, a person may write
•9
off persistent negative job performance reviews rather than accepting that his wor k abilities are deficient .
• 10 Job performance

· 11
B is not correct. 3°/o chose this .
• 12
Humor is a matu re defense mechan ism wh ich involves expressing uncomfortable emotions in a comfortab le way
• 13 (ie, as funny) : an overweight patient might ma ke jokes about "fat people."
• 14 C is not correct. 11% chose this .
• 15 Reaction formation has been ca lled unconscious hypocrisy. The person adopts opposite attitudes to unconscious
• 16 emotions. A burnt out docto r who dislikes medicine may spend his t ime encouraging med ical students in their
• 17
professional development .
Reaction formation Hypocrisy
• 18
• 19 E is not correct. 7°/o chose this .
• 20
Sublimation is a matu re defense mechanism that involves the expression of uncomfortable emotions (anger) in
a social ly useful way, such as becoming an advocate for socia l justice .
• 21 Sublimation (psychology) Sublimation (phase transition) •

6
lock
s
Suspend
0
End Block
Item:8of73 ~. , . M k <:] t> al ~· ~
QIO: 1133 .l. ar Previous Next lab 'lifllues Notes Calculator

1 •
Deni al involves the refusal t o accept a rea lit y that a person deems unsettl ing. For inst ance, a person may write
2 off persistent negative job performance reviews rather than accepting that his wor k abi lit ies are deficient .
3 Job performance

4 B is not correct. 3°/o chose this.


5 Humor is a mat ure defense mechanism wh ich involves expressing uncomfortable emotions in a comfortable way
6 ( ie, as funny) : an overweight patient might make jokes about "fat people. "
7 C is not correct . 11% chose this.
8 React ion formation has been called unconscious hypocrisy. The person adopts opposite attitudes to unconscious
•9
emotions. A bu r nt out doctor who disl ikes medicine may spend his t ime encou raging med ica l students in their
professional development .
• 10 Reaction formation Hypocrisy
· 11
E is not correct. 7°/o chose this .
• 12
Subl imation is a mat ure defense mechanism that involves the expression of uncomfortable emotions (anger ) in
• 13
a socia lly useful way, such as becoming an advocate for social j ustice .
• 14 Sublimation (psychology) Sublimation (phase transition)

• 15
• 16
Bottom Line:
• 17
Individuals with border line personality disorder are characterized by their labile mood and unstable
• 18
relationships. They often engage in self -destr uctive behaviors in an attempt to gain attention . Splitting is a
• 19 maladaptive coping mechan ism employed by t hese individuals that paints others as al l good or all bad .
• 20 Borderline personality disorder Emotional dysregulation Coping (psychology) Personality disorder lability Adaptive behavior
• 21

6
lock
s
Suspend
0
End Block
Item:8of73 ~. , . M k <:] t> al ~· ~
QIO: 1133 .l. ar Previous Next lab 'lifllues Notes Calculator

1 • •
FA17 p 524.4
2
Ego defenses [ental processes (unconscious or conscious) used to resolve confl ict and JXe,·ent undesirable
3
feelings (eg, anxiety, depression).
4
IMMATURE DEFENSES DESCRIPTION EXAMPLE
5
Acting out Expressing unacceptable feelings and thoughts Tantrums.
6
through actions.
7
Denial Avoiding the awareness of some painful reality. A patient with cancer plans a full-time work
8 schedule despite being warned of significant
•9 fatigue during chemotherapy.
0 10 Displacement Redirection of emotions or impulses to a neutral A teacher is yelled at by the principa I. Instead of
· 11 person or object (vs projection). confronting the principal directly, the teacher
• 12 goes home and criticizes her husband's dinner
• 13
selection .
• 14 Dissociation Temporary, drastic change in personality, A victim of sexual abuse suddenly appears numb
memory, consciousness, or motor behavior to and detached when she is exposed to her
0 15
avoid emotional stress. Patient has incomplete abuser.
• 16
or no memory of traumatic event.
• 17
Fixation Partially remaining at a more childish level of A surgeon throws a tantrum in the operating
• 18 development (vs regression). room because the last case ran very lal·e.
• 19 Idealization Expressing extremely positi,·e thoughts of self A patient boasts about his physician and his
0 20 and others while ignoring negative thoughts. accomplishments while ignoring any Aaws.
• 21 Identification Largely unconscious assumption of the A resident starts putting his stethoscope in his •

6
lock Suspend
s 0
End Block
Item: 8 of 7 3 ~ 1 • M k -<:J 1>- Jil ~· !:';-~
QIO: 1133 ..L ar Pre v ious Next Labfli!llues Not es Calcula t o r
& &
1 Idealization Expressing extremely positive thoughts of self A patient boasts about his physician and his
2 and others while ignoring negal i' e thoughts. accomplishments while ignoring any na\\ s.
3 Identification Largely unconscious assumption of the A resident starts putting his stethoscope in his
4 characteristics, qualities, or trails of another pocket like his favorite attending, instead of
5
person or group. wearing it around his neck like before.
6
Intellectualization Using facts and logic to emotionall) distance In a therapy session, patient diagnosed'' ith
oneself from a stressful situation. cancer focuses onh on rates of survi,·al.
7
Isolation (of affect) Separating feelings from ideas and C\Cnts. Describing murder in graphic detail "ith no
8
emotional response.
.9
Passive aggression Demonstrating hostile feelings in a Disgruntled employee is repeatedly late to work,
• 10
nonconfrontational manner; sho'' ing indirect but won't admit it is a way to get back at the
• 11 opposition. manager.
• 12
Projection Attributing an unacceptable interna l impulse to A man "ho wants to cheat on his" ife accuses
• 13 an external source (vs displacement). his wife of being unfaithful.
• 14 Rationalization Proclaiming logical reasons for actions actually After getting fired, claiming that the job was not
• 15 performed for other reasons, usually to avoid important anyway.
• 16 self-blame.
• 17 Reaction formation Replacing a warded-off idea or feeling by an A patient with libidinous thoughts enters a
• 18 (unconsciously derived) emphasis on its monastery.
opposite (vs sublimation).
• 19
Regression Involuntarily turning back the maturational Seen in children under stress such as illness,
• 20
clock and going back to earlier modes of punishment, or birth of a new sibling (eg,
• 21
• dealine with the world (,s fi>.ationl. bed\\etline in a orevioush toilet-trained child
a
Lock
s
Suspend
8
End Bl ock
Item: 8 of 7 3 ~ 1 • M k -<:J 1>- Jil ~· !:';-~
QIO: 1133 ..L ar Pre v ious Next Labfli!llues Not es Calcula t o r
. .. . .
& &
1
Regression Involuntarily turning back the mat urationa I Seen in children under stress such as illness,
2
clock and going back to earlier modes of punishment, or birth of a new sibling (eg,
3 dealing with the world (' s fixa lion). bech,etting in a previously toilet-trained child
4 when hospitalized).
5 Repression lrwolunlarily withhold ing an idea or feeling A 20-year-old does not remember going to
6 from conscious awareness (vs suppression). counseling during his parents' di,orce 10 years
7 earlier.
8 Splitting Belie\'ing that people are either all good or A patient sars that all the nurses are cold and
.9
all bad at different times due to intolerance insensiti,·e but that the doctors are warm and
of ambiguity. Commonly seen in borderline friendly.
• 10
personality disorder.
• 11 MATURE DEFENSES
• 12 Sublimation Replacing an unacceptable wi~h with a course 1ccnagcr's aggression toward his fath er is
• 13 of action that is similar to the \\ ish but does redirected lo perform well in sports.
• 14 not conAict with one's value system (vs
• 15
reaction formation).
• 16 Altruism Alleviating negative feel ings via unsolicited Mafia boss makes large donation to charity.
generosity.
• 17
Suppression Intentionally withholding an idea or feeling from Choosing to not worry about the big game until
. 18
conscious awareness (vs repression); temporary. il is lime to play.
• 19
Humor Appreciating the amusing nature of an anxiety- l\ervous medical student jokes about the boards.
• 20
pro,·oking or ach-erse situation .
• 21
• l<>horp <>rlnlt< R'P<>r" '- .\ '-H
a
Lock
s
Suspend
8
End Bl ock
Item: 8 of 7 3 ~ 1 • M k -<:J 1>- Jil ~· !:';-~
QIO: 1133 ..L ar Pre v ious Next Labfli!llues Not es Calcula t o r

2
3 FA17 p 535.2
4 Cluster B personality Dramatic, emotional, or erratic; genetic "\\"ild."
5 disorders association with mood disorders and substance Cluster B: Bad, Borderline, AamBoyant, must
6 abuse. be the Best
7 An tisocial Disregard for and \'iolation of rights of others Anti~ocial =~ociopath.
8 with lack of remorse, criminal it}', impulsivitr; Bad.
males > females; must be~ 18 years old and
.9
ha,·e history of conduct disorder before age 15.
• 10
Conduct disorder if< 18 years old.
• 11
Borderline Unstable mood and interpersonal relationships, Treatment: dialectical beha\'ior therapy.
• 12 impulsivity, self-mutilation, suicidality, sense Borderline.
• 13 of emptiness; females > males; splilling is a
• 14 major defense mechanism .
• 15 Histrionic Excessive emotionality and excitabi lity, FlamBoyant.
• 16 attention seeking, sexually provoc<ttivc, overly
concerned with appearance.
• 17
Narcissist ic Grandiosity, sense of entitlement; lacks empathy 1ust be the Best.
• 18
and requires excessive admiration; often
• 19
demands the "best" and reacts to criticism
• 20 with rage.
• 21

a
Lock
s
Suspend
8
End Bl ock
Item:9of73 ~. , . M k <:] t> al ~· ~
QIO: 1469 .l. ar Previous Next lab 'lifllues Notes Calculator

1 •
A 39-year-old man is seen by a psychiatrist after reports that he has been locking himself in his apartment ~~AI
2 because "the devi l is trying to put thoughts into my head." He responded we ll to pharmacologica l
3 management initia lly and was able to resume norma l dai ly activities . However, after 3 weeks, he presents to
4 the hospita l with muscu lar rigidity and hyperthermia.
5
6 A med ication that is also used to t reat ma lignant hyperther mia is administered . What is the mechan ism of action of
this drug?
7

8 :

•9
• 10 A. Blocks y- aminobutyric acid receptor
· 11 B. Blocks dopamine D2 receptor
• 12
C. Potentiates y-aminobutyric acid effects by increasing the duration of ch loride channe l opening
• 13
• 14 D . Potentiates y-aminobutyric acid effects by increasing the frequency of chlor ide channel open ing
• 15 E. Uncouples muscle excitation - contraction cycles
• 16
• 17

• 18
• 19
• 20
• 21

6
lock
s
Suspend
0
End Block
Item:9of73 ~. , . M k <:] t> al ~· ~
QIO: 1469 .l. ar Previous Next lab 'lifllues Notes Calculator

1 •
The correct answer is E. 57°/o chose this.
2
This patient like ly has neu roleptic mal ignant syndrome, a serious adverse eff ect of antipsychotic agents. Classic
3
symptoms include hyperpyrexia, autonomic instabi lity, and severe muscle rigidity. Treatment requi res
4 immed iate discontinuation of al l neuroleptics, supportive care, and the administration of dantrolene. Dantro lene
5 uncoup les muscle excitation-contraction coup ling by binding to the ryanodine receptor and preventing
accumulation of intracell ular calcium that is needed to sustain contraction. Dantrolene is also indicated in
6
ma lignant hyperther mia.
7 Neuroleptic malignant syndrome Ryanodine receptor Dantrolene Malignant hyperthermia Hyperpyrexia Muscle contraction Antipsychotic Fever Hyperthermia

8 Dysautonomia Hypertonia Adverse effect Autonomic nervous system Calcium Intracellular Muscle Cancer Receptor (biochemistry)
9
A is not correct. 8°/o chose this.
0 10
This is the mechanism of action of f lumazenil. As a competitive antagonist at the y-aminobutyr ic acid receptor,
· 11 it is used to treat an overdose of benzodiazepines. It is not ind icated for the treatment of neuro leptic mal ignant
0
12 synd rome .
Neuroleptic malignant syndrome Flumazenil Benzodiazepine Receptor antagonist Mechanism of action Antipsychotic Competitive antagonist Cancer
0
13
0 14 B is not correct. 14% chose this.
0 15 This is the mechanism of typica l antipsychotic med ications such as haloperidol. Administration of additional
0
16 neuro leptic medications wou ld worsen the symptoms of neu roleptic mal ignant syndrome and therefore is
contra indicated.
0 17
Neuroleptic malignant syndrome Typical antipsychotic Haloperidol Antipsychotic Contraindication Cancer Malignancy
0
18
C is not correct. 10% chose this.
0 19
This is the mechanism of action of phenobarbital , a barbitu rate. Barbitu rates bind the y-aminobutyric acid
20
( GABA)A receptor at a site unique from both the GABA and benzodiazepine binding sites. Phenobarbita l would
0

0
21

not be helpfu l in a patient who is experiencing neuroleptic malignant synd rome. It is used as a third-line agent

6
lock
s
Suspend
0
End Block
Item:9of73 ~. , . M k <:] t> al ~· ~
QIO: 1469 .l. ar Previous Next lab 'lifllues Notes Calculator

1 Neuroleptic malignant syndrome Typical antipsychotic Haloperidol Antipsychotic Contraindication Cancer Malignancy

2
C is not correct. 10% chose this.
3
This is the mechanism of action of phenobarbital, a barbitu rate. Barbitu rates bind the y -aminobutyric acid
4 (GABA)A receptor at a site unique from both the GABA and benzodiazepine binding sites. Phenobarbital would
5 not be helpfu l in a patient who is experiencing neuroleptic malignant synd rome. It is used as a third-line agent
6
for status epilepticus.
Benzodiazepine Neuroleptic malignant syndrome Barbiturate Phenobarbital Status epilepticus Gamma-Aminobutyric acid Mechanism of action Antipsychotic
7
Receptor (biochemistry) Cancer Malignancy
8
9 Dis not correct. 11% chose this.
0 10
This is the mechanism of action of benzodiazepines (eg, diazepam). They bind the y-aminobutyric acid (GABA)A
receptor at a site unique from GABA itself and potentiate the effects of this neurotransmitter. Benzodiazepines
· 11 are not indicated for the treatment of neuroleptic ma lignant synd rome. Diazepam is the first-line treatment for
0
12 status epi lepticus and is used in most alcohol withdrawal protocols.
Neuroleptic malignant syndrome Diazepam Status epilepticus Neurotransmitter Gamma-Aminobutyric acid Alcohol withdrawal syndrome Mechanism of action
0
13
0 14 Benzodiazepine Antipsychotic Alcohol Alcoholic beverage Cancer First-line treatment Therapy Malignancy Receptor (biochemistry)

0 15
0
16 Bottom Line:
0 17
Neuroleptic mal ignant syndrome is a serious and sometimes fatal adverse effect of antipsychotic use. It is
0
18 character ized by hyperthermia, muscle rigidity, and autonomic instability. I f it is suspected, al l neuroleptics
0 19 should be discontinued immediately and dantrolene shou ld be administered intramuscularly .
Neuroleptic malignant syndrome Dantrolene Hyperthermia Antipsychotic Dysautonomia Hypertonia Adverse effect Intramuscular injection
0 20
Autonomic nervous system Cancer Malignancy
0
21

6
lock
s
Suspend
0
End Block
Item: 9 of 7 3 ~ 1 • M k -<:J 1>- Jil ~· !:';-~
QIO: 1469 ..L ar Pre v ious Next Lab fli!ltues Notes Calcula t o r
& &
1
FA17 p 543.1
2 Typical antipsychotics Haloperidol, pimozide, triAuopera.rinc, Auphena:.-ine, thiorida~in e, chlorprom a~in e.
3
MECHANISM Block dopamine 0 2 receptor (f c 1 P).
4
CLINICAL USE Schizophrenia (1° positi,·c symptoms), psychosi~, bipolar disorder, delirium, Tourcttc syndrome,
5 Huntington disease, OCD.
6
POTENCY H igh potency: Trifluoperazine, fluphena7ine, llaloperidol (Try to FJ~, Iligh)-neurologic side
7 effects (eg, extrapyramidal S) mptoms [ EPS]).
8 Low potencr: C h lorproma7ine, T h iorida7ine (C heating Th ieves are low)-anticholinergic,
9 antihistamine, a 1-blockade effects.

• 10 ADVERSEEFFECTS Lipid soluble .... stored in body fat .... slow to be removed from body.
• 11 Endocrine: dopamine receptor antagonism .... hypc rprolactincmia .... galactorrhea,
• 12 oligomenorrhea, gynccomast i<L
• 13 l\letabolic: dyslipidemia, weight gain, hyperglycemia .
Antimuscarinic: dry mo uth, constipation .
• 14
Antihistamine: sedation .
• 15
a 1-blockade: orthostatic hypotension.
• 16 Cardiac: QT prolongation .
• 17 Ophthalmologic: C hlorpromazine- C orneal deposits; T hioridazine- reT inal deposi ts.
. 18 EPS- ADAPT:
• 19 Hours to days: Acute D ystonia (muscle spasm, stiffness, oculogyric crisis).
• 20 Days to months: \ kathisia (restlessness), Parkinsonism (bradykinesia).
Months to years: Tardive dyskinesia (orofacial chorea).
• 21
• 'T'ra.""h-nAnt-· h A t'l? frnn;n A (,..,.. .. t A ,1, dnn;,.. l 11rrliv~ rl, <'lrinAc-i.,, h An?nAi'l?A•'\inAC' fLl,Jn,..l..A r ('t

a
Lock
s
Suspend
8
End Bl ock
Item: 9 of 7 3 ~ 1 • M k -<:J 1>- Jil ~· !:';-~
QIO: 1469 ..L ar Pre v ious Next Lab fli!ltues Notes Calcula t o r

1
&
Treatment: benztropine (acute dyslon ia, lard ive d) skinesia}, benzodiazepines, ~-bl ockers &

2 (akathisia).
3 Neuroleptic malignant syndrome (NMS)- \ 1alignant FEVER: :\lyoglobinuria, Fever,
4 Encephalopathy, unstable Vitals, f Enzymes, muscle Rigidity. Treatment: dantrolene, 0 2 agonist
5
(eg, bromocriptine).

6
7 FA17 p 520.2

8 Dantrolene
9 MECHANISM Prevents release of Ca 2+ from the sarcoplasmic reticulum of skeletal muscle by binding to Ihe
• 10
ryanodine receptor.
• 11 CLINICAl USE ~ lalignant hyperthermia and neuroleptic malignant S) ndrome (a toxicity of antipsychotic drugs).
• 12
• 13 FA17p515.1
• 14 Barbiturates Phenobarbital, pentobarbital, thiopental , secobarbital.
• 15 MECHANISM Facil itate GAB!\A action by f dure~tion of Cl- channel opening, thus l neuron firing (barbiduratcs
• 16 f duration). C ontraindicated in porphyria .

• 17 CliNICAl USE edative for anxiety, seizures, insomnia, induction of anesthesia (thiopenta l).
• 18 ADVERSE EFFECTS Respiratory and cardiovascular depression (can be fatal ); CNS depression (can be exacerbated by
• 19 alcohol use); dependence; drug interactions (induces cytochrome P-450).
Overdose treatment is supportive (assist respiration and maintain BP).
• 20
• 21

a
Lock
s
Suspend
8
End Bl ock
Item: 10 of 73 ~ 1 • M k -<:J 1>- Jil ~· !:';-~
QIO: 1152 ..L ar Pre v ious Next Labfli!llues Not es Calcula t o r

IAA]
& &
1
A 42-year-old man presents to the local crisis center requesting alcohol detoxifica t ion . He has a 20-year
2
history of heavy drinking, wit h the longest period of abstinence being 4 months. His last drink was 2 nights
3 ago, and he now complains of discomfort and anxiety. Physica l examination revea ls coarse tremors, facial
4 flush ing, palmar erythema, and spider angiomas. His blood pressure is m ildly elevated at 145/95 mm Hg, he is
tachycardic at 115/min, and he has a temperature of 38.3°C ( 100.9°F). Thiamine is admin istered.
5
6
Which of the following drugs is the best treatment of this patient's condition?
7
8 :
A. Chlordiazepoxide
9
• 10 B. Disulfiram
• 11 C. Haloperidol
• 12
D. Lorazepam
• 13
• 14
E. Methadone
• 15 F. Na ltrexone
• 16
• 17
• 18
• 19
• 20
• 21

a
Lock
s
Suspend
8
End Bl ock
Item: 10 of 73 ~. I • M k <:] t> al ~· ~
QIO: 1152 .l. ar Previous Next lab 'lifllues Notes Calculator

1 •

2 The correct answer is D. 51°/o chose this.


3 This patient is showing signs of alcohol withdrawa l (manifested by tachycard ia, fever, nausea, vomiting,
4 tremors, and hypertension) and is at risk for del irium t remens . Delirium tremens is an extreme and life-
threatening form of withdrawa l, characterized by perceptual disturbances and confusion. I ntravenous
5
benzodiazepines are indicated in the t reatment of both mild withdrawal and de lirium tremens. Used early, they
6 can prevent progression to withdrawal-induced seizures, psychosis, and coma via stimulation of GABA
7 receptors. Given the patient's liver disease (evidenced by palma r erythema and spider angiomas), he should be
8
given lorazepam, oxazepam, or temazepam, because these drugs undergo phase II metabol ism and are not
contra indicated in patients with liver disease. You can remember them with the mnemonic OTL : Out The Liver;
9
Oxazepam, Temazepam, Lorazepam.
10 Temazepam lorazepam Oxazepam Delirium tremens Alcohol withdrawal syndrome Gamma-Aminobutyric acid Tachycardia Benzodiazepine Psychosis Delirium

· 11 Hypertension Intravenous therapy Nausea Erythema Vomiting liver Coma Epileptic seizure Metabolism Alcoholic beverage Alcohol Clinical trial GABA receptor

• 12 Drug metabolism liver disease Fever

• 13
A is not correct. 18% chose this .
• 14
Although chlordiazepoxide is a long-acting benzodiazepine that can be used to treat alcohol withd rawal and
• 15 de lir ium tremens, chlordiazepoxide shou ld be avoided in patients with liver disease since its metabolism is liver-
• 16 dependent. Therefore, intermed iate-acting benzodiazepines such as lorazepam that do not requi re liver
• 17
metabolism are the best choice for th is patient .
Benzodiazepine lorazepam Delirium tremens Chlordiazepoxide Alcohol withdrawal syndrome Delirium Alcohol liver Metabolism Alcoholic beverage
• 18
• 19 B is not correct. 12% chose this .
• 20
Disulfiram inhibits acetaldehyde dehyd rogenase, wh ich causes accumu lation of aceta ldehyde with ingestion of
alcohol. This bu ildup of alcoho l byproducts leads to extremely unpleasant adverse effects, includ ing f lush ing,
• 21 headache, diaphoresis, nausea, and vomit ing. Essentially, disulfiram produces many of the effects of a •

6
lock
s
Suspend
0
End Block
Item: 10 of 73 ~. I • M k <:] t> al ~· ~
QIO: 1152 .l. ar Previous Next lab 'lifllues Notes Calculator

1 •
B is not correct. 12% chose this.
2 Disulfiram inhibits acetaldehyde dehyd rogenase, wh ich causes accumulation of aceta ldehyde with ingestion of
3 alcohol. This bu ildup of alcohol byproducts leads to extremely unpleasant adverse effects, includ ing flush ing,
4
headache, diaphoresis, nausea, and vomiting. Essentially, disulfiram produces many of the effects of a
"hangover," which discourages alcohol ics from drinking, therefore helping them ma intain sobriety. Although
5
disulfiram would help the patient with his chronic alcoholism, he is presenting acutely to the t reatment center
6 with delir ium tremens. Therefore, he would benefit from an I V benzodiazepine, with special consideration given
7 to his liver disease.
Benzodiazepine Delirium tremens Disulfiram Acetaldehyde Acetaldehyde dehydrogenase Perspiration Nausea Headache Vomiting Delirium liver Alcohol
8
Alcoholism liver disease Alcoholic beverage Flushing (physiology)
9
10 C is not correct. 4°/o chose this.
· 11 Haloperidol is a typical antipsychotic . It can be used in patients withd rawing from alcohol who suffer psychotic
symptoms such as hallucinations .
• 12
Typical antipsychotic Haloperidol Antipsychotic Psychosis Hallucination Alcoholic beverage Alcohol
• 13
• 14 E is not correct. 5°/o chose this .
• 15
Methadone is a potent, long -acting opioid agonist used in the treatment of opioid addiction. This patient is an
alcoholic and does not requ ire a methadone taper.
• 16 Methadone Opioid Agonist Opioid use disorder Alcoholism
• 17
F is not correct. 10% chose this .
• 18
Naltrexone is an opioid antagonist used to help mainta in opioid sobr iety. I t also is used to help prevent alcohol
• 19 relapses in alcohol dependence. This patient, however, requi res acute ca re, not maintenance treatment.
• 20 Naltrexone Opioid Opioid antagonist Receptor antagonist Alcohol dependence Alcoholic beverage Alcohol

• 21

6
lock
s
Suspend
0
End Block
Item: 10 of 73 ~. I • M k <:] t> al ~· ~
QIO: 1152 .l. ar Previous Next lab 'lifllues Notes Calculator

1 • p • p I p g y
wit h delirium tremens. Therefore, he wou ld benefit from an IV benzodiazepine, with special considerat ion given
2
to his liver disease.
3 Benzodiazepine Delirium tremens Disulfiram Acetaldehyde Acetaldehyde dehydrogenase Perspiration Nausea Headache Vomiting Delirium liver Alcohol

4 Alcoholism liver disease Alcoholic beverage Flushing (physiology)

5
C is not correct. 4°/o chose this.
6
Ha loperidol is a typical antipsychotic. It can be used in patients withdrawing from alcohol who suffer psychotic
7 sympt oms such as hal lucinat ions.
8 Typical antipsychotic Haloperidol Antipsychotic Psychosis Hallucination Alcoholic beverage Alcohol

9 E is not correct. 5°/o c hose this.


10 Methadone is a potent, long -acting opioid agonist used in the t reat ment of opioid add iction . This patient is an
· 11 alcohol ic and does not requ ire a methadone taper.
Methadone Opioid Agonist Opioid use disorder Alcoholism
• 12
• 13 F is not correct. 10% chose this .
• 14 Na ltrexone is an opioid antagon ist used to help mainta in opioid sobriet y . I t also is used t o help prevent alcohol
• 15
relapses in alcohol dependence. This patient, however, requires acute care, not ma int enance treatment .
Naltrexone Opioid Opioid antagonist Receptor antagonist Alcohol dependence Alcoholic beverage Alcohol
• 16
• 17

• 18 Bottom Line:
• 19 Benzod iazepines are ind icated in the treatment of alcohol wit hdrawa l. Patient s with liver disease shou ld receive
• 20 oxazepam, lorazepam, or temazepam, because other benzod iazepines are contraindicated in t his sett ing .
Temazepam lorazepam Oxazepam Alcohol withdrawal syndrome Benzodiazepine liver disease liver Alcohol Alcoholic beverage
• 21

6
lock
s
Suspend
0
End Block
Item: 10 of 7 3 ~ 1 • M k -<:J 1>- Jil ~· !:';-~
QIO: 1152 ..L ar Pre v ious Next Labfli!llues Not es Calcula t o r
& &
1
FA17 p542.1
2
3
Delirium tremens Life-threatening alcohol'' ithdrawal S) ndrome that peaks 2-4 days after last drink.
Characterized by autonomic hyperactivity (eg, tachycardia, tremors, anxiety, seizures), electrolyte
4
disturbances, respiratory alkalosis. Classically occurs in hospital setting (eg, 2-4 days postsurgery)
5
in alcoholics not able to drinl.. as inpatients. Treatment: benzodiat.epines (eg, chlordiazepoxide,
6 lorazepam, diazepam).
7
8 FA17 p 515.2
9
Benzodiazepines Diazepam, lorazcpam, triazolam, tcmazcpam, oxazepam, midazolam, chlordiazepoxide,
10 alprazolam .
• 11 "Frcnzodiazepines" t frequency.
MECHANISM Facilitate GABA,, action by t frequency of
• 12 CJ- channel opening. ! RE~I sleep. Most Benzos, barbs, and alcohol all bind the
• 13 have long half-lives and active metabolites GABAA receptor, which is a ligand-gated Cl-
• 14 {exceptions [ATOl\1]: Alprazolam, Triazolam, channel.
Oxazepam, and Midazolam are short acting Oxazepam, Temazepam, and Lorazepam are
• 15
-+ higher addictive potential}. metabol ized O utside T he Liver
• 16
CLINICAL USE Anxiety, spasticity, status epilepticus (lora:tepam
• 17
and diazepam), eclampsia, detoxification
• 18 (especially alcohol withdrawal-DTs}, night
• 19 terrors, sleepwalking, general anesthetic
• 20 {amnesia, muscle relaxation), hypnotic
• 21
(insomnia).

a
Lock
s
Suspend
8
End Bl ock
Item: 10 of 7 3 ~ 1 • M k -<:J 1>- Jil ~· !:';-~
QIO: 1152 ..L ar Pre v ious Next Labfli!llues Not es Calcula t o r

1
&
-- - -- .. -- . &

(amnesia, muscle relaxation), hypnotic


2 (insomnia).
3 ADVERSE EFFECTS Dependence, additive C · depression effects
4 with alcohol. Less risk of respiratory depression
5 and coma than " ith barbiturates.
6
Treat o,·erdose with Aumazenil (competiti'e
antagonist at CABA benzodiazepine receptor).
7
Can precipitate seizures b~ causing acute
8 benzodiazepine withdrawal.
9
10 FA17 p 541.2
• 11
Alcoholism Physiologic tolerance and dependence on alcohol with symptoms of withdrawal when intake is
• 12
interrupted.
• 13 Complications: alcoholic cirrhosis, hepatitis, pancreatitis, peripheral neuropathy, testicular atrophy.
• 14 Treatment: disulfiram (to condition the patient to abstain from alcohol use), acamprosate,
• 15 naltrexone, supportive care. Support grottps such as Alcoholics Anonymous are helpfu l in
sustaining abstinence and supporting patient and fami ly.
• 16
• 17
Wernicke-Korsakoff Caused by vitamin B1 deficiency. Triad of conrusion, ophthalmoplegia, ataxia (Wernicke
syndrome encephalopathy). May progress to irreversible memory loss, confabulation, personality change
. 18
(Korsakoff syndrome). Symptoms may be precipitated by giving dextrose before admin istering
• 19 ,·itamin B1 to a patient with thiamine deficiency. Associated with peri,·entricular hemorrhage/
• 20 necrosis of mammillary bodies. Treatment : I vitamin B1•
• 21

a
Lock Suspend
s 8
End Bl ock
Item: llof73 ~. , . M k <:] t> al ~· ~
QIO: 1134 .l. ar Previous Next lab 'lifllues Notes Calculator

1 •
A 40-year-old woman presents to a psychiatrist after a referral from her family physician because she feels ~~AI
2 anxious al l the time . She wor ks as a computer prog rammer at home, where she lives alone. She qu it her job
3 as a programmer working in a large company due to concerns of constantly being crit icized despite good
4 performance reviews . She does not participate in socia l activities actively because she prefers to avoid conflicts
and potential rejection . She joined an on line matchmaking website but deleted her profile because no one had
5
contacted her after 1 week, which she attr ibuted to "being unattractive ."
6
7
Wh ich of the fol lowing is the most likely diagnosis?
8
:
9
A. Avoidant
10
B. Dependent
• 11
• 12 C. Histrion ic
• 13 D. Narcissistic
• 14
E. Schizoid
• 15
• 16
F. Schizotypa l
• 17

• 18
• 19
• 20
• 21

6
lock
s
Suspend
0
End Block
Item: llof73 ~. , . M k <:] t> al ~· ~
QIO: 1134 .l. ar Previous Next lab 'lifllues Notes Calculator

1 •

2
The correct answer is A. 80°/o chose this.
Cl uster C persona lity disorders incl ude avoidant, o bsessive-compulsive, and dependent types. Patients with
3
these diso rders are characterized as anxious or wo rried and have abnor mal fears about relationships,
4 sepa ration, and control. Patients with avoidant pe rsonality disorder are sensit ive to rej ection, socia lly inh ibited,
5 and timid, with overwhelming feelings of inadequacy. Clea r ly, th is patient shows that she has avoidant
6
pe rsona lity disorde r, given her social inhibitions lead ing to a j ob in w hich she does not interact regularly with
othe rs as well as he r sensitivity to rej ection in ta king down her dating profi le afte r a wee k. She does see m to be
7
seeking co mpanionship though, thus diffe ring he r fro m someone w ho has schizoid personal ity disorder.
8 Schizoid personality disorder Avoidant personality disorder Personality disorder Obsessive-compulsive disorder

9
B is not correct. 4°/o chose this.
10
Cluster C persona lity disorde rs incl ude avoidant, obsessive-compulsive, and dependent types. Patients with
11 dependent pe rsonal ity disorder are submissive and cl inging; t hey have low self-esteem and have an excessive
• 12 need to be ta ken ca re of. These patients have difficulty ma king decisions on their own and require othe rs to
share the responsibi lity f or most areas or tasks in thei r lives .
• 13
Dependent personality disorder Personality disorder Obsessive-compulsive disorder Self-esteem
• 14
• 15 C is not correct. 3°/o chose this .
Cluster 8 persona lity disorders include antisocial, borde rline, histr ionic, and narcissistic types. These patients
• 16
are best descr ibed as dramatic, emotional, or erratic. There is an association wit h mood diso rders and
• 17 subst ance abuse. Patients with histrionic personal ity diso rder display excessive emotional ity, somatization, and
• 18 attention-seeking and sexual ly provocative behavio r. These symptoms are not seen in this patient .
Histrionic personality disorder Cluster B personality disorders Mood disorder Personality disorder Narcissistic personality disorder Somatization Attention seeking
• 19
Narcissism Substance abuse Anti-social behaviour Antisocial personality disorder
• 20
• 21

D is not correct. 2°/o chose this .
6
lock
s
Suspend
0
End Block
Item: llof73 ~. , . M k <:] t> al ~· ~
QIO: 1134 .l. ar Previous Next lab 'lifllues Notes Calculator

1 •
D is not correct. 2°/o chose this.
2 Cluster B persona lity disorders include antisocial, borderl ine, histr ionic, and narcissistic types. Patients with
3 narcissistic personal ity disorder are grand iose and have a sense of entitlement. They frequently demand the
4
best of everyth ing, includ ing physicians and hea lth ca re. These symptoms are not seen in th is patient.
Narcissistic personality disorder Cluster B personality disorders Narcissism Personality disorder Histrionic personality disorder Anti-social behaviour
5
Antisocial personality disorder
6
7 E is not correct. 8°/o chose this.
8
Cluster A persona lity disorders include paranoid, schizoid, and sch izotypal types. These patients may be seen as
odd or eccentric. There is an association with schizophrenia. Patients with schizoid persona lity disorder exhibit
9
voluntary social withdrawa l and have limited emotiona l expression . Their lack of meaningful relationsh ips does
10 not concern them the way it concerns the patient with avoidant persona lity disorder.
Schizoid personality disorder Avoidant personality disorder Schizophrenia Personality disorder Schizotypal personality disorder Solitude Paranoia
11
• 12 F is not correct. 3°/o chose this .
• 13 Cluster A persona lity disorders incl ude paranoid, schizoid, and sch izotypal types. Patients with schizotypa l
• 14 persona lity disorder demonstrate interpersonal awkwardness, odd thought patterns (ie, mag ical thinking), and
• 15
an eccentr ic appearance .
Schizotypal personality disorder Magical thinking Personality disorder Schizoid personality disorder Paranoia
• 16
• 17

• 18 Bottom Line:
• 19 Patients with avoidant persona lity disorder are sensitive to rejection and are socially inhibited and t imid . In
• 20 contrast to patients with schizoid personal ity disorder who voluntarily seek iso lation, avoidant people strong ly
desi re to interact social ly, but have an overwhelming sense of inadequacy that holds them back .
• 21 Schizoid personality disorder Avoidant personality disorder Personality disorder

6
lock
s
Suspend
0
End Block
Item: 11 of 7 3 ~ 1 • M k -<:J 1>- Jil ~· !:';-~
QIO: 1134 ..L ar Pre v ious Next Labfli!llues Not es Calcula t o r
& &
1
FA17 p 535.3
2
Cluster C personality Anxious or fearful ; genetic association " ith "\\'orricd."
3 disorders anxiety disorders. Cluster C: Cowardly, obsessive-Compulsi\ c,
4 C lingr
5 Avoidant Hypersensiti,·e to rejection, socially inhibited, Cowardly.
6 timid, feelings of inadequacy, desires
7 relationships with others (vs schizoid).
8 Obsessive-Compulsive Preoccupation " ith order, perfectionism, and
9
control; ego-S}1ltonic: behavior consistent with
one's own bel iefs and attitudes (vs OCO).
10
Dependent Submissive and C lingy. excessive need to be Patients often get stuck in abusive relationships.
11
taken care of, low self-confidence .
• 12
• 13
FA1 7 p 535.2
• 14
Cluster 8 personality Dramatic, emotional, or erratic; genetic "Wild."
• 15
disorders association with mood disorders and substance Cluster B: Bad, Borderline, fl amBoyant, must
• 16 abuse. be the Best
• 17
Antisocial Disregard for and violation of rights of others Antisocial =~ociopath.
. 18 with lack of remorse, criminality, impulsivity; Bad.
• 19 males > females; must be~ 18 years old and
• 20 have history of conduct disorder before age 15.
Conduct disorder if< 18 years old.
• 21
• . .. T T • I 1
' . I • • I •
'. ' ' ' ' ..
a
Lock
s
Suspend
8
End Bl ock
Item: 11 of 7 3 ~ 1 • M k -<:J 1>- Jil ~· !:';-~
QIO: 1134 ..L ar Pre v ious Next Labfli!llues Not es Calcula t o r
& &
1 Borderline Unstable mood and interpersonal relat ionships, Treatment·: dialectical behavior therapy.
2 impu lsivity, self-mutilation, su icida lit y, sense Border! in e.
3 of emptiness; females> males; splilling is a
major defense mechanism.
4
5
Histrionic Exccssi,·e emotionality and excitability, FlamBoyant.
attention seeking, sexually provocati\'e, O\'erly
6
concerned with appearance.
7
Narcissistic Grandiosity, sense of entitlement; lacks empathy \lust be the Best.
8 and requires excessive admiration; often
9 demands the ''best" and reacts to criticism
10 with rage.
11
• 12 FA17 p 535.1

• 13 Cluster A personality Odd or eccentric; inabil ity to develop . I"


"\ \'CJrC.
• 14 disorders meaningful social relationships. o psyc h o~i s; Clu~ler A: Accusatory, Aloof, Awkward.

• 15
genetic association with schizophrenia .
• 16
Paranoid Pervasive d islrust (Accusatory) and
suspiciousness of others and a profound ly
• 17
cynical view of the world .
• 18
Schizoid oluntary social wilhdra\\ al (Aloof), limited
• 19
emotional expression, content with social
• 20 isolation (,·s a,·oidant).
• 21 Schizotypal Eccentric appearance, odd beliefs or magical Schizotr pal = magical thinking.

a
Lock
s
Suspend
8
End Bl ock
Item: 12 of 73 ~ 1 • M k -<:J 1>- Jil ~· !:';-~
QIO: 4972 ..L ar Pre v ious Next Lab fli!ltues Not es Calcula t o r

IAA]
& &
1
A 28-year-old man decides to quit drinking "cold turkey." He has been consuming up to 15 drinks per day for
2
the past year and occasiona lly usi ng cocaine on the weekends. One day after his last drink, he begins to
3 notice smal l monkeys on surfboa rds and continues to hear car horns even though he is 10 mi les from the
4 nearest road. He otherwise feels well and is alert.
5
6 This patient is most likely suffering from what?
7 :

8 A. Alcoholic hallucinosis
9 B. Cocaine withdrawal
10
C. Delirium tremens
11
D. Panic disorder
• 12
• 13 E. Schizophren ia
• 14
• 15
• 16
• 17
• 18
• 19
• 20
• 21

a
Lock
s
Suspend
8
End Bl ock
Item: 12 of 73 ~. I • M k <:] t> al ~· ~
QIO: 4972 .l. ar Previous Next lab 'lifllues Notes Calculator

1 •
The correct answer is A. 60°/o chose this.
2
Alcohol ic hallucinosis refers to the onset of visua l, auditory, and tacti le hal lucinations with in 12-24 hours after
3
the most recent drink. The hal lucinations are unstructured and short lived. These specific ha llucinations usua lly
4 resolve within 24-48 hours and are not associated with a global clouding of the sensorium or dysautonomia, as
5 in delirium tremens. Benzodiazepines are the treatment of choice, but in some cases antipsychotic medications
6
may be needed.
Alcoholic hallucinosis Delirium tremens Antipsychotic Benzodiazepine Alcoholism Sensorium Hallucination Delirium Dysautonomia Somatosensory system
7
Pseudohallucination
8
9 B is not correct. 7°/o chose this.
10
Cocaine withd rawal does not cause specific aud itory and visual ha llucinations, though it is associated with
depressed or dysphor ic mood and unpleasant dreams. Cocaine intoxication may cause a tacti le hal lucination
11 described as the sensation of bugs craw ling over the skin.
12 Cocaine intoxication Cocaine dependence Cocaine Hallucination Alcohol intoxication Dysphoria

• 13
C is not correct. 31% chose this .
• 14
Delirium tremens (DTs) usually begins within 48-96 hou rs of the most recent dr ink and last 1- 5 days.
• 15 Characteristics of DTs include visual or tactile hallucinations, delir ium, tachycard ia, hypertension, fever,
• 16 agitation or letha rgy, and diaphoresis. Particular ly important in estab lish ing the diagnosis of DTs is the presence
of autonomic instability. There is a 5%-20% associated mortality, mostly due to cardiac ar rhythmias. Treatment
• 17
is with benzod iazepines, hydration, and supportive care . Risk factors include age between 30 and 40 yea rs, 5-
• 18 15 years of heavy drinking, and physica l il lness such as hepatitis or pancreatitis.
• 19 Delirium tremens Perspiration Pancreatitis Tachycardia Benzodiazepine Delirium Hypertension Dysautonomia Hepatitis Cardiac arrhythmia Hallucination lethargy

• 20 Autonomic nervous system Fever Somatosensory system

• 21 Dis not correct. 1°/o chose this .


6
lock
s
Suspend
0
End Block
Item: 12 of 73 ~. I • M k <:] t> al ~· ~
QIO: 4972 .l. ar Previous Next lab 'lifllues Notes Calculator

1 •
C is not correct. 31% chose this.
2 Delirium tremens (DTs) usually begins within 48-96 hou rs of the most recent dr ink and last 1-5 days.
3 Characteristics of DTs include visual or tactile hallucinations, delir ium, tachycard ia, hypertension, fever,
4 agitation or letha rgy, and diaphoresis. Particular ly important in estab lish ing the diagnosis of DTs is the presence
of autonomic instability. There is a 5% -20% associated mortality, mostly due to cardiac ar rhythmias. Treatment
5
is with benzod iazepines, hydration, and supportive care . Risk factors include age between 30 and 40 yea rs, 5-
6 15 years of heavy drinking, and physica l il lness such as hepatitis or pancreatitis.
7 Delirium tremens Perspiration Pancreatitis Tachycardia Benzodiazepine Delirium Hypertension Dysautonomia Hepatitis Cardiac arrhythmia Hallucination lethargy

8 Autonomic nervous system Fever Somatosensory system

9 Dis not correct. 1°/o chose this.


10 Panic disorder does not cause specific ha llucinations.
11 Panic disorder Hallucination

12 E is not correct. 1°/o chose this .


• 13 This patient does not fit the criteria for sch izophrenia with on ly 1 day of ha llucinations and no other signs or
• 14 symptoms .
Schizophrenia Hallucination
• 15
• 16
• 17 Bottom Line:
• 18 Alcohol ic hall ucinosis refers to the onset of visua l, auditory , and tacti le hal lucinations 12-24 hours after
• 19 consumption of the last alcohol ic beverage .
Alcoholic hallucinosis Alcoholic beverage Hallucination Alcoholism Somatosensory system
• 20
• 21

6
lock
s
Suspend
0
End Block
Item: 12 of 7 3 ~ 1 • M k -<:J 1>- Jil ~· !:';-~
QIO: 4972 ..L ar Pre v ious Next Lab fli!ltues Not es Calcula t o r
& &
1
2 FA17 p542.1
3 Delirium tremens Life-threatening alcohol'' ithdrawal S) ndrome that peaks 2-4 days after last drink.
4 C haracterized by autonomic hr peractivity (eg, tachycardia, tremors, anxiety, seizures), electrolyte
5 disturbances, respiratory alkalosis. Classically occurs in hospital setting (eg, 2-4 days postsurgery)
6 in alcoholics not able to drinl- as inpatients. Treatment: benzodiat.epines (eg, chlordiazepoxide,
lorazepam, diazepam).
7
8
FA17 p 540.1
9
Psychoactive drug intoxication and withdrawal
10
DRUG INTOXICATION WITHDRAWAL
11
Depressants
12
1onspecific: mood elevation, l anxiety, onspecific: anxiety, tremor, seizures,
• 13 . .
sedation, behavioral disinhibition, respiratory 111SOI11nl a .
• 14 depression .
• 15 Alcohol Emotional labil ity, slurred speech, ataxia, Time from last drink:
• 16 coma, blackouts. Serum y-glutamyll ra nsfcrase 3- 36 hr: minor symptoms similar to other
• 17 (GGT)- sensitivc indicator of alcohol usc. depressants
• 18
AST value is twice ALT va lue. 6-48 hr: withdrawal seizures
12-48 hr: alcoholic hallucinosis (usually visual)
• 19
48-96 hr: delirium tremens (DTs) in 5% of
• 20
cases
• 21 Treatment: benzod iazepi nes.

a
Lock
s
Suspend
8
End Bl ock
~ -<:J 1>- Jil ~· !:';-~

.
Item: 12 of 7 3 1 • M k
QIO: 4972

1
..L
p .
ar Pre v ious Next

p p
Lab fli!ltues

} ~
p
Not es Calcula t o r

g, p p 'p (
2
l gag reAex, pupillary constriction (pinpoint turkey"), fe\·er, rhinorrhea, yawn ing, nausea,
pupils), seizures (overdose). i\ lost common stomach cramps, diarrhea ("Au-like" S) mptoms).
3
cause of drug overdose death. Treatment: Treatment: long-term support, methadone,
4 naloxone. buprenorphine.
5 Delirium, life-threatening cardiovascular
Barbiturates Low safety margin, marked respirator)
6 depression. Treatment: symptom management collapse.
7 (eg, assist respiration, t BP).
8 Benzodiazepines Greater safety margin. taxia, minor Jeep disturbance, depression, rebound amiet),
9 respiratory depression. Treatment: Oumazcnil Seizure.
10
(benzodiazepine receptor antagonist, but
rarely used as it can precipitate seizures).
11
Stimulants
12
Nonspeci fie: mood elevation, psychomotor 1onspecific:post-use "crash," including
• 13
agitation, insomnia, ca rd iac arrhythmias, depression, lethargy, t appetite, sleep
• 14
tachycardia, anxiety. disturbance, vivid nightmares.
• 15
Amphetamines Euphoria, grandiosity, pupillary dilation,
• 16 prolonged wakefulness and attention,
• 17 hypertension, tachycardia, anorexia, paranoia,
. 18 fe\·er. Se\·ere: cardiac arrest, seizures.
• 19 Treatment: benzodiazepines for agitation and
se1zures.
• 20
Cocaine Impaired judgment, pupillar)' dilation,
• 21
• h~llut"in~f-inn~ l inf'lnrlincr t6.l,...f-iJI"o) n-:ar.,nnirl

a
Lock
s
Suspend
8
End Bl ock
Item: 12 of 73 ~ 1 • M k -<:J 1>- Jil ~· !:';-~
QIO: 4972 ..L ar Pre v ious Next Lab fli!ltues Not es Calcula t o r
& &
1
Caffeine Restlessness, t diuresis, muscle twitching. lleadache, difficulty concentrating, Au-like
2 symptoms.
3 Nicotine Restlessness. Irritability, anxiety, restlessness, difficulty
4 concentrating. Treatment: nicotine patch,
5 gum, or lozenges; bupropion/varcnicl inc.
6 Phencyclidine iolence, impulsi,ity, psychomotor agitation,
7 nystagmus, tach) cardia, h) perlension,
8
analgesia, psychosis, delirium, sci~ures.
Trauma is most common complication.
9
Treatment: benzodiazcpincs, rapid-acting
10 antipsychotic.
11 lysergic acid Perceptual distortion (visual, auditory),
12 diethylamide (LSD) depersonalization, am.iety, paranoia,
• 13 psychosis, possible Aashbacks.
• 14 Marijuana Euphoria, anxiety, paranoid delusions, Irritability, anxiety, depression, insomnia,
• 15 (cannablnoid) perception of slowed time, impa ired judgment, restlessness, ! appetite.
• 16
social withdrawal, t appetite, dry mouth,
conjunctival injection, hallucinations.
• 17
Pharmaceutical form is dronabinol: used
• 18 as antiemetic (chemotherapy) and appetite
• 19 stimulant (in AIDS).
• 20 MDMA (ecstasy) Hallucinogenic stimulant: euphoria, Depression, fatigue, change in appetite, difficulty
• 21 disinhibition, hyperacti' it), distorted sensory concentrating, anxiety.

a
Lock
s
Suspend
8
End Bl ock
Item: 13 of 73 ~ 1 • M k -<:J 1>- Jil ~· !:';-~
QIO: 250 5 ..L ar Pre v ious Next Lab fli!ltues Notes Calcula t o r

IAA]
& &
1
A 35-year-old woman presents to the urgent care clin ic because she is concerned about her trembling hands
2
and a facial tic. According to her mother, who accompanies her, the patient has been seeing a psych iatrist for
3 the past several years. On examination the patient exhibits repetitive facial grimaces with occasional
4 protrusion of the tongue. She denies that she is purposefu lly producing these facial expressions.
5
6 What is the mechanism of action of the medications that this patient is most likely taking?
7 :

8 A . Antagonism at dopamine, serotonin, and adrenergic receptors


9 B. Antagonism of Drdopamine receptors
10
C. Antagon ism of noradrenergic and serotonin receptors
11
D. I nhibition of serotonin reuptake
12
• 13 E. Inhibition of serotonin-norepinephrine reuptake
• 14 F. I nteraction with cation transport
• 15
• 16
• 17
• 18
• 19
• 20
• 21

a
Lock
s
Suspend
8
End Bl ock
Item: 13 of 73 ~. I • M k <:] t> al ~· ~
QIO: 2505 .l. ar Previous Next lab 'lifllues Notes Calculator

1 •
The correct answer is B. 77°/o chose this.
2 Typica l antipsychotics may lead to tardive dyskinesia, which is implicated in the D2 signal ing pathway. By
3 decreasing dopamine signaling in the brain, they reduce positive symptoms of schizophren ia includ ing aud itory
4 hallucinations but do not affect the negative symptoms. Tardive dyskinesia results from the blockade of D2
dopamine receptors. This symptom is also an adverse effect of the anti psychotics haloperidol, fluphenazine,
5
chlorpromazine, perphenazine, and th ioridazine. Tardive dyskinesia often worsens over time and is not always
6 reversible on discontinuation of the drug.
7 Fluphenazine Chlorpromazine Haloperidol Thioridazine Tardive dyskinesia Perphenazine Schizophrenia Dopamine Antipsychotic Dyskinesia Typical antipsychotic

8 Hallucination Adverse effect Positive symptoms Auditory hallucination Symptom Signal transduction Brain Receptor (biochemistry) Negative symptoms

9 A is not correct. 8°/o chose this.


10 Quetiapine is an atypical antipsychotic used to t reat psych iatric disorders such as schizophrenia . Atypical
11 antipsychotics act on mu ltiple receptors includ ing dopamine, seroton in, and ad renergic receptors. Although
12
atypical antipsychotics can have extrapyramidal adverse effects, the risk is less than with typical antipsychotics
such as chlor promazine. The most common adverse effect of quetiapine is sedation, along with weight gain,
13 constipation, and headaches .
• 14 Atypical antipsychotic Chlorpromazine Quetiapine Serotonin Dopamine Schizophrenia Constipation Antipsychotic Mental disorder Extrapyramidal symptoms

• 15 Sedation Typical antipsychotic Extrapyramidal system Adrenergic receptor Adverse effect Weight gain Adrenergic Psychiatry

• 16
C is not correct. 4°/o chose this .
• 17
Mirtazapine is an antidepressant whose common adverse effects include sedation and weight gain. The
• 18 hypothesized mechanism of action of mirtazapine combines 5-hydroxytryptaminerreceptor and a-adrenoceptor
• 19 antagon ism .
Mirtazapine Antidepressant Mechanism of action Sedation Serotonin Weight gain
• 20
• 21 D is not correct. 5°/o chose this .

6
lock
s
Suspend
0
End Block
Item: 13 of 73 ~. I • M k <:] t> al ~· ~
QIO: 2505 .l. ar Previous Next lab 'lifllues Notes Calculator

1 •
D is not correct. 5°/o chose this.
2 Sertra line is a selective seroton in reuptake inhibitor (SSRI ) associated with anxiety, insomnia, tremor, and
3 nausea . I f sertraline is taken in combination with monoamine oxidase inhibitors, the result can be serotonin
4 synd rome, which is cha racte rized by progression to severe headaches, dizziness, vomit ing, coma, and death.
Decreased libido is a common complaint of SSRI users.
5 Selective serotonin reuptake inhibitor Serotonin syndrome Sertraline libido Insomnia Serotonin Serotonin reuptake inhibitor Tremor Monoamine oxidase Nausea
6 Monoamine oxidase inhibitor Reuptake Dizziness Vomiting Anxiety Monoamine neurotransmitter Coma Reuptake inhibitor Enzyme inhibitor
7
E is not correct. 5°/o chose this.
8
I mipramine is a t r icycl ic antidepressant that works via inhibition of serotonin and norepineph r ine reupta ke,
9
although othe r neurotransmitters are also affected. Adverse effects of TCAs incl ude sedation, orthostatic
10 hypotension, weight gain, and sexual disturbances . Severe adverse effects can be remembered by the 3 Cs:
11 Convu lsions, Coma, and Ca rdiotoxicity (conduction defects and ar rhythmias). Tricycl ic antidep ressants exhibit
antichol inergic adverse effects as wel l, includ ing dry mouth, mydriasis, constipation, and urinary retention .
12
These dr ugs can be extremely dangerous in overdose.
13 Tricyclic antidepressant Anticholinergic Orthostatic hypotension Mydriasis Imipramine Serotonin Antidepressant Norepinephrine Urinary retention Hypotension
• 14 Constipation Neurotransmitter Sedation Reuptake Xerostomia Cardiac arrhythmia Weight gain Tricyclic Drug overdose
• 15
F is not correct. 1 °/o chose this .
• 16
Whereas the mechanism of action of lithium is unknown, it has been suggested that lithium acts via interaction
• 17
with cation transport in the brain. Lith ium inta ke has been associated with t remor, hypothyroidism, and
• 18 neph rogenic diabetes insipidus. Lithium does not resu lt in dopaminergic blockade, and thus is not associated
• 19 with extrapyramida l symptoms .
Hypothyroidism Diabetes insipidus Extrapyramidal symptoms Diabetes mellitus Tremor Mechanism of action lithium Dopaminergic Cation Brain Human brain
• 20
• 21

6
lock
s
Suspend
0
End Block
Item: 13 of 73 ~. I • M k <:] t> al ~· ~
QIO: 2505 .l. ar Previous Next lab 'lifllues Notes Calculator

1
E is not correct. 5°/o chose this.
2
I mipramine is a tricycl ic antidepressant that works via inhibition of serotonin and norepinephrine reuptake,
3 although othe r neurotransmitters are also affected. Adverse effects of TCAs incl ude sedation, orthostatic
4 hypotension, weight gain, and sexual disturbances . Severe adverse effects can be remembered by t he 3 Cs:
5 Convu lsions, Coma, and Cardiotoxicity (conduction defects and arrhythmias). Tricyclic antidep ressants exhibit
antichol inergic adverse effects as wel l, includ ing dry mouth, mydriasis, constipation, and urinary retention .
6
These drugs can be extremely dangerous in overdose.
7 Tricyclic antidepressant Anticholinergic Orthostatic hypotension Mydriasis Imipramine Serotonin Antidepressant Norepinephrine Urinary retention Hypotension

8 Constipation Neurotransmitter Sedation Reuptake Xerostomia Cardiac arrhythmia Weight gain Tricyclic Drug overdose

9
F is not correct. 1 °/o chose this.
10
Whereas the mechanism of action of lithium is unknown, it has been suggested that lithium acts via interaction
11 with cation transport in the brain. Lith ium intake has been associated with t remor, hypothyroidism, and
12 neph rogenic diabetes insipidus. Lithium does not resu lt in dopaminergic blockade, and thus is not associated
13
with extrapyramida l symptoms.
Hypothyroidism Diabetes insipidus Extrapyramidal symptoms Diabetes mellitus Tremor Mechanism of action lithium Dopaminergic Cation Brain Human brain
• 14
• 15
• 16 Bottom Line:
• 17 Common adverse effects associated with antipsychotic use are tre mors (short-term or long-term use), facial
• 18 tics, and tardive dyskinesia (long-term use), often as a resu lt of inhibition of dopamine pathways in the brain .
These extrapyramidal adverse effects are more com mon with typical antipsychotics such as ch lorpromazine
• 19
than with atypical agents such as quetiapine .
• 20 Chlorpromazine Tardive dyskinesia Quetiapine Dopamine Antipsychotic Dopaminergic pathways Dyskinesia Extrapyramidal symptoms Typical antipsychotic

• 21 Extrapyramidal system Human brain Tic Brain

6
lock
s
Suspend
0
End Block
Item: 13 of 7 3 ~ 1 • M k -<:J 1>- Jil ~· !:';-~
QIO: 250 5 ..L ar Pre v ious Next Lab fli!ltues Notes Calcula t o r
& &
1
FA17 p 543.1
2 Typical antipsychotics Haloperidol, pimozide, triAuopera /inc, Auphena:rine, thioridazine, chlorpromazine.
3
MECHANISM Block dopamine 0 2 receptor (f c 1 P).
4
CliNICAl USE Schizophrenia (1° positi,·c symptoms), psychosis, bipolar disorder, delirium, Tourcttc syndrome,
5 Huntington disease, OCD.
6
POTENCY H igh potency: TriAnoperazine, flnphcna7ine, llaloperidol (Try to Fly H igh)- neurologic side
7 effects (eg, extrapyramidal S) mploms [F.PS]).
8 Lo'' potency: Chlorpromazine, Thioridazine (Cheating l'hieves are low)-anticholinergic,
9 antihistamine, a 1-blockade effects.
10 ADVERSEEFFECTS Lipid soluble .... stored in body fat .... slow to be removed from body.
11 Endocrine: dopamine receptor antagonism .... hyperprolaetinemia .... galactorrhea,
12 oligomenorrhea, gynecomastia.
13
fetabolic: dyslipidemia, weight gain, hypcrglyccm ia.
Antimuscarinic: dry mouth, constipation .
• 14
Antihistamine: sedation .
• 15
a .-blockade: orthostatic hypotension.
• 16 Cardiac: QT prolongation .
• 17 Ophthalmologic: C hlorpromazine-C orneal deposits; T hioridazine-reT inal deposi ts.
. 18 EPS-ADAPT:
• 19 Hours to days: Acute D ystonia (muscle spasm, stiffness, oculogyric crisis).
• 20 Days to months: Akathisia (restlessness), Parkinsonism (bradykinesia).
.\tfonths to years: Tardive d)slinesia (orofacial chorea).
• 21

a
Lock
s
Suspend
8
End Bl ock
Item: 13 of 7 3 ~ 1 • M k -<:J 1>- Jil ~· !:';-~
QIO: 250 5 ..L ar Pre v ious Next Lab fli!ltues Notes Calcula t o r

1 p g p p
2 EPS-ADAPT:
3 Hours to days: Acute D ystonia (muscle spasm, stiffness, oculogyric crisis).
4 Days to months: \ kathisia (restlessness), Parkinsonism (bradykinesia).
~lonths to years: Tardive dyskinesia (orofacial chorea).
5
Treatment: benztropine (acute dystonia, tardive dyskinesia), benzodiazepines, ~-blockers
6
(akathisia).
7
Neuroleptic malignant syndrome (NMS)- \ 1alignant FEVER: \ lyoglobinuria, Fe,·er,
8
E ncephalopathy, unstable Yitals, t Enz) mes, muscle Rigidity. Treatment: dantrolene, 0 2 agonist
9 (eg, bromocriptine).
10
11
FA17 p 54 3.2
12
Atypical Aripiprazole, asenapinc, clozapinc, olanzapinc, quctiapinc, ilopcridone, paliperidonc,
13
anti psychotics risperidone, lurasidonc, ziprasidonc.
• 14
MECHANISM ot completely understood. \!lost arc D2
• 15
antagon ists; aripiprazolc is 0 2 partial agonist.
• 16 Varied effects on 5-I IT 2, dopam ine, and
• 17 a- and H 1-receptors.
. 18 CLINICALUSE Schizophrenia-both positi\ c and negative Use clozapine for treatment-resistant
• 19 symptoms. Also used for bipolar disorder, sch izophrenia or schizoaffective disorder and
• 20 OCD, anxiety disorder, depression, mania, for suicidality in schizophrenia.
Tourette syndrome.
• 21
• ... • r

a
Lock
s
Suspend
8
End Bl ock
Item: 13 of 7 3 ~ 1 • M k -<:J 1>- Jil ~· !:';-~
QIO: 250 5 ..L ar Pre v ious Next Lab fli!ltues Notes Calcula t o r

1
g, 1
2

3 FA17 p 543.2

4 Atypical Aripiprazole, asenapinc, clozapinc, olanzapinc, quctiapinc, iloperidone, paliperidone,


5 antipsychotics rispcridone, lurasidonc, ziprasidonc.
6 MECHANISM Not completely understood. \ lost arc 0 2
7 antagonists; aripiprazole is D2 partial agonist.
Varied effects on 5-I IT 2, dopamine, and
8
a- and H1-reccptors.
9
CliNICAL USE Schizophrenia-both posith·e and negative Use clozapine for treatment-resistant
10
symptoms. Also used for bipolar disorder, schizophrenia or schizoaffecti\'e disorder and
11 for su icidality in schizophrenia.
OCD, anxiety disorder, depression, mania,
12 Tourette syndrome.
13 ADVERSE EFFECTS Al l-prolonged QT interval, fewer EPS and
• 14 antichol inergic side effects than typical
• 15 antipsychotics.
• 16
"-pines"- metabolic syndrome (weight gain, Olanzapine .... O besity
diabetes, hyperlipidem ia) .
• 17
C lozapine-agranulocytosis (monitor WBCs Must· watch bone marrow clo7cly with clo1.apinc.
. 18 frequently) and seizures (dose related).
• 19 Risperidone-hyperprolactinemia (amenorrhea,
• 20 galactorrhea, gynecomastia).
• 21

a
Lock
s
Suspend
8
End Bl ock
Item: 14 of 73 ~. I • M k <:] t> al ~· ~
QIO: 4632 .l. ar Previous Next Lab 'lifllues Notes Calculator

1 •
An obese 88-year -old man with a history of hypertension and cerebrovascu lar accident is admitted to the
2 hospital because of a new ly diagnosed arrhythmia. Neurolog ic examination reveals left leg pa raesthesia,
3 weakness of flexor and extensor muscles, and dorsiflexion of the ha llux on stimu lation of the so le of the foot.
4 In add it ion, his grown ch ildren are concerned about the deter ioration of his mental function over the past year.
They claim that he routinely started misplacing objects around his home about 6 months ago . He was relatively
5
stable after that but as of about 3 months ago he can barely remember how to get home when he wa lks to the
6 park . In the last 2 weeks, he has begun to have an increased difficu lty find ing the right words.
7

8 What is the most like ly cause of the patient's change in cogn it ive capacity over the last 6 months'
9
:
10 A. "Sundown ing"
11
B. Alzheimer disease
12
C. Amnestic disorder
13
• 14 D . Delirium due to high blood pressure
• 15 E. Mu lti -infarct vascu lar dementia
• 16
F. Substance-induced del irium
• 17

• 18
• 19
• 20
• 21

6
lock
s
Suspend
0
End Block
Item: 14 of 73 ~. I • M k <:] t> al ~· ~
QIO: 4632 .l. ar Previous Next Lab 'lifllues Notes Calculator

1
The correct answer is E. 67°/o chose this.
2
Because the patient's children notice a prog ressive cogn itive decl ine, he most likely suff ers from dementia (a
3 long-ter m illness of gradua l onset) rather than del irium (a more acute cogn it ive disorder that reflects an
4 underlying medical condition) . Although Alzheimer dementia is one of the most common forms of dementia, th is
patient's history of hypertension and cerebrovascu lar accident and his neurologic symptoms suggest that he
5
suffers from vascular dementia (the second most common form of dementia in the United States). Vascu lar t
6 dementia typica lly manifests with a step-wise decline in neurologic function with interim per iods of relative
7 stability as seen in th is patient.
Cognitive disorder Vascular dementia Stroke Dementia Hypertension Delirium Cerebrovascular disease Neurology Cognition Blood vessel
8
9 A is not correct. 3°/o chose this.
10 "Sundowning" refers to the increased severity of symptoms seen at night in patients with dementia, especial ly
11
Alzheimer disease. Because the chi ldren noticed a general ized change in their father 's cognitive function that did
not occur at a specific t ime, the patient is un likely to be "sundown ing. "
12 Sundowning Dementia Alzheimer' s disease Cognition Sundowning (album)
13
B is not correct. 20% chose this.
14
Patients with Alzheimer disease (AD) ra rely have foca l neurologic deficits. I n AD, a gradua l decline in cognitive
• 15
function over a few years is seen . Th is is in contrast with the step-wise decl ine in cogn it ive function seen in th is
• 16 patient noted over the previous 6 months. The cl inical course, along with the history of cerebrovascular disease
• 17 suggests that vascular dementia is more likely than AD .
Cerebrovascular disease Vascular dementia Alzheimer' s disease Dementia Focal neurologic signs Neurology Cognition Blood vessel
• 18
• 19 C is not correct. 4°/o chose this .
• 20 Amnestic disorder is usually caused by alcoholism or brain injury and is cha racterized by memory loss, with
• 21
preservation of other cognitive functions. This patient clearly has a change in global cognitive function and
th<:>r<:>fnr<:> c:r off<:>rc: frnn"l rl<=>IYI<=>nti;::r r;::rth<:>r th;::rn ;::rn ::rn"ln<:>c:tir rl ic:nrrl<=>r

6
lock
s
Suspend
0
End Block
Item: 14 of 73 ~. I • M k <:] t> al ~· ~
QIO: 4632 .l. ar Previous Next Lab 'lifllues Notes Calculator
• • ... - - -" .. - . -- • - u - -." - .- - •• :..- • • --
1
patient noted over the previous 6 months. The cl inical course, along with the history of cerebrovascular disease
2 suggests that vascular dementia is more likely than AD.
3 Cerebrovascular disease Vascular dementia Alzheimer' s disease Dementia Focal neurologic signs Neurology Cognition Blood vessel

4
C is not correct. 4°/o chose this.
5 Amnestic disorder is usually caused by alcoholism or brain injury and is cha racterized by memory loss, with
6 preservation of other cognitive functions. This patient clearly has a change in global cognitive function and
7 therefore suffers from dementia rather than an amnestic disorder.
Amnesia Dementia Cognition Traumatic brain injury Alcoholism Brain injury Acquired brain injury Brain Human brain
8
9 D is not correct. 4°/o chose this.
10 Although med ical conditions are a common underlying cause of delirium, th is patient's prog ressive change in
cognitive function appears to be due to dementia rather than delir ium.
11
Delirium Dementia Cognition
12
F is not correct. 2°/o chose this.
13
Although substance abuse is a common cause of delir ium, there is no mention of substance abuse in the
14
vignette, and the patient appea rs to be suffering from dementia rather than delirium .
• 15 Dementia Substance abuse Delirium Vignette (literature)

• 16
• 17
Bottom Line:
• 18
• 19
Multi-infarct vascu lar dementia is a progressive, step-wise cognitive decline. History of hypertension,
cerebrovascu lar accident, neu rologic symptoms, and arrythmia make this diagnosis more likely .
• 20 Cardiac arrhythmia Vascular dementia Stroke Dementia Hypertension Blood vessel Cerebrovascular disease Neurology Cognition

• 21

6
lock
s
Suspend
0
End Block
Item: 14 of 7 3 ~ 1 • M k -<:J 1>- Jil ~· !:';-~
QIO: 4632 ..L ar Pre v ious Next Labfli!llues Not es Calcula t o r
& &
1
FA17 p490.1
2

3
Neurodegenerative I in cognitive ability, memory, or funct ion with intact consciousness.
disorders
4
DISEASE DESCRIPTION HISTOLOGIC/GROSS FINDINGS
5
Parkinson disease Parkinson TR. \PS your body: Loss of dopaminergic neurons (ie,
6
Tremor (pill-rolling tremor at rest) depigmentation) of substantia nigra pars
7 Rigidity (cogwheel) compacta.
8 Akinesia (or bradykinesia) Lc"} bodies: composed of a -synuclein
9 Postural instabilit) (intracellular eosinoph ilic inclusions ).
Shuffling gait
10
~ IPTP, a contaminant in illegal drugs, is
11
metabolized to l\ IPP+, which can cause
12 parkinsonian symptoms.
13 Huntington disease Autosomal dominant trinucleotide (CAG)n Atrophy of caudate and putamen with ex ,·acuo
14 repeat disorder on chromosome 4. Symptoms ven tricu lomega Iy.
• 15 manifest between ages 20 and 50: chorea, t dopamine, I GABA, I ACh in brain. 1 euronal
• 16 athetosis, aggression, depression, dementia death via MDA-R binding and glutamate
(sometimes initially mistaken for substance excitoloxicity.
• 17
abuse).
. 18 Anticipation results from expansion of CAG
• 19 repeats. Caudate loses ACh and GABA .
• 20 Alzheimer disease lost common cause of dementia in elderl) . Widespread cortical atrophy (normal cortex
• 21 Down syndrome patients ha,·e t risk of cortex in Alzheimer disease ), especially

a
Lock
s
Suspend
8
End Bl ock
Item: 14 of 7 3 ~ 1 • M k -<:J 1>- Jil ~· !:';-~
QIO: 4632 ..L ar Pre v ious Next Labfli!llues Not es Calcula t o r
& &
1
2
3
4 FA17 p 528.4
5
Delirium "\ Vaxing and waning" lc\·cl of consciousness Delirium =changes in sensorium.
6 with acute onset; rapid l in alieniion span and Ia~ be caused by medications (eg,
7 b·el of arousal. Characterized by disorgani7ed anticholinergics), especially in the elderly.
8 thinking, hallucinations (often visual), Reversible.
9
illusions, misperceptions, disturbance in sleep-
wake cycle, cognitive dysfunction.
10
zo
Usually to other illness (eg, C i S
11 disease, infection, trauma, subst<mce
12 abuse/withdrawal, metabolic/electrolyte
13 disturbances, hemorrhage, urinary/fecal
14
retention).
Most common presentation of altered mental
• 15
status in inpatient setting, especially in the
• 16 intensive care unit and with prolonged hospital
• 17 stays. Common ly, diffuse slowing EEC .
• 18 Treatment is aimed at identifying and
• 19 addressing underlying condition .
Antipsychotics may be used acutely as needed.
• 20
• 21

a
Lock
s
Suspend
8
End Bl ock
Item: 15 of 73 ~. I • M k <:] t> al ~· ~
QIO: 1488 .l. ar Previous Next lab 'lifllues Notes Calculator

1 •
A 42-year-old woman has a 6-month history of insomn ia due to intense wor ry over her f inancia l situation.
2 After a thorough physica l examination and wor k-up to exclude organ ic disease, the physician prescribes a 2-
3 month supply of a common ly prescribed anxiolytic agent with low potential for life-threatening CNS
4 depression . The patient requests a follow-up appointment a month later, as she has nearly run out of medication,
saying that by now she needs twice her origina l dose to achieve the same anxiolytic effect. At her appointment,
5
the patient appears much more anxious than at the previous office visit . She is sweating profusely, rapidly tapping
6 her feet, and reports becoming symptomatic when she does not take her medication .
7

8 Wh ich of the fo llowing short-acting anxiolytic agents was most likely prescribed?
9
:
10 A. Buspirone
11
B. Diazepam
12
C. Hyd roxyzine
13
14 D . Oxazepam
0 15 E. Th iopental
0
16
0 17
0
18
0 19
0 20
0
21

6
lock
s
Suspend
0
End Block
Item: 15 of 73 ~. I • M k <:] t> al ~· ~
QIO: 1488 .l. ar Previous Next lab 'lifllues Notes Calculator

1 •

2 The correct answer is D. 51°/o chose this.


Substance dependence is defined as th ree or more of the fol lowing in a 12-month period:
3
4 1. Cha racteristic withdrawal symptoms; substance taken to rel ieve withd rawal
2. Substance taken in la rger amount and for longer period than intended
5
3. Persistent desire or repeated unsuccessfu l attempt to quit
6 4 . Much time/activity spent to obtain, use, recover
7 5. Important social, occupational, or recreationa l activities given up or reduced
8
6. Continued use, despite knowledge of adverse consequences (eg, failure to fu lfill role obl igation, use when
physical ly hazardous)
9
The patient demonstrates signs of benzodiazepine dependence. As a general rule, physiologic dependence is
10
more likely to occu r with short-acting agents (ie, those with a shorter half-life). Oxazepam is one of short-acting
11 benzodiazepines that can be used to treat anxiety (triazo lam and midazolam, and alprazolam are other
12 examples) . Note that benzodiazepines are not common ly used as solo agents for t reating anxiety disorders.
13
I nstead, they are often used as a bridge during the first few weeks of treatment whi le SSRis (the first-line
agents for anxiety disorder treatment) take effect. Although very eff ective for the symptoms of anxiety, the
14
potential for dependence and tolerance mean that benzodiazepines should be used with great caution and very
15 rarely as so lo therapy for general anxiety disorder. Due to the potential for abuse, it is not advisable to
• 16 prescribe such a large quantity of a benzodiazepines with an init ial prescr iption .
• 17 Benzodiazepine Alprazolam Oxazepam Midazolam Benzodiazepine dependence Triazolam Generalized anxiety disorder Anxiety disorder Substance dependence

• 18 Half-life Selective serotonin reuptake inhibitor Benzodiazepine withdrawal syndrome Anxiety Drug withdrawal

• 19
A is not correct. 11% chose this .
• 20 Buspirone is an anxiolytic agent that is used to treat general ized anxiety disorder. It has a slow onset of action
• 21 ( it typical ly ta kes 2-3 wee ks to show any effect) and a longer duration of action. It is not pharmacologically

6
lock
s
Suspend
0
End Block
Item: 15 of 73 ~. I • M k <:] t> al ~· ~
QIO: 1488 .l. ar Previous Next lab 'lifllues Notes Calculator

1 •

2
A is not correct. 11% chose this.
Buspirone is an anxiolytic agent that is used to treat general ized anxiety disorder. It has a slow onset of action
3
( it typical ly takes 2-3 weeks to show any effect) and a longer duration of action. It is not pharmacologically
4 re lated to barbiturates, benzodiazepines, or other sedatives or anxiolytics. There is no risk of dependence.
5 Anxiolytic Buspirone Generalized anxiety disorder Anxiety disorder Benzodiazepine Barbiturate Anxiety Sedative Onset of action Pharmacology

6 B is not correct. 27% chose this.


7 Diazepam is the benzodiazepine with the longest half-life. It is used mainly in the treatment of status epi lepticus
8 and del irium tremens. I t does have the potential for causing dependency, but its long half-l ife reduces onset and
9 severity of withdrawa l symptoms, and thus reduces physical dependence.
Benzodiazepine Diazepam Delirium tremens Physical dependence Status epilepticus Half-life Delirium Benzodiazepine withdrawal syndrome
10
11 C is not correct. 6°/o chose this.
12 Hydroxyzine is an antihistamine with H1 -receptor antagonist effects. In addition to treating pruritus, it is a
useful as an anxiolytic with no addictive potential.
13 Antihistamine Anxiolytic Hydroxyzine Receptor antagonist Itch Antagonist
14
E is not correct. 5°/o chose this.
15
Thiopental is a barbiturate that can be used to treat anxiety. I t is not preferable to benzod iazepines for the
• 16
treatment of anxiety because of its toxicity profile .
• 17 Barbiturate Benzodiazepine Sodium thiopental Anxiety Toxicity

• 18
• 19
Bottom Line:
• 20
Al l benzodiazepines have a relatively high potential for causing dependency. Shorter-acting agents, such as
• 21
• I"'Y;:!7,:>n::~m tri::~?nl;:~m ::~nri miri;:~?nl;:~m ;:~r,:> n;:~rtirr d ::~rlv li~""lv tn r::~rrc::,:> rJ,..n,.. n rl""nrv R,..m,..m h,:>r th"" c::hnrt-;=~rti nn

6
lock
s
Suspend
0
End Block
Item: 15 of 73 ~. I • M k <:] t> al ~· ~
QIO: 1488 .l. ar Previous Next lab 'lifllues Notes Calculator

1 ( it typical ly takes 2-3 weeks to show any effect) and a longer duration of action. It is not pharmacologically
2 related to barbitu rates, benzodiazepines, or other sedatives or anxio lytics. There is no risk of dependence.
Anxiolytic Buspirone Generalized anxiety disorder Anxiety disorder Benzodiazepine Barbiturate Anxiety Sedative Onset of action Pharmacology
3
4 B is not correct. 27% chose this.
5 Diazepam is the benzodiazepine with the longest half-life. It is used mainly in the treatment of status epi lepticus
and del irium tremens. I t does have the potential for causing dependency, but its long ha lf-l ife reduces onset and
6
severity of withdrawal symptoms, and thus reduces physica l dependence.
7 Benzodiazepine Diazepam Delirium tremens Physical dependence Status epilepticus Half-life Delirium Benzodiazepine withdrawal syndrome

8
C is not correct. 6°/o chose this.
9
Hyd roxyzine is an antihistamine with H1 -receptor antagonist effects. I n addition to treating pruritus, it is a
10 usefu l as an anxiolytic with no addictive potential.
11 Antihistamine Anxiolytic Hydroxyzine Receptor antagonist Itch Antagonist

12 E is not correct. 5°/o chose this.


13 Thiopental is a barbiturate that can be used to treat anxiety. I t is not preferable to benzod iazepines for the
14 treatment of anxiety because of its toxicity profile.
Barbiturate Benzodiazepine Sodium thiopental Anxiety Toxicity
15
• 16
• 17 Bottom Line:
• 18
Al l benzodiazepines have a relatively high potential for causing dependency. Shorter -acting agents, such as
• 19 oxazepam, triazolam, and midazo lam, are particu larly like ly to cause dependency. Remember the short-acting
• 20 benzos with the mnemonic ATOM (Aiprazolam, T r iazo lam, Oxazepam, Midazolam) .
Oxazepam Midazolam Triazolam Benzodiazepine
• 21

6
lock
s
Suspend
0
End Block
Item: 15 of 7 3 ~ 1 • M k -<:J 1>- Jil ~· !:';-~
QIO: 1488 ..L ar Pre v ious Next Lab fli!ltues Not es Calcula t o r
& &
1
FA17 p 515.2
2
Benzodiazepines Diazepam, lorazepam, triazolam, lcmazcpam, oxazepam, midazolam, chlordiazepoxide,
3
alprazolam.
4
MECHANISM Facilitate GABA,\ action by t frequency of "Frcnzodiazcpines" t frequency.
5
c J- channel opening. l REJ\ 1sleep. .Niost 13enzos, barbs, and alcohol all bind the
6 have long haJf-Ji,·es and acti,·e metabolites GABA \ receptor, which is a ligand-gated CJ-
7 (exceptions (ATO~I]: Alprazolam, Triazolam, channel.
8 Oxazepam, and ~ Jidazolam are short acting Oxazepam, Temazepam, and Lorazepam are
- higher addictive potential). metabolized O utside T he Li,·er
9
10
CLINICAL USE Anxiety, spasticity, status epileplicus (lorazepam
and diazepam), eclampsia, detoxifica tion
11
(especiaJly alcohol withdrawai-Dls), night
12 terrors, sleepwalking, general anesthetic
13 (<lmncsia, muscle relaxation), hypnotic
14 (insomnia).
15 ADVERSE EFFECTS Dependence, additive CNS depression effects
• 16 with alcohol. Less risk of respiratory depression
and coma than with barbiturates .
• 17
Treat overdose with Aumazenil (compclit i\ C
. 18
antagonist at GAI3A benzodiazepine receptor).
• 19 Can precipitate seizures by causing acute
• 20 benzodiazepine withdrawal.
• 21

a
Lock
s
Suspend
8
End Bl ock
Item: 15 of 7 3 ~ 1 • M k -<:J 1>- Jil ~· !:';-~
QIO: 1488 ..L ar Pre v ious Next Lab fli!ltues Not es Calcula t o r
& &
1
FA17 p 239.1
2
Specific toxicity TOXIN TREATMENT
3
treatments Acetaminophen N-acetylcysteine (replenishes glutathione)
4
AChE inhibitors, organophosphates Atropine> pralidoxime
5
Antimuscarinic, anticholinergic agents Physostigmine, control hyperthermia
6
Arsenic Dimercaprol. succi mer
7
8
Benzodiazepines Flumazenil
9 ~-blockers Atropine, glucagon
Carbon monoxide 100% 0 , , hyperbaric 0 ,
10 - -
11 Copper Penicillamine, trientine (Copper penny)
12 Cyanide Nitrile+ thiosulfate, hydroxocobalamin
13 Digitalis (digoxin) Anti-dig Fab fragmen ts
14 Heparin Protamine sulfate
15 Iron Defe roxam ine, deferasirox, deferiprone
• 16 Lead ED'l 1\, dimercaprol, succimer, penicil lamine
• 17 Dimercaprol, succimcr
l\lcrcury
• 18
Methanol, ethylene glycol (antifreeze) F'omepizole >ethanol, dialysis
• 19
~ lethemoglobin 1\ lethvlene blue, ,·itamin C
• 20 '
OpiO ids 'aiOxO nc
• 21
• C::::., J;,..,.J... f.-.oC" ~ ... urn /. . JL.,_ J; ..,;'?O ur; no\ ,..J;., J•.C';C"

a
Lock
s
Suspend
8
End Bl ock
Item: 15 of 7 3 ~ 1 • M k -<:J 1>- Jil ~· !:';-~
QIO: 1488 ..L ar Pre v ious Next Lab fli!ltues Not es Calcula t o r
& &
1
FA17 p 239.1
2
Specific toxicity TOXIN TREATMENT
3
treatments Acetaminophen N-acetylcysteine (replenishes glutathione)
4
AChE inhibitors, organophosphates Atropine> pralidoxime
5
Antimuscarinic, anticholinergic agents Physostigmine, control hyperthermia
6
Arsenic Dimercaprol. succi mer
7
8
Benzodiazepines Flumazenil
9 ~-blockers Atropine, glucagon
Carbon monoxide 100% 0 , , hyperbaric 0 ,
10 - -
11 Copper Penicillamine, trientine (Copper penny)
12 Cyanide Nitrile+ thiosulfate, hydroxocobalamin
13 Digitalis (digoxin) Anti-dig Fab fragmen ts
14 Heparin Protamine sulfate
15 Iron Defe roxam ine, deferasirox, deferiprone
• 16 Lead ED'l 1\, dimercaprol, succimer, penicil lamine
• 17 Dimercaprol, succimcr
l\lcrcury
• 18
Methanol, ethylene glycol (antifreeze) F'omepizole >ethanol, dialysis
• 19
~ lethemoglobin 1\ lethvlene blue, ,·itamin C
• 20 '
OpiO ids 'aiOxO nc
• 21
• C::::., J;,..,.J... f.-.oC" ~ ... urn /. . JL.,_ J; ..,;'?O ur; no\ ,..J;., J•.C';C"

a
Lock
s
Suspend
8
End Bl ock
Item: 16 of 73 ~ 1 • M k -<:J 1>- Jil ~· !:';-~
QIO: 1444 ..L ar Pre v ious Next Lab fli!ltues Not es Calcula t o r

2
A 31-year-old man was recently brought to the hospital by his wife because he was "acting crazy." She IAA]
reported that her husband "hasn't slept in days," has lost $20,000 gambling, and init iated an affair with his
3 secretary. The patient speaks rapidly and demonstrates f light of ideas. He is prescribed a medication to help
4 prevent future episodes. The image shows a characteristic of this new medication's properties.
1
5
6 80% probability
of therapeutic
7 0.8 response
8
9 ~ 0.6
10 .,
~

~
Therapeutic
window
e ..
11 Cl. 0.4 •
12
13
0.2
14
15
• 16
0 10 100
• 17 Concentrati on
. 18
• 19 Wh ich of the fol lowing agents has this patient most likely been prescribed?
• 20
:
• 21 A. Gabapentin

a
Lock
s
Suspend
8
End Bl ock
2
3 Therapeutic
window
4
5
.. ..
6
7
8
9
0 10 100
10
Concentration
11
12
Which of the follow ing agents has t his patient most likely been prescr ibed?
13
:
14
A. Gabapentin
15
• 16
B. Lamotr igine

• 17 C. Lithium
• 18 D. Oxca rbamazepine
• 19
E. Risperidone
• 20
• 21

a
Lock
s
Suspend
8
End Block
Item: 16 of 73 ~. I • M k <:] t> al ~· ~
QIO: 1444 .l. ar Previous Next lab 'lifllues Notes Calculator

1 •

2 The correct answer is C. 79°/o chose this.


The patient has signs and symptoms of acute mania, as detai led in
3 Sign s an d Sym ptom s of Sig ns an d Sym ptom s of
the table, and likely has bipolar disorder. The image in the vignette Acute Mania Lithium Toxic ity
4 demonstrates a drug with a na r row therapeutic index/window. Th is
Pressured speech Nausea, vomiting, diarrhea
5 character istic, which is a measure of drug safety, is the ratio of the
Impulsive behavior Tremor
6 dose that produces toxic effects to the dose that is needed to (gambling, spending sprees,
produce a therapeutic response. Lithium, used for the treatment of sexual promiscuity) Ataxia
7 Nystagmus
acute man ia and mood stabilization in bipo lar disorder, has a very Flight of ideas, raci ng
8 narrow therapeutic index and thus requ ires close monitoring of thoughts Polyuria
9 ser um levels in patients who are treated with it. The toxicity curve Grandiosity Polydipsia

10
starts before 100% of the therapeutic response is achieved . Decreased need for sleep Confusion
Lithium can cause nephrogenic diabetes insipidus, electrolyte Increase in goal-directed
11 abnorma lities, hypothyroidism, and weight gain. Patients should be activity
12 monitored for tremor, ataxia, nystagmus, and salt intake since
13 high-salt diets can increase lithium excretion and low-salt diets can increase lith ium reabsor bance in the kidney,
leading to toxicity. It is also teratogenic and can cause Ebstein anoma ly in the infants of mothers who ta ke
14
lithium.
15 Therapeutic index Bipolar disorder Nystagmus Hypothyroidism Diabetes insipidus Electrolyte Ataxia Teratology Diabetes mellitus Mania Tremor Kidney

16 Nephrogenic diabetes insipidus Blood plasma Water-electrolyte imbalance Excretion lithium Toxicity Serum (blood) Salt (chemistry)
• 17
A is not correct. 4°/o chose this .
• 18
Gabapentin is a GABA analog used as adjunctive treatment for seizure disorder and as primary t reatment for
• 19 neuropath ic pain. It is not used to treat bipola r disorder and has a wide therapeutic index.
• 20 Gabapentin Therapeutic index Bipolar disorder Gamma-Aminobutyric acid Neuropathic pain Structural analog Epilepsy Peripheral neuropathy Epileptic seizure

• 21 B is not correct. 5°/o chose this .


6
lock
s
Suspend
0
End Block
Item: 16 of 73 ~. I • M k <:] t> al ~· ~
QIO: 1444 .l. ar Previous Next lab 'lifllues Notes Calculator

1 •
B is not correct. 5°/o chose this .
2 Lamotr igine is an anticonvulsant that is increasingly used for maintenance therapy in patients with bipolar
3 disorder. Although lamotrigine is associated with a hypersensitivity reaction that can lead to Stevens-Johnson
synd rome, slow t itration can minimize this risk . Aside from this adverse eff ect, it does not requ ire weekly or
4
month ly mon itoring to be used safely .
5 Stevens-Johnson syndrome Anticonvulsant lamotrigine Bipolar disorder Hypersensitivity Titration Adverse effect Maintenance therapy

6
D is not correct. 4°/o chose this.
7
Oxcarbamazepine is an anticonvulsant agent used to treat bipo lar disorder (off -label/investigational use). One
8 advantage of oxcar bamazepine over other anticonvulsants, such as carbamazepine and lithium, used to treat
9 bipolar disorder is that it has a wide therapeutic index.
Carbamazepine Anticonvulsant Therapeutic index Bipolar disorder lithium
10
11 E is not correct. 8°/o chose this.
12 Risperidone is an atypical antipsychotic agent approved as first-l ine monotherapy for bipolar disorder. Although
13 r isperidone can cause adverse effects over t ime, such as weight gain and elevated liver enzymes, the
therapeutic index is not narrow and it does not require close mon itoring to be used safely.
14
Atypical antipsychotic Risperidone Therapeutic index Bipolar disorder Antipsychotic Combination therapy liver Weight gain liver function tests
15
16
• 17 Bottom Line:
• 18 The therapeutic index is the ratio of the toxic dose to the therapeutic dose. The lower the number, the more
closely the patient shou ld be monitored for toxicity . Lith ium is one drug with a nar row therapeutic index. Signs
• 19
of toxicity include tremor, ataxia, nystagmus, and renal dysfunction .
• 20 Therapeutic index Nystagmus Ataxia Tremor Toxicity lithium Kidney

• 21

6
lock
s
Suspend
0
End Block
Item: 16 of 7 3 ~ 1 • M k -<:J 1>- Jil ~· !:';-~
QIO: 1444 ..L ar Pre v ious Next Lab fli!ltues Not es Calcula t o r
& &
1

2 FA17 p 228.2

3 Therapeutic index Measurement of drug safety. TITE: T herapeutic Index = T D ; 0 I E D;o·


4 TD 50 _ median toxic dose Safer drugs ha,·e higher T I \'a lues. D rugs with
ED;o median effecti\ e dose lower Tl \'a lues frequently require monitoring
5
(eg, \\ arfarin, T heophylline, D igoxin,
6 Therapeutic window-dosage range that can Lithium; \\ arning! T hese D rugs are Lethal!).
7 safely and effectively treat disease. LD 50 (lethal median dose) often replaces TD;o
8 in animal studies.
9 Efficacy Toxicity
100
10 en
c:
11 i5
c:
0
a.
12
13
-"'
CIJ

"'
c:
CIJ
50
·.:; ED = Etfecllve dose
14 "'
a. TO =Tox1c dose
0
15 ~

16 0
• 17 Log (drug concentration)

. 18
• 19 FA17p544.1
Lithium
• 20
MECHANISM i\'ol established; possibly related to inhibition of L iT III U~ I :
• 21
• - L - - L -:-- ....:&.-1 ----- ..l -

a
Lock
s
Suspend
8
End Bl ock
Item: 16 of 7 3 ~ 1 • M k -<:J 1>- Jil ~· !:';-~
QIO: 1444 ..L ar Pre v ious Next Lab fli!ltues Not es Calcula t o r
& &
1
2 FA17 p 544.1

3 Lithium
4 MECHANISM i'\ol established; possibly related lo inhibition of L iT III U~ I :

5
phosphoinositol cascade. Low T hyroid (hrpoth)Toidism)
CliNICAL USE ~lood stabilize r for bipolar disorder; blocks H eart (Ebstein anomaly)
6
rclapse.and acule manic C\ ents. I nsipidus (nephrogenic diabetes insipidus)
7 Unwanted ,\Jo,·eme nts (tremor)
ADVERSE EffECTS Tremor, hrpoth}Toidism , polyuria (causes
8
nephrogenic diabetes insipidus), teratogenesis.
9
Causes Ebstein anomaly in newborn if taken
10 by pregnant mother. arrow therape utic
11 window requires close monitoring of serum
12 Je,·els. Almost exclusively excreted by
kidneys; most is reabsorbed at PCT wi th Na+.
13
Thiazides (and other nephrotoxic agents) are
14
implicated in lithium toxicity.
15
16 FA17 p 543.2
• 17
Atypical Aripiprazole, ascnapinc, clozapinc, olanzapinc, quctiapinc, ilopcridonc, palipcridonc,
• 18 antipsychotics risperidone, lurasidonc, ziprasidonc.
• 19 1ol
MECHANISM completely understood. \los! arc D2
• 20 antagonists; aripiprazole is D2 partial agonist.
• 21 Va ried effects on 5-HT2, dopamine, and

a
Lock
s
Suspend
8
End Bl ock
Item: 16 of 7 3 ~ 1 • M k -<:J 1>- Jil ~· !:';-~
QIO: 1444 ..L ar Pre v ious Next Lab fli!ltues Not es Calcula t o r

1
&
implicated in lithium toxicity. &

2
3 FA 17 p 543.2

4 Atypical Aripiprazole, asena pinc, clozapinc, olanzapinc, quctiapinc, ilopcridone, palipcridonc,


5 antipsychotlcs risp eridone, lurasidonc, ziprasidonc.
6 MECHANISM Not completely understood. \lost arc 0 2
7 antagonists; aripiprazole is 0 2 partial agonist.
Varied effects on 5-HT 2, dopamine, and
8
<X- and I-1 1-reccptors.
9
CliNICAL USE Schizophrenia- both positi,·e and negative Use clozapine for treatment-resistant
10
S}'lnptoms. Also used for bipolar disorder, schizophrenia or schizoaffective disorder and
11 OCD, anxiety disorder, depression, mania, for suicida lity in schizophrenia.
12 'Iourette syndrome.
13 ADVERSE EFFECTS All-prolonged QT inlcmd, fewer EPS and
14 antichol inergic side effects than typical
15 anti psychotics.
"-pines"-metabolic syndrome (weight gain, O hmzapinc -+ O besity
16
diabetes, hyperli pidem ia).
• 17
C lozapine-agranulocytosis (mon itor W BCs Must watch bone ma rrow clozely with clozapinc.
• 18 frequently) and seizures (dose related).
• 19 Risperidone- hyperprolactinemia (amenorrhea,
• 20 galactorrhea, gynecomastia).
• 21

a
Lock
s
Suspend
8
End Bl ock
Item: 17 of 73 ~ 1 • M k -<:J 1>- Jil ~· !:';-~
QIO: 1135 ..L ar Pre v ious Next Labfli!llues Not es Calcula t o r
& &
1
A 20-year-old patient has been brought in by her parents for a psychiatric evaluation. For about a year, she
2
has stopped grooming herself and engag ing in social activities. Her family has had increasing difficulty trying
3 to commun icate with her. There is no known history of alcohol or drug abuse. For the past 6 months, the
4 patient has been stating that she is married to the Beatles singer, Paul McCartney. She gets very upset when her
parents try to tell her otherwise.
5
6
Which of the following terms best describes this new symptom that started 6 months ago?
7
8 :
A. Delusion
9
10 B. Hallucination
11 C. Idea of reference
12
D. I llusion
13
14
E. Loose association
15
16
• 17
• 18
• 19
• 20
• 21

a
Lock
s
Suspend
8
End Bl ock
Item: 17 of 73 ~. I • M k <:] t> al ~· ~
QIO: 1135 .l. ar Previous Next lab 'lifllues Notes Calculator

1 • The correct answer is A. 86°/o chose this .


2 Disorders of thought content are of two types: ideas of reference and delusions (seen here). Delusions are false
be liefs not correctable by logic or reason. These bel iefs are fir mly he ld by the individual only and not shared by
3
the cu lture . De lusions of persecution are the most common. Maintaining that one is marr ied to a celebrity is
4 another example of de lusional th inking . Other than a delusion, this patient also has the fol lowing symptoms of
5 schizoph renia: disorganized speech and negative symptoms (socia l withdrawa l, apathy manifesting as poor
6 grooming. Ha llucinations are another character istic symptom of schizophren ia are not seen in this case.
Diagnosis of schizophren ia also requ ires these symptoms to be present for at least 6 months and significantly
7
impa ir the patient's function.
8
Delusions are also present in de lusional disorder, but they are nonbizarre, and there cannot be other symptoms
9 of sch izophrenia, a previous diagnosis of schizoph renia, or other medica l conditions or med ications present that
10 e
may be causing the delusions. In folie deux, or shared psychotic disorder, two peop le may develop a shared
11 de lusion.
12 Diagnosis of schizophrenia Delusional disorder Ideas of reference and delusions of reference Psychosis Schizophrenia Delusion Hallucination Symptom

13 Persecutory delusion Shared psychotic disorder Negative symptoms Apathy

14 B is not correct. 4°/o chose this.


15 Hal lucinations are fa lse sensory perceptions in absence of an externa l stimu lus. In the case of schizophrenia,
16 these are usual ly auditory, but they can also be visua l, tacti le, gustatory, or olfactory. An example is the
17 "hearing of voices" giving a patient specific tasks or instructions to fo llow.
Schizophrenia Taste Hallucination Olfaction Sound Hearing
• 18
• 19 C is not correct. 4°/o chose this .
• 20 An idea of reference is the misinterpretation that other peoples' statements (or actions) or neutral objects in the
environment are directed toward one's self, when in fact they are not. Those with ideas of reference common ly
• 21
• have false convictions that thev are the subiect of attention bv other oeoole or the media. when th is is not the •
6
lock
s
Suspend
0
End Block
Item: 17 of 73 ~. I • M k <:] t> al ~· ~
QIO: 1135 .l. ar Previous Next lab 'lifllues Notes Calculator

1 •
C is not correct. 4°/o chose this.
2
An idea of reference is the misinterpretation that other peoples' statements (or act ions) or neutral objects in the
3
environment are di rected toward one's self, when in fact they are not . Those with ideas of reference common ly
4 have fa lse convictions t hat t hey are the subject of attention by other people or t he media, when t his is not t he
5 case.
Ideas of reference and delusions of reference
6
7 D is not correct. 4°/o c hose this.
8 I llusions are a disorder of perception, along with ha llucinations. Hal lucinations are fa lse sensory perceptions in
the absence of externa l st imu lus, whereas illusions are misperceptions of rea l ext ernal st imu li ( eg, a woman
9
interprets t he appea rance of a dress in a da r k closet as a person).
10 Hallucination Stimulus (physiology) Stimulus (psychology)
11
E is not correct. 2°/o c hose this.
12
Disorders of thought process include ( 1) clang associat ions, or associat ing words by their sounds, not because
13 of their log ica l meaning (eg, a patient says, "I 'm very su re I 've got a cure, and I 'm not pure."); (2) loose
14 associations, in wh ich ideas sh ift from one subj ect to another in an un related or only partially related fash ion
15 (eg, a patient beg ins to answer a question about her parents, then launches into a diat ribe about world
hunger ); (3) neolog isms, or invent ing new words; (4) perseveration, or repeating the same word or ph rase over
16
and over; (5) tangentiality, or beg inn ing a response in a logica l fash ion but t hen getting further and further
17 from t he point (going off on a t angent); (6) t hought blocking, or an abrupt ha lt in the train of th inking, often
• 18 because of ha llucinations; and (7) "word salad, " or uttering unrelated combinat ions of words or phrases (eg, a
• 19
patient says, "I 'm not so utt erly pu re that I 'm going to anyway to break it .") .
Thought blocking Perseveration Tangentiality Neologism
• 20
• 21

6
lock
s
Suspend
0
End Block
Item: 17 of 73 ~. I • M k <:] t> al ~· ~
QIO: 1135 .l. ar Previous Next lab 'lifllues Notes Calculator

1 •

2
D is not correct. 4°/o chose this.
I llusions are a disorder of perception, along with hal lucinations. Hallucinations are fa lse sensory perceptions in
3
the absence of externa l stimulus, whereas illusions are misperceptions of real external stimul i ( eg, a woman
4 interprets the appearance of a dress in a dark closet as a person).
5 Hallucination Stimulus (physiology) Stimulus (psychology)

6 E is not correct. 2°/o chose this.


7 Disorders of thought process include ( 1) clang associations, or associating words by their sounds, not because
8 of their logica l meaning (eg, a patient says, "I'm very sure I 've got a cure, and I 'm not pure."); (2) loose
9 associations, in wh ich ideas sh ift from one subject to another in an unre lated or only partially re lated fash ion
(eg, a patient beg ins to answer a question about her parents, then launches into a diatribe about world
10
hunger); (3) neologisms, or inventing new words; (4) perseveration, or repeating the same word or phrase over
11 and over; (5) tangentia lity, or beginn ing a response in a logica l fash ion but then getting further and further
12 from the point (going off on a tangent); (6) thought blocking, or an abrupt ha lt in the train of thinking, often
because of ha llucinations; and (7) "word salad, " or uttering unrelated combinations of words or phrases (eg, a
13
patient says, "I 'm not so utterly pure that I 'm going to anyway to break it .").
14 Thought blocking Perseveration Tangentiality Neologism
15
16
Bottom Line:
17
• 18
Delusions are fixed, fa lse beliefs that are not correctable by logic or reason. I deas of reference are fa lse
convictions that one is the subject of attention by other people or the med ia. Both are disorders of thought
• 19
content and can be seen in psychotic disorders like schizophrenia .
• 20 Schizophrenia Ideas of reference and delusions of reference Psychosis Delusion

• 21

6
lock
s
Suspend
0
End Block
Item: 17 of 7 3 ~ 1 • M k -<:J 1>- Jil ~· !:';-~
QIO: 1135 ..L ar Pre v ious Next Labfli!llues Not es Calcula t o r
& &
1
FA17 p 530.2
2
3 Delusional disorder F'ixed, persistent, false belief srstem lasting > 1 month. Functioning otherwise not impaired
(eg, a woman who genuinely believes she is married to a celebrity when, in fact, she is not).
4
Can be shared by individm1ls in close relationships (folic a deux).
5
6
FA17 p 530.1
7
Schizophrenia Chronic mental disorder with periods of Frequent cannabis use is associated" ith
8
psychosis, disturbed bchm ior and thought, ps}chosis/schizophrenia in teens.
9 and decline in functioning Ia ting > 6 Lifetime pre\·alence-1.5% (males= females,
10 months. Associated with f dopamincrgic frican Americans= Caucasians). Presents
11 activity, ! dendritic branching. earlier in men (late teens to early 20s \Slate
Diagnosis requires at least 2 of the following, 20s to early 30s in women). Patients are at f
12
and at least I of these should include 1- 3 risk for suicide.
13
(first 4 are "positive symptoms"): cntriculomegaly on brain imaging.
14 Treatment: atypical antipsychotics (eg,
I. Delusions
15 2. llallucinations- often auditory risperidone) are first line.
16 3. Disorganized speech egative symptoms often persist after treatment ,
4. Disorganized or catatonic behavior despite resolution of positive symptoms.
17
. 18 5. legati\·e symptoms (affect ive Oatlening,
avolition, anhedonia, asociality, alogia)
. 19
Brief psychotic disorder- lasting< l month,
• 20
usually stress related .
. 21

a
Lock
s
Suspend
8
End Bl ock
Item: 17 of 7 3 ~ 1 • M a rk -<:J 1>- Jil ~· !:';-~
QIO: 1135 ..L Pre v ious Next Labfli!llues Not es Calcula t o r

1 & -
months. Associated with t dopamincrgic
- fr ican Americans= Caucasians). Presents
&

2 activity, ' dendritic branch in g. earlier in men (late teens to early 20s \ Slate
3 Diagnosis requires at least 2 of the following, 20s to early 30s in women). Patients are at t
4 and at least I of these should include 1- 3 risk for suicide.
(first-+ are "positive symptoms"): Ventriculomegaly on brain imaging.
5
I. Delusions Treatment: atypical antipsychotics (eg,
6 risperidone) are fi rst line.
Z. llallucinat ions- often auditory
7 3. Disorganized speech 1\:egali,·e s~ mptoms often persist after treatment,
8 -+. Disorganized or catatonic behavior despite resolution of positiveS} mptoms.
9 5. 1\:egati,·e symptoms (affective Aatlening,
10 avolition, anhedonia, asocialily, alogia)
11 Brief psychotic disorder- lasting< 1 month,
12
usually stress related.

13 Schizophreniform disorder- lasting 1-6


months.
14
15
Schizoaffective disorder- Meets criteria for
schizophrenia in addition lo major mood
16
disorder (major depressive or bipolar). To
17 differentiate from a major mood disorder
. 18 with psychotic fea tures, pal ienl mmt have
. 19 > 2 weeks of hallucinations or delusions
• 20 without major mood episode.
. 21

a
Lock
s
Suspend
8
End Bl ock
Item: 18 of 73 ~ 1 • M k -<:J 1>- Jil ~· !:';-~
QIO: 1454 ..L ar Pre v ious Next Lab fli!ltues Not es Calcula t o r

IAA]
& &
1
A 73-year-old man being treated for depression is found to have a blood pressure of 170/100 mm Hg. On
2
exam, he general ly appears uncomfortable, confused, and is noted to smel l of alcohol. Fundoscopic exam
3 revea ls bilateral papilledema with f lame hemorrhages. There is no evidence of traumatic injury.
4
5 Which of the following drugs is the patient likely taking for depression?
6 :
7 A. Amitriptyline
8
B. Fluoxetine
9
C. Phenelzine
10
11 D. Trazodone
12 E. Ven lafaxine
13
14
15
16
17
. 18
. 19
• 20
. 21

a
Lock
s
Suspend
8
End Bl ock
Item: 18 of 73 ~. I • M k <:] t> al ~· ~
QIO: 1454 .l. ar Previous Next lab 'lifllues Notes Calculator

1 •

2 The correct answer is C. 52°/o chose this.


3 Monoamine oxidase inh ibitors (MAOis) are an older class of antidepressants now rarely used. They act by
inhibiting monoamine oxidase, an enzyme responsible for inactivating excess neurotransmitter. The net resu lt is
4
an increase in neurotransmitter concentration, wh ich helps alleviate symptoms of depression . Of note, patients
5 taking MAOis are told to avoid tyramine-containing foods, wh ich include aged cheeses and red wines. Tyramine
6 causes release of norepinephrine from nerve-end ings and increased blood pressu re. Normal ly, monoamine
7 oxidase inactivates tyramine and excess norepineph rine and prevents an increase in blood pressu re. When an
MAOI is coingested with wine (alcohol sme ll is noted from the patient), these neurotransmitters remain active,
8 and norepinephine increases blood pressure, leading to a hypertensive cr isis. Th is severe systemic hypertension
9 can cause end-organ damage, such as the increased intracran ial pressure and hypertensive encephalopathy
10 seen in this patient. Additiona lly, MAOis are contraind icated in patients taking many other medications,
including SSRis, TCAs, St. John 's wort, and dextromethorphan, due to the r isk of serotonin syndrome .
11
Phenelzine is the only MAOI listed in the answer choices.
12 Phenelzine Serotonin syndrome Tyramine Oextromethorphan Serotonin Norepinephrine Hypertensive crisis Enzyme Hypertension Hypericum perforatum

13 Intracranial pressure Neurotransmitter Hypertensive emergency Monoamine oxidase inhibitor Monoamine oxidase Blood pressure
14 Selective serotonin reuptake inhibitor Tricyclic antidepressant Antidepressant Monoamine neurotransmitter Major depressive disorder Contraindication
15 Depression (mood) Encephalopathy Oxidase Alcohol
16
A is not correct. 20% chose this.
17
Amitr iptyl ine is a tricycl ic antidepressant that acts by blocking reuptake of norepinephrine and seroton in from
18 the synaptic cleft. It does not cause an increase in blood pressure or additiona l adverse reactions when
• 19 coingested with foods or dr ink containing tyramine .
• 20 Tricyclic antidepressant Amitriptyline Tyramine Serotonin Antidepressant Synaptic cleft Norepinephrine Chemical synapse Reuptake Blood pressure Tricyclic

• 21

B is not correct. 10% chose this .

6
lock
s
Suspend
0
End Block
Item: 18 of 73 ~. I • M k <:] t> al ~· ~
QIO: 1454 .l. ar Previous Next lab 'lifllues Notes Calculator

1 •

2
B is not correct. 10% chose this.
Fluoxetine, a selective serotonin reuptake inhibitor, acts by selectively inhibiting the presynaptic serotonin
3
reuptake pump. It does not cause an increase in blood pressure or additional adverse reactions when coingested
4 with foods or drink conta ining tyramine.
5 Selective serotonin reuptake inhibitor Tyramine Fluoxetine Serotonin Serotonin reuptake inhibitor Chemical synapse Blood pressure Reuptake Reuptake inhibitor

6 Enzyme inhibitor

7
D is not correct. 6°/o chose this.
8 Trazodone, a heterocyclic antidepressant, binds at 5-HT2 receptor, it acts as a serotonin agonist at high doses
9 and a serotonin antagonist at low doses. Trazodone's antidepressant activity results from blockage of serotonin
10 reuptake by inhibiting seroton in reuptake pump at the presynaptic neu rona l membrane. It does not cause an
increase in blood pressure or additional adverse reactions when coingested with foods or drink containing
11
tyramine.
12 Tyramine Antidepressant Trazodone Serotonin Serotonin antagonist Receptor antagonist Agonist Chemical synapse Serotonin receptor agonist

13 Heterocyclic compound Serotonin reuptake inhibitor Blood pressure Reuptake Antagonist Heterocyclic antidepressant Receptor (biochemistry)

14
E is not correct. 12% chose this.
15
Venlafaxine is a potent inhibitor of neu rona l serotonin and norepinephrine reuptake and a weak inhibitor of
16 dopamine reuptake . It is used to treat depression and generalized anxiety disorder. Although ven lafaxine can
17 cause an increase in blood pressure, it is not associated with increased blood pressure or add it ional adverse
18
reactions when coingested with foods or drinks containing tyramine.
Tyramine Venlafaxine Generalized anxiety disorder Serotonin Norepinephrine Dopamine Dopamine reuptake inhibitor Anxiety disorder Reuptake Blood pressure
• 19
Anxiety Major depressive disorder Depression (mood) Enzyme inhibitor
• 20
• 21

6
lock
s
Suspend
0
End Block
Item: 18 of 73 ~. I M k <:] t> al ~· ~

. ..

QIO: 1454

1
.l. ar
..
Previous
.. .
Next
..
lab 'lifllues Notes Calculator

Trazodone, a heterocyclic antidepressant, binds at 5-HT2 receptor, it acts as a serotonin agonist at high doses
2
and a serotonin antagonist at low doses. Trazodone's antidepressant activity results from blockage of serotonin
3 reuptake by inhibiting seroton in reuptake pump at the presynaptic neu rona l membrane. It does not cause an
4 increase in blood pressure or additional adverse reactions when coingested with foods or drink containing
5 tyramine.
Tyramine Antidepressant Trazodone Serotonin Serotonin antagonist Receptor antagonist Agonist Chemical synapse Serotonin receptor agonist
6
Heterocyclic compound Serotonin reuptake inhibitor Blood pressure Reuptake Antagonist Heterocyclic antidepressant Receptor (biochemistry)
7
8 E is not correct. 12% chose this.
9 Venlafaxine is a potent inhibitor of neu rona l serotonin and norepinephrine reuptake and a weak inhibitor of
dopamine reupta ke. It is used to treat depression and generalized anxiety disorder. Although ven lafaxine can
10
cause an increase in blood pressure, it is not associated with increased blood pressure or add it ional adverse
11 reactions when coingested with foods or drinks containing tyramine.
12 Tyramine Venlafaxine Generalized anxiety disorder Serotonin Norepinephrine Dopamine Dopamine reuptake inhibitor Anxiety disorder Reuptake Blood pressure

13 Anxiety Major depressive disorder Depression (mood) Enzyme inhibitor

14
15
Bottom Line:
16
Monoamine oxidase inh ibitors ( MAOis) are an older class of antidepressants used to t reat atypica l or refractory
17 depression. People taking an MAOI shou ld avoid foods or drink containing tyramine (eg, aged cheese and red
18 wi ne), due to the adverse effect of hy pertensive crisis, in w hich blood pressure can get dangerously high due
• 19 to excess circu lating catecholamines .
Tyramine Monoamine oxidase inhibitor Hypertensive crisis Hypertensive emergency Monoamine oxidase Catecholamine Monoamine neurotransmitter
• 20 Antidepressant Treatment-resistant depression Blood pressure Major depressive disorder Cheese ripening Adverse effect Depression (mood) Red wine Cheese
• 21

6
lock
s
Suspend
0
End Block
Item: 18 of 7 3 ~ 1 • M k -<:J 1>- Jil ~· !:';-~
QIO: 1454 ..L ar Pre v ious Next Lab fli!ltues Not es Calcula t o r
& &
1
FA17 p546.1
2
Monoamine oxidase Tranylcypromine, P henelzine, Isocarboxazid, Selegil inc (selecti,·e YL\0-B inhibitor).
3
inhibitors (~lAO Takes Pride In Shanghai).
4
MECHANISM 'onselecti,·e .\If 0 inhibition t b·el~ of amine neurotransmitters (norepinephrine, ;-I IT,
5
dopamine).
6
CLINICAL USE Al}'pical depression, an-:iel). Parl.inson disease (selegiline).
7
ADVERSE EFFECTS CNS stimulation; hrpertensive crisis, most notably ''ith ingestion of tyramine, which is found in
8
mam· foods such as aged cheese and wine. Tyramine displaces other neurotransmitters (eg, NE)
9 into the sp1aptic cleft - t sympathetic stimulation. Contraindicated with SSRls, TCAs, St. John's
10 wort, meperidine, dextromethorphan (to prevent serotonin syndrome).
11 Wait 2 \\·eeks after stopping MAO inhibitors before starting serotonergic drugs or stopping dietary
restrictions.
12
13
14 FA17 p 544.3
15 Antidepressants
16
NORADRENERGIC SEROTONERGIC
17
18 AXON AXON
• 19
- - Metabolites
• 20 Metabollt<!S - -

• 21

.,r:;- Buproplon \
A.-~ <UT

a
Lock
s
Suspend
8
End Bl ock
Item: 18 of 7 3 ~ 1 • Ma r k -<:J I> ~ £!1}>'
• !!":-~
QIO: 1454 ..L Prev ious Next Lab lues Not es Calculat o r

1 &
' AAUN &

--rr-- MAO Inhibitors -


2 ~ ~
-0----~-f MAO - - • Metabolites
Metabolites - - - MAO
3
4
NE 0
I 00
Buproplon

5
6
(/o
0
7
I
8
TCAs,SNRis ~'tlt.- Mlrtazaplne TCAs. SSRis.
9
t NEreupta~ JY SNRis, trazodone

10 '-.._o o
11
5-HT re<ept or
12
13
14
15
16
POSTSYNAPTIC NEURON
17
18 FA17 p 545.4
. 19
Tricyclic Amitriptyline, nortriptyline, imipramine, desipramine, clomipramine, doxepin, amoxapine.
• 20 antidepressants
. 21
• MECHANISM Inhibit ' E and )-l iT reuotake.

a
Lock
s
Suspend
8
End Bl ock
Item: 18 of 7 3 ~ 1 • M k -<:J 1>- Jil ~· !:';-~
QIO: 1454 ..L ar Pre v ious Next Lab fli!ltues Not es Calcula t o r
-
2 ~ NE receptor
5-HT receptor

3
4
5
6
7 POSTSYNAPTIC NEURON

8
FA17 p 545.4
9
10 Tricyclic Amitriptyline, nortriptyline, imipramine, dcsipmminc, clomipramine, doxcpin, amoxapine.
antidepressants
11
12 MECHANISM Inhibit NE and ) -l iT reuptake.
13 CLINICAL USE lajor depression, OCD (clomipramine), peripheral neuropathy, chronic pain, migraine
prophylaxis. octurnal enuresis (imipramine, although adverse effects may limit use).
14
15
ADVERSE EFFECTS Sedation, a 1-blocking effects inclnding postural hypotension, and atropine-like (anticholinergic)
side effects (tachycardia, minary retention, dry mouth). 3° TCAs (amitriptyline) have more
16
antichol inergic effects than 2° TCAs (nortriptyline). Can prolong QT interval.
17 '!'ri-C 's: Convulsions, Coma, C ardiotoxicity (arrhythmia due to 1 a+ channel inhibition);
18 also respiratory depression, hyperpyrexia. Confusion and hallucinations in elderly due to
. 19 anticholinergic side effects (nortriptyline beller tolerated in the elderly). Treatment: 1al JC03 to
• 20 prevent arrhythmia .
. 21

a
Lock
s
Suspend
8
End Bl ock
Item: 19 of 73 ~. I • M k <:] t> al ~· ~
QIO: 4601 .l. ar Previous Next lab 'lifllues Notes Calculator

1 •
A 2 1-year-old woman presents to her male family practit ioner comp laining of sleep depr ivation and severe
2 depression. When asked about her sleeping habits in greater depth, she reports sleeping for 7-9 hou rs per
3 night, but states that it is "just not right." I n addition, she later claims to not have any problems with her
4 personal life and is happy and excited about her recent promotion . Throughout the course of the visit, the patient
becomes progressively more an imated and begins making aggressive sexual advances toward the physician and
5
the staff. She reports that her mood has been good, but not overly elevated, expansive, or elated.
6
7
Wh ich of the fo llowing is the most like ly diagnosis?
8
:
9
A. Bipo lar disorder type I
10
B. Histrionic persona lity disorder
11

12 C. Obsessive-compu lsive personality disorder


13 D. Paranoid personal ity disorder
14
E. Schizotypa l personal ity disorder
15
16
17
18
• 19
• 20
• 21

6
lock
s
Suspend
0
End Block
Item: 19 of 73 ~. I • M k <:] t> al ~· ~
QIO: 4601 .l. ar Previous Next lab 'lifllues Notes Calculator

1 •
The correct answer is B. 71°/o chose this.
2 Histrionic persona lity disorder is one of the cluster 8 personality disorders (the "wild" group) that manifests
3 early in adu lthood . It is best cha racterized as a pattern of excessive emotionality and attention seeking, and it is
4 often accompanied by somatoform disorders (somatization is a process by wh ich an ind ividua l uses his or her
body or symptoms for a range of psycho log ic purposes and gains) . An especia lly important character istic to
5
remember about histrion ic persona lity disorder is the often overtly sexual nature of those affected. Relationsh ips
6 with physicians in particular are affected by the patient's attention-seeking behavior, as is evident in this case
7 by the patient reporting both sleep deprivation and depression without objective signs of either.
Histrionic personality disorder Attention seeking Cluster B personality disorders Somatic symptom disorder Personality disorder Sleep deprivation
8
Depression (mood) Somatization Major depressive disorder
9
10 A is not correct. 20% chose this.
11 Type 1 bipola r disorder is cha racterized by manic episodes (periods of elevated or irritable mood that must last
at least 1 wee k) . Sleep of 7-9 hours per night is not usual ly consistent with a manic syndrome. Although
12
patients with bipolar I disorder may have depressive episodes or symptoms, depression is not part of the
13 diagnostic criteria .
14 Bipolar disorder Bipolar I disorder Major depressive disorder Mania Depression (mood) Medical diagnosis

15
C is not correct. 2°/o chose this.
16 Obsessive-compu lsive personality disorder is cha racterized by an excessive preoccupation with control, order,
17 and perfection ism. Th is patient presents none of these characteristics.
Obsessive-compulsive personality disorder Obsessive-compulsive disorder Personality disorder Perfectionism (psychology) Obsessive-compulsive
18
19 Dis not correct. 1°/o chose this.
0 20 Paranoid personal ity disorder is character ized by the inherent be lief that the world is a dangerous and
0 21 threatening place. Upon meeting these individua ls they often project strength and capability, and their distrust

6
lock
s
Suspend
0
End Block
Item: 19 of 73 ~. I • M k <:] t> al ~· ~
QIO: 4601 .l. ar Previous Next lab 'lifllues Notes Calculator

1
p I p I I y I p p I y p I p p
diagnostic criteria .
2 Bipolar disorder Bipolar I disorder Major depressive disorder Mania Depression (mood) Medical diagnosis
3
C is not correct. 2°/o chose this.
4
Obsessive-compu lsive personality disorder is cha racterized by an excessive preoccupation with control, order,
5
and perfection ism. Th is patient presents none of these characteristics.
6 Obsessive-compulsive personality disorder Obsessive-compulsive disorder Personality disorder Perfectionism (psychology) Obsessive-compulsive

7
Dis not correct. 1°/o chose this.
8
Paranoid personal ity disorder is character ized by the inherent be lief that the world is a dangerous and
9 threatening place. Upon meeting these individua ls they often project strength and capability, and their distrust
10 and suspiciousness of everyone is evident. These ind ividuals tend to bel ieve in various conspiracy theories.
Paranoid personality disorder Personality disorder Conspiracy theory Paranoia
11

12 E is not correct. 6°/o chose this.


13 Schizotypa l personal ity disorder is characterized by interpersonal awkwa rdness, odd bel iefs or magical thinking,
14
and an eccentric appearance .
Schizotypal personality disorder Magical thinking Personality disorder
15
16
17 Bottom Line:
18 Histrionic persona lity disorder is one of the cluster 8 personal ity disorders. It manifests early in adu lthood as
19 excessive emotiona lity, attention-seeking, and sexually provocative behavior.
Histrionic personality disorder Cluster B personality disorders Personality disorder Attention seeking
0 20
0 21

6
lock
s
Suspend
0
End Block
Item: 19 of 7 3 ~ 1 • M k -<:J 1>- Jil ~· !:';-~
QIO: 460 1

1
..L
. ···-. -·-
FIRST AID FAC T S
ar
. Pre v ious
' -. . Next Lab fli!ltues Notes Calcula t o r

2
3 FA17 p 535.2

4 Cluster B personality Dramatic, emotional, or erratic; genetic "\\'ild."


5 disorders association with mood disorders and substance Cluster B: Bad, Borderline, AamBo}'ant, must
abuse. be the Best
6
7
Antisocial Disregard for and \'iolation of rights of others Anti~ocial =~ociopath.
with lack of remorse, criminalit}', impulsivitr; Bad.
8
males> females; must be~ 18 •years old and
9 ha,·e history of conduct disorder before age 15.
10 Conduct disorder if< 18 years old.
11 Borderline Unstable mood and interpersonal relationships, Treatment: dialectical beha\'ior therapy.
12 impulsivity, self-mutilation, suicidality, sense Borderline.
13
of emptiness; females> males; splilting is a
major defense mechan ism.
14
Histrionic Excessive emotionality and excitability, FlamBoyant.
15
<tttention seeking, sexually provocative, overly
16
concerned with appearance.
17
Narcissist ic Grandiosity, sense of entitlement; lacks empathy 1ust be the Best.
18 and requires excessive admiration; often
19 demands the "best" and reacts to criticism
• 20 with rage.
. 21

a
Lock
s
Suspend
8
End Bl ock
Item: 20 of 73 ~ 1 • M k -<:J 1>- Jil ~· !:';-~
QIO: 4123 ..L ar Pre v ious Next Labfli!llues Not es Calcula t o r

IAA]
& &
1
A 34-year-old man is somnolent and difficult to arouse on arrival at the emergency department. He has
2
ataxia and poor memory function. He recently was treated for sleeplessness with a medication that is in the
3 same class as some of the anticonvu lsants.
4
5 What is the mechanism of action of the drug this patient should receive as treatment for his current symptoms?
6 :
7 A . Alkalinizes the urine to enhance elimination
8
B. Antagonist at serotonin receptors
9
C. Competitive antagonist at y-am inobutyric acid receptors
10
11 D. High affinity for opioid 1-1-receptors
12 E. Modulates function of neuronal Na+ channels
13
14
15
16
17
18
19
• 20
. 21

a
Lock
s
Suspend
8
End Bl ock
Item: 20 of 73 ~ 1 • M k -<:J 1>- Jil ~· !:';-~
QIO: 4123 ..L ar Prev ious Next Labfli!ll ues Not es Calculat o r

& &
1
2 The correct answer is C. 65°/o chose this.
3
Flumazeni l acts as a compet it ive antagon ist at the benzodiazepine-bind ing
site on the GABA receptor. It can reverse the action of benzodiazepines such
4
as temazepam, diazepam, and midazolam. Based on his history and recent
5 treatment for sleeplessness, the patient is most likely suffering from
6 benzodiazepine intoxication. Benzodiazepines increase y-aminobutyric acid
(GABA) receptor action by increasing the frequency of chloride channel
7
opening, as illustrated in the diagram. Intoxication with benzodiazepines
8 causes symptoms of somnolence, ataxia, minor respiratory depression, and
9 amnesia.
Benzodiazepine Diazepam Temazepam Midazolam GABA receptor Hypoventilation Flumazenil Somnolence
10
Ataxia Gamma·Aminobutyric acid Receptor antagonist Chloride channel Amnesia Competitive antagonist
11
Insomnia Major depressive disorder Depression (mood) Antagonist Receptor (biochemistry) Chloride
12
13
14 Image courtesy of Wikimedia
Commons
15
16 A is not correct. 7°/o chose this.
17 Sod ium bicarbonate is given for a significant aspirin overdose (salicylate level >35 mg/d l 6 hours after
18 ingestion), regardless of the serum pH because it enhances elim ination of aspirin in the urine. I t is given until a
urine pH of 7.5-8.0 is achieved .
19
Sodium bicarbonate Aspirin PH Sodium Urine Bicarbonate Salicylic acid Blood plasma Drug overdose Serum (blood)
20
. 21 B is not correct. 7 °/o chose this .
• ,-. •••••~• • •••• 4 - ...l: .... ... ·· - - --1-: . . :L..... -L ............... - 1 ........... . ... 1- ... . . ... 1..... 4 ! &.. ! .............. .... ..... 4 - l ... l ... £ ... . ... : L ... ___ .,_ _ _ _ __ : .......... -"- ~ I I"T" . ....... ....... .L ..... ...

Lock
a Suspend
s 8
End Block
Item: 20 of 73 ~. I • M k <:] t> al ~· ~
QIO: 4123 .l. ar Previous Next Lab 'lifllues Notes Calculator
I
1 •

2 B is not correct. 7°/o chose this.


3
Cyproheptad ine has activity at severa l receptors, but it is most notable for its antagonism at 5-HT receptors .
Although it has several indications, it is included in th is question because of its ro le in the treatment of serotonin
4
synd rome .
5 Serotonin syndrome Cyproheptadine Serotonin Receptor (biochemistry)

6
D is not correct. 6°/o chose this.
7
Naloxone is the reversal agent for opioid intoxication, w hich is characterized by centra l nervous system
8 depression, nausea and vomit ing, constipation, papillary constriction, and seizures. Naloxone acts at opioid
9 receptors.
Naloxone Central nervous system Opioid Constipation Toxidrome Nausea Vomiting Epileptic seizure Central nervous system depression Substance intoxication
10
Major depressive disorder Receptor (biochemistry) Depression (mood) Nervous system Alcohol intoxication Opioid receptor
11

12 E is not correct. 15% chose this.


13 Severa l drugs used in the treatment of epi lepsy, including phenytoin, carbamazepine, and va lproate, modu late
14
activity at Na+ channels. However, these agents are not generally used to t reat benzodiazepine overdose.
Benzodiazepine Carbamazepine Phenytoin Benzodiazepine overdose Epilepsy Valproate Drug overdose
15
16
17 Bottom Line:
18 Flumazenil is the reversa l agent for benzodiazepine intoxication, which is character ized by somnolence,
19 respiratory depression, amnesia, and ataxia.
Benzodiazepine Flumazenil Hypoventilation Somnolence Ataxia Amnesia Major depressive disorder Depression (mood)
20
• 21

6
lock
s
Suspend
0
End Block
Item: 20 of 73 ~ 1 • M k -<:J 1>- Jil ~· !:';-~
QIO: 4123 ..L ar Pre v ious Next Labfli!llues Not es Calcula t o r
& &
1
FA17 p 515.2
2
Benzodiazepines Diazepam, lorazepam, triazolam, lemazepam, oxazepam, midazolam, chlordiazepoxide,
3
alprazolam.
4
MECHANISM Facilitate GABA,\ action by t frequency of "Frenzodiazepines" t frequency.
5
Cl- channel opening. l REX I sleep. .Most 13enzos, barbs, and alcohol all bind the
6 have long haJf-li,·es and acti,·e metabolites GABA \ receptor, which is a ligand-gated CJ-
7 (exceptions (ATOM]: Alprazolam, Triazolam, channel.
8 Oxazepam, and ~ Jidazolam are short acting Oxazepam, Temazepam, and Lorazepam are
9
- higher addictive potential). metabolized O utside T he Liver
10 CLINICAL USE Anxiety, spasticity, status epileplicus (lorazepam
and diazepam), eclampsia, detoxifica tion
11
(especially alcohol withdrawal-Dls), night
12 terrors, sleepwalking, general anesthetic
13 (amnesia, muscle relaxation), hypnotic
14 (insomnia).
15 ADVERSE EFFECTS Dependence, additive CNS depression effects
16 with alcohol. Less risk of respiratory depression
and coma than with barbiturates.
17
Treat overdose with Aumazenil (compclit i\ C
18
antagonist at GAI3A bcnzodiazepine receptor).
19 Can precipitate seizures by causing acute
20 benzodiazepine withdrawal.
. 21

a
Lock
s
Suspend
8
End Bl ock
Item: 20 of 73 ~ 1 • Ma rk -<:J 1>- Jil ~· !:';-~
QIO: 4123 ..L Pre v ious Next Labfli!llues Not es Calcula t o r

1
FA17 p228.1
2

3 Receptor binding
4
5 •. . r-----::~---- Z:~ 100 r-------------7'1------ 100 r-------------------~--

6
7
~
§ I alone competitive

·i ~---------1------J--
antagonist E
AgoniSt ""'-
alone
A
':1
.....-Effect of
antagomst ~E
=>
AgoniSt""'-
alone
lower
efficacy

8
9
!,. ---------r:_g_ "'''"' -o
~
so
II
-~

0
Partial aoomst
~ so r----- ~~-
~

10 j .I
j !1
competitive
antagonist
~
I
I
Agon1st plus
noncompebtive
antagomst
j
11 I

12 0 0 0
01 1.0 10 100 1000 01 10 10 100 1000 0.1 1.0 10 100 1000
13 Agonist dose Agonist dose Agonist dose Ill
14 AGONIST WITH EFFECT EXAMPLE
15 0 Competitive Sh ifts curve right {l potency), no change Diazep<lm {agonist) + Anmazenil (competitive
16 antagonist in efficacy. Can be overcome by t the antagon ist) on G BA receptor.
concentration of agonist substrate.
17
18 ~ Noncompetitive Shifts curve down (l efficacy). Cannot be orepinephrine (agonist) + phenoxybcnzaminc
antagonist overcome by t agonist substrate concentration. {noncompetitive antagonist) on a -receptors.
19
20 ~ Partial agonist Acts at same site as full agonist, but with lower lorphine {full agonist) \ 'S buprcnorphinc
• 21 (alone) maximal effect {l efficacy). Potency is an (partial agonist) at opioid Jl-receptors.

a
Lock
s
Suspend
8
End Bl ock
Item: 20 of 73 ~ 1 • M k -<:J 1>- Jil ~· !:';-~
QIO: 4123 ..L ar Pre v ious Next Labfli!llues Not es Calcula t o r
& &
1
2 FA17p239.1

3 Specific toxicity TOXIN TREAIMENT

4
treatments Acetaminophen '-acetylcysteine (replenishes glutathione)
5 AChE inhibitors, organophosphates Atropine> pralidoxime
6 Antimuscarinic, anticholinergic agents Physostigmine, control hyperthermia
7 Arsenic Dimercaprol. succimer
8 Benzodiazepines Flumazenil
9 ~-blockers Atropine, glucagon
10 Carbon monoxide 100% 0 7 , hyperbaric 0 7
11
- -
Copper Pe nicillamine, trientine (Copper pe nn))
12
Cyanide Nitrile+ thiosulfate, hydroxocobalamin
13
Digitalis (digoxin) Anti-dig Fab fragmen ts
14
Heparin Protamine sulfate
15
Iron Defe roxam ine, deferasirox, defe riprone
16
Lead EO'lA, dimercaprol, succimer, penicil lamine
17
l\lcrcury Dime rcaprol, succi mcr
18
Methanol, ethylene glycol (antifreeze) Fomepizole >ethanol, dialysis
19
20
l\ lethemoglobin 1\lethylene blue, vitamin C

. 21 O piO ids 'alO xO ne



a
Lock
s
Suspend
8
End Bl ock
Item: 21 of 73 ~ 1 • M k -<:J 1>- Jil ~· !:';-~
QIO: 1463 ..L ar Pre v ious Next Lab fli!ltues Not es Calcula t o r
& &
1
A 35-year-old man with a history of depression and schizophren ia presents for follow- up. His med ications
2
were changed a few months ago . He is frustrated due to new-onset symptoms that have significantly
3 interfered with his activities of daily living. Specifical ly, he has noticed restlessness and unsteadiness in his
4 feet. He den ies any alcohol or drug use. On examination, the patient adopts a stooped posture and a shuffl ing gait.
Prominent muscle r igidity is noted. There is no galactorrhea or gynecomastia.
5
6
These adverse effects are due to blockade of dopamine in which of the following pathways?
7
8 :
A. Mesocortical
9
10 B. Mesolimbic
11 C. Nigrostriata l
12
D. Papez circuit
13
14
E. Tuberoinfundibular
15
16
17
18
19
20
. 21

a
Lock
s
Suspend
8
End Bl ock
Item: 21 of 73 ~. I • M k <:] t> al ~· ~
QIO: 1463 .l. ar Previous Next lab 'lifllues Notes Calculator

1 •

2 The correct answer is C. 71°/o chose this.


3 This patient is experiencing a parkinsonian-type extrapyramidal How Antlpsychotlcs
Function
adverse eff ect from antipsychotic use to manage his schizoph renia. Affect Pathway
4
Antipsychotics block the dopamine D2 receptor in the meso limbic Involved in brain's Decrease positive
reward system; symptoms of schizophrenia
5 pathway in order to decrease the positive symptoms associated Mesolimblc responsible tor
6 with sch izophrenia . However, these drugs also block dopamine posrtive symptoms
in schizophrenia
7
receptors in other pathways. The nigrostr iatal pathway is a Extrapyramidal symptoms:
Extrapyramidal
dopaminergic pathway invo lved in the extrapyramida l motor loop motor system
parkinsonism (rigidity,
Nigrostriatal tremor, stooped posture,
8 that connects the substantia nigra to the str iatum. This motor loop (background motor
shuffling gait); akathisia
tone)
(resuessness); dystonia
9 regu lates backg round motor tone . Blockade of dopamine in the
Galactorrhea, amenorrhea
10 nigrostriata l pathway causes parkinsonian symptoms of motor Tubuloinfundibular Prolactin regulation in females: gynecomastia,
impa irment such as r ig idity, stooped posture, shuffling gait, and decreased libido in males
11 Motivation; Typical antipsychotics do
tremor. The medications most like ly to cause this toxicity are the emotion: not affect (atypical
12 high-potency antipsychotics (eg, haloperidol and trif luoperazine) . Mesocortical responsible tor
negative symptoms
antipsychotics help alleviate
negative symptoms
13 The table ind icates how dopaminergic pathways are aff ected by in schizophrenia somewhat)

14
typical antipsychotic agents .
Typical antipsychotic Haloperidol Mesolimbic pathway Nigrostriatal pathway Trifluoperazine Substantia nigra Schi zophrenia Dopamine Dopaminergic pathways
15
Striatum Dopaminergic Antipsychotic Extrapyramidal system Tremor Extrapyramidal symptoms Adverse effect Receptor (biochemistry) Parkinson's disease
16
Dopamine receptor Toxicity
17
18 A is not correct. 11% chose this.
19 The mesocortical pathway is a dopaminergic pathway in the brain connecting the midbrain to the cerebral
cortex, particu larly the frontal lobes . This pathway is invo lved in motivation and emotion . It is postulated that
20
abnorma lities in this pathway are responsible for the negative symptoms of schizophren ia, such as apathy,
21

avol it ion, and socia l withdrawal. However, typical antipsychotics are generally ineffective at t reating these

6
lock
s
Suspend
0
End Block
Item: 21 of 73 ~. I • M k <:] t> al ~· ~
QIO: 1463 .l. ar Previous Next lab 'lifllues Notes Calculator

1 • •
A is not correct. 11% chose this.
2
The mesocortical pathway is a dopaminergic pathway in the brain connecting the midbrain to the cerebral
3 cortex, particu larly the frontal lobes. This pathway is invo lved in motivation and emotion. It is postulated that
4 abnorma lities in this pathway are responsible for the negative symptoms of schizophren ia, such as apathy,
avol it ion, and socia l withdrawal. However, typical antipsychotics are generally ineffective at t reating these
5
symptoms.
6 Avolition Mesocortical pathway Dopaminergic pathways Cerebral cortex Midbrain Schizophrenia Antipsychotic Dopaminergic Frontal lobe Typical antipsychotic
7 Brain Emotion

8
B is not correct. 13% chose this.
9
The mesol imbic pathway is a dopaminergic pathway in the brain that connects areas of the midbrain to the
10 limbic system . I t is theorized that an overabundance of dopamine in th is pathway is responsible for the positive
11 symptoms of sch izophrenia . Typica l antipsychotics target positive symptoms by blocking dopamine receptors in
12 this pathway. Thus, blockade of dopamine in th is pathway is not the cause of th is patient's adverse effects.
Mesolimbic pathway limbic system Dopaminergic pathways Midbrain Dopamine Schizophrenia Antipsychotic Dopaminergic Typical antipsychotic Human brain
13
Brain
14
15 D is not correct. 2°/o chose this.
16 The Papez circuit is a limbic system pathway involved in memory storage. I t is not affected by antipsychotic
drugs.
17
limbic system Papez circuit Antipsychotic James Papez
18
19
E is not correct. 3°/o chose this.
The tuberoinfundibu lar pathway is a dopaminerg ic pathway that regu lates the production of the hormone
20
prolactin. In this pathway, dopamine functions to block release of prolactin. Thus, antipsychotic drugs that block
21 dopamine receptors can cause an increase in prolactin, resulting in adverse effects such as ga lactorr hea and •

6
lock
s
Suspend
0
End Block
Item: 21 of 73 ~. I • M k <:] t> al ~· ~
QIO: 1463 .l. ar Previous Next lab 'lifllues Notes Calculator

1
- . - - -- .. . --- -- - ---- --
symptoms of sch izophrenia . Typica l antipsychotics target positive symptoms by blocking dopamine receptors in
.. .. . ..... - - -- .
2 this pathway. Thus, blockade of dopamine in th is pathway is not the cause of th is patient's adverse effects.
3 Mesolimbic pathway limbic system Dopaminergic pathways Midbrain Dopamine Schizophrenia Antipsychotic Dopaminergic Typical antipsychotic Human brain

4 Brain

5 D is not correct. 2°/o chose this.


6 The Papez circuit is a limbic system pathway involved in memory storage. I t is not affected by antipsychotic
7 drugs.
limbic system Papez circuit Antipsychotic James Papez
8
9 E is not correct. 3°/o chose this.
10 The tuberoinfundibu lar pathway is a dopaminerg ic pathway that regu lates the production of the hormone
11
prolactin. I n this pathway, dopamine functions to block release of prolactin. Thus, antipsychotic drugs that block
dopamine receptors can cause an increase in prolactin, resulting in adverse effects such as ga lactorrhea and
12 amenorrhea in women and gynecomastia in men. These are not the symptoms this patient is experiencing.
13 Gynecomastia Galactorrhea Tuberoinfundibular pathway Prolactin Dopaminergic pathways Antipsychotic Dopamine Amenorrhoea Dopaminergic Hormone

14 Receptor (biochemistry)

15
16
Bottom Line:
17
Al l antipsychotics can cause extrapyramidal adverse effects such as akinesia, rigidity, stooped posture,
18
akathisia, and dystonia due to dopamine receptor blockage in the nigrostriatal pathway. The risk and severity
19 of these adverse effects are higher with typical neuroleptic agents.
20 Nigrostriatal pathway Akathisia Dopamine receptor Hypokinesia Dopamine Dystonia Antipsychotic Extrapyramidal system Extrapyramidal symptoms
Receptor (biochemistry)
21

6
lock
s
Suspend
0
End Block
Item: 21 of 73 ~ 1 • M k -<:J 1>- Jil ~· !:';-~
QIO: 1463 ..L ar Pre v ious Next Lab fli!ltues Not es Calcula t o r
& &
1
2 FA17 p 543.1

3 Typical antipsychotics Haloperidol, pimozide, triAuopera tine, Auphen;u ine, thiorida:tine, chlorpromaz;in e.

4 MECHANISM Block dopamine 0 2 receptor (f c 1P).


5 CliNICAl USE Schizophrenia (1° positi,·e symptoms), psychosis, bipolar disorder, delirium, Tourette syndrome,
6
Huntington disease, OCD.

7 POTENCY H igh potency: TriAnoperazine, fluphena1ine, llaloperidol (Try to Fly H igh)- neurologic side
effects (eg, extrapyramidal S) mploms [F:PS)).
8
Lo" potency: C hlorpromazine, Thioridazine (C heating l'h ieves are low)-anticholinergic,
9
antihistamine, a .-blockade effects.
10
ADVERSEEFFECTS Lipid soluble .... stored in body fat .... slow to be removed from body.
11
Endocrine: dopamine receptor antagonism .... hypcrprolactinemia .... galactorrhea,
12
oligomenorrhea, gynecomastia.
13 fetabolic: dyslipidemia, weight gain, hyperglycemia.
14 Antimuscarinic: dry mouth, constipation.
15 Antihistamine: sedation.
16 a.-blockade: orthostatic hypotension.
Cardiac: QT prolongation.
17
Ophthalmologic: C hlorpromazine- C orneal deposits; T hioridazine-re'J'inal deposi ts.
18
EPS-ADAPT:
19
Hours to days: Acute D ystonia (muscle spasm, stiffness, oculogyric crisis).
20
Days to months: \ kathisia (restlessness), Parkinsonism (bradykinesia).
21 .\lfonths to years: Tardive d) sl-inesia (orofacial chorea).

a
Lock
s
Suspend
8
End Bl ock
Item: 21 of 73 ~ 1 • M k -<:J 1>- Jil ~· !:';-~
QIO: 1463 ..L ar Pre v ious Next Lab fli!ltues Not es Calcula t o r

1 &
Ophthalmologic: C hlorpromazine-C orneal deposits; T hioridazine-reT inal deposits. &

2 EPS-ADAPT:
3 Hours to days: Acute D ystonia (muscle spasm , stiffness, oculogyric crisis).
4 Days to months: \ kathisia (restlessness), Parkinsonism (bradykinesia).
5 \llonths to yea rs: Tardive dyskinesia (orofacial chorea).
Treatment: benztropine (acute dystonia, tardive dyskinesia), benzodiazepines, ~-blockers
6
(akathisia).
7
Neuro leptic malignant syndro me (NMS)- \1alignant FEVER: \lyoglobinuria, Fe,·er,
8
E ncephalopathy, unstable Vitals, t E nz) mcs, muscle R igidity. Treatment: dantrolcne, 0 2 agonist
9
(eg, bromocriptine).
10
11
FA17 p 543.2
12
Atypical Aripiprazole, asenapinc, clozapinc, olanzapinc, quctiapinc, ilopcridone, paliperidone,
13
anti psychotics rispcridone, lurasidonc, ziprasidonc.
14
MECHANISM ot completely understood . Vlost arc D2
15
antagon ists; aripiprazolc is 0 2 partial agon ist.
16 Varied effects on 5-IIT2, dopam ine, and
17 a- and H 1-receptors.
18 CLINICAL USE Schizophrenia- both positive and negative Use clozapine for treatment-resistant
19 symptoms. Also used for bipolar disorder, schizophrenia or schizoaffective disorder and
20 OCD, anxiety disorder, depression, mania, for suicidality in schizophrenia.
Tourelte syndrome.
21

a
Lock
s
Suspend
8
End Bl ock
Item: 21 of 7 3 ~ 1 • M k -<:J 1>- Jil ~· !:';-~
QIO: 1463 ..L ar Pre v ious Next Lab fli!ltues Not es Calcula t o r

1
g, 1
2
3 FA17 p 543.2

4 Atypical Aripiprazole, asenapine, clozapinc, olanzapinc, quctiapinc, iloperidone, paliperidone,


5 antipsychotics risperidone, lurasidonc, ziprasidonc.
6 MECHANISM Not completely understood. \ lost arc 0 2
7 antagonists; aripiprazole is D2 partial agonist.
Varied effects on 5-I IT 2, dopamine, and
8
a- and H1-reccptors.
9
CliNICAL USE Schizophrenia-both posith·e and negative Use clozapine for treatment-resistant
10
symptoms. Also used for bipolar disorder, schizophrenia or schizoaffecti\'e disorder and
11 OCD, anxiety disorder, depression, mania, for su icidality in schizophrenia.
12 Tourette syndrome.
13 ADVERSE EFFECTS Al l- prolonged QT interval, fewer EPS and
14 antichol inergic side effects than typical
15 antipsychotics.
"-pines"- metabolic syndrome (weight gain, Olanzapine .... O besity
16
diabetes, hyperlipidemia).
17
C lozapine-agranulocytosis (monitor WBCs Must· watch bone marrow clo7cly with clo1.apinc.
18 frequently) and seizures (dose related).
19 Risperidone-hyperprolactinemia (amenorrhea,
20 galactorrhea, gynecomastia).
21

a
Lock
s
Suspend
8
End Bl ock
Item: 22 of 73 ~ 1 • M k -<:J 1>- Jil ~· !:';-~
QIO: 1489 ..L ar Pre v ious Next Lab fli!ltues Not es Calcula t o r

IAA]
A A

20 An unresponsive 30-year-old man is brought to the emergency department from the streets . Blood pressure
21 is 92/60 mm Hg, pu lse is 58/m in, temperature is 37.0°C (98.6°F), and respiratory rate is 6/min. His pupils
appear small and slugg ish. An emergency blood panel shows:
• 22
• 23 pH: 7 .30
• 24 Na +: 135 mEq/L
• 25
K+: 4.1 mEq/L
cl-: 105 mEq/L
• 26
HC0 3- : 22 mEq/L
• 27 BUN: 8 mg/dl
• 28 Creatinine: 0.8 mg/dl
Glucose: 70 mg/dl
• 29
o30
Which of the fol lowing is an antidote for the substance that the patient most likely ingested?
• 31
• 32 :
A. Flumazenil
• 33
o34 B. N-Acetylcysteine
o35 C. Na loxone
• 36
D. Penacillamine
• 37
E. Phentolamine
• 38
o39
• 40 •
a
Lock
s
Suspend
8
End Bl ock
Item: 22 of 73 ~ 1 • M k -<:J 1>- Jil ~· !:';-~
QIO: 1489 ..L ar Pre v ious Next Lab fli!ltues Not es Calcula t o r
A A

20 The correct answer is C. 79°/o chose this.


21
22
• 23
This patient is exhibiting classic signs of opioid toxicity: pinpoint pupils
(shown in the image), respi ratory depression, and coma. These effects arise
f rom the agonist activity of these agents at endogenous opioid 1-1-receptors in
the central nervous system. (Miosis is from stimulation of both 1-1- and K-
I
• 24 receptors.) Naloxone is used in the acute reversal of opioid toxicity, as it is a
• 25 pure opioid antagonist that competes with and displaces narcotics at the
opioid receptor. Naloxone is rapid acting and not as bioavailable, whereas
• 26
naltrexone (the longer word) is longer acting and is more bioavailable (PO).
• 27 Because of these properties, the former is more likely used in acute opioid
• 28 overdose, and the latter is used more in longer-term management .
• 29
Furthermore, naltrexone has shown some clinical uti lity in, and has been
prescribed for, the treatment of alcohol dependence. Image copyright © 2010 Ferrari
o30 . . . . et at· licensee BioMed Central
Lastly, regard ing the mterpretrat 1on of the blood gas, although a Pco 2 va lue 1s Ltd '
• 31
not given, it is safe to assume that w it h a respirato ry rate of 6, there is likely ·
• 32 C0 2 ret enti on, t hus indicati ng respirat ory acidosis . The slightly low HC03 -
• 33 Ievel would t hen suggest a concomitant (non-gap) metabolic acidosis as well .
o34 Naloxone Naltrexone Miosis Metabolic acidosis Hypoventilation Central nervous system Respiratory acidosis Opioid antagonist Opioid overdose Opioid

o35 Receptor antagonist Agonist Opioid receptor Major depressive disorder Depression (mood) Respiratory rate Endogeny (biology) Coma Nervous system
• 36 Alcohol dependence Toxicity Opioid peptide Bioavailability Alcohol Narcotic Alcoholic beverage Acidosis Metabolism Receptor (biochemistry)
• 37
A is not correct. 12°/o chose this.
• 38
Flumazenil is a competitive inhibitor of cent ra l nervous system benzodiazepine receptors, and it is used to
o39 reverse the sedat ive effects of benzodiazepines and general anesthesia . It does not reverse narcotic symptoms
• 40 • nr :lrY\noci ~

a
Lock
s
Suspend
8
End Bl ock
Item: 22 of 73 ~. I • M k <:] t> al ~· ~
QIO: 1489 .l. ar Previous Next lab 'lifllues Notes Calculator


20
A is not correct. 12% chose this.
21 Flumazenil is a competitive inhibitor of centra l nervous system benzodiazepine receptors, and it is used to
22 reverse the sedative effects of benzodiazepines and general anesthesia. It does not reverse narcotic symptoms
• 23 or amnesia .
Benzodiazepine Flumazenil Central nervous system Sedative Competitive inhibition General anaesthesia Anesthesia GABAA receptor Narcotic Amnesia
• 24
Nervous system Receptor (biochemistry)
• 25

• 26 B is not correct. 5°/o chose this .


• 27 N-Acetylcysteine is a mucolytic agent that acts as a sulfhydryl donor, substituting for the liver's usua l sulfhydryl
donor glutathione. It is used to prevent acetaminophen-induced liver inju ry . Th is patient is not displaying typical
• 28
signs and symptoms of acetaminophen toxicity, such as nausea, vomiting, and anorexia .
• 29 Glutathione Paracetamol Mucokinetics Thiol Nausea Vomiting Paracetamol toxicity Hepatotoxicity Anorexia (symptom) liver Toxicity Anorexia nervosa

o30
D is not correct. 2°/o chose this .
• 31
Penicil lamine can be used to treat heavy-metal poisoning caused by lead, mercury, and copper, and it may be
• 32 used in patients with Wilson disease to prevent copper accumu lation .
• 33 Penicillamine Wilson' s disease Toxic heavy metal Mercury (element) Copper

o34 E is not correct. 2°/o chose this.


o35 Phentolamine is an a-adrenerg ic receptor blocker that produces peripheral vasodilation. It is used to treat
• 36 hypertensive crisis associated with stimulant drug overdose (eg, cocaine, amphetamines, and ephedrine),
• 37 monoamine oxidase inhibitor/tyramine toxicity, or sudden withd rawa l of sympatholytic antihypertensive drugs
( eg, clonidine) . Th is patient is not hypertensive and wou ld not benefit from t reatment with phento lamine .
• 38
Clonidine Sympatholytic Ephedrine Phentolamine Vasodilation Hypertensive crisis Antihypertensive drug Hypertensive emergency Monoamine oxidase Cocaine
o39
Substituted amphetamine Stimulant Monoamine neurotransmitter Drug overdose Receptor (biochemistry)
• 40 •

6
lock Suspend
s 0
End Block
Item: 22 of 73 ~. I • M k <:] t> al ~· ~
QIO: 1489 .l. ar Previous Next lab 'lifllues Notes Calculator


20
donor glutathione. It is used to prevent acetaminophen-induced liver inju ry . Th is patient is not displaying typical
signs and symptoms of acetaminophen toxicity, such as nausea, vomit ing, and anorexia .
21 Glutathione Paracetamol Mucokinetics Thiol Nausea Vomiting Paracetamol toxicity Hepatotoxicity Anorexia (symptom) liver Toxicity Anorexia nervosa
22
D is not correct. 2°/o chose this .
• 23
Penicil lamine can be used to treat heavy-metal poisoning caused by lead, mercury, and copper, and it may be
• 24
used in patients with Wilson disease to prevent copper accumu lation .
• 25 Penicillamine Wilson' s disease Toxic heavy metal Mercury (element) Copper

• 26
E is not correct. 2°/o chose this .
• 27
Phentolamine is an a-adrenerg ic receptor blocker that produces peripheral vasodilation. It is used to treat
• 28 hypertensive crisis associated with stimulant drug overdose (eg, cocaine, amphetamines, and ephedrine),
• 29 monoamine oxidase inhibitor/tyramine toxicity, or sudden withd rawa l of sympatholytic antihypertensive drugs
(eg, clonidine) . Th is patient is not hypertensive and wou ld not benefit from t reatment with phento lamine.
o30
Clonidine Sympatholytic Ephedrine Phentolamine Vasodilation Hypertensive crisis Antihypertensive drug Hypertensive emergency Monoamine oxidase Cocaine
• 31
Substituted amphetamine Stimulant Monoamine neurotransmitter Drug overdose Receptor (biochemistry)
• 32

• 33

o34 Bottom Line:


o35 Classic signs of opioid toxicity include pinpoint pupils, respiratory depression, and coma caused by the agonist
• 36
activity of opioids at endogenous opioid j.J-receptors in the centra l nervous system. Naloxone is used in the
acute reversal of opioid toxicity, because it is a pure opioid antagonist that competes with narcotics, displacing
• 37
them at the opioid receptor.
• 38 Naloxone Hypoventilation Central nervous system Opioid Opioid antagonist Receptor antagonist Miosis Agonist Opioid receptor Major depressive disorder
Depression (mood) Opioid peptide Narcotic Endogeny (biology) Nervous system Coma Toxicity Antagonist Receptor (biochemistry)
o39
• 40 •

6
lock
s
Suspend
0
End Block
Item: 22 of 73 ~. I • M k <:] t> al ~· ~
QIO: 1489 .l. ar Previous Next lab 'lifllues Notes Calculator

• •
20
FA17 p 540.1
21
Psychoactive drug intoxication and withdrawal
22
DRUG ltiTOXICATION WITHDRAWAL
• 23
Depressants
• 24
onspecific: mood elevation, l anxiety, onspecific: anxiety, tremor, seizures,
• 25 . .
sedation, behavioral disinhibition, respiratory lllSOmnaa.
• 26 depression .
• 27 Alcohol Emotional labil ity, slurred speech, ataxia, Time from last drink:
• 28 coma, blackouts. Serum y-glutamyltransferase 3-36 hr: minor symptoms similar to other
• 29 (GGT)-sensitive indicator of alcohol usc. depressants
AST value is twice ALT value. 6- 48 hr: withdrawal seizures
o30
12-48 hr: alcoholic hallucinosis (usually visual)
• 31
48-96 hr: delirium tremens (DTs) in 5% of
• 32 cases
• 33 Treatment: benzodiazepines.
o34 Opioids Euphoria, respiratory and CNS depression, Sweating, dilated pupils, piloerection ("cold
o35 l gag reAex, pupillary constriction (pinpoint turkey"), fever, rhinorrhea, yawning, nausea,
• 36
pupils), seizures (overdose). Most common stomach cramps, diarrhea {"Au-like" symptoms).
cause of drug overdose death. Treatment: Treatment: long-term support, methadone,
• 37
naloxone. buprenorphine.
• 38
Barbiturates Low safety margin, marked respiratory Delirium, life-threatening cardiovascular
o39
depression. Treatment: symptom management collapse.
• • nn\ •
• 40 (

6
lock
s
Suspend
0
End Block
Item: 22 of 73 ~. I • M k <:] t> al ~· ~
QIO: 1489 .l. ar Previous Next lab 'lifllues Notes Calculator

• •
20
FA17 p 520.5
21
Opioid analgesics Morph ine, oxycodone, fentanyl, codeine, loperamide, methadone, meperidine, dextromethorphan,
22
diphenoxylate, pentazocine.
• 23
MECHANISM o
Act as agonists at opioid receptors (ll = ~-endorphin, = cnkephalin, K = dynorphin) to modulate
• 24 synaptic transmission- open K+ channels, close Ca 2+ channels -+ ! synaptic transmission. Inhibit
• 25 release of ACh, norepinephrine, 5-HT, glutamate, substance P.
• 26 CliNICAL USE Pain, cough suppression (dextromethorphan), diarrhea (loperamide, diphenoxylate), acute
• 27 pulmonary edema, maintenance programs for heroin addicts (methadone, bupre norphine +
• 28
naloxone).

• 29 ADVERSE EFFECTS lausea, vomiting, pruritus, addiction, respiratory depression, constipation, miosis (except
meperidine - mydriasis), additive C IS depression with other drugs. Tolerance docs not cb·clop
o30
to miosis and constipation. Toxicity treated with naloxone (opioid receptor antagonist) and relapse
• 31
prevention with naltrexone once detoxified .
• 32

• 33
FA17p239.1
o34
Specific toxicity TOXIN TREATMENT
o35
treatments Acetaminophen N-acetylcysteine (replenishes glutathione)
• 36
AChE inhibitors, organophosphates Atropine> pralidoxime
• 37
Antimuscarinic, anticholinergic agents Physos tigmine, control hyperthermia
• 38
Arsenic D imercaprol, succimer
o39
• 40 • Benzod iazepines Flumazenil •

6
lock
s
Suspend
0
End Block
Item: 22 of 73 ~ 1 • M k -<:J 1>- Jil ~· !:';-~
QIO: 1489 ..L ar Pre v ious Next Lab fli!ltues Not es Calcula t o r


20 FA17 p239.1
21
Specific toxicity TOXIN TREAIMENT
22 treatments Acetaminophen N-acetylcysteine (replenishes glutathione)
• 23
AChE inhibitors, organophosphates Atropine> pralidoxime
• 24
Antimuscarinic, anticholinergic agents Phrsostigmine, control hyperthermia
• 25
Arsenic Dimercaprol. succimer
• 26
Benzodiazepines Flumazenil
• 27
~-blockers Atropine, glucagon
• 28
Carbon monoxide 100% 0 2, hyperbaric 0 2
• 29

o30
Copper Penicillamine, trientine (Copper penny)

. 31 Cyanide Nitrile + thiosulfate, hydroxocobalami n


• 32 Digitalis (digoxin) Anti-dig Fab fragments
• 33 Heparin Protamine sulfate
o34 Iron Dcfe roxamine, deferasirox, deferiprone
o35 Lead ED'lA, dimercaprol, succimer, penicillami ne
• 36 l\lcrcury Dimercaprol, succimcr
• 37 1\ !ethanol, ethylene glycol (antifreeze) F'omepizole >ethanol, dialysis
• 38 l\ lethemoglobin 1\ lethvlene blue, vitamin C
'
o39 O piO ids 'aiO xO nc
• 40 •
a
Lock
s
Suspend
8
End Bl ock
Item: 23 of 73 ~ 1 • M k -<:J 1>- Jil ~· !:';-~
QIO: 4839 ..L ar Pre v ious Next Lab fli!ltues Not es Calcula t o r

IAA]
A A

20 A 50-year-old man with schizophrenia recently started using haloperidol as part of his t reatment regimen. He
21 mista ken ly took extra doses of his medication and was found to be confused, with rigidity in his arms and
legs as well as reddish-brown discoloration of his urine. Upon presentation to the emergency department his
22
tem peratu re is 38.9°C ( 102°F) and his creatine kinase level is 5000 U/ L.
• 23
• 24
Which of the following is most useful in treating this man's acute condition?
• 25
:
• 26
A. Benztropine
• 27
B. Bromocriptine
• 28
• 29 C. Chlorpromazine
o30 D. Cyproheptadine
• 31
E. Diphenhydramine
• 32
• 33
o34
o35
• 36
• 37
• 38
o39
• 40 •
a
Lock
s
Suspend
8
End Bl ock
Item: 23 of 73 ~. , . M k <:] t> al ~· ~
QIO: 4839 .l. ar Previous Next lab 'lifllues Notes Calculator


20 The correct answer is B. 49°/o chose this.
21 This patient is suffering from the rare condition of neuroleptic malignant synd rome (NMS), which is
22
character ized by fevers, muscle rigid ity, and myog lobinuria. This is most likely due to the decrease in dopamine
signal ing caused by his treatment with haloperidol, an antipsychotic agent that blocks dopamine receptors. NMS
23
is treated with dantrolene . Bromocriptine, a dopamine agon ist common ly used to treat Parkinson disease, can
• 24 also reverse this process .
• 25 Neuroleptic malignant syndrome Bromocriptine Dopamine agonist Haloperidol Dantrolene Dopamine Antipsychotic Hypertonia Agonist Parkinson' s disease Cancer

• 26 Muscle Receptor (biochemistry) Malignancy

• 27 A is not correct. 22% chose this.


• 28 Benztropine is an antimuscarinic drug that can be used to treat extrapyramidal side effects of antipsychotics,
• 29 including treatment of acute dyston ia. Acute dyston ia is character ized by involuntary, sustained muscle
o30
contractions in major muscle groups, resulting in symptoms such as torticol lis. These symptoms often lead to
abnorma l posturing of the patient. I n contrast to neuro leptic mal ignant syndrome, patients with acute dystonia
• 31
do not typically exhibit fever, elevated creatine kinase, myog lobinuria, or rigidity. Benztropine also improves
• 32 tremor and rigidity in patients with Pa r kinson disease and wou ld have no effect on dopamine signal ing .
Neuroleptic malignant syndrome Muscarinic antagonist Benzatropine Creatine kinase Dopamine Extrapyramidal symptoms Dystonia Antipsychotic Tremor
• 33
Torticollis Extrapyramidal system Parkinson' s disease Creatine Fever Side effect Muscle Abnormal posturing
o34

o35 C is not correct. 7°/o chose this.


• 36 Ch lorpromazine is an antipsychotic agent in the same class as ha loperidol. These two drugs are the
• 37 antipsychotic agents associated with the highest incidence of neu roleptic mal ignant syndrome .
Neuroleptic malignant syndrome Chlorpromazine Haloperidol Antipsychotic Malignancy Cancer
• 38

o39 D is not correct. 14% chose this.


• 40 • Cvoroheotad ine is a histamine and seroton in antaaonist used to treat serotonin svndrome if suooortive ca re and •
6
lock
s
Suspend
0
End Block
Item: 23 of 73 ~. , . M k <:] t> al ~· ~
QIO: 4839 .l. ar Previous Next lab 'lifllues Notes Calculator
I g I y p • I I I I p g g
20 Neuroleptic malignant syndrome Muscarinic antagonist Benzatropine Creatine kinase Dopamine Extrapyramidal symptoms Dystonia Antipsychotic Tremor

21 Torticollis Extrapyramidal system Parkinson' s disease Creatine Fever Side effect Muscle Abnormal posturing

22
C is not correct. 7°/o chose this.
23
Ch lorprom azi ne is an antipsychotic agent in the same class as ha loperidol. These t wo drugs are t he
• 24 ant ipsychotic agents associated with the highest incidence of neu roleptic mal ignant sy ndrom e .
• 25 Neuroleptic malignant syndrome Chlorpromazine Haloperidol Antipsychotic Malignancy Cancer

• 26 D is not correct. 14% chose this .


• 27 Cy proheptad ine is a histami ne and seroton in antagonist used to treat serotonin sy ndrom e if supportive care an d
• 28 other methods of treatment have failed .
Serotonin syndrome Cyproheptadine Serotonin Histamine Serotonin antagonist Receptor antagonist
• 29

o30 E is not correct. 8°/o chose this.


• 31 Diphen hydramine is an antihistamine used primarily to treat allergies and insom nia. It is not indicated for use in
NMS, t hough it can be used to treat acute dystonias related to antipsychotics .
• 32
Antihistamine Diphenhydramine Insomnia Antipsychotic Allergy
• 33

o34

o35 Bottom Line:


• 36 Neuroleptic mal ignant sy ndrom e is characterized by fevers, muscle r igidit y, and myog lobinuria. It can be
• 37
caused by antipsychotic med ications that block dopa mine receptors. Dantrolene or brom ocriptine can be used
as t reatm ent .
• 38 Neuroleptic malignant syndrome Bromocriptine Dantrolene Antipsychotic Dopamine Hypertonia Malignancy Cancer Muscle
o39
• 40 •

6
lock
s
Suspend
0
End Block
Item: 23 of 73 ~ 1 • M k -<:J 1>- Jil ~· !:';-~
QIO: 4839 ..L ar Pre v ious Next Lab fli!ltues Not es Calcula t o r
A A

20 FA17 p 543.1
21 Typical antipsychotics Haloperidol, pimozide, triAuopera tine, Auphen;u ine, thiorida:tine, chlorpromatin e.
22 MECHANISM Block dopamine 0 2 receptor (f c 1P).
23 CLINICAL USE Schizophrenia (1° positi,·e symptoms), psychosis, bipolar disorder, delirium, Tourctte syndrome,
• 24 Huntington disease, OCD.
• 25 POTENCY H igh potency: Trifluoperazine, Fluphena7ine, llaloperidol (Try to Fly Iligh)-neurologic side
• 26 effects (eg, extrapyramidal S) mptoms [ EPS]).
• 27 Low potency: C hlorproma7ine, T h iorida7ine (C heating Th ieves are low)-anticholinergic,
antihistamine, a rblockade effects.
• 28
• 29 ADVERSEEFFECTS Lipid soluble ..... stored in body fat ..... slow to be removed from body.

o30 Endocrine: dopamine receptor antagonism ..... hypcrp rolactinemia ..... galactorrhea,
• 31 oligomenorrhea, gynecomast i;1.
l\letabolic: dyslipiclemia, weight gain, hyperglycemia .
• 32
Anti muscarinic: dry mouth, constipation .
• 33
Antihistami ne: sedation.
o34 a 1-blockade: orthostatic hypotension.
o35 C;,nd iac: QT prolongation.
• 36 Ophthalmologic: C hlorpromazine- C orneal deposits; T hioridazine-reT inal deposi ts.
• 37 EPS- ADAPT:
• 38 Hours to days: Acute D ystonia (muscle spasm, stiffness, oculogyric crisis).
Days to months: Akathisia (restlessness), Parkinsonism (bradykinesia).
o39
Months to years: Tardive dyskinesia (orofacial chorea).
• 40 •
a
Lock
s
Suspend
8
End Bl ock
Item: 23 of 73 ~ 1 • M k -<:J 1>- Jil ~· !:';-~
QIO: 4839 ..L ar Pre v ious Next Lab fli!ltues Not es Calcula t o r
A A

20
FA17 p 520.2
21 Dantrolene
22 MECHANISM Pre,·ents release of Ca 2~ from the sarcoplasmic reticulum of skeletal muscle by binding to the
23 ryanodine receptor.
• 24 CliNICAL USE lalignant hyperthermia and neuroleptic malignant syndrome (a toxicity of antipsychotic drugs).
• 25
• 26
FA17 p 530.1
• 27
Schizophrenia Chronic mental disorder with periods of Frequent cannabis use is associated\\ ith
• 28
ps}chosis, disturbed bchm ior and thought, ps}chosis/schizophrenia in teens.
• 29 and decline in functioning lasting> 6 Lifetime pre,·alence- 1.5% (males= females,
o30 months. Associated with f dopaminergic fr ican Americans= Caucasians). Presents
• 31 activity, ! dendritic branching. earlier in men (late teens to early 20s \'Slate
• 32 Diagnosis requires at least 2 of the following, 20s to early 30s in women). Patients arc at f
and at least I of these should include 1-3 risk for suicide.
• 33
(first 4 are "positive symptoms"): Vcntricnlomcgaly on brain imaging.
o34 Treatment: atypical antipsychotics (eg,
I. Delusions
o35 2. llallucinations- often auditory risperidone) are first line.
• 36 3. Disorganized speech egative symptoms often persist after treatment ,
• 37 4. Disorganized or catatonic bcha\'ior despite resolution of positive symptoms.
• 38 5. Negati,·e symptoms (affccti,·c Aallcning,
ami ilion, anhedonia, asociality, alogia)
o39
• 40 • Brief psychotic disorder- lasting< I month ,

a
Lock
s
Suspend
8
End Bl ock
Item: 24 of 73 ~. I • M k <:] t> al ~· ~
QIO: 3480 .l. ar Previous Next lab 'lifllues Notes Calculator


20 A 27-year-old man is brought to the emergency department by ambu lance . Paramedics report that he was
21 found sitting on the sidewa lk speaking as though engaged in a heated argument, but nobody else was
around . They say that the patient appeared to be in distress and that he was quite disheveled . The man is
22
eva luated by a psychiatrist, admitted to the hospita l, and started on a med ication to t reat his symptoms. Two days
23 later a med ical student notices that the patient has pa inful spasms in his neck muscles.
• 24

• 25 Wh ich of the fol lowing med ications can be used to the treat the adverse effect the patient is exper iencing?
• 26
:
• 27 A. Benztropine
• 28
B. Diazepam
• 29
C. Fluphenazine
o30
• 31 D . Methadone
• 32 E. Prochlorperazine
• 33

o34

o35
• 36
• 37

• 38

o39
• 40 •

6
lock
s
Suspend
0
End Block
Item: 24 of 73 ~. I • M k <:] t> al ~· ~
QIO: 3480 .l. ar Previous Next lab 'lifllues Notes Calculator


20 The correct answer is A. 52°/o chose this.
21 The patient has classic signs of schizophrenia and was likely given ha loperidol, a typical antipsychotic agent that
acts by blocking dopamine receptors. Ha loperidol has a high affinity for the Drdopaminergic receptor. The
22
patient is experiencing an acute dyston ic reaction soon after receiving the med ication. The painful muscle spasm
23 of the neck is known as torticollis. This acute extrapyramidal adverse effect is the result of unopposed
24 chol inergic activity in the central nervous system fo llowing blockade of dopaminergic transmission. The
• 25
treatment for this adverse effect is init iation of an anticho linerg ic agent such as benztropine .
Typical antipsychotic Anticholinergic Haloperidol Benzatropine Schizophrenia Central nervous system Dopamine Antipsychotic Cholinergic Extrapyramidal system
• 26
Extrapyramidal symptoms Dopaminergic Torticollis Spasm Adverse effect Nervous system Dopamine receptor Receptor (biochemistry) Affinity (pharmacology)
• 27
Pharmaceutical drug
• 28
• 29 B is not correct. 20% chose this .
o30 Diazepam can be used as an hypnotic, a sedative, an anticonvulsant, and a muscle relaxant. As a muscle
relaxant, diazepam is used to treat chorea, an invo luntary abnormal movement disorder or dyskinesia that is a
• 31
hallmark of Huntington's disease .
• 32 Huntington' s disease Anticonvulsant Diazepam Muscle relaxant Sedative Chorea Hypnotic Dyskinesia Movement disorder Hypnosis Muscle

• 33
C is not correct. 13% chose this.
o3 4
Fluphenazine, like haloper ido l, can induce potent Drdopaminergic receptor blockade. It is a high-potency
o35 typical antipsychotic that is sometimes used as an alternative to ha loperidol fo r patients suffe ring from
• 36 schizoph renia or bipolar diso rder. Administration of f luphenazine wi ll like ly exacerbate this patient's symptoms
• 37
rather than al leviate them .
Fluphenazine Typical antipsychotic Haloperidol Bipolar disorder Antipsychotic Schizophrenia Receptor (biochemistry)
• 38

o39
D is not correct. 6°/o chose this.
• 40 • Methadone is a long-acting oral opiate used for management of heroin withdrawa l. The re is no evidence this •
6
lock
s
Suspend
0
End Block
Item: 24 of 73 ~. I • M k <:] t> al ~· ~
QIO: 3480 .l. ar Previous Next lab 'lifllues Notes Calculator

• Huntington' s disease Anticonvulsant Diazepam Muscle relaxant Sedative Chorea Hypnotic Dyskinesia Movement disorder Hypnosis Muscle
20

21 C is not correct. 13% chose this.


22 Fluphenazine, like haloper idol, can induce potent Drdopaminergic receptor blockade. It is a high-potency
typical antipsychotic that is sometimes used as an alternative to haloperidol for patients suffering from
23
schizoph renia or bipolar disorder. Administration of fluphenazine wi ll likely exacerbate this patient's symptoms
24 rather than al leviate them.
• 25 Fluphenazine Typical antipsychotic Haloperidol Bipolar disorder Antipsychotic Schizophrenia Receptor (biochemistry)

• 26
D is not correct. 6°/o chose this .
• 27
Methadone is a long-acting oral opiate used for management of heroin withdrawal. There is no evidence this
• 28 patient has taken heroin, and methadone is not indicated .
Methadone Opiate Heroin Opioid use disorder
• 29

o30 E is not correct. 9°/o chose this.


• 31 Prochlorperazine is a typical antipsychotic agent with potent antidopaminergic effects. It can also be used to
• 32 treat nausea because of its weak anticholinerg ic and antihistaminic effects. In this case it would be of no benefit
for the patient, and could make his symptoms worse .
• 33
Typical antipsychotic Anticholinergic Prochlorperazine Antipsychotic Antihistamine Dopamine antagonist Nausea
o3 4

o35
• 36
Bottom Line:
• 37 One of the adverse effects of haloper idol treatment is acute dystonia. The treatment of choice for this adverse
effect is an anticholinergic such as benztropine .
• 38
Anticholinergic Haloperidol Benzatropine Dystonia Adverse effect
o39
• 40 •

6
lock
s
Suspend
0
End Block
Item: 24 of 73 ~ 1 • M k -<:J 1>- Jil ~· !:';-~
QIO: 3480 ..L ar Pre v ious Next Lab fli!ltues Not es Calcula t o r
A A
FA17 p 543.1
20
Typical antipsychotics Haloperidol, pimozide, triAuopera:~ine, Auphen :t:~ine, thioridazine, chlorpromazine.
21
MECHANISM Block dopamine 0 2 receptor (f c M P).
22
CLINICAL USE Schizophrenia {1° positi,·e symptoms), psychosi~. bipolar disorder, delirium, Tourette syndrome,
23
Huntington disease, OCD.
24
POTENCY Il igh potency: Trifluoperazine, fluphena7ine, llaloperidol (Try to Fl)' Iligh)- neurologic side
• 25
effects (eg, extrapyramidal S) mptoms [ EPS]).
• 26 Low potencr: Chlorproma7ine, T h iorida7ine (C heating Thieves are low)-anticholinergic,
• 27 antihistamine, a 1-blockade effects.
• 28 ADVERSE EFFECTS Lipid soluble .... stored in body fat .... slow to be removed from body.
• 29
Endocrine: dopamine receptor antagonism .... hypcrprolactinemia .... galactorrhea,
o30 oligomenorrhea, gynccomast i<L
. 31 l\letabolic: dyslipidemia, weight gain, hyperglycemia .
• 32 Antimuscarinic: dry mouth, constipation .
Antihistamine: sedation .
• 33
a 1-blockade: orthostatic hypotension.
o34
Cardiac: QT prolongation.
o35 Ophthalmologic: C hlorpromazine- C orneal deposits; T hioridazine-reT inal deposi ts.
• 36
EPS- ADAPT:
• 37 Hours to days: Acute D ystonia (muscle spasm, stiffness, oculogyric crisis).
• 38 Days to months: \ kathisia (restlessness), Parkinsonism (bradykinesia).
o39 Months to years: Tardive dyskinesia (orofacial chorea).
• 40 • Treatment: benztropine (acu te d\stonia, tardive chskinesia), benzodiazepines, 13-blockers

a
Lock
s
Suspend
8
End Bl ock
Item: 24 of 7 3 ~ 1 • M a rk -<:J 1>- Jil ~· !:';-~
QIO: 3480 ..L Prev ious Next Lab fli!ltues Not es Cal culat o r


20 FA17 p530.1

21 Schizophrenia Chronic mental disorder with periods of Frequent cannabis use is associated "ith
22 psychosis, disturbed behavior and thought, psychosis/schizophrenia in teens.
23
and decline in functioning lasting> 6 Lifetime pre\'alence-l.)% (males= females,
months. Associated with t dopamincrgic frican Americans= Caucasians). Presents
24
acti,·ity, l dendritic branching. earlier in men (late teens to early 20s ,.s late
• 25
Diagnosis requires at least 2 of the following, 20s to early 30s in women). Patients arc at t
• 26 risk for suicide.
and at least I of these should include 1-3
• 27 (first 4 are "positi\'e symptoms"): Vcntriculomcgaly on brain imaging.
• 28 I. Delusions Treatment: atypical antipsychotics (cg,
2. Hallucinations-often auditory risperidone) are first line.
• 29
3. Disorgan ized speech 'egative symptoms often persist after treatment,
o30
4. Disorganized or catatonic bcha, ior despite resolution of positive symptoms.
. 31
5. egative symptoms (affective Aattcn ing,
• 32 avolition, anhedonia, asocia lity, alogia)
• 33
Brief psychotic disorder- lasting< I month,
o34 usually stress related.
o35 Schizophreniform disorder- hasting 1-6
• 36 months.
• 37 Schizoaffective disorder- \tlccts criteria for
• 38 schizophrenia in addition to major mood
o39 disorder (major depressi\'e or bipolar). To
differentiate from a major mood disorder
• 40 •
a
Lock Suspend
s 8
End Bl ock
Item: 25 of 73 ~ 1 • M k -<:J 1>- Jil ~· !:';-~
QIO: 4685 ..L ar Pre v ious Next Lab fli!ltues Not es Calcula t o r

IAA]
A A

20 A 22-year-old man with schizophrenia is being treated as an inpatient with a first-generation antipsychotic
21 medication . Several hours after his initial dose, he develops involuntary muscle spasms, particular ly in his
facial muscles.
22
23
To which class of drugs does the indicated treatment for this patient's adverse effects belong?
24
0 25 :
A . 13-Biockers
• 26
• 27 B. Anticholinergic agents
• 28 C. Antiepileptic drugs
• 29
D. Atypical antipsychotic agents
o30
E. Dopami ne agon ists
• 31
• 32 F. Mood stabilizers
0 33
0 34
0 35
• 36
• 37
• 38
o39
• 40 •
a
Lock
s
Suspend
8
End Bl ock
Item: 25 of 73 ~. , . M k <:] t> al ~· ~
QIO: 4685 .l. ar Previous Next lab 'lifllues Notes Calculator


20 The correct answer is B. 41 °/o chose this.
21 This pat ient has acute dyston ia, wh ich is an ext rapyramidal adverse effect of antipsychotics, usual ly occurring
22 after 4 hours of therapy; it is easi ly t reat able. Benztropine, an anticholinergic agent, is the first-line t reatment
for ant ipsychotic-i nduced dyston ia and for most other extrapyramidal symptoms associated with antipsychotic
23
medications. Antichol inergics may be given prophylactical ly wit h the initia l administration of antipsychotics t o
24 prevent the onset of acute dyston ia. Acut e dyst on ia is usual ly most pronounced in the facial muscles and
25 involves involuntary, susta ined, and repetitive muscle spasms, which may man ifest as torticol lis or ocu logyric
• 26
crisis. Other extrapyram idal adverse effects of antipsychotic agents include akathisia (after 4 days),
bradykinesia (after 4 weeks), and ta rdive dyskinesia (after 4 months) .
• 27 Anticholinergic Tardive dyskinesia Akathisia Benzatropine Bradykinesia Extrapyramidal symptoms Oculogyric crisis Dystonia Antipsychotic Extrapyramidal system
• 28 Torticollis Dyskinesia Adverse effect Spasm Muscle Preventive healthcare
• 29
A is not correct. 3°/o chose this.
o30
~ -Bl ocke rs are used to t reat extrapyram idal symptoms, particu lar ly in pat ients with akat hisia. They will not
• 31
remedy the pat ient's acute dystonic symptoms .
• 32 Akathisia Extrapyramidal symptoms Extrapyramidal system Dystonia

• 33
C is not correct. 10% chose this.
o3 4
Antiepi leptics are used to t reat seizure disorders . Acute dyston ia is not related to seizure activity; therefore,
o35 antiepilept ic therapy will not remedy t he patient 's acute dyst onic symptoms .
• 36 Dystonia Anticonvulsant Epilepsy Epileptic seizure

• 37 D is not correct. 15% chose this .


• 38 Atypical antipsychotics have a lower associated incidence of ext rapyramidal adverse effect s. However, switching
o39 antipsychotics will not remedy t he patient 's acut e dyst on ic symptoms .
Antipsychotic Extrapyramidal system Extrapyramidal symptoms Atypical antipsychotic Dystonia
• 40 •

6
lock
s
Suspend
0
End Block
Item: 25 of 73 ~. , . M k <:] t> al ~· ~
QIO: 4685 .l. ar Previous Next lab 'lifllues Notes Calculator
r r 1 r r

20
D is not correct. 15% chose this.
21 Atypical antipsychotics have a lower associated incidence of extrapyramida l adverse effects. However, switching
22 antipsychotics will not remedy the patient's acute dyston ic symptoms.
Antipsychotic Extrapyramidal system Extrapyramidal symptoms Atypical antipsychotic Dystonia
23
24 E is not correct. 23% chose this.
25 Dopamine agonists, like bromocr iptine, are not used in the t reatment of antipsychotic-induced extrapyramida l
• 26 symptoms . Bromocriptine is used to treat prolactin-secreting pituitary adenomas or neu roleptic ma lignant
synd rome .
• 27
Neuroleptic malignant syndrome Bromocriptine Dopamine Extrapyramidal symptoms Dopamine agonist Pituitary gland Antipsychotic Pituitary adenoma Malignancy
• 28
Agonist Adenoma
• 29
F is not correct. 8°/o chose this.
o30
Mood stabilizers are often concurrently administered to psych iatr ic patients with schizoaffective disorde rs .
• 31
However, they will not remedy the patient's acute dystonic symptoms .
• 32 Schizoaffective disorder Mood stabilizer Dystonia

• 33

o34
Bottom Line:
o35
• 36
Typica l antipsychotic med ications cause extrapyramidal effects, such as dystonia, akath isia, bradykinesia, and,
eventually, tardivie dyskinesia. Anticholinergics, such as benztropine, a musca rinic antagonist, relieve the
• 37
symptoms of excess choline rgic activity.
• 38 Typical antipsychotic Akathisia Benzatropine Bradykinesia Antipsychotic Muscarinic antagonist Dystonia Cholinergic Anticholinergic Receptor antagonist
Muscarinic acetylcholine receptor Dyskinesia Extrapyramidal symptoms Extrapyramidal system
o39
• 40 •

6
lock
s
Suspend
0
End Block
Item: 25 of 73 ~ 1 • M k -<:J 1>- Jil ~· !:';-~
QIO: 4685 ..L ar Pre v ious Next Lab fli!ltues Not es Calcula t o r
A A

20 FA17 p 543.1

21 Typical antipsychotics Haloperidol, pimozide, triAuopera /inc, Auphena:rine, thioridazine, chlorpromazine.

22 MECHANISM Block dopamine 0 2 receptor (f c 1P).


23 CliNICAl USE Schizophrenia (1° positi,·c symptoms), psychosis, bipolar disorder, delirium, Tourcttc syndrome,
24
Huntington disease, OCD.

25 POTENCY H igh potency: TriAnoperazine, flnphena1ine, llaloperidol (Try to Fly H igh)- neurologic side
effects (eg, extrapyramidal S) mploms [F.PS]).
• 26
Lo'' potency: Chlorpromazine, Thioridazine (Cheating l'hieves are low)-anticholinergic,
• 27
antihistamine, a 1-blockade effects.
• 28
ADVERSEEFFECTS Lipid soluble .... stored in body fat .... slow to be removed from body.
• 29
Endocrine: dopamine receptor antagonism .... hyperprolaetinemia .... galactorrhea,
o30
oligomenorrhea, gynecomastia .
. 31
fetabolic: dyslipidemia, weight gain, hypcrglyccm ia.
• 32 Antimuscarinic: dry mouth, constipation .
• 33 Antihistamine: sedation .
o34 a.-blockade: orthostatic hypotension.
Cardiac: QT prolongation.
o35
Ophthalmologic: C hlorpromazine-C orneal deposits; T hioridazine-reT inal deposi ts.
• 36
EPS-ADAPT:
• 37
Hours to days: Acute D ystonia (muscle spasm, stiffness, oculogyric crisis).
• 38
Days to months: Akathisia (res tlessness), Parkinsonism (bradykinesia).
o39 .\tfonths to years: Tardive d)slinesia (orofacia l chorea).
• 40 •
a
Lock
s
Suspend
8
End Bl ock
Item: 25 of 73 ~ 1 • M k -<:J 1>- Jil ~· !:';-~
QIO: 4685 ..L ar Pre v ious Next Lab fli!ltues Not es Calcula t o r
A A

20 POTENCY High potency: TriAuoperazine, Fluphena7ine, llaloperidol (Try to Fly High}- neurologic side
effects (eg, extrapyramidal S) mptoms [EPS]).
21
Lo" potency: C h lorpromazine, Thioridazine (C heat ing Thieves are low)-anticholinergic,
22
antihistam ine, a 1-blockade effects.
23
ADVERSHFFECTS Lipid soluble - stored in body fat - slow to be removed from body.
24
Endocrine: dopamine receptor antagonism - hypcrprolactinemia - galactorrhea,
25
oligomenorrhea, gp1ecomastia.
• 26
~ letabolic: dyslipidemia, weight gain, hyperglycemia.
• 27 Antimuscarinic: dry mouth, constipation.
• 28 Antihistamine: sedation .
• 29 a 1-blockade: orthostatic hypotension .
Cardiac: QT prolongation.
o30
Ophthalmologic: C hlorpromazine-C orneal deposits; T hioridazine-reT inal deposits.
• 31
EPS-ADAPT:
• 32
Hours to days: Acute D ystonia (muscle spasm , stiffness, oculogyric crisis).
• 33
Days to months: Akathisia (restlessness), Parkinsonism (bradykinesia).
o34 Months to years: Tardive dyskinesia (orofacia l chorea).
o35 Treatment: benztropine (acute d)•Stonia, tardive dyskinesia}, benzodiazepines, ~-blockers
• 36 (akathisia).
• 37 Neuroleptic malignant syndrome (NMS)- \1a lignant FEVER: ~1roglobinuria, Fever,
• 38 E ncephalopathy, unstable Vitals, t Enzymes, muscle Rigidity. Treatment: dantrolene, 0 2 agonist
o39 (eg, bromocriptine).
• 40 •
a
Lock
s
Suspend
8
End Bl ock
Item: 25 of 73 ~ 1 • M k -<:J 1>- Jil ~· !:';-~
QIO: 4685 ..L ar Pre v ious Next Lab fli!ltues Not es Calcula t o r
: , - • • - 4

20
21
FA17 p233.1
22
Muscarinic antagonists
23 DRUGS ORGAN SYSTEMS APPLICATIONS
24 Atropine, Eve Produce mydriasis and cycloplegia.

25 homatropine,
• 26 tropicamide
• 27 Benztropine, CK Parkinson disease ("park my Bcn7").
trihexyphenidyl Acute dystonia.
• 28
• 29 Glycopyrrolate Cl, respiratory Parenteral: preoperative use to reduce airwar
secretions.
o30
Oral: drooling, peptic ulcer.
. 31
Hyoscyamine, Cl Antispasmodics for irritable bowel syndrome.
• 32
dicyclomine
• 33
lpratropium, Respiratory COPD, asthma ("' pray I can breathe soon!").
o34 tiotropium
o35
Oxybutynin, Genitourinary Reduce bladder spasms and urge urinary
• 36 solifenacin, incontinence (o\·eractive bladder).
• 37 tolterodine
• 38 Scopolamine CNS lotion sickness.
o39
• 40 •
a
Lock
s
Suspend
8
End Bl ock
Item: 26 of 73 ~. I • M k <:] t> al ~· ~
QIO: 1470 .l. ar Previous Next lab 'lifllues Notes Calculator


20 A 22-year-old ma le comes to the clinic with his mother. She says that he was a poor student in high schoo l,
21 but played on the football team and graduated at age 18. He has become increasingly withdrawn since his
graduation. He quit his part-t ime job, stopped attend ing his col lege classes, and no longer sees his fr iends.
22
He now spends most of his time alone in his room . Recently, he has become convinced that the government can
23 hear all the thoughts in his head and he sometimes hears voices speaking back to him. The patient is started on a
24 drug that acts as a competit ive inhibitor at dopamine receptors in the central nervous system .
25
• 26 With th is med ication, wh ich symptom of the patient's schizoph ren ia wou ld likely show the most benefit'
• 27 :
• 28 A. Auditory ha llucinations
• 29 B. Cogn it ive impa irment
o30
C. Flattened affect
• 31

• 32
D . Poor grooming and appetite
• 33 E. Socia l withd rawa l
o34

o35
• 36
• 37

• 38

o39
• 40 •

6
lock
s
Suspend
0
End Block
Item: 26 of 73 ~. I • M k <:] t> al ~· ~
QIO: 1470 .l. ar Previous Next lab 'lifllues Notes Calculator


20
The correct answer is A. 84°/o chose this.
21 Psychotic disorders ( eg, schizoph renia) are thought to arise from an imbalance in central dopaminergic
22 neurotransmission . The hypothesis that psychotic disorders result from an excess of dopamine in the centra l
23 nervous system led to the successful use of dopamine receptor-blocking agents (neuroleptics) in the t reatment
of psychosis. Traditional antipsychotics such as haloperidol that block central dopaminergic receptors effectively
24
treat positive symptoms, which include disorders of perception (illusions and hal lucinations), distu r bed thought
25 content (delusions and idea of reference), agitation, and talkativeness .
26 Haloperidol Schizophrenia Central nervous system Dopamine Antipsychotic Neurotransmission Dopaminergic Psychosis Hallucination Nervous system

• 27 Positive symptoms

• 28 B is not correct. 4°/o chose this .


• 29 Cognitive impairment generally precedes the positive symptoms of schizophren ia. Symptoms include deficits in
o30 memory, concentration, and processing speed. There is some evidence that antipsychotics can improve
• 31 cognitive symptoms if given early in the disease course . However, the t reatment of the cognitive impai r ment of
schizoph renia is still poorly understood, although improvement of positive symptoms with dopamine receptor
• 32
antagon ists is wel l establ ished. As a result, the symptom most likely to improve with the medication described
• 33 in this question is disturbed thought content .
o34 Dopamine receptor Dopamine Schizophrenia Antipsychotic Cognitive deficit Dopamine antagonist Symptom Positive symptoms Pharmaceutical drug

o35 C is not correct. 7°/o chose this.


• 36 Nontraditiona l or "atypical" antipsychotic medications, which appear to modu late central serotonergic activity
• 37 and dopamine receptors, have long been used to treat both positive and negative symptoms of schizoph renia .
• 38
Negative symptoms include flat affect, amotivation, poverty of speech, and disinterest in forming relationships .
Although atypical antipsychotics are generally thought to be more effective than typicals at treating negative
o39 symptoms, this is an area of ongoing research and debate, with some recent studies suggesting that both
• 40 • rl:!\eeQe nf 1"'\""\c~ir:~~ti l""tne =a ...c. Ql"111=a l l\/ i ncffQ,...ti\/Q =at il"'\""\nr""l"\\/ i nl"1 nc.t'"1=a+'iHc. e\/n"'\nti'\I"'Y'\e

6
lock
s
Suspend
0
End Block
Item: 26 of 73 ~. I • M k <:] t> al ~· ~
QIO: 1470 .l. ar Previous Next lab 'lifllues Notes Calculator

20
Nontraditiona l or "atypica l" antipsychotic medications, wh ich appear to modu late central se rotonergic activity
and dopamine receptors, have long been used to treat both posit ive and negative symptoms of schizoph renia .
21 Negative symptoms include flat affect, amotivation, poverty of speech, and disinterest in fo rming relationships.
22 Although atypical antipsychotics are generally thought to be more effective than typica ls at treating negative
23 symptoms, this is an area of ongoing research and debate, with some recent studies suggesting that both
classes of medications are equally ineffective at improving negative symptoms.
24
Schizophrenia Dopamine Serotonin Antipsychotic Alogia Serotonergic Atypical antipsychotic Receptor (biochemistry) Dopamine receptor Reduced affect display
25
Positive and Negative Syndrome Scale Negative symptoms
26
• 27
D is not correct. 2°/o chose this .
Poor grooming and appetite are negative symptoms and would be best treated with atypical antipyschotic
• 28
agents .
• 29 Child grooming Schizophrenia Social grooming

o30
E is not correct. 3°/o chose this .
• 31
Socia l withd rawa l is a negative symptom and would be best treated with atypical antipyschotic agents .
• 32 Schizophrenia Negative symptom Symptom Solitude
• 33

o34
Bottom Line:
o35
• 36
Typica l antipsychotics, wh ich potently block centra l nervous system dopamine receptors, are more effective at
treating positive symptoms. Atypical agents, wh ich block dopamine and serotonin receptors, are better at
• 37
alleviating negative symptoms .
• 38 Central nervous system Serotonin Dopamine Antipsychotic Typical antipsychotic Nervous system Schizophrenia Receptor (biochemistry) Dopamine receptor
Negative symptoms
o39
• 40

6
lock
s
Suspend
0
End Block
Item: 26 of 73 ~ 1 • M k -<:J 1>- Jil ~· !:';-~
QIO: 1470 ..L ar Pre v ious Next Lab fli!ltues Not es Calcula t o r
A A

20
FA17 p 543.1
21 Typical antipsychotics Haloperidol, pimozide, triAuopera tine, Auphen:uine, thioridazine, chlorproma;rine.
22
MECHANISM Block dopamine 0 2 receptor (t eA 1P).
23
CLINICAL USE Schizophrenia (1° positi,·c symptoms), psychosis, bipolar disorder, delirium, Tourctte syndrome,
24 Huntington disease, OCD.
25
POTENCY H igh potency: TriAuoperazine, rtuphenazine, llaloperidol (Try to Fly Iligh)-neurologic side
26 effects (eg, extrapyramidal S) mptoms [ EPS)l.
. 27 Low potency: C h lorproma7ine, Thiorida1ine (C heating Thie\'es are low)-anticholinergic,
. 28 antihistamine, a 1-blockade effects.
. 29 ADVERSEEFFECTS Lipid soluble .... stored in body fat .... slow to be removed from body.
o30 Endocrine: dopamine receptor antagonism .... hyperp rolactinemia .... galactorrhea,
. 31 oligomenorrhea, gynceomas t i<L
• 32 Metabolic: dyslipidemia, weight gain, hyperglycemia .
Anti muscarinic: dry mo uth, constipation .
• 33
Antihistamine: sedation.
o34 a 1-blockade: orthostatic hypotension.
o35 Cardiac: QT prolongation.
. 36 Ophthalmologic: C hlorpromazine- C orneal deposits; T hioridazine-reT inal deposi ts.
. 37 EPS- ADAPT:
. 38 Hours to days: Acute D ystonia (muscle spasm, stiffness, oculogyric crisis).
o39 Days to months: \ kathisia (restlessness), Parkinsonism (bradykinesia).
• Months to years: Tardive dyskinesia (orofacial chorea).
• 40
a
Lock
s
Suspend
8
End Bl ock
Item: 26 of 73 ~ 1 • M k -<:J 1>- Jil ~· !:';-~
QIO: 1470 ..L ar Pre v ious Next Lab fli!ltues Not es Calcula t o r
A A

20 FA17 p 543.2

21 Atypical Aripiprazole, asenapine, clozapinc, olanzapinc, quetiapinc, ilopcridone, palipcridonc,


22
antipsychotics risperidone, lurasidone, ziprasidonc.
23 MECHANISM 1otcompletely understood. \lost arc D2
antagonists; aripiprazolc is D2 partial agonist.
24
Varied effects on 5-I IT 2, dopamine, and
25
a- and H1-receptors.
26
CLINICAL USE Schizophrenia-both posithe and negati\'e Use clozapine for treatment-resistant
• 27 srmptoms. Also used for bipolar disorder, schizophrenia or schizoaffecti\'e disorder and
• 28 OCD, anxiety disorder, depression, mania, for suicidality in schizophrenia.
• 29 Tourette syndrome.
o30 ADVERSE EFFECTS Al l-prolonged QT inter\'al, fewer EPS and
. 31 antichol inergic side effects than typical
antipsychotics.
• 32
"-pines" - metabolic syndrome (weight gain, Olanzapine .... O besity
• 33
diabetes, hyperlipidem ia).
o34 C lozapinc-agranulocytosis (monitor WBCs Ivlust· watch bone marrow clozcly with ci<JJ:apine.
o35 frequen tly) and seizures (dose related).
• 36 Risperidone-hyperprolactinemia (amenorrhea,
• 37
galactorrhea, gynecomastia).
• 38
FA17 p 530.1
o39
• 40 • Schizophrenia Chronic mental disorder with periods of Frequent cannabis use is associated'' ith
a
Lock
s
Suspend
8
End Bl ock
Item: 26 of 73 ~ 1 • Ma rk -<:J 1>- Jil ~· !:';-~
QIO: 1470 ..L Pre v ious Next Lab fli!ltues Not es Calcula t o r


20 FA17 p530.1
21
Schizophrenia Chronic mental disorder with periods of Frequent cannabis use is associated'' ith
22 psychosis, disturbed beha' ior and I hough!, ps}chosis/schizophrenia in teens.
23 and decline in functioning lasting> 6 Lifetime pre,·alence-1.5% (males= females,
24 months. Associated with t dopaminergic African Americans= Caucasians). Presents
acti,·ity, l dendritic branching. earlier in men (late teens to early 20s ' s late
25
Diagnosis requires at least 2 of the following, 20s to early 30s in women). Patients are at t
26
and at least I of these should include 1-3 risk for suicide.
• 27 Vcntriculomegaly on brain imaging.
(first-+ are "positive symptoms"):
• 28
I. Delusions Treatment: atypical antipsychotics (eg,
• 29 2. llallucinations-often auditory risperidone) are first line.
3. Disorganized speech Negative symptoms often persist after treatment,
o30
4. Disorgan ized or catatonic behavior despite resolution of positi,·e symptoms.
. 31
5. legati,·c symptoms (affect ivc Aallen ing,
• 32
avolition, anhedonia, asociality, alogia)
• 33
Brief psychotic disorder- lasting< l month,
o34
usually stress related.
o35
Schizophreniform disorder- las ti ng l -6
• 36 months.
• 37
Schizoaffective disorder- \!lects criteria for
• 38 schizophrenia in addition to major mood
o39 disorder (major depressive or bipolar). 'lb
• 40 • differentiate from a maior mood disorde r

a
Lock
s
Suspend
8
End Bl ock
Item: 27 of 73 ~. I • M k <:] t> al ~· ~
QIO: 1128 .l. ar Previous Next Lab 'lifllues Notes Calculator


20 A 26-year-old woman presents to her primary ca re physician due to fatigue for the past 2 weeks . Her
21 physica l examination is unremarkable. Over the past 4 years, she has sought medical attention on mu ltip le
occasions with complaints of episod ic nausea and vomiting, as well as dyspareun ia, low back pain, hand
22
numbness, and menorrhag ia. She had a negative resu lt on ultrasonography of the gal lbladder after complaining of
23 severe right upper quadrant pa in. She also had a norma l colonoscopy with biopsies after comp laining of rectal pa in
24 and frequent diarrhea . She reports to her physician that she can tell something is wrong with her and hopes that a
25
diagnosis can final ly be made.
26
Wh ich of the fo llowing is th is patient's most like ly diagnosis?
• 27
• 28 :

• 29
A. Factitious disorder

o30 B. Fibromyalgia
• 31 C. I llness anxiety disorder
• 32
D. Mal inger ing
• 33
o34
E. Somatic symptom disorder
o35
• 36
• 37
• 38
o39
• 40 •

6
lock
s
Suspend
0
End Block
Item: 27 of 73 ~. I • M k <:] t> al ~· ~
QIO: 1128 .l. ar Previous Next lab 'lifllues Notes Calculator


20 The correct answer is E. 53°/o chose this.
21 Patients with somatic symptom disorder present with one or more persistent physica l compla ints and symptoms
22 that are distressing or result in significant disruption of da ily life. The patient must also have excessive
thoughts, feel ings, or behaviors related to the somatic symptoms . Whereas in factitious disorder the symptoms
23
are created by the patient for a primary gain, in somatic symptom disorder the complaints are not consciously
24 produced . Patients with somatic symptom disorder will often have a history of symptoms in multiple organ
25 systems (GI, GU), as described in this vignette, suggesting somatic symptom diso rder as the diagnosis rather
than factitious diso rder.
26
Somatic symptom disorder Factitious disorder Somatization disorder Primary and secondary gain Symptom Somatic nervous system
27
• 28
A is not correct. 19% chose this .
• 29
Patients with factitious disorder deliberately feign physical symptoms for a primary gain such as attention or
being able to fi ll the "sick ro le." This is in contrast to patients with somatic symptom diso rder, who do not
o30 de libe rately create their symptoms for a primary gain. Furthermore patients with somatic symptom diso rder
• 31 often exh ibit a variety of symptoms invo lving numerous o rgans as is the case he re .
Factitious disorder Somatic symptom disorder Sick role Somatization disorder Primary and secondary gain Symptom Somatic nervous system
• 32
• 33 B is not correct. 4°/o chose this .
o34 Patients with fibromyalgia have ch ronic pain in their joints, muscles, tendons, etc. It is most common among
o35 young wo men . Diagnosis is supported by at least 3 months of widespread pa in involving multiple areas on the
body. Othe r than low back pain, th is patient is not comp laining of diffuse pain throughout the body .
• 36
Fibromyalgia low back pain Chronic pain Back pain Tendon
• 37
C is not correct. 17% chose this .
• 38
Patients with il lness anxiety disorde r are preoccupied with having a serious medica l illness despite minimal or
o39
nonexistent somatic symptoms. The diagnosis is made after their concerns persist for more than 6 months. The
• 40 • -•! --· ·- - - : - - •-- ·-- - · ·:·-- - ....._ _ ·- ·-- - - ·- - - _J: - · · ·--L.-.-.L.! - 1 - · - · · ! - .1... . __ _ __ _ __ _. : ____ . _ _ _ . ....... _ _ __ _. _ : ...... _ __ _ - · · - -- - : . . _ •--- ·.L ·- ___._.._ __.

6
lock
s
Suspend
0
End Block
Item: 27 of 73 ~. I • M k <:] t> al ~· ~
QIO: 1128 .l. ar Previous Next lab 'lifllues Notes Calculator

• Fibromyalgia low back pain Chronic pain Back pain Tendon


20
21 C is not correct. 17% chose this.
22 Patients with il lness anxiety diso rder are preoccupied with having a serious medica l illness despite minimal or
nonexistent somatic symptoms. The diagnosis is made after their concerns persist for more than 6 months. The
23
diagnosis also requires the presence of substantial anxiety regard ing health and either excessive hea lth -related
24 behaviors (checking oneself for signs repeatedly) or ma ladaptive avoidance of situations or activities thought to
25 be harmful to health. These patients tend to be midd le-aged, and the incidence is roughly equal between men
26
and women .
Anxiety disorder Hypochondriasis Anxiety Somatic symptom disorder Somatic nervous system
27
• 28 D is not correct. 7°/o chose this .
• 29
Patients who are malinger ing del iberately feign illness for a secondary ga in such as money, disabil ity, or
medications. The patient does not appea r to be making any particula r demands during her visits (signing
o30 disability for ms, pain medication prescription demands, etc), so a secondary gain appea rs to be an un like ly
• 31 motivator here. Mal inge ring disorder is diffe rent from somatic symptom diso rder, where both production of
• 32 symptoms and motivation are unconscious drives . t
Somatic symptom disorder Malingering Somatization disorder Symptom Analgesic Primary and secondary gain Somatic nervous system
• 33
o3 4
o35 Bottom Line:
• 36 Somatic symptom disorder is diagnosed when an individual presents with several physica l complaints in
• 37 multiple organ systems. Such patients have often undergone several tests, none of wh ich indicate an organic
• 38
cause. The symptoms are not deliberately produced, as in factitious disorder or mal inger ing .
Factitious disorder Somatic symptom disorder Somatization disorder Malingering Symptom Somatic nervous system
o39
• 40 •

6
lock
s
Suspend
0
End Block
Item: 27 of 73 ~. I • M k <:] t> al ~· ~
QIO: 1128 .l. ar Previous Next Lab 'lifllues Notes Calculator

• •
20
FA17 p 536.3
21
So matic symptom and Category of disorders characterized by physical symptoms causing significant distress and
22
related disorders impairment. Both illness production and motivation are uncon scious drives. Symptoms not
23
intentionally produced or feigned. More common in women.
24
Somatic symptom Variety of bodi ly complaints (eg, pain, fat igue) lasting for mont hs to years. Associated with
25
disorder excessive, persistent thoughts and anxiety abou t symptoms. lay co-occur with medical illness.
26 Treatment: regular office visits with the same physician in combination with psychotherapy.
27 Conversion disorder Loss of sensory or motor function (eg, paralysis, blindness, mutism), often following an acute
• 28 (functional stressor; patient is aware of but sometimes indifferent toward symptoms ("Ia belle indifference");
• 29 neurologic symptom more common in females, adolescents, and young adults.
disorder)
o30
• 31
Illness anxiety Excessive preoccupation with acq uiring or having a serious illness, often despite med ical
disorder evaluation and reassurance; minimal somatic symptoms.
• 32
(hypochondriasis)
• 33
0 34
FA17 p 536.1
0 35
Malingering Patient consciously fakes, profoundly exaggerates, or claims to have a disorder in order to attain a
• 36
specificzo (external) gain (eg, avoiding work, obtaining compensation). Poor compliance with
0 37 treatment or follow-up of diagnostic tests. Complaints cease after gain (vs factitious disorder).
• 38

0 39 FA17 p 536.2
• •
0 40

6
lock
s
Suspend
0
End Block
Item: 27 of 73 ~ 1 • M k -<:J 1>- Jil ~· !:';-~
QIO: 1128 ..L ar Pre v ious Next Labfli!llues Not es Calcula t o r
A A

20 Conversion disorder Loss of sensory or motor function (eg, paralysis, blindness, mutism), often following an acute
(functional stressor; patient is aware of but sometimes indifferent toward symptoms ("Ia belle indifference");
21
neurologic symptom more common in females, adolescents, and young adults.
22 disorder)
23 Illness anxiety Excessi,·e preoccupation with acquiring or having a serious illness, often despite medical
24 disorder e,-aJuation and reassurance; minimal somatic symptoms.
25 (hypochondriasis)
26
27 FA17 p 536.1

• 28 Malingering Patient consciously fakes, profoundly exaggerates, or claims to have a disorder in order to attain a
• 29 specific zo(external) gain (eg, avoiding \\Ork, obtaining compensation). Poor compliance" ith
o30
treatment or follow-up of diagnostic tests. Complaints cease after gain (vs factitious disorder).
. 31
• 32 FA1 7 p 536.2

• 33 Factitious disorders Patient consciously creates ph)•sical and/or psychologica l symptoms in order to assume "sick role"
o34
and to get medical attention and sympathy {1° [internal) gain).
o35
Factitious disorder Also known as Munchauscn syndrome. Chronic factitious disorder with predominantly physical
imposed on self signs and symptoms. Characterized by a history of multiple hospital admissions and willingness to
• 36
undergo invasive procednres. lore common in women and healthcare workers.
• 37
Factitious disorder Also known as Munchausen S) ndrome by JJrOX). Illness in a chi ld or elderly patient is caused or
• 38
imposed on another fabricated by the caregi,·er. J\ loti vat ion is to assume a sick role by proxy. Form of child/elder abuse.
o39
• 40 •
a
Lock Suspend
s 8
End Bl ock
Item: 28 of 73 ~ 1 • M k -<:J 1>- Jil ~· !:';-~
QIO: 3737 ..L ar Pre v ious Next Labfli!llues Not es Calcula t o r
A A

20 A 29-year-old f inancia l trader is brought to the emergency department by local po lice after getting in several
21 verbal altercations at a local bar. The patient is disoriented and sweating excessively. On physica l
examination, heart rate is 115/min and blood pressure is 168/90 mm Hg; he is breathing at a rate of
22
18/min. He is admitted for overnight observation. The next morning, he refuses to cooperate w ith hospital staff. It
23 is difficult to wake him up, but after a prolonged effort, he comments "either let me sleep or just kill me. "
24
25 What substance had he most likely been abusing?
26
:
27 A. Barbiturates
• 28
B. Cocaine
• 29
C. Heroin
o30
• 31 D. Marijuana
• 32 E. Methadone
• 33
0 34
0 35
• 36
• 37
• 38
0 39
0 40 •
a
Lock
s
Suspend
8
End Bl ock
Item: 28 of 73 ~. , . M k <:] t> al ~· ~
QIO: 3737 .l. ar Previous Next Lab 'lifllues Notes Calculator


20 The correct answer is B. 72°/o chose this.
21
Stimulant intoxication m ay be associated with tachycardia, hypertension, di lated pupils, and a sense of
restlessness. Cocaine abuse is associated wit h a post-use "crash," wh ich may include fatigue, ir ritability,
22 hyperso mnolence, and seve re depression wit h suicidality. This patient's presentation is most consistent wit h
23 stimu lant intoxication and withdrawa l, making cocaine the most likely cause .
Tachycardia Cocaine Stimulant Hypertension Suicidal ideation Mydriasis Hypersomnia Fatigue (medical) Suicide Major depressive disorder Irritability
24
25 Depression (mood) Substance intoxication Alcohol intoxication Boredom Pupillary response

26 A is not correct. 8°/o chose this.


27 Barbitu rates are central nervous system depressants. I ntoxication may be evidenced by constricted pupils,
28 confusion, hypotension, and poor coordination. The most f ea red result of intoxication is respiratory dep ression,
• 29
which could lead to coma and death .
Central nervous system Hypoventilation Hypotension Barbiturate Depressant Coma Nervous system Substance intoxication Major depressive disorder Miosis
o30
Depression (mood) Alcohol intoxication
• 31
• 32 C is not correct. 12% chose this .
• 33
Heroin is associated wit h a br ief rush of euphoria. Signs of intoxication include nausea, vomiting, pupi llary
constr iction, seizure, and central ne rvous syste m depression. Withdrawa l is associated with "fl u-like" symptoms,
o34 sweating, insomnia, and ano rexia.
o35 Central nervous system Insomnia Heroin Euphoria Nausea Vomiting Anorexia nervosa Perspiration Major depressive disorder Anorexia (symptom)

• 36 Central nervous system depression Nervous system Depression (mood) Epileptic seizure

• 37
D is not correct. 3°/o chose this .
• 38
Marijuana use may present with injected conj unctivae, impa ired motor coordination, increased appetite, dry
o39 mouth, and anxiety . Withdrawa l symptoms include irritabi lity, dep ression, insomnia, and nausea; symptoms
• 40 • usua llv oeak in 48 hours. Serious comolications are ve rv rare . •
6
lock
s
Suspend
0
End Block
Item: 28 of 73 ~. , . M k <:] t> al ~· ~
QIO: 3737 .l. ar Previous Next lab 'lifllues Notes Calculator


20 Heroin is associated with a brief rush of euphoria. Signs of intoxication include nausea, vomiting, pupi llary
constriction, seizure, and central nervous system depression. Withdrawa l is associated with "flu-like" symptoms,
21
sweating, insomnia, and anorexia.
22 Central nervous system Insomnia Heroin Euphoria Nausea Vomiting Anorexia nervosa Perspiration Major depressive disorder Anorexia (symptom)

23 Central nervous system depression Nervous system Depression (mood) Epileptic seizure

24
D is not correct. 3°/o chose this.
25
Marij uana use may present with injected conjunctivae, impa ired motor coordination, increased appetite, dry
26 mouth, and anxiety . Withdrawa l symptoms include irritabi lity, dep ression, insomnia, and nausea; symptoms
27 usua lly peak in 48 hours. Serious complications are ve ry rare . t
Insomnia Cannabis {drug) Nausea Xerostomia Anxiety Motor coordination Irritability Major depressive disorder Depression (mood)
28
Benzodiazepine withdrawal syndrome Polyphagia Conjunctiva Marijuana
• 29
o30 E is not correct. 5°/o chose this.
• 31 Methadone is a long-acting opiate used for he roin detoxification or fo r long-term care during heroin withdrawal.
• 32 Methadone abuse wou ld demonstrate symptoms similar to those of he roin abuse .
Methadone Opiate Heroin Detoxification Drug detoxification Opioid use disorder long-term care
• 33
o34
o35 Bottom Line:
• 36 Stimulant intoxication presents with sweating, tachyca rdia, high blood pressu re, and hyperactivity. Withdrawal
• 37 presents with hypersomno lence and irritabi lity, and it may be associated with severe depressive episodes .
Tachycardia Perspiration Stimulant Blood pressure Hypersomnia Attention deficit hyperactivity disorder Irritability Hypertension Substance intoxication
• 38
Alcohol intoxication
o39
• 40 •

6
lock
s
Suspend
0
End Block
Item: 28 of 73 ~. , . M k <:] t> al ~· ~
QIO: 3737 .l. ar Previous Next Lab 'lifllues Notes Calculator

• •
20 FA17 p 540.1
21 Psychoactive drug intoxication and withdrawal
22 DRUG ltiTOXICATION WITHDRAWAL

23 Depressants
24 onspecific: mood elevation, l anxiety, onspecific: anxiety, tremor, seizures,
. .
25 sedation, behavioral disinhibition, respiratory lllSOmnaa.
depression.
26
27
Alcohol Emotional labil ity, slurred speech, ataxia, Time from last drink:
coma, blackouts. Serum y-glutamyltransferase 3-36 hr: minor symptoms similar to other
28
(GGT)-sensitive indicator of alcohol usc. depressants
• 29
AST value is twice ALT value. 6- 48 hr: withdrawal seizures
o30 12-48 hr: alcoholic hallucinosis (usually visual)
• 31 48-96 hr: delirium tremens (DTs) in 5% of
• 32 cases
Treatment: benzodiazepines.
• 33
0 34
Opioids Euphoria, respiratory and CNS depression, Sweating, dilated pupils, piloerection ("cold
l gag reAex, pupillary constriction (pinpoint turkey"), fever, rhinorrhea, yawning, nausea,
0 35
pupils), seizures (overdose). Most common stomach cramps, diarrhea {"Au-like" symptoms).
• 36 cause of drug overdose death. Treatment: Treatment: long-term support, methadone,
• 37 naloxone. buprenorphine.
• 38 Barbiturates Low safety margin, marked res piratory Delirium, life-threatening cardiovascular
0 39 depression. Treatment: symptom management collapse.
0 40 • (eg, assist respiration, t BP). •

6
lock
s
Suspend
0
End Block
Item: 29 of 73 ~ 1 • M k -<:J 1>- Jil ~· !:';-~
QIO: 1475 ..L ar Pre v ious Next Lab fli!ltues Not es Calcula t o r

IAA]
A A

20 A 23-year-old man is taken to his family physician because his parents are worried about his behavior. He
21 ta kes hours to shower and frequently spends long periods of t ime washing his hands. He tells the physician
he has to m ake sure that he removes all the germs from his body before he leaves for work. His hands are
22
chapped w ith areas of cracking and bleeding.
23
24
In combination with psychotherapy, which of the following agents is most useful for the treatment of this condition?
25
:
26
A. Amitriptyline
27
B. Bupropion
28
• 29 C. Ch lorpromazine
o30 D. Diazepam
• 31
E. Sertra line
• 32
• 33
0 34
0 35
• 36
• 37
• 38
0 39
0 40 •
a
Lock
s
Suspend
8
End Bl ock
Item: 29 of 73 ~. I • M k <:] t> al ~· ~
QIO: 1475 .l. ar Previous Next lab 'lifllues Notes Calculator

• The correct answer is E. 63°/o chose this .


20
Se lective serotonin reuptake inhibitors (SSRis ), such as sertraline and fluoxetine, are effective treatments for
21 obsessive-compulsive disorder (OCD) . Serotonin is a key neurotransmitter in impu lse control, and thus agents
22 such as SSRis that increase brain serotonin levels have been effective at t reating this disorder. Sertraline also
23
has been used in the treatment of panic disorder, OCD, posttraumatic stress disorder, and social anxiety
disorder. Reported adverse effects include changes in weight, insomn ia, fatigue, tremor, dry mouth, nausea,
24
decreased libido, ejaculatory dysfunction, and hepatotoxicity.
25 Posttraumatic stress disorder Sertraline Fluoxetine Social anxiety disorder Obsessive-compulsive disorder Panic disorder Serotonin Hepatotoxicity libido

26 Neurotransmitter Insomnia Anxiety disorder Selective serotonin reuptake inhibitor Tremor Nausea Reuptake Xerostomia Social anxiety Anxiety Fatigue (medical)

27 Delayed gratification Brain

28
A is not correct. 10% chose this.
29
Amitr iptyl ine is a tricycl ic antidepressant (TCA) that inhibits seroton in and norepinephrine reuptake .
o30 Amitr iptyl ine is used in the treatment of depression, ch ron ic pain, peripheral neuropathy, and tension headache.
• 31 The on ly approved TCA for obsessive-compulsive disorder is clomipramine. However, se lective serotonin
• 32
reuptake inhibitors (SSRis) remain as first-line treatment because of the more favorable side-effect profi le .
Tricyclic antidepressant Amitriptyline Clomipramine Obsessive-compulsive disorder Tension headache Serotonin Antidepressant Norepinephrine
• 33
Peripheral neuropathy Selective serotonin reuptake inhibitor Headache Chronic pain Reuptake Major depressive disorder Depression (mood) Tricyclic
o34
First-line treatment Therapy
o35
• 36 B is not correct. 7°/o chose this .
• 37 Bupropion is a heterocycl ic antidepressant that is indicated primarily for depression and smoking cessation . It is
a selective norepineph r ine and dopamine reuptake inhibitor with no serotonergic activity. As a resu lt, it is not
• 38
associated with the sexua l dysfunction, weight gain, and sedation seen with SSRis. However, there is a risk of
o39 seizures in patients with electro lyte abnormalities.
• 40 • Buorooion Antideoressant Dooamine Noreoineohrine Dooamine reuotake inhibitor Serotonin Smokina cessation Electrolyte Serotoneraic Sexual dysfunction

6
lock
s
Suspend
0
End Block
Item: 29 of 73 ~. I • M k <:] t> al ~· ~
QIO: 1475 .l. ar Previous Next lab 'lifllues Notes Calculator


20 Amitr iptyl ine is a tricycl ic antidepressant (TCA) that inhibits seroton in and norepinephrine reuptake .
Amitr iptyl ine is used in the treatment of depression, ch ron ic pain, peripheral neuropathy, and tension headache.
21
The on ly approved TCA for obsessive-compulsive disorder is clomipramine. However, se lective serotonin
22 reuptake inhibitors (SSRi s) remain as first-line treatment because of the more favorable side-effect profi le.
23 Tricyclic antidepressant Amitriptyline Clomipramine Obsessive-compulsive disorder Tension headache Serotonin Antidepressant Norepinephrine

24 Peripheral neuropathy Selective serotonin reuptake inhibitor Headache Chronic pain Reuptake Major depressive disorder Depression (mood) Tricyclic

25 First-line treatment Therapy

26
B is not correct. 7°/o chose this.
27 Bupropion is a heterocycl ic antidepressant that is indicated primarily for depression and smoking cessation . It is
28 a selective norepineph r ine and dopamine reupta ke inhibitor with no serotonergic activity. As a resu lt, it is not
29 associated with the sexua l dysfunction, weight gain, and sedation seen with SSRis. However, there is a risk of
seizures in patients with electro lyte abnormalities.
o30
Bupropion Antidepressant Dopamine Norepinephrine Dopamine reuptake inhibitor Serotonin Smoking cessation Electrolyte Serotonergic Sexual dysfunction
• 31
Major depressive disorder Selective serotonin reuptake inhibitor Sedation Reuptake Depression (mood) Heterocyclic compound Epileptic seizure Reuptake inhibitor
• 32
Enzyme inhibitor Weight gain Tobacco smoking Water-electrolyte imbalance
• 33
C is not correct. 13% chose this.
o34
Ch lorpromazine, a neuroleptic agent, is used to treat schizophren ia. I t is a typical antipsychotic that has
o35
antichol inergic, antidopaminergic, and antihistaminic effects .
• 36 Typical antipsychotic Anticholinergic Chlorpromazine Schizophrenia Antipsychotic Antihistamine Dopamine antagonist
• 37
D is not correct. 7°/o chose this .
• 38
Diazepam is a benzodiazepine that is used to t reat anxiety, alcoho l withdrawa l, muscle spasm, status
o39 epilepticus, and some panic disorders.
• R.on?nrli~?.onin.o nL~?.on~m C:::t~toc: .onil.ontiroc: blrnhnl withrlr~w~l c:vnrlrnm.o C:::n~c:m D~nir rlic:nrrl.or bnvi.otv blrnhnlir h.ov.or;:::an.o blrnhnl
• 40
6
lock
s
Suspend
0
End Block
Item: 29 of 73 ~. I • M k <:] t> al ~· ~
QIO: 1475 .l. ar Previous Next lab 'lifllues Notes Calculator

• associated with the sexua l dysfunction, weight gain, and sedation seen with SSRis. However, there is a risk of
20 seizures in patients with electro lyte abnormalities.
21 Bupropion Antidepressant Dopamine Norepinephrine Dopamine reuptake inhibitor Serotonin Smoking cessation Electrolyte Serotonergic Sexual dysfunction

22 Major depressive disorder Selective serotonin reuptake inhibitor Sedation Reuptake Depression (mood) Heterocyclic compound Epileptic seizure Reuptake inhibitor

23 Enzyme inhibitor Weight gain Tobacco smoking Water-electrolyte imbalance

24
C is not correct. 13% chose this.
25
Ch lorpromazine, a neuroleptic agent, is used to treat schizophren ia. It is a typical antipsychotic that has
26 antichol inergic, antidopaminergic, and antihistaminic effects.
27 Typical antipsychotic Anticholinergic Chlorpromazine Schizophrenia Antipsychotic Antihistamine Dopamine antagonist

28 D is not correct. 7°/o chose this.


29 Diazepam is a benzod iazepine that is used to t reat anxiety, alcoho l withdrawa l, muscle spasm, status
o30 epilepticus, and some panic disorders.
Benzodiazepine Diazepam Status epilepticus Alcohol withdrawal syndrome Spasm Panic disorder Anxiety Alcoholic beverage Alcohol
• 31
• 32
• 33 Bottom Line:
o34 SSRis, such as sertra line and fluoxetine, are f irst- line treatments for OCD . They should be used in combination
o35 with behaviora l therapy, specifical ly exposure and response prevention, to achieve the greatest benefit.
• 36
Clomipramine is the on ly approved tricyclic antidepressant (TCA) for OCD. However, SSRis remain as first-line
treatment because of the more favorable side-effect profile .
• 37 Tricyclic antidepressant Sertraline Clomipramine Fluoxetine Antidepressant Exposure and response prevention Selective serotonin reuptake inhibitor Tricyclic
• 38 Therapy

o39
• 40 •

6
lock
s
Suspend
0
End Block
Item: 29 of 73 ~ 1 • M k -<:J 1>- Jil ~· !:';-~
QIO: 1475 ..L ar Pre v ious Next Lab fli!ltues Not es Calcula t o r
A A

20 FA17 p 545.1
21 Selective serotonin Fluoxetine, Am·oxamine, paroxetine, sertraline, escitalopram, citalopram.
22 reuptake inhibitors
23 MECHANISM Inhibit 5-HT reuptakc. It normally takes 4-8 weeks for antidepressants
24 CLINICAL USE Depression, generalized anxiety disorder, to have an effect.
25 panic disorder, OCD, bulimia, social an,iety
26 disorder, PTSD, premature ejaculation,
premenstrual dysphoric disorder.
27
28
ADVERSE EFFECTS Fe\\er than TC s. Cl di~tress, Sl Dl l, se,ual
dysfunction (anorgasmia, l libido).
29
o30
. 31 FA17 p 542.2

• 32 Preferred medications PSYCHIATRICCONDITION PREFERRED DRUGS

• 33 for selected ADHD Stimulants (methylphenidate, amphetam ines)


psychiatric conditions
o34 Alcohol withdrawal Benzodiazepines (eg, chlordiazepoxide,
o35 lorazcpam, diazepam)
. 36 Bipolar disorder Lithium, valproic acid, carbamazcpinc,
. 37
lamotrigine, atypical antipsychotics
. 38 Bulimia nervosa SSRis
o39 Depression SRis
• 40 • C:ener:~li7.f'rl :mxif'tv rli~nrriN SSRis S lRis
a
Lock
s
Suspend
8
End Bl ock
Item: 29 of 73 ~ 1 • M k -<:J 1>- Jil ~· !:';-~
QIO: 1475 ..L ar Pre v ious Next Lab fli!ltues Not es Calcula t o r
A A

20
Bulimia ner\'osa SSRis
21 Depression SSRis
22 Generalized anxiety disorder SSRis, SNRis
23 Obsessi,·e-compulsi\'e disorder SSRis, venlafaxine, clomipramine
24 Panic disorder Rls, \'enlafaxine, benzodiazepines
25 PTSD SSRis, 'enlafaxine
26 Schizophrenia Atypical antipsychotics
27 Social anxiety disorder SSRis. \'enlafaxine
28 Performance only: ~-blockers, benzodiazepines
29 Tourette syndrome Antipsychotics (eg, fluphenazine, pimozide),
o30 tel rabenazi ne
. 31
• 32 FA1 7 p 533.4

• 33 Obsessive-compulsive Recurring intrusive thoughts, feelings, or sensations (obsessions) that cause severe distress;
o34 disorder relieved in part by the performance of repetitive actions (compulsions). Ego-dystonic: behavior
o35 inconsistent with one's own beliefs and attitudes (vs obsessive-compulsive personality disorder).
Associated " ·ith Tourette syndrome. Treatment: C BT, SSRis, and clomipramine are first line.
. 36
. 37 Body dysmorphic disorder-preoccupation with minor or imagined defect in appearance
... significant emotional distress or impaired functioning; patients often repeatedly seek cosmetic
. 38
treatment. Treatment: C BT.
o39
• 40 •
a
Lock
s
Suspend
8
End Bl ock
Item: 30 of 73 ~. I • M k <:] t> al ~· ~
QIO: 4410 .l. ar Previous Next lab 'lifllues Notes Calculator


20 A 23-year-old woman with a history of pan ic disorder and general ized anxiety disorder is brought to the
21 emergency department after being found unconscious in her room by her pa rents . She is letha rgic and can
barely be aroused. She nods "yes" when asked if she has had any alcohol, and "yes" when asked if she has
22
taken any pi lls. Her parents are sure the only pi lls in the house are those prescr ibed by her psychiatrist. Her vital
23 signs are normal, and her pupils are di lated to 2 mm and nor mal ly reactive. Her blood alcohol level is 100 mg/dl.
24 Resu lts of her urine toxicology screen are pending, when sudden ly she beg ins having episodes of bradypnea that
25
cause her oxygen saturation to dip.
26
What should the physician give to treat this patient's condition?
27
28 :
A. Benztropine
29
o30 B. Flucytosine
• 31 C. Flumazeni l
• 32
D . Na loxone
• 33
o34
E. Na lt rexone
o35
• 36
• 37
• 38
o39
• 40 •

6
lock
s
Suspend
0
End Block
Item: 30 of 73 ~. I • M k <:] t> al ~· ~
QIO: 4410 .l. ar Previous Next lab 'lifllues Notes Calculator


20 The correct answer is C. 64°/o chose this.
21 This woman is exh ibiting symptoms of central nervous system (CNS) depression that cannot be exp lained by
her blood alcohol leve l alone (this level of CNS depression would typical ly be seen in a nonchronic drinker at
22
blood alcoho l levels of 250-300 mg/dL) . It is reasonab le, based on her psychiatric diagnoses, to think she may
23 have ingested a benzodiazepine along with the alcohol, resu lt ing in a synergistic effect. To reverse the effect of
24 the benzodiazepine, the drug of choice is flumazeni l, a competitive GABA antagonist.
Benzodiazepine Flumazenil Central nervous system Gamma-Aminobutyric acid Major depressive disorder Depression (mood) Alcohol Alcoholic beverage
25
Receptor antagonist Central nervous system depression Nervous system GABA receptor antagonist Blood alcohol content Synergy
26
27 A is not correct. 7°/o chose this.
28 Benztropine is a centra lly acting anticho linergic agent that acts as an acetylcholine receptor antagon ist. It is
29 used to treat parkinsonism as wel l as extrapyramidal and dystonic reactions. It is not used to treat
benzodiazepine overdose.
30
Benzodiazepine Anticholinergic Benzatropine Benzodiazepine overdose Acetylcholine Parkinsonism Dystonia Extrapyramidal symptoms Receptor antagonist
• 31
Extrapyramidal system Drug overdose Acetylcholine receptor Receptor (biochemistry)
• 32
• 33
B is not correct. 5°/o chose this .
Flucytosine is a potent antifunga l that inhibits DNA and RNA biosynthesis. It is used, in combination with
o34
amphotericin B, to treat systemic funga l infections, including meningitis caused by Cryptococcus.
o35 Amphotericin B Flucytosine Meningitis Antifungal Fungicide Mycosis Cryptococcus Fungus RNA DNA Biosynthesis
• 36
D is not correct. 16% chose this .
• 37
Naloxone is a competitive opioid antagon ist used to reverse opioid overdose. Opioid intoxication typically
• 38 manifests with drowsiness, slurred speech, constricted pupi ls, and respiratory depression, which can lead to
o39 coma or death.
Naloxone Hypoventilation Opioid Opioid antagonist Opioid overdose Toxidrome Somnolence Receptor antagonist Miosis Major depressive disorder Coma
• 40 •

6
lock
s
Suspend
0
End Block
Item: 30 of 73 ~. I • M k <:] t> al ~· ~
QIO: 4410 .l. ar Previous Next lab 'lifllues Notes Calculator


20 B is not correct. 5°/o chose this.
21 Flucytosine is a potent antifunga l that inhibits DNA and RNA biosynthesis. It is used, in combination with
amphotericin B, to t reat systemic funga l infections, including meningitis caused by Cryptococcus.
22
Amphotericin B Flucytosine Meningitis Antifungal Fungicide Mycosis Cryptococcus Fungus RNA DNA Biosynthesis
23
24
D is not correct. 16% chose this.
Naloxone is a competitive opioid antagon ist used to reverse opioid overdose. Opioid intoxication typically
25
manifests with drowsiness, slurred speech, constricted pupi ls, and respiratory depression, which can lead to
26 coma or death.
27 Naloxone Hypoventilation Opioid Opioid antagonist Opioid overdose Toxidrome Somnolence Receptor antagonist Miosis Major depressive disorder Coma

28 Depression (mood) Drug overdose Alcohol intoxication

29
E is not correct. 8°/o chose this.
30 Naltrexone is a long-acting opioid antagon ist. It is used to prevent relapse in opioid users once they have been
• 31 detoxified. Naltrexone can precipitate withdrawal if used within 7 days of heroin use .
Naltrexone Opioid antagonist Opioid Heroin Receptor antagonist Relapse
• 32
• 33
o3 4 Bottom Line:
o35
I ngesting alcohol potentiates the effects of benzodiazepines and can lead to dangerous respiratory depression .
• 36 Flumazenil acts as a competit ive antagon ist at the GABA receptor and is used to t reat benzodiazepine toxicity .
• 37 Benzodiazepine Flumazenil GABA receptor Hypoventilation Gamma-Aminobutyric acid Receptor antagonist Competitive antagonist Major depressive disorder
Alcohol Alcoholic beverage Depression (mood) Toxicity Antagonist
• 38
o39
• 40 •

6
lock
s
Suspend
0
End Block
Item: 30 of 7 3 ~ 1 • M k -<:J 1>- Jil ~· !:';-~
QIO: 4410 ..L ar Pre v ious Next Lab fli!ltues Not es Calcula t o r
A A

20
FA17 p 515.2
21
Benzodiazepines Diazepam, lorazepam, triazolam, lcma7cpam, oxazepam, midazolam, chlordiazepoxide,
22
alprazolam.
23
MECHANISM Facilitate GABA,\ action by t frequency of "Frcnzodiazepines" t frequency.
24 CJ- channel opening. l RE\ 1sleep. lost Benzos, barbs, and alcohol all bind the
25 have long haJf-li,·es and acti,·e metabolites GABA \ receptor. which is a ligand-gated CJ-
26 {exceptions (ATOl\1]: Alprazolam, 1 riazolam, channel.
27
Oxazepam, and ~ fidazolam are short acting Oxazepam, Temazepam, and Lorazepam are
- higher addictive potential). metabolized O utside T he Liver
28
CliNICAL USE Anxiety, spasticity, status epilepl icus (lorazepam
29
and diazepam), eclampsia, detoxification
30
(especially alcohol withdrawai-DTs), night
. 31 terrors, sleepwalking, general anesthetic
• 32 (amnesia, muscle relaxation), hypnotic
• 33 (insomnia).
o34 ADVERSE EFFECTS Dependence, additive CNS depression effects
o35
with alcohol. Less risk of respiratory depression
and coma than with barbiturates .
. 36
Treat overdose with Aumazenil (compclil i\C
. 37 antagonist at GABA benzodiazepine receptor).
. 38 Can precipitate seizures by causing acute
o39 benzodiazepine withdrawal.
• 40 •
a
Lock
s
Suspend
8
End Bl ock
Item: 30 of 73 ~. I • M k <:] t> al ~· ~
QIO: 4410 .l. ar Previous Next lab 'lifllues Notes Calculator

• •
20 FA17 p 540.1

21 Psychoactive drug intoxication and withdrawal


DRUG ltiTOXICATION WITHDRAWAL
22
Depressants
23
24 onspecific: mood elevation, l anxiety, onspecific: anxiety, tremor, seizures,
. .
sedation, behavioral disinhibition, respiratory lllSOmnaa.
25
depression.
26
Alcohol Emotional labil ity, slurred speech, ataxia, Time from last drink:
27
coma, blackouts. Serum y-glutamyltransferase 3-36 hr: minor symptoms similar to other
28 (GGT)-sensitive indicator of alcohol usc. depressants
29 AST value is twice ALT value. 6- 48 hr: withdrawal seizures
30 12-48 hr: alcoholic hallucinosis (usually visual)
• 31
48-96 hr: delirium tremens (DTs) in 5% of
cases
• 32
Treatment: benzodiazepines.
• 33
Opioids Euphoria, respiratory and CNS depression, Sweating, dilated pupils, piloerection ("cold
0 34 l gag reAex, pupillary constriction (pinpoint turkey"), fever, rhinorrhea, yawning, nausea,
0 35 pupils), seizures (overdose). Most common stomach cramps, diarrhea {"Au-like" symptoms).
• 36 cause of drug overdose death. Treatment: Treatment: long-term support, methadone,
• 37 naloxone. buprenorphine.
• 38 Barbiturates Low safety margin, marked res piratory Delirium, life-threatening cardiovascular
depression. Treatment: symptom management collapse.
0 39
(eg, assist respiration, t BP). •
0 40 •

6
lock
s
Suspend
0
End Block
Item:31of73 ~. , . M k <:] t> al ~· ~
QIO: 1139 .l. ar Previous Next lab 'lifllues Notes Calculator


20 A 23-year-old woman is admitted to the psychiatr ic ward after demonstrating er ratic behavior over the past
21 several months. Her fami ly reports she has been leaving town regula r ly on unscheduled vacations. Her mood
has been unstable, sometimes feel ing "h igh as a kite" and other t imes sobbing because she is "all alone ." On
22
morning rounds, she requests to be cared for by a particular nurse, who she says is the most attentive person she
23 has ever met. She bitterly comp lains that the other nurses t reat her miserably, and she refuses to cooperate with
24 them.
25
26 This patient's behavior is most consistent which of the fol lowing?
27 :
28 A. Borderl ine personality disorder
29 B. Dependent personal ity disorder
30
C. Dissociative identity disorder
· 31
• 32
D. Histrionic persona lity disorder
• 33 E. Obsessive-compu lsive personality disorder
o34 F. Paranoid personality disorder
o35
• 36
• 37
• 38
o39
• 40 •

6
lock
s
Suspend
0
End Block
Item:31of73 ~. , . M k <:] t> al ~· ~
QIO: 1139 .l. ar Previous Next lab 'lifllues Notes Calculator


The correct answer is A. 73°/o chose this .
20
Borderl ine personality disorder (BPD) is a cluster B disorder, cha racterized by dramatic and emotional behavior,
21
impulsivity, labile moods, and unstable relationships. Patients often engage in self-destructive behaviors in an
22 attempt to gain attention. Splitting is a maladaptive coping mechanism used by these ind ividua ls that paints
23 others as all "good" or all "bad ." In this case, the patient perceives that the one nu rse is wonderfu l, but the
24
others are bad to an extreme. Associated with this condition are mood disorders and substance abuse (heroin
use). Treatment for BPD includes psychotherapy along with possible adjunctive pharmacotherapy, wh ich may
25 include a mood stabilizer such as valproate or an atypica l antipsychotic such as olanzapine.
26 Atypical antipsychotic Borderline personality disorder Olanzapine Mood stabilizer Antipsychotic Valproate Mood disorder Pharmacotherapy Coping (psychology)

27 Personality disorder Psychotherapy Heroin Substance abuse Impulsivity

28
B is not correct. 7°/o chose this.
29 I nd ividuals with dependent personal ity disorder are cha racterized by low self-confidence, cling ing, and the need
30 to be taken care of. It is a cluster C personal ity disorder.
Dependent personality disorder Personality disorder
31
• 32 C is not correct. 6°/o chose this .
• 33 Dissociative identity disorder (previous ly known as mu ltip le personal ity disorder) is associated with the presence
o34 of multiple distinct persona lities or identities. The patient in th is question acts differently toward the one nu rse,
but there is no evidence given that she demonstrates mu ltiple identities.
o35
Dissociative identity disorder Dissociative Dissociation (psychology) Personality disorder
• 36
• 37
D is not correct. 10% chose this .
Histrionic persona lity disorder is a cluster B personality disorder characterized by excessive emotionality and
• 38
excitability, attention seeking, sexua lly provocative behavior, and excessive concern with appearance. Although
o39 this patient appears to have great emotiona l liabi lity, her display of "spl itting" makes another diagnosis more
• 40 • l iv.::.l"

6
lock
s
Suspend
0
End Block
Item:31of73 ~. , . M k <:] t> al ~· ~
QIO: 1139 .l. ar Previous Next lab 'lifllues Notes Calculator

• D is not correct. 10% chose this .


20
Histrionic persona lity disorder is a cluster B personality disorder characterized by excessive emotionality and
21 excitability, attention seeking, sexua lly provocative behavior, and excessive concern with appearance. Although
22 this patient appears to have great emotiona l liabi lity, her display of "spl itting" makes another diagnosis more
23 likely.
Histrionic personality disorder Attention seeking Personality disorder Cluster B personality disorders
24
25 E is not correct. 1°/o chose this.
26 Obsessive-compu lsive personality disorder is a cluster C disorder characterized by a preoccupation with
perfectionism . In contrast to obsessive-compulsive disorder, in which behaviors are often distressing to the
27
individual, patients with obsessive-compulsive personal ity disorder demonstrate behaviors consistent with their
28 underlying be liefs and attitudes (egosyntonic).
29 Obsessive-compulsive personality disorder Obsessive-compulsive disorder Perfectionism (psychology) Personality disorder Obsessive-compulsive

30 F is not correct. 3°/o chose this.


31 Paranoid personal ity disorder is a cluster A disorder. I ndividuals often exhibit an under lying distrust of others
• 32 and often emp loy projection as a defense mechanism .
Paranoid personality disorder Defence mechanisms Personality disorder Paranoia
• 33
o34
o35 Bottom Line:
• 36 I nd ividuals with border line personality disorder are characterized by their labile moods and unstable
• 37 relationships. They are often impu lsive and engage in self-destr uctive behaviors in an attempt to gain
• 38 attention . Spl itting is a maladaptive coping mechan ism used by these individuals that paints others as al l
" good" or all " bad."
o39 Borderline personality disorder Coping (psychology) Personality disorder lability Maladaptation Adaptive behavior
• 40 •

6
lock
s
Suspend
0
End Block
Item: 31 of 7 3 ~ 1 • M k -<:J 1>- Jil ~· !:';-~
QIO: 1139 ..L ar Pre v ious Next Labfli!llues Notes Calcula t o r
A A

20 FA17 p 535.2
21 Cluster B personality Dramatic, emotional, or erratic; genetic "\\'ild."
22 disorders association with mood disorders and substance Cluster B: Bad, Borderline, AamBoyant, must
23 abuse. be the Best
24 Antisocial Disregard for and violation of rights of others Anli~ocial =~oeiopath.
25 ,,·ith lack of remorse, criminality, impulsi,·ity; Bad.
males> females; must be~ 18 years old and
26 '
ha,·e history of conduct disorder before age 15.
27 Conduct disorder if< 18 years old.
28
Borderline Unstable mood and interpersonal relationships, Treatment: dialectical behavior therapy.
29 impulsivity, self-mutilation, suicidality, sense Borderline.
30 of emptiness; females> males; splitting is a
31 major defense mechanism.
• 32 Histrionic Excessive emotionality and excitability, FlamBoyant.
• 33 attention seeking, sexually pro,•ocativc, overly
concerned with appearance.
o34
Narcissistic G randiosity, sense of entitlement; lacks emp<1thy Must be the Best.
o35
and requires excessive admiration; often
. 36
demands the "best" and reacts to criticism
. 37 with rage.
. 38
o39 FA11 p 524.4
• 40 • . . ~- . . ..
a
Lock
s
Suspend
8
End Bl ock
Item:31of73 ~. , . M k <:] t> al ~· ~
QIO: 1139 .l. ar Previous Next lab 'lifllues Notes Calculator

FA17 p 524.4
20

21 Ego defenses [ental processes (unconscious or conscious) used to resolve confl ict and prevent undesirable
feelings (eg, anxiety, depression).
22

23
IMMATURE DEFENSES DESCRIPTION EXAMPLE

24 Acting out Expressing unacceptable feelings and thoughts Tantrums.


through actions.
25
Denial Avoiding the awareness of some painfu l reality. A patient with cancer plans a full-time work
26
schedule despite being warned of significant
27
fatigue during chemotherapy.
28
Displacement Redirection of emotions or impulses to a neutral A teacher is yelled at by the principaI. Instead of
29 person or object (vs projection). confronting the principal directly, the teacher
30 goes home and criticizes her husband's dinner
31 selection.
• 32 Dissociation Temporary, drastic change in personal ity, A victim of sexua l abuse suddenly appears numb
• 33 memory, consciousness, or motor behavior to and detached when she is exposed to her
avoid emotional stress. Patient has incomplete abuser.
0 34
or no memory of traumatic event.
0 35
Fixation Partially remaining at a more chi ldish level of A surgeon throws a tantrum in the operating
• 36
development (vs regression). room because the last case ran very late.
0 37
Idealization Expressing extremely positive thoughts of self A patient boasts about his physician and his
• 38 and others while ignoring negative thoughts. accomplishments while ignoring any Raws.
0 39
Identification Largely unconscious assumption of the A resident starts putting his stethoscope in his
0 40 • 0 • • 1 •• • ., r .. I o 1 •1 I • r .. ,. 1 r

6
lock
s
Suspend
0
End Block
Item: 32 of 73 ~ 1 • M k -<:J 1>- Jil ~· !:';-~
QIO: 3531 ..L ar Pre v ious Next Labfli!llues Not es Calcula t o r

IAA]
A A

20 A 60-year-old African -American man has been reclusive, ra rely leaving his home for the past 40 years. His
21 family describes him as an emotionally cold person with few friends. Growing up, he preferred sol itary
activities like read ing to engag ing in activities with others. Members of his church have del ivered groceries to
22
his front door once a week for the past 20 years, but he never opens the door to greet t hem.
23
24
Which of the following is the most likely diagnosis?
25
:
26
A . Avoidant personality disorder
27
B. Paranoid personality disorder
28
29 C. Schizoid personal ity disorder
30 D. Schizophren ia
31
E. Schizophren iform disorder
• 32
F. Schizotypa l personal ity disorder
• 33
o34
o35
. 36
. 37
. 38
o39
• 40 •
a
Lock
s
Suspend
8
End Bl ock
Item: 32 of 73 ~. I • M k <:] t> al ~· ~
QIO: 3531 .l. ar Previous Next Lab 'lifllues Notes Calculator


20
The correct answer is C. 60°/o chose this.
This man has sch izoid personal ity disorder, marked by a lifelong pattern of socia l withdrawal. People with this
21
disorder experience discomfort with human interaction and therefore isolate themselves, avoid close
22 relationships, and engage in solitary activit ies. These patients often are viewed as eccentric, isolated, lonely,
23 and emotional ly co ld .
Schizoid personality disorder Solitude Personality disorder
24
25 A is not correct. 24% chose this.
26 Avoidant patients are like schizoid patients in their pe rvasive pattern of socia l inhibition. Howeve r, they do
27 desi re companionship; an intense fea r of rejection leads to avoiding any situation where the re is a pe rceived
r isk of rejection. Think avoidant persona lity disorde r in patients inh ibited by feel ings of inadequacy and socia l
28
ineptness to the extent they will participate social ly only when they are certain to be liked.
29 Avoidant personality disorder Social inhibition Personality disorder Schizoid personality disorder

30
B is not correct. 4°/o chose this.
31
Patients with paranoid personal ity disorder tend to be more social ly engaged than those with schizoid
32 pe rsona lity disorde r, even though they have a lifelong history of suspiciousness and mistr ust of othe r peop le.
• 33 Examples include recurrent suspicion of a sexual partner 's fidel ity or blaming others for their problems .
Schizoid personality disorder Paranoid personality disorder Personality disorder Paranoia
o34
o35 D is not correct. 2°/o chose this.
• 36 Patients with schizoph ren ia exhibit a fo rmal thought disorde r with ha llucinations, or delusional thin king. In
• 37
contrast, patients with schizoid persona lity disorde r have intact rea lity testing .
Schizoid personality disorder Thought disorder Schizophrenia Personality disorder Hallucination Reality testing
• 38
o39 E is not correct. 3°/o chose this.
• 40 •
Schizoph renifo rm disorder is identica l to schizoph ren ia except that symptoms last for at least 1 month but less •

6
lock
s
Suspend
0
End Block
Item: 32 of 73 ~. I • M k <:] t> al ~· ~
QIO: 3531 .l. ar Previous Next Lab 'lifllues Notes Calculator


20 D is not correct. 2°/o chose this.
21 Patients with schizophren ia exhibit a formal thought disorder with hallucinations, or delusional thinking. In
contrast, patients with schizoid personality disorder have intact reality testing.
22 Schizoid personality disorder Thought disorder Schizophrenia Personality disorder Hallucination Reality testing
23
E is not correct. 3°/o chose this.
24
Schizoph reniform disorder is identical to schizophrenia except that symptoms last for at least 1 month but less
25
than 6 months. Patients with schizophren iform disorder have a better prognosis than do most patients with
26 schizoph renia, and may return to their baseline mental functioning .
27 Schizophreniform disorder Schizophrenia Prognosis

28 F is not correct. 7°/o chose this.


29 Schizotypa l and schizoid personality disorder are rather simi lar, but the former is distingu ished in that these
30 patients tend to be more similar to schizophrenics. Patients with sch izotypal disorder are strikingly odd, with
pecu liar notions, ideas of reference, illusions, magical thinking, and derealization.
31
Schizoid personality disorder Derealization Schizotypal personality disorder Magical thinking Ideas of reference and delusions of reference Schizophrenia
32
Personality disorder
• 33
o34
o35 Bottom Line:
• 36 Schizoid personality disorder is characterized by social withdrawa l, avoidance of close relationships, and
• 37 tendency to be viewed by others as eccentric and emotionally cold, but without increased likelihood of having
schizoph renic relatives .
• 38 Schizoid personality disorder Schizophrenia Solitude Personality disorder
o39
• 40 •

6
lock Suspend
s 0
End Block
Item: 32 of 73 ~ 1 • M k -<:J 1>- Jil ~· !:';-~
QIO: 3531 ..L ar Pre v ious Next Labfli!llues Not es Calcula t o r
A A

20 FA17 p535.1

21 Cluster A personality Odd or eccentric; inability to dc,·clop . d."


"\\'Clf
22 disorders meaningful social relationships. 'o pS)Cho~i~; Cluster .\: Accusatory, .\ loof, A" kward.
23 genetic association with schizophrenia.
24 Paranoid Pen·asive distrust (Accusatory) and
25
suspiciousness of others and a profound I}
C)'11ical ,·iew of the world.
26
Schizoid Voluntary social" ithdrawal (Aloof), limited
27
emotional expression, content with social
28 isolation (vs m·oidant).
29
Schizotypal Eccentric appearance, odd beliefs or magical Schizotrpal = magical thinking.
30 thinking, interpersonal \ wkwardncss.
31
32 FA17 p 534.3
• 33 Personality
-34 Personality trait An enduring, repetitive pattern of perceiving, relating to, and thinking about the environment and
-35 oneself.
. 36 Personality disorder InAexible, maladaptive, and rigidly pervasive pat tern of behavior causing subjecti,•e distress
. 37 and/or impaired functioning; person is usuallr not aware of problem. Usually presents by early
. 38 adulthood .
Three clusters, A, B, and C; remember as Weird, \\ ild, and \\'orricd based on symptoms.
-39
• 40 •
a
Lock
s
Suspend
8
End Bl ock
Item: 33 of 73 ~ 1 • M k -<:J 1>- Jil ~· !:';-~
QIO: 5113 ..L ar Pre v ious Next Labfli!llues Notes Calcula t o r

IAA]
A A

20 A 19-year-old man is brought to t he physician by his mother, who reports that her son has been acting
21 strangely over the past 8 months . Lately, he has been qu iet, with short responses, and sometimes makes
strange comments, for example about the television sending him messages. He denies suicida l or homicida l
22
ideat ion, and he has not been using any substances. Further med ical work-up and cran ial imaging are with in
23 normal li m its.
24
25 Along which brain pathway(s) does the patient most likely have abnormal neurotransmission?
26
:
27 A. Locus coeruleus
28
B. Mesoli mbic and mesocortical
29
C. Nigrostriata l pathway
30
31 D. Paramedian pontine reticular formation
32 E. Raphe nuclei
• 33
0 34
0 35
0 36
. 37
. 38
0 39
0 40 •
a
Lock
s
Suspend
8
End Bl ock
Item: 33 of 73 ~. I • M k <:] t> al ~· ~
QIO: 5113 .l. ar Previous Next lab 'lifllues Notes Calculator


20 The correct answer is B. 68°/o chose this.
21 This patient like ly has schizoph renia, wh ich is associated with increased dopamine leve ls. The mesocortica l
22 pathway is a major dopaminergic pathway that connects the ventral tegmentum to the cortex, and dysfunction
of this pathway is be lieved to play a role in the negative symptoms of schizophren ia (avolition and alog ia). The
23
mesolimbic pathway, which begins in the midbrain and connects to the limbic system, is a major dopaminergic
24 pathway that is involved in addiction and rewa rd. It is also believed that dysfunction of this pathway is
25 responsible for the positive symptoms of sch izophrenia (delusions, ha llucinations, and disorgan ized thought) . A
26
diagnosis of schizophrenia requi res the presence of two or more of the fol lowing, each present for a significant
period of t ime during a 1-month period over 6 months: (1) delusions, (2) ha llucinations, (3) disorganized
27
thought such as incoherent answers, (4) grossly disorganized or cataton ic behavior, (5) and negative symptoms
28 such as flat affect, social withdrawal, and lack of motivation. Continuous signs of the disturbance (including the
29 1 month described) must persist for at least 6 months, with socia l and occupational dysfunction. The symptoms
should not be due to substance abuse or a general medical condition. Schizoph ren ia typical ly presents earl ier
30
and has a worse prognosis in men, with a pea k age of onset in the ea r ly 20s . The med ical treatment includes
31 anti-psychotic medications, which often function to block dopamine D2 receptors.
32 Mesocortical pathway Mesolimbic pathway limbic system Alogia Schizophrenia Dopaminergic pathways Ventral tegmental area Midbrain Dopamine Antipsychotic

33 Dopaminergic Reduced affect display Hallucination Tegmentum Substance abuse Catatonia Cerebral cortex Delusion Prognosis Anatomical terms of location

34
0

A is not correct. 7°/o chose this.


0 35 The locus coeru leus is a nucleus in the pons that is the major site of norepinephrine production and secretion to
0
36 the rest of the brain. It is involved in the body's physiologic response to stress, and plays a role in depression
0 37 and anxiety disorders .
locus coeruleus Pons Norepinephrine Anxiety Anxiety disorder Major depressive disorder locus (genetics) Human brain Cell nucleus Secretion Depression (mood)
0 38
Physiology
0 39
0 40 • C is not correct. 150/o chose this.
6
lock
s
Suspend
0
End Block
Item: 33 of 73 ~. I • M k <:] t> al ~· ~
QIO: 5113 .l. ar Previous Next lab 'lifllues Notes Calculator

• locus coeruleus Pons Norepinephrine Anxiety Anxiety disorder Major depressive disorder locus (genetics) Human brain Cell nucleus Secretion Depression (mood) •
20
Physiology
21
C is not correct. 15% chose this.
22
The nigrostria l pathway, which connects the substantia nigra to the str iatum, is also a major dopaminergic
23
pathway, but it is primarily associated with Parkinson disease. I n Pa r kinson, there is a decrease in dopamine
24 levels from loss of dopaminergic neu rons in the substantia nigra, resu lt ing in the cl inica l picture of rigidity,
25 tremo r, and bradykinesia. Treatments aim to increase dopamine levels.
Substantia nigra Bradykinesia Dopamine Dopaminergic pathways Striatum Dopaminergic Tremor Parkinson' s disease Neuron
26
27 D is not correct. 4°/o chose this.
28 The paramedian pontine reticu lar formation is a gaze cente r that is responsible for the extraocu lar eye
29
movements that occur du ring rapid eye movement sleep .
Paramedian pontine reticular formation Reticular formation Rapid eye movement sleep Eye movement Pons
30
31 E is not correct. 6°/o chose this.
32
The raphe nuclei, located in the brain stem around the reticu lar formation, are the major source of se rotonin in
the brain. Se roton in plays a role in a variety of functions, including sleep, mood, appetite, and sexua lity.
33 Raphe nuclei Reticular formation Serotonin Brainstem Nucleus (neuroanatomy) Human brain Raphe Cell nucleus Brain
0 34
0 35
Bottom Line:
0
36
0 37 Schizoph renia is associated with increased dopamine levels; it has been proposed that dysfunction of the
mesolimbic pathway is related to the positive symptoms, whereas dysfunction of the mesocortica l pathway is
38
related to the negative symptoms.
0

0 39 Mesocortical pathway Mesolimbic pathway Schizophrenia Dopamine

0 40 •

6
lock
s
Suspend
0
End Block
Item: 33 of 7 3 ~ 1 • Ma rk -<:J 1>- Jil ~· !:';-~
QIO: 5113 ..L Pre v ious Next Labfli!llues Notes Calcula t o r
A A

20 FA17 p 530.1
21 Schizophre nia Chronic mental disorder'' ith periods of Frequent cannabis use is associated "it h
22 psychosis, disturbed behavior and thought, psychosis/schizophrenia in teens.
23 and decline in functioning lasting> 6 Lifetime pre,·alence-1.5% (males= females,
24
months. Associated with f dopaminergic African mericans =Caucasians). Presents
acti,·ity, l dendritic branching. earlier in men (late teens to earlv• 20s \ 'S late
25
Diagnosis requires at least 2 of the folio\\ ing, 20s to earlr 30s in women). Patients are at f
26 ri~k for suicide.
and at least 1 of these should include 1-3
27 (first 4 are ~positive symptoms"}: Venlriculomegaly on brain imaging.
28 I. Delusions Treatment: atypical antipsychotics (eg,
29 2. Hallucinations-often auditor) risperidone) are first line.
3. Disorganized speech Negative symptoms often persist after treatment,
30
4. Disorganized or ca1<1tonic behavior despite resolution of positi,·e symptoms.
31
5. 1egati,·c symptoms (affective flallening,
32 a,·olition, anhedonia, asociality, alogia)
33
Brief psychotic disorder- lasting< 1 month,
0 34 usual ly stress related.
0 35 Schizophreniform disorder- lasting 1-6
0 36 months.
. 37 Schizoaffective disorder- leets criteria for
. 38 schizophren ia in addition to major mood
0 39 disorder (major depressive or bipolar). To
0 40 • differentiate from a major mood disorder

a
Lock Suspend
s 8
End Bl ock
Item: 34 of 73 ~ 1 • M k -<:J 1>- Jil ~· !:';-~
QIO: 4633 ..L ar Pre v ious Next Lab fli!ltues Notes Calcula t o r

IAA]
A A

20 A 60-year-old man has been feeling depressed after the death of his wife 1 month ago. He has had a hard
21 t ime sleeping and has had brief moments where he thought he saw his wife still in their home. He often
thinks that he would be better off dead with his wife and bought a pistol with ammun ition 4 days ago. He
22
blames himself for the death of his wife and has difficulty motivating himself to resume his normal activities.
23 Additionally, he mentions that he has started to get frequent headaches. He has lost 7 lb (3.2 kg) since his last
24 visit 6 weeks ago.
25
26 Which of his symptoms would compel the physician to diagnose this patient with abnorma l grieving?
27 :
28 A. Difficulty falling asleep
29 B. Feelings of guilt
30
C. Seeing his wife for brief moments
31
D. Tension headaches
32
33 E. Thoughts of being better off dead
0 34 F. Weight loss
0 35
0 36
. 37
. 38
0 39
0 40 •
a
Lock
s
Suspend
8
End Bl ock
Item: 34 of 73 ~. I • M k <:] t> al ~· ~
QIO: 4633 .l. ar Previous Next lab 'lifllues Notes Calculator

• •
20 The correct answer is E. 78°/o chose this.
Normal grief often encompasses symptoms of depression . However, norma l grief shou ld not encompass suicida l
21
thoughts as an attempt to reach loved ones, particula r ly if they are accompanied by a plan. This man just
22 bought a gun and ammun it ion days ago, so it can be assumed that he has a plan to actual ly attempt suicide .
23 This suggests active su icida lity and is much different from passive thoughts of death that can be associated with
24 norma l bereavement. In addition, th is man's increased age places him at increased risk for suicide .
Major depressive disorder Depression (mood) Suicide Suicidal ideation Suicide crisis Bereavement
25
26 A is not correct.
27 Changes in sleep patterns is a depressive symptom. Depressive symptoms can commonly occur in the setting of
grief. However, if these depressive symptoms last for >2 months after the death of the loved one, and are
28
causing sign if icant functiona l impai r ment, then the survivor may be experiencing complicated (patholog ic) gr ief.
29 Symptom Pathology

30
B is not correct.
31
It is norma l for a gr ieving person to fee l some gu ilt after the death of a loved one. However, extreme gui lt or
32 feel ings of worthlessness are symptoms of pathologic grief, which warrants psychiatric t reatment.
33 Psychiatry Pathology

34 C is not correct. 14% chose this.


o35 Brief aud ito ry and visua l hal lucinations di rectly related to a recently deceased loved one are symptoms of
• 36 norma l grief. However, if hal lucinations last for longer than 2 months, cause functional impairment, or involve
• 37 imagery or voices un related to the pe rson that has died, the su rvivor may be experiencing complicated grief.
Hallucination Sound Auditory system Hearing
• 38
o39 D is not correct. 6°/o chose this.
• 40 • It is common fo r grieving persons to present with somatic complaints relating to the increased stress of losing a •

6
lock
s
Suspend
0
End Block
Item: 34 of 73 ~. I • M k <:] t> al ~· ~
QIO: 4633 .l. ar Previous Next lab 'lifllues Notes Calculator
• I I -

20 It is norma l for a grieving person to fee l some gu ilt after the death of a loved one. However, extreme gui lt or
21 feel ings of worthlessness are symptoms of pathologic grief, which warrants psychiatric treatment.
Psychiatry Pathology
22

23 C is not correct. 14% chose this.


24 Brief aud itory and visua l hal lucinations directly related to a recently deceased loved one are symptoms of
25
norma l grief. However, if hal lucinations last for longer than 2 months, cause functional impairment, or involve
imagery or voices unre lated to the person that has died, the survivor may be experiencing complicated grief.
26 Hallucination Sound Auditory system Hearing
27
D is not correct. 6°/o chose this.
28
It is common for grieving persons to present with somatic complaints relating to the increased stress of losing a
29 loved one. Tension headaches are often one such manifestation.
30 Tension headache Somatic symptom disorder

31
F is not correct. 2°/o chose this.
32 Mild weight loss, most likely attributable to decreased appetite, is normal in the first 2 months after losing a
33 loved one.
34 Anorexia (symptom) Weight loss

o35
• 36 Bottom Line:
• 37
Normal grief includes den ial, gui lt, and somatic symptoms that typical ly last <2 months to 1 year. However,
• 38 grief shou ld not encompass su icida l thoughts, particularly if they are accompanied by a plan .
o39 Somatic symptom disorder

• 40

6
lock
s
Suspend
0
End Block
Item: 34 of 7 3 ~ 1 • M k -<:J 1>- Jil ~· !:';-~
QIO: 4633 ..L ar Pre v ious Next Lab fli!ltues Notes Calcula t o r
A A

20 FA17 p 532.2

21 Grief The fi ve stages of grief per the Kiibler-Ross model arc denial, anger, bargaining, depression, and
22 acceptance, not ncccssaril) in that order. Other normal grief S) mptoms include shock, guilt ,
sadness, anxiety, yearn ing, and somal ic S) mploms. Simple hallucinations of the deceased person
23
are common (eg, hearing the deceased speaking). Duration ,·aries widely; usually< 6 months.
24
Pathologic grief is persistent and causes functional impairment. Can meet criteria for major
25 depressi\'e episode.
26
27 FA17 p 531.4

28 Depression with Characterized by mood reacti\'ity (being able to experience impro,·ed mood in response to positive
29 atypical features events, albeit brieRy), "reversed" vegelali\e symptoms (hypersomnia, hyperphagia), leaden
30 paralysis (hea\'}' feeling in arms and legs), long-standing interpersonal rejection sensitivity. r-. lost
common subtype of depression. Treatment: C BT and SSRis are first line. lAO inhibitors are
31
effecti,·e but not first line because of their risk profile.
32
33
FA17 p 531.3
34
o35
Major depressive Episodes characterized by at least 5 of Diagnostic symptoms:
disorder the 9 diagnostic symptoms lasting :!: 2 SIC E CAP S:
• 36
weeks (symptoms must include patient- • Depressed mood
• 37
reported depressed mood or anhedonia). Sleep disturbance
• 38 Treatment: C BT and SSRis arc first line. Loss of Interest (anhedonia)
o39 S ' Rls, mirtazapine, bupropion can also be Guilt or feelings of worth lessness
• 40 • considered. ntidepressants are indicated if Energy loss and fatigue
a
Lock
s
Suspend
8
End Bl ock
Item: 34 of 7 3 ~ 1 • M k -<:J 1>- Jil ~· !:';-~
QIO: 4633 ..L ar Pre v ious Next Lab fli!ltues Notes Calcula t o r
--- ' - ' ·-
20
21 FA17 p 531 .3
22
Major depressive Episodes characterized by al least 5 of Diagnostic symptoms:
23 disorder the 9 diagnostic symptoms lasting ~ 2 SIC L CAPS:
24 weeks (symptoms must include patient- Depressed mood
25 reported depressed mood or anhedonia). Sleep disturbance
26
Treatment: CBT and SSRis arc first line. Loss of Interest (anhedonia)
S ' Ris, mirtazapine, bupropion can also be Guilt or feelings of worthlessness
27
considered. Antidepressants are indicated if Energy loss and fatigue
28 bipolar disorder is ruled oul. Eleclrocom ulsive Concentration problems
29 therapy (ECT) in select patients. \ ppetite/weight changes
30 Psychomotor retardation or agitation
Persistent depressive disorder (dysthymia)-
31
Suicidal ideations
depression, often mi lder, lasting at least
Patients with depression typically have the
32 2 years.
following changes in their sleep stages:
33 • ! slow-wave sleep
34 • ! REM latency
o35 • t REM early in sleep cycle
. 36 • t total REM sleep
Repeated nighttime awakenings
. 37
Early-morning awakening (terminal
. 38
insomnia)
o39
• 40 •
a
Lock
s
Suspend
8
End Bl ock
Item: 35 of 73 ~ 1 • M k -<:J 1>- Jil ~· !:';-~
QIO: 1464 ..L ar Pre v ious Next Lab fli!ltues Not es Calcula t o r

IAA]
A A

20 A 33-year-old woman with schizoaffective disorder presents to her physician with a 2-week history of thin,
21 wh ite discharge from her nipples. She reports that these symptoms began after she was prescribed a new
medication.
22
23
Which of the following agents is most likely responsible for the symptoms described?
24
:
25
A. Amitriptyline
26
27 B. Bromocriptine
28 C. Chlorpromazine
29
D. Fluoxetine
30
E. Selegiline
31
32
33
34
0 35
. 36
. 37
. 38
0 39
0 40 •
a
Lock
s
Suspend
8
End Bl ock
Item: 35 of 73 ~. I • M k <:] t> al ~· ~
QIO: 1464 .l. ar Previous Next lab 'lifllues Notes Calculator


20
The correct answer is C. 60°/o chose this.
21
Ch lorpromazine is a typical antipsychotic agent that blocks D2 dopamine receptors. In the pituitary, dopamine
22 prevents the release of prolactin. Therefore an agent that prevents dopamine from interacting with its receptors
23 in the pitu itary causes an uncontrolled re lease of pro lactin and leads to galactorrhea . Other examples of typical
24 dopamine receptor antagonists are thioridazine, haloperidol, and f luphenazine. Risperidone, an antipsychotic
agent, can also cause galactorrhea.
25 Fluphenazine Typical antipsychotic Risperidone Chlorpromazine Haloperidol Thioridazine Galactorrhea Prolactin Antipsychotic Dopamine receptor Dopamine
26
Dopamine antagonist Pituitary gland Receptor (biochemistry)
27
A is not correct. 7°/o chose this.
28
Amitriptyl ine is a tricycl ic antidepressant that prevents the reuptake of norepinephrine and serotonin from the
29
synaptic cleft. It has no effect on dopamine receptors and thus would not affect prolactin secretion .
30 Tricyclic antidepressant Amitriptyline Prolactin Serotonin Antidepressant Dopamine Norepinephrine Synaptic cleft Chemical synapse Reuptake Tricyclic Secretion

31
B is not correct. 18% chose this.
32
Bromocriptine, a dopamine ana log, acts as an agonist at pitu itary dopamine receptors and therefore prevents
33 prolactin release. This is an agent used to treat prolactin-secreting adenomas that often cause galactorr hea.
34 Bromocriptine Galactorrhea Prolactin Dopamine Agonist Structural analog Pituitary gland Adenoma Receptor (biochemistry) Dopamine receptor

35 D is not correct. 8°/o chose this.


• 36 Fluoxetine is a selective serotonin reuptake inh ibitor that is used to treat depression. This agent has no effect on
• 37 dopamine receptors and wou ld not affect the release of pro lactin .
Selective serotonin reuptake inhibitor Fluoxetine Prolactin Serotonin Dopamine Serotonin reuptake inhibitor Reuptake Reuptake inhibitor Major depressive disorder
• 38
Depression (mood) Enzyme inhibitor
o39
• 40 •

6
lock
s
Suspend
0
End Block
Item: 35 of 73 ~. I • M k <:] t> al ~· ~
QIO: 1464 .l. ar Previous Next lab 'lifllues Notes Calculator
. .

20
B is not correct. 18% chose this.
21
Brom ocriptine, a dopa mine analog, acts as an agonist at pituitary dopamine receptors and therefore prevents
22 prolactin release. This is an agent used to treat prolactin-secreting adenomas that often cause galactorr hea.
23 Bromocriptine Galactorrhea Prolactin Dopamine Agonist Structural analog Pituitary gland Adenoma Receptor (biochemistry) Dopamine receptor

24 D is not correct. 8°/o chose this.


25 Fluoxetine is a selective serotonin reuptake inh ibitor that is used to treat depression. This agent has no effect on
26 dopamine receptors and would not affect the release of prolactin.
Selective serotonin reuptake inhibitor Fluoxetine Prolactin Serotonin Dopamine Serotonin reuptake inhibitor Reuptake Reuptake inhibitor Major depressive disorder
27
Depression (mood) Enzyme inhibitor
28
29 E is not correct. 7°/o chose this.
30 Seleg iline, which acts as an inh ibitor of the mitochondrial enzyme, monoamine oxidase 8, results in elevated
dopamine levels in the synaptic cleft and as a result, this class of drug is often used to t reat Parkinson disease.
31
Thus, selegiline would prevent prolactin release and wou ld not cause galactorr hea as described in th is vignette .
32 Selegiline Prolactin Monoamine oxidase B Galactorrhea Dopamine Synaptic cleft Enzyme Chemical synapse Monoamine oxidase Parkinson' s disease

33 Monoamine neurotransmitter Mitochondrion Synapse Enzyme inhibitor


34
35
• 36
Bottom Line:
• 37 Dopamine inh ibits the release of a pituitary hormone cal led prolactin, which promotes milk production . Thus
neuroleptic agents, wh ich block dopamine receptors, pred ictably may cause galactor rhea as an adverse effect .
• 38 Galactorrhea Prolactin Dopamine Hormone Pituitary gland Antipsychotic Adverse effect Hypothalamic-pituitary hormone Milk
o39
• 40 •

6
lock
s
Suspend
0
End Block
Item: 35 of 73 ~ 1 • M k -<:J 1>- Jil ~· !:';-~
QIO: 1464 ..L ar Pre v ious Next Lab fli!ltues Not es Calcula t o r
A A

20
FA17 p543.1
21 Typical antipsychotics Haloperidol, pimozide, triAuopera,ine, Auphemu ine, thioridazine, chlorpromazin e.
22 MECHANISM Block dopamine 0 2 receptor (f c 1P).
23 Schizophrenia (1° positi,·e symptoms), psychosis, bipolar disorder, delirium, Tourette syndrome,
CliNICAl USE
24 Huntington disease, OCD.
25 POTENCY H igh potency: TriAuoperazine, fl uphena1ine, llaloperidol (Try to Fly H igh)- neurologic side
26 effects (eg, extrapyramidal S) mploms [F:PS)).
27 Lo'' potency: C hlorpromazine, T h ioridazine (C heating l'hieves are low)-anticholinergic,
antihistamine, a 1-blockade effects.
28
29 ADVERSEEFFECTS Lipid soluble .... stored in body fat .... slow to be removed from body.
30 Endocrine: dopamine receptor antagonism .... hyperprolactinemia .... galactorrhea,
31 oligomenorrhea, gyn ecomastia.
fetabolic: dyslipidemia, weight gain, hyperglycem ia.
32
Antimuscarinic: dry mouth, constipation.
33
Antihistamine: sedation.
34 a.-blockade: orthostatic hypotension.
35 Cardiac: QT prolongation.
. 36 Ophthalmologic: C hlorpromazine-C orneal deposits; T hioridazine-re'l'inal deposi ts.
. 37 EPS-ADAPT:
. 38 Hours to days: Acute D ystonia (muscle spasm, stiffness, oculogyric crisis).
o39 Days to months: \ kathisia (restlessness), Parkinsonism (bradykinesia).
.Vfonths to years: Tardive d)slinesia (orofacial chorea).
• 40 •
a
Lock
s
Suspend
8
End Bl ock
Item: 35 of 7 3 ~ 1 • M k -<:J 1>- Jil ~· !:';-~
QIO: 1464 ..L ar Pre v ious Next Lab fli!ltues Not es Calcula t o r
A A

20 FA17 p 543.2

21 Atypical Aripiprazole, asenapine, clozapine, olanzapine, quctiapinc, ilopcridone, palipcridonc,


22
antipsychotics rispcridone, lurasidone, ziprasidonc.
23 MECHANISM 1otcompletely understood. \lost arc D2
antagonists; aripiprazole is D2 partial agonist.
24
Varied effects on 5-I IT 2, dopamine, and
25
a- and H1-receptors.
26
CLINICAL USE Schizophrenia-both posithc and negati\'e Use clozapine for treatment-resistant
27 srmptoms. Also used for bipolar disorder, schizophrenia or schizoaffecti\'e disorder and
28 OCD, anxiety disorder, depression, mania, for suicidality in schizophrenia.
29 Tourette syndrome.
30 ADVERSE EFFECTS Al l-prolonged QT inter\'al, fewer EPS and
31 antichol inergic side effects than typical
antipsychotics.
32
"-pines" - metabolic syndrome (weight gain, Olanzapine .... O besity
33 diabetes, hyperlipidem ia).
34 C lozapinc-agranulocytosis (monitor WBCs Ivlust· watch bone marrow clozcly with ci<JJ:apine.
35 frequen tly) and seizures (dose related).
• 36 Risperidone-hyperprolactinemia (amenorrhea,
• 37
galactorrhea, gynecomastia).
• 38
FA17 p 545.4
o39
• 40 • Tricyclic Amitriptyline, nortriptyline, imipramine, desipramine, clomipramine, doxepin, amoxapine.
a
Lock
s
Suspend
8
End Bl ock
Item: 35 of 7 3 ~ 1 • M k -<:J 1>- Jil ~· !:';-~
QIO: 1464 ..L ar Pre v ious Next Lab fli!ltues Not es Calcula t o r

g }I
20 antipsychotics.
21 "-pines"- metabolic syndrome (weight gain, O lanzapine .... Obesity
22 diabetes, hyperlipidemia).
Clozapine-agranulocytosis (monitor WBC~ ~ l ust watch bone marrow clo7ely with clcuapine.
23
frequently) and seizures (dose related).
24
Risperidone- hyperprolactinem ia (amenorrhea,
25
galactorrhea, gynecomastia).
26
27 FA17 p 545.4
28
Tricyclic Amitriptyline, nortriptyline, imipramine, desipramine, clomipramine, doxcpin, amoxapine.
29 antidepressants
30 MECHANISM Inhibit NE and )-liT reuptake.
31
CLINICAL USE lajor depression, OCD (clomipramine), peripheral neuropathy, chronic pain, migraine
32 prophylaxis. octurnal enuresis (imipram ine, although adverse effects may limit use).
33 ADVERSE EFFECTS Sedation, a 1-blocking effects inclnding postural hypotension, and atropine-like (anticholinergic)
34 side effects (tachycardia, minary retention, dry mouth). 3° TCAs (amitriptyline) have more
35 antichol inergic effects than 2° TCAs (nortriptyline). Can prolong QT interval.
• 36 Tri-C's: Convulsions, Coma, C ardiotoxieity (arrhythmia due to 1 a+ channel inhibition);
also respiratory depression, hyperpyrexia. Confusion and hallucinations in elderly due to
• 37
anticholinergic side effects (nortriptyline beller tolerated in the elderly). Treatment: 1al JC03 to
• 38
prevent arrhythmia.
o39
• 40 •
a
Lock
s
Suspend
8
End Bl ock
Item: 36 of 73 ~ 1 • M k -<:J 1>- Jil ~· !:';-~
QIO: 250 7 ..L ar Pre v ious Next Lab fli!ltues Notes Calcula t o r

IAA]
A A

20 A man has felt depressed for severa l months. He is prescribed an antidepressant by his primary care
21 physician but continues to have difficulty sleeping and still does not enjoy his usual da ily activities. A friend
recommends that he try an over-the-counter supplement to help treat his persistent depression symptoms.
22
In the next few days, he beg ins experiencing fever, menta l status changes, and muscle rigidity. His core body
23 temperature is elevated to 40.1 °C (104°F).
24
25 Which of the following supplements did this man likely add to his antidepressant treatment?
26
:
27 A. Echinacea
28
B. Fish oil
29
C. Glucosamine and chondroitin
30
31 D. Melatonin
32 E. St. John's wort
33
34
35
. 36
. 37
. 38
o39
• 40 •
a
Lock
s
Suspend
8
End Bl ock
Item: 36 of 73 ~ 1 • M k -<:J 1>- Jil ~· !:';-~
QIO: 250 7 ..L ar Pre v ious Next Lab fli!ltues Notes Calcula t o r

IAA]
A A

20 A man has felt depressed for severa l months. He is prescribed an antidepressant by his primary care
21 physician but continues to have difficulty sleeping and still does not enjoy his usual da ily activities. A friend
recommends that he try an over-the-counter supplement to help treat his persistent depression symptoms.
22
In the next few days, he beg ins experiencing fever, menta l status changes, and muscle rigidity. His core body
23 temperature is elevated to 40.1 °C (104°F).
24
25 Which of the following supplements did this man likely add to his antidepressant treatment?
26
:
27 A. Echinacea
28
B. Fish oil
29
C. Glucosamine and chondroitin
30
31 D. Melatonin
32 E. St. John's wort
33
34
35
. 36
. 37
. 38
o39
• 40 •
a
Lock
s
Suspend
8
End Bl ock
Item: 36 of 73 ~. I • M k <:] t> al ~· ~
QIO: 2507 .l. ar Previous Next lab 'lifllues Notes Calculator


20 The correct answer is E. 83°/o chose this.
21 Hyperforin and other naphthodianth rones in St. John's wort {SJW) inhibit 5-HT reuptake. When used in
22 conjunction with selective seroton in reuptake inhibitors (SSRI ) like sertral ine, SJW can cause serotonin
synd rome . Thus, the major mechan ism by which SJW contributes to serotonin synd rome is via 5-HT reuptake
23
inhibition. Serotonin syndrome is potentially life-th reatening and is characterized by mental status changes,
24 autonomic changes (eg, fever, diarrhea, diaphoresis, tachycardia), and neuromuscular changes (eg,
25 hyperreflexia, tremor, rigidity).
26 Other drugs that can cause serotonin syndrome when used with SSRis include monoamine oxidase
27 inhibitors, meperidine, and cocaine.
28 Hyperforin Serotonin syndrome Sertraline Hyperreflexia Selective serotonin reuptake inhibitor Hypericum perforatum Perspiration Tachycardia Serotonin Diarrhea

29 Tremor Monoamine oxidase Pethidine Monoamine oxidase inhibitor Cocaine Reuptake inhibitor Monoamine neurotransmitter Reuptake Autonomic nervous system

30 Fever

31
A is not correct. 5°/o chose this.
32
Echinacea is used for prevention or treatment of viral respiratory illnesses. Patients taking Echinacea may
33 experience allergic reactions to the product, and there is a risk of exacerbation of autoimmune disease in
34 patients using Echinacea. There is no known risk of seroton in syndrome.
Serotonin syndrome Autoimmune disease Serotonin Allergy Virus
35
36 B is not correct. 2°/o chose this.
• 37 Fish oil has been shown to have a positive effect in the prevention of coronary artery disease and in the
• 38
treatment of hyperlipidem ia. There is not a r isk of serotonin syndrome in patients combining fish oil and anti-
depressants; however, patients may experience gastrointestinal side effects or changes in International
o39 Normalized Ratio with fish oil use .
• 40 •

6
lock
s
Suspend
0
End Block
Item: 36 of 73 ~. I • M k <:] t> al ~· ~
QIO: 2507 .l. ar Previous Next lab 'lifllues Notes Calculator


20 A is not correct. 5°/o chose this.
21 Echinacea is used for prevention or treatment of vi ral respi ratory illnesses. Patients ta king Ech inacea may
experience al lergic reactions to the product, and the re is a risk of exace rbation of autoimmune disease in
22
patients using Echinacea. There is no known risk of se roton in syndrome.
23 Serotonin syndrome Autoimmune disease Serotonin Allergy Virus
24
B is not correct. 2°/o chose this.
25
Fish oil has been shown to have a positive effect in the prevention of coronary artery disease and in the
26 treatment of hyperl ipidem ia. There is not a r is k of serotonin syndrome in patients combining fish oil and anti-
27 depressants; however, patients may experience gastrointestinal side effects or changes in International
28 Normalized Ratio with fish oi l use.
Serotonin syndrome Coronary artery disease Serotonin Hyperlipidemia Fish oil Antidepressant Coronary circulation Gastrointestinal tract Side effect Fish
29
Prothrombin time
30
31 C is not correct. 4°/o chose this.
32 Glucosamine and chond roitin have been used to treat osteoarth ritis th rough promotion of cartilage formation.
No increased risk of serotonin synd rome is associated with combining glucosamine and chondroitin and SSRi s,
33
although patients can expe rience interaction with anticoagu lant med ications .
34 Serotonin syndrome Anticoagulant Osteoarthritis Serotonin Glucosamine Selective serotonin reuptake inhibitor Cartilage

35
D is not correct. 6°/o chose this.
36
Melatonin is used as a sleep aid . Synthesized by the pineal g land, melatonin acts through effects on the
• 37 circadian sleep-wa ke cycle. Melatonin does not increase the risk of serotonin syndrome; however, it may
• 38 promote fee lings of depressed mood .
Serotonin syndrome Pineal gland Melatonin Serotonin Circadian rhythm Sleep aid Sedative Depression (mood) Gland
o39
• 40 •

6
lock
s
Suspend
0
End Block
Item: 36 of 73 ~. I • M k <:] t> al ~· ~
QIO: 2507 .l. ar Previous Next lab 'lifllues Notes Calculator


Normalized Ratio with fish oi l use.
20
Serotonin syndrome Coronary artery disease Serotonin Hyperlipidemia Fish oil Antidepressant Coronary circulation Gastrointestinal tract Side effect Fish
21
Prothrombin time
22
23 C is not correct. 4°/o chose this.
24
Glucosamine and chondroitin have been used to treat osteoarthritis th rough promotion of cartilage formation.
No increased risk of serotonin syndrome is associated with combining glucosamine and chondroitin and SSRi s,
25 although patients can experience interaction with anticoagu lant med ications .
26 Serotonin syndrome Anticoagulant Osteoarthritis Serotonin Glucosamine Selective serotonin reuptake inhibitor Cartilage

27
D is not correct. 6°/o chose this.
28 Melatonin is used as a sleep aid . Synthesized by the pineal g land, melatonin acts through effects on the
29 circadian sleep-wa ke cycle. Melatonin does not increase the risk of serotonin syndrome; however, it may
30 promote fee lings of depressed mood.
Serotonin syndrome Pineal gland Melatonin Serotonin Circadian rhythm Sleep aid Sedative Depression (mood) Gland
31
32
33 Bottom Line:
34 I n a patient taking drugs affecting serotonin, think serotonin syndrome if there is autonomic dysfunction along
35 with menta l status changes and neuromuscu lar hyperactivity. St. John's wort can cause herb-drug interactions
by inducing the CYP450 system . With SSRis, St. John's wort has an additive effect by further inhibition of 5-HT
36
reuptake .
• 37 Serotonin syndrome Serotonin Hypericum perforatum Cytochrome P450 Selective serotonin reuptake inhibitor Reuptake Autonomic nervous system Dysautonomia
• 38 Attention deficit hyperactivity disorder

o39
• 40 •

6
lock
s
Suspend
0
End Block
Item: 36 of 7 3 ~ 1 • M k -<:J 1>- Jil ~· !:';-~
QIO: 250 7 ..L ar Pre v ious Next Lab fli!ltues Notes Calcula t o r
A A

20
FA17 p 545.3
21
Serotonin syndrome Can occur with any drug that t 5-IIT (eg, 'v1A01s, SSRis, SNRis, TCAs, tramadol, ondansetron,
22 triptans, linezolid, .\ IDJ\ IA, dextromethorphan). Characterized by 3 A's: neuromuscular
23 h) perActivity (clonus, h~ perrcAexia, h) pertonia, tremor. seizure), Autonomic stimulation
24 (hyperthermia, diaphoresis, diarrhea), and \ gitation. Treatment: cyproheptadine (5-HT1 receptor
25 antagonist).
26
27 FA17 p 545.1

28 Selective serotonin Fluoxetine, Aumxamine, paro~etine, sert ral inc, cscitalopram, cita lopram.
29 reuptake inhibitors

30 MECHANISM Inhibit 5-HT reuptakc. It normally takes 4-8 weeks for antidepressants
31 CLINICAL USE Depression, generalized anxiety disorder, to have an effect.
32 panic disorder, OCD, bulimia, social anxiety
disorder, PTSD, premature ejaculation,
33
premenstrua I dysphoric disorder.
34
ADVERSE EFFECTS Fewer than TCAs. G I distress, SIADI I, sexual
35
dysfunction (anorgasmia, l libido).
36
• 37
FA17 p 546.1
• 38
o39
Monoamine oxidase Tranylcypromine, Phenelzine, l socarboxazid, Selegilinc (selecti,·e M.\ 0 -B inhibitor).
inhibitors (.\lAO Takes Pride In Shanghai).
• 40 •
a
Lock
s
Suspend
8
End Bl ock
Item: 36 of 7 3 ~ 1 • M k -<:J 1>- Jil ~· !:';-~
QIO: 250 7 ..L ar Pre v ious Next Lab fli!ltues Notes Calcula t o r
A
.. . . . A

20 MECHANISM Inhibit 5-HT re uptake. It normally takes 4-8 weeks for antide pressa nts
21 CLI NICAL USE De pression, generalized anxiety disorder, lo have an effect.
22 panic disorde r, O C D , bulimia, social anxiety
23 disorder, PTSD, premature ejaculation,
24
preme nstrual dysphoric d isorder.

25 ADVERSE EFFECTS Fe we r than T CAs. Cl distress, SI 0 11, sc:~.u a l


dysfunction (anorgasm ia, l libido).
26
27
28 FA17 p 546.1

29 Monoamine oxidase Tranylcypromine, P he nelzine, lsocarboxazid, Sclegiline (selecti,·e ~1AO -B inhibitor).


30 inhibitors ( ~ lAO Takes P ride I n Shanghai).

31 MECHANISM t'-:onselecti,·e .\!JAO inhibition t le \'cls of amine ncm otransmitte rs (norepinephrine, 5-HT,
32
dopamine).

33 CLINICAL USE Atypical depression, an xiety. Parkinson disease (sclcgiline).


34 ADVERSEEFFECTS C 1 S stimulation; hypertensive crisis, most notably with ingestion of tyramine, wh ich is found in
many foods such as aged cheese and wine. Tyramine displaces other neurotransmitters (cg, E)
35
into the synaptic cleft - t sympathetic stimulation. Contraindicated with SSRls, TCAs, St. John's
36
wort, meperidine, dextromethorphan (to preve nt serotonin syndrome) .
• 37 Wait 2 weeks afte r stopping .\!JAO inhibitors before starting serotone rgic drugs or stopping dietary
• 38 restrictions .
o39
•40 •
a
Lock
s
Suspend
8
End Bl ock
Item: 37 of 73 ~. I • M k <:] t> al ~· ~
QIO: 5171 .l. ar Previous Next lab 'lifllues Notes Calculator


20 A patient presents 4 weeks after giving birth to her fi rst chi ld. She admits to feeling "hope less" and "help less" I"' Aj
21 since the delivery. She has not slept well and has difficu lty concentrating, wh ich she attributes to frequent A
nighttime awakenings because she worr ies about the baby . She states that she has not been eating much
22
and has had a poor appetite, but wonders if it is because she has " been so busy with the baby ." She feels like she
23 "can't handle a chi ld right now" because she is so t ired. She sometimes feels angry at the baby and finds herself
24 bursting into tears "for no apparent reason ." She used to enjoy read ing, even up to the baby's birth, but f inds no
25
joy in th is presently . She assu res the physician that she is taking good ca re of the baby, but has recently asked
her mother to come stay with her for assistance. A review of her medical record reveals that she had an
26 uncomplicated spontaneous vaginal del ivery of a hea lthy baby boy weigh ing 7 lb 7 oz (3.37 kg) at 39 3/7 wee ks'
27 gestation. There was moderate loss of blood during the delivery. Her hemoglobin level fo llowing the delivery was
28 12.0 g/dl.
29
30
What is the most like ly diagnosis?
31 :

32 A. I ron-deficiency anemia
33 B. Maj or depressive disorder with peripartum onset
34
C. Postpartum blues
35
D . Postpartum psychosis
36
• 37 E. Sheehan syndrome
• 38
o39
• 40 •

6
lock
s
Suspend
0
End Block
Item: 37 of 73 ~. I • M k <:] t> al ~· ~
QIO: 5171 .l. ar Previous Next lab 'lifllues Notes Calculator

20 The correct answer is B. 51°/o chose this.


21 Majo r depressive disorder with peripartum onset is a cl inical diagnosis listed in the 5th edition of the Diagnostic
22 and Statistical Manual of Mental Disorders (DSM-5), also referred to as postpartum depression. I t is not listed as
23
a separate diagnosis from major depressive disorder, with the t iming of the symptoms being its only distinction .
Symptoms are identical to the DSM-5 symptoms for major depression, wh ich can be recal led using the
24 mnemonic SIGECAPS : Sleep disturbances, loss of Interest, Guilt, loss of Energy, difficulty with Concentration,
25 loss of Appetite, Psychomotor ag itation or retardation, and Su icidal ideation . Th is cond it ion occurs in 10% of
26 women, but can be difficu lt to diagnose because many symptoms can be confused with normal sequelae of labor
and del ivery. According to the DSM-5, major depressive disorder with peripartum onset is defined by the
27
occurrence of at least one major depressive episode du r ing pregnancy or with in 4 wee ks of de livery.
28 Major depressive disorder DSM-5 Diagnostic and Statistical Manual of Mental Disorders Postpartum depression Major depressive episode Depression (mood)

29 Postpartum period Mnemonic Pregnancy Childbirth Sequela


30
A is not correct. 1 °/o chose this.
31
Hemor rhage may cause iron-deficiency anemia, which may manifest with a pictu re simila r to that of major
32 depression. Psychiatric diseases no rma lly requi re that medica l conditions with overlapping symptomatology
33 have been excluded before the diagnosis is made. The patient's prominent psychiatric symptoms and no rmal
34 hemog lobin rule out iron-deficiency anemia as a cause of her il lness.
Iron-deficiency anemia Hemoglobin Bleeding Anemia Major depressive disorder Depression (mood) Mental disorder
35
36 C is not correct. 35% chose this.
37 Twenty-five pe rcent to 85% of women experience mino r mood fluctuations afte r del ivery. These "postpartum
blues" tend to peak around day 5 postde livery and resolve by day 10 . Persistence of symptoms beyond
• 38
postdel ivery day 10 wa rrants further investigation. Postpartum blues is a risk factor for the development of
o39 major depressive disorde r with peripartum onset.
• 40 Maior deoressive disorder

6
lock
s
Suspend
0
End Block
Item: 37 of 73 ~. I • M k <:] t> al ~· ~
QIO: 5171 .l. ar Previous Next lab 'lifllues Notes Calculator
•• •- :.:-:.: ,_:.: • u_ - • - :t:- •- • -, •- • • •- - ., - • • - e-:.:- •• - •
20 major depressive disorder with peripartum onset.
Major depressive disorder
21
22 D is not correct. 9°/o chose this.
23 Less than 0.5% of chi ldbearing women experience postpartum psychosis, usual ly with in 1 month of del ivery. It
24 is characterized by a manic- like episode of agitation, expansive or irritable mood, no sleep for severa l nights,
and avoidance of the infant. Delusions and hal lucinations are present and often involve the baby (eg, command
25
hallucinations to harm the infant).
26 Postpartum psychosis Psychosis Postpartum period Hallucination

27
E is not correct. 4°/o chose this.
28
The most common presentation of postpartum hypopituitarism (Sheehan syndrome) is failure to lactate, and it
29 is caused by severe puerpera l hemorrhage. Eventual ly the patient experiences the symptoms of hypothyroidism
30 (fatigue, constipation, and nonresumption of menses), and hypoadrenalism (hyponatremia and hyperka lemia
due to decreased aldosterone, and loss of pubic and axillary hai r because androgens in women are produced in
31
the adrena l cortex) .
32 Sheehan' s syndrome Hyperkalemia Hypopituitarism Hypothyroidism Hyponatremia Aldosterone Adrenal cortex Adrenal insufficiency Constipation Bleeding
33 Fatigue (medical) Androgen Postpartum period lactic acid Menstruation Adrenal gland
34
35
Bottom Line:
36
37 Major depressive disorder with peripartum onset is a psychiatric illness distinguished from norma l postpartum
blues (" baby blues") by its severity and duration. Symptoms of th is cond it ion are identical to the DSM-V
• 38
symptoms for diagnosis of major depression.
o39 Major depressive disorder Mental disorder Childbirth Depression (mood) DSM-5 Maternity blues Psychiatry Blues

• 40 •

6
lock
s
Suspend
0
End Block
Item: 37 of 73 ~ 1 • M k -<:J 1>- Jil ~· !:';-~
QIO: 5171 ..L ar Pre v ious Next Labfli!llues Notes Calcula t o r
A A

20 FA17p532.1
21 Postpartum mood O nset within 4 weeks of de]i,·cry.
22 disturbances
23 Maternal 50-85% incidence rate. Characteri1ed by depressed affect, tearfulness, and fati gue starting 2-3
24 (postpartum) "blues" days after del ivery. Usuall} resoh es \\ it hin 10 da)S. Treatment: supportive. Follow up to assess for
25 possible postpartum depression.
26 Postpartum 10- 15% incidence rate. Charactcri1cd b) depressed affect, anxiety, and poor concentration for ~ 2
27
depression weeks. Treatment: CBT and SSRis arc first line.
28 Postpartum psychosis 0.1-0.2% incidence rate. Characterized by mood-congruent delusions, hallucinations, and
thoughts of harming the bab) or self. Risl.. factors include history of bipolar or psychotic disorder,
29
first pregnane) , famil) histOr), recent discontinuation of psychotropic medication. Treatment:
30
hospitalization and initiation of atypical antipsychotic; if insufficient, ECT may be used.
31
32
FA17 p 531 .3
33
Major depressive Episodes characterized by at least 5 of D i<~g nosti c
symptoms:
34
disorder the 9 diagnostic symptoms lasting :!: 2 SIC E CAPS:
35 weeks (symptoms must include patient·- • Depressed mood
36 reported depressed mood or anhedon ia). ~ Sleep disturbance
37 Treatment: CBT and SSRis arc first line. Loss of Interest (anhedonia)
. 38 S ' Ris, mirtazapine, bupropion can also be Guilt or feelings of worthlessness
considered. ntidepressants are indicated if Energy loss and fatigue
o39
bipolar disorder is ruled out. Electrocom ulsi\ e C oncentration problems
• 40 •
a
Lock
s
Suspend
8
End Bl ock
Item: 37 of 7 3 ~ 1 • M k -<:J 1>- Jil ~· !:';-~
QIO: 5171 ..L ar Pre v ious Next Labfli!llues Notes Calcula t o r
f • I f f 1: f • . .. , • •

20
21
FA17 p 531 .3
22
Major depressive Episodes characterized by al least 5 of Diagnostic symptoms:
23 disorder the 9 diagnostic symptoms lasting <!: 2 SIC L CAPS:
24 weeks (spnptoms must include patient- Depressed mood
25 reported depressed mood or anhedonia). Sleep disturbance
26 Treatment: CBT and SSRis arc first line. Loss of Interest (anhedonia)
Si':Ris, mirtazapine, bupropion can also be Guilt or feelings of worthlessness
27
considered. ntidepressants are indicated if Energy loss and fatigue
28 bipolar disorder is ruled out. Eleclrocom ulsive Concentration problems
29 therapy (ECT) in select patients. \ ppetite/weight changes
30 Psychomotor retardation or agitation
Persistent depressive disorder (dysthymia)-
31 Suicidal ideations
depression, often mi lder, lasting at least
Patients with depression typically have the
32 2 years.
following changes in their sleep stages:
33 • ~ slow-wave sleep
34 • ~ REM latency
35 • t REM early in sleep cycle
36 • t total REM sleep
Repeated nighttime awakenings
37
Early-morning awakening (terminal
. 38
insomnia)
o39
• 40 •
a
Lock
s
Suspend
8
End Bl ock
Item: 38 of 73 ~ 1 • M k -<:J 1>- Jil ~· !:';-~
QIO: 1455 ..L ar Pre v ious Next Lab fli!ltues Not es Calcula t o r

IAA]
A A

20 A 28-year-old man who has been experiencing delusions, hallucinations, f lattened affect, social withd rawal,
21 and thought disorders for the past 6 months is started on a medication to help control these symptoms. This
medication requires weekly blood work to monitor for a potentially serious side effect.
22
23
What is the name of this medication and this side effect?
24
:
25
A. Clozapine, agranulocytosis
26
27 B. Fluphenazine, agranulocytosis
28 C. Haloperidol, galactorrhea
29
D. Olanzapine, weight gain
30
E. Risperidone, galactorrhea
31
32
33
34
35
36
37
. 38
o39
• 40 •
a
Lock
s
Suspend
8
End Bl ock
Item: 38 of 73 ~. I • M k <:] t> al ~· ~
QIO: 1455 .l. ar Previous Next lab 'lifllues Notes Calculator


20 The correct answer is A. 87°/o chose this.
21 Clozapine is an atypical antipsychotic that blocks both serotonin and dopamine receptors . Drugs in this class are
noted for their ability to treat symptoms of schizoph renia without causing as many extrapyramida l side effects
22
as typical antipsychotics. Clozapine has the potentia l to cause ag ranulocytosis, which leads to seve re reduction
23 in WBC production. Patients should therefore receive weekly blood wo rk to monitor their WBC count. Clozapine
24 may also cause weight gain, seizu res, and QT pro longation.
Atypical antipsychotic Agranulocytosis Clozapine Serotonin Schizophrenia Antipsychotic Dopamine long QT syndrome Extrapyramidal symptoms Epileptic seizure
25
Typical antipsychotic Extrapyramidal system Blood test Weight gain Adverse drug reaction Dopamine receptor Receptor (biochemistry) Side effect
26
27 B is not correct. 5°/o chose this.
28 Fluphenazine is a typical antipsychotic that blocks on ly dopamine recepto rs and can be used to t reat
29 schizoph renia . Ag ranulocytosis is not a side effect of typical antipsychotics. Typical antipsychotics display more
extrapyramida l side eff ects, may cause hyperprolactinemia and ga lactorrhea, anti-musca r inic effects (dry
30
mouth, etc.) and QT prolongation.
31 Fluphenazine Typical antipsychotic Galactorrhea Agranulocytosis Muscarinic antagonist Hyperprolactinaemia Dopamine Schizophrenia long QT syndrome

32 Antipsychotic Extrapyramidal symptoms Xerostomia Side effect Adverse effect Receptor (biochemistry) Extrapyramidal system Dopamine receptor

33 Adverse drug reaction

34
C is not correct. 5°/o chose this.
35
Haloperidol is a typical antipsychotic that can be used to treat schizoph renia; however, it does not lead to
36 agranulocytosis. Like other typica l antipsychotics, ha loperidol can cause extrapyramidal side effects,
37 hyperprolactinemia and ga lactorrhea, antimuscar inic effects (dry mouth, etc.) and QT pro longation.
Typical antipsychotic Muscarinic antagonist Haloperidol Galactorrhea Agranulocytosis Hyperprolactinaemia Schizophrenia Antipsychotic long QT syndrome
38
Extrapyramidal symptoms Xerostomia Extrapyramidal system Adverse drug reaction Side effect
o39
0 40 •

6
lock
s
Suspend
0
End Block
Item: 38 of 73 ~. I • M k <:] t> al ~· ~
QIO: 1455 .l. ar Previous Next lab 'lifllues Notes Calculator
- -- -- - - - - - - - - - - - - - - -- - - -- -- -
20 agranulocytosis. Like other typica l antipsychotics, ha loperidol can cause extrapyramidal side effects,
hyperprolactinemia and ga lactorrhea, antimuscar inic effects (dry mouth, etc.) and QT pro longation.
21
Typical antipsychotic Muscarinic antagonist Haloperidol Galactorrhea Agranulocytosis Hyperprolactinaemia Schizophrenia Antipsychotic long QT syndrome
22
Extrapyramidal symptoms Xerostomia Extrapyramidal system Adverse drug reaction Side effect
23
D is not correct. 2°/o chose this.
24
Olanzapine, like clozapine, is an atypica l antipsychotic that could be used to t reat this patient's symptoms.
25
Un like clozapine, ag ranu locytosis is not a side effect of olanzapine. Olanzapine does often cause weight gain as
26 a side effect.
27 Atypical antipsychotic Olanzapine Agranulocytosis Clozapine Antipsychotic Adverse effect Side effect Weight gain

28 E is not correct. 1°/o chose this.


29 Risperidone, lik clozapine, is an atypical antipsychotic that cou ld be used to treat th is patient's symptoms.
30 Un like clozapine, ag ranu locytosis is not a side effect of risperidone. Risperidone, un like the other atypical
31
antipsychotics, may cause galactorrhea due to an increased prolactin and can also cause problems with ferti lity
due to dysregu lation of LH and FS H levels.
32 Atypical antipsychotic Risperidone Galactorrhea Agranulocytosis Clozapine Prolactin Antipsychotic Adverse effect Side effect luteinizing hormone
33
34
Bottom Line:
35
36 Atypical antipsychotics such as clozapine are used in the t reatment of schizoph renia and general ly have a more
favorable side effect profile than typical antipsychotics. However, clozapine has the potential to cause
37
agranulocytosis, wh ich requires weekly blood work to monitor patient's WBC counts.
38 Agranulocytosis Clozapine Schizophrenia Antipsychotic Adverse effect Typical antipsychotic Atypical antipsychotic Side effect Blood test

o39
0 40 •

6
lock
s
Suspend
0
End Block
Item: 38 of 7 3 ~ 1 • M k -<:J 1>- Jil ~· !:';-~
QIO: 1455 ..L ar Pre v ious Next Lab fli!ltues Not es Calcula t o r
A A

20
FA17 p 543.2
21
Atypical Aripiprazole, asenapinc, clozapinc, olanzapinc, quctiapinc, ilopcridonc, pali pcridonc,
22
antipsychotics risperidone, lurasidonc, ziprasidonc.
23
MECHANISM 1 ot completely understood. \lost arc 0 1
24 antagonists; aripiprazole is 0 1 partial <•gonist.
25 Varied effects on 5-HT 2, dopamine, and
26 a - and l-1 1-rcceptors.
27 CLINICAL USE chizophrenia-both positi\'e and negative Use clozapine for treatment-resistant
28 symptoms. Also used for bipolar disorder, schizophrenia or schizoaffective disorder and
OCD, anxiety disorder, depression, mania, for suicidality in schizophrenia.
29
Tourette syndrome.
30
ADVERSE EFFECTS All-prolonged QT internd, fewer EPS and
31
antichol inergic side effects than trpical
32 anti psychotics.
33 "-pines"-metabolic syndrome (weight gain, O lanzapinc -+ O besity
34 diabetes, hyperlipidem ia).
C lozapine-agranulocytosis (mon itor W BCs · 1ust· watch bone marrow clozely with clozapinc.
35
frequently) and seizures (dose related).
36
Risperidone-hyperprolactinemia (amenorrhea,
37 galactorrhea, gynecomastia).
38
o3 9 FA17 p 543.1
• 40 • u .rot ............... - :...1 . . . 1 ..... :.- ...... _; ......... ._ .. :o................. . ... :_ ... o ...,.,t, ........ .. ... :_ .... a.t... : ...... :..l ..... .. : ........ ..... t..l ........-. ....... _... ....... :- ....

a
Lock
s
Suspend
8
End Bl ock
Item: 38 of 7 3 ~ 1 • M k -<:J 1>- Jil ~· !:';-~
QIO: 1455 ..L ar Pre v ious Next Lab fli!ltues Not es Calcula t o r
A A

FA17 p 543.1
20
Typical antipsychotics Haloperidol, pimozide, triAuopera:tin e, Auphena:tine, thioridazine, chlorpromazin e.
21
MECHANISM Block dopamine 0 2 receptor (f e 1 P).
22
23 CliNICAL USE Schizophrenia (1° positive symptoms), psychosis, bipolar disorder, delirium, Tourette syndrome,
Huntington disease, OCD.
24
POTENCY High potency: Trifluoperazine, fluphena7ine, llaloperidol (Try to Fly H igh)- neurologic side
25
effects (eg, extrapyramidal S) mploms [F:PS]).
26
Lo'' potency: C hlorpromazine, T h ioridazine (C heating l'hieves are low)-anticholinergic,
27 antihistamine, a 1-blockade effects.
28 ADVERSEEFFECTS Lipid soluble - stored in body fat - slow to be removed from body.
29
Endocrine: dopamine receptor antagonism - hyperprolactinemia ..... galactorrhea,
30
oligomenorrhea, gynecomastia.
31 fetabolic: dyslipidemia, weight gain, hyperglycem ia.
32 Antimuscarinic: dry mouth, constipation.
33 Antihistamine: sedation.
a.-blockade: orthostatic hypotension.
34
Cardiac: QT prolongation.
35
Ophthalmologic: C hlorpromazine-C orneal deposits; T hioridazine-reT inal deposi ts.
36
EPS-ADAPT:
37
Hours to days: Acute D ystonia (muscle spasm , stiffness, oculogyric crisis).
38 Days to months: \ kathisia (restlessness), Parkinsonism (bradykinesia).
o39 .\lfonths to years: Tardive d)sl.inesia (orofacia l chorea).
• 40 • Trf':~tmf'nt: hf'nztroninf' (:ll'lltf' th ~tnni:~ t:~rdivf' d, ~kinf'si:~) hf'nzndi:~zf'ninf'_~ _ R-hlnl'kf'rs

a
Lock
s
Suspend
8
End Bl ock
Item: 38 of 7 3 ~ 1 • Ma rk -<:J 1>- Jil ~· !:';-~
QIO: 1455 ..L Prev ious Next Lab fli!ltues Not es Cal culat o r


20
FA17p530.1
21
Schizophrenia Chronic mental disorder with periods of Frequent cannabis usc is associated" ith
22
psychosis, disturbed beha' ior and I hough!, ps}chosis/schizophrenia in teens.
23 and decline in functioning lasting> 6 Lifetime pre,·alence-1.5% (males= females ,
24 months. Associated" ith t dopaminergic African Americans= Caucasians). Presents
25 acti,·ity, l dendritic branching. earlier in men (late teens to early 20s ' s late
Diagnosis requires at least 2 of the following, 20s to early 30s in women). Patients arc at t
26
and at least I of these should include 1-3 risk for suicide.
27
(first-+ are "positive symptoms"): Ventriculomegaly on brain imaging.
28 Treatment: atypical antipsychotics (eg,
I. Delusions
29 2. llallucinations-often auditory risperidone) are first line.
30 3. Disorganized speech Negative symptoms often persist after treatment,
4. Disorgan ized or catatonic behavior despite resolution of positi,·e symptoms.
31
32
5. legati,·c symptoms (affect ive Aattcn ing,
avolition, anhedonia, asociality, alogia)
33
Brief psychotic disorder- lasting< l month,
34
usually stress related.
35
Schizophreniform disorder- lasting l-6
36
months.
37
Schizoaffective disorder- \llects criteria for
38
schizophrenia in addition to major mood
o3 9 disorder (major depressive or bipolar). 'lo
• 40 • rliU'earPnti!lh~ frnn'l !I 1n!l1nr rnnnrl rl;cnrrl~r

a
Lock
s
Suspend
8
End Bl ock
Item: 39 of 73 ~. I • M k <:] t> al ~· ~
QIO: 1132 .l. ar Previous Next Lab 'lifllues Notes Calculator


20 A 45-year-old man presents to a psychiatrist at his wife's prompting . He is a col lege professor and regu larly ~~AI
21 wins accolades for his well-organ ized and articu late lectures . In the past 3 months, he has become convinced
that his wife is having an affair with her coworker, despite her protests to the contrary . His wife recently
22
discovered that he hired a private investigator to track her whereabouts. He is very defensive when the counselor
23 questions his suspicions about his wife. Mental status exam ination reveals a wel l-dressed, middle-aged man
24 without ha llucinations or other mood disturbances. His speech is norma l and displays an appropriate affective
25
range. He states that he has a great relationship with his supervisor and enjoys spend ing t ime with his neighbors
on the weekends.
26
27
Wh ich of the fol lowing is the most likely diagnosis?
28
:
29
A. Antisocial persona lity disorder
30
31
B. Delusional disorder
32 C. Paranoid persona lity disorder
33 D . Schizoid personal ity disorder
34
E. Schizophren ia
35
36 F. Schizophren iform disorder
37
38
0 39
0 40 •

6
lock
s
Suspend
0
End Block
Item: 39 of 73 ~. I • M k <:] t> al ~· ~
QIO: 1132 .l. ar Previous Next Lab 'lifllues Notes Calculator

• The correct answer is B. 54°/o chose this .


20
Delusiona l disorder is diagnosed fo llowing 1 month of nonbizarre de lusions that are usually focused around a
21 particular topic, in this case, the fidelity of the patient's wife. The de lusions are not attributable to another
22 psychiatric disorder such as schizophrenia. Delusional disorder does not markedly impair the person's
23
function ing in dai ly activities . The ramifications are limited to the delusional content.
Delusional disorder Schizophrenia Mental disorder
24
25 A is not correct. 1 °/o chose this.
26 Cl uster 8 persona lity disorders include antisocial , borde rline, histr ionic, and narcissistic types. Patients with
antisocial pe rsona lity disorder show a disrega rd for and often violate the rights of othe rs. These ind ividuals often
27
have a cr iminal histo ry. This is the only personality disorde r with an age limit (>18 yea rs). Mino rs with simila r
28 behavior are classified as having conduct disorder.
29 Antisocial personality disorder Conduct disorder Cluster B personality disorders Personality disorder Histrionic personality disorder Narcissistic personality disorder

30 Narcissism Anti -social behaviour

31 C is not correct. 35% chose this.


32 Paranoid personal ity diso rder is a cluster A persona lity disorde r. These individuals have pervasive, long -standing
33 distrust of others. This patient has a specific delusion about his wife and his distrust is not pervasive. Other
34
cl uste r A pe rsonality disorders are schizoid and sch izotypal persona lity disorde rs. I n the for mer, individuals are
content with social isolation and voluntarily withdraw f rom socia l interactions. Individua ls with sch izotypal
35 pe rsona lity disorde rs usua lly have eccentric appearances and they are fixated on odd bel iefs and mag ical
36 thinking.
Paranoid personality disorder Magical thinking Schizotypal personality disorder Personality disorder Delusion Schizoid personality disorder Social isolation Paranoia
37
38 D is not correct. 4°/o chose this.
39 Schizoid persona lity disorde r is a cluster A personal ity diso rder. Patients with sch izoid personal ity diso rder
0 40 • exh ibit voluntarv socia l withd rawal (un like avoidant oersona litv disorde r oatients) and have limited emotiona l •
6
lock
s
Suspend
0
End Block
Item: 39 of 73 ~. I • M k <:] t> al ~· ~
QIO: 1132 .l. ar Previous Next Lab 'lifllues Notes Calculator

• content with social isolation and voluntarily withdraw from socia l interactions. Individua ls with sch izotypal
20
persona lity disorders usua lly have eccentric appearances and they are fixated on odd bel iefs and mag ical
21 thinking.
22 Paranoid personality disorder Magical thinking Schizotypal personality disorder Personality disorder Delusion Schizoid personality disorder Social isolation Paranoia

23 D is not correct. 4°/o chose this.


24 Schizoid persona lity disorder is a cluster A personal ity disorder. Patients with sch izoid personal ity disorder
25 exh ibit voluntary socia l withdrawal (un like avoidant persona lity disorder patients) and have limited emotiona l
26
expressions . They lack strange beliefs and thoughts characteristic of schizotypal personal ity disorder.
Schizoid personality disorder Avoidant personality disorder Schizotypal personality disorder Personality disorder Solitude
27
28 E is not correct. 2°/o chose this.
29
Schizophrenia is a chronic psychiatric cond ition. It affects 1% of the population and usual ly begins before 25
years of age . The diagnosis requires at least two of the fo llowing symptoms to be present for 6 months,
30 including at least 1 month of active symptoms : de lusions, hal lucinations, disorganized speech, grossly
31 disorganized or cataton ic behavior, and negative symptoms ( eg, f lat affect, lack of motivation, or poverty of
32 speech) .
Schizophrenia Alogia Catatonia Reduced affect display Negative symptoms Hallucination Psychiatry Mental disorder
33
34 F is not correct. 4°/o chose this.
35 Schizophreniform disorder is simi lar to sch izophrenia except that its symptoms have lasted between 1 and 6
months. In contrast, patients with schizophrenia must have had symptoms for longer than 6 months.
36
Schizophreniform disorder is different from schizotypa l persona lity disorder. Individuals with sch izotypal
37 persona lity disorders usua lly have eccentric appearances and they are fixated on odd bel iefs and mag ical
38 thinking. Individuals with schizotypal persona lity disorde rs, howeve r, do not experience symptoms of
39
schizophrenia .
Schizotypal personality disorder Schizophreniform disorder Schizophrenia Magical thinking Personality disorder
0 40 •

6
lock
s
Suspend
0
End Block
Item: 39 of 73 ~. I • M k <:] t> al ~· ~
QIO: 1132 .l. ar Previous Next Lab 'lifllues Notes Calculator

20
E is not correct. 2°/o chose this.
21
Schizoph renia is a chronic psychiatric cond ition. I t affects 1% of the population and usual ly begins before 25
22 years of age . The diagnosis requires at least two of the fo llowing symptoms to be present for 6 months,
23 including at least 1 month of active symptoms : de lusions, hal lucinations, disorganized speech, grossly
24
disorganized or cataton ic behavior, and negative symptoms ( eg, f lat affect, lack of motivation, or poverty of
speech) .
25 Schizophrenia Alogia Catatonia Reduced affect display Negative symptoms Hallucination Psychiatry Mental disorder
26
F is not correct. 4°/o chose this.
27
Schizoph reniform disorder is simi lar to sch izophrenia except that its symptoms have lasted between 1 and 6
28 months. I n contrast, patients with schizophrenia must have had symptoms for longer than 6 months.
29 Schizoph reniform disorder is different from schizotypa l persona lity disorder. I ndividuals with sch izotypal
30 persona lity disorders usua lly have eccentric appearances and they are fixated on odd bel iefs and mag ical
thinking. I ndividuals with schizotypal persona lity disorders, however, do not exper ience symptoms of
31
schizoph renia .
32 Schizotypal personality disorder Schizophreniform disorder Schizophrenia Magical thinking Personality disorder

33
34
Bottom Line:
35
Delusiona l disorder is diagnosed fo llowing 1 month of nonbizarre de lusions that are focused around a particu lar
36
topic. I mportantly, delusional disorder does not markedly impair the person's function ing in daily activities,
37 and the ramifications are limited to the delusiona l content.
38 Delusional disorder

39
0 40 •

6
lock
s
Suspend
0
End Block
Item: 39 of 7 3 ~ 1 • M k -<:J 1>- Jil ~· !:';-~
QIO: 1132 ..L ar Pre v ious Next Labfli!llues Not es Calcula t o r
A A

20 FA17 p 530.2

21 Delusional disorder Fixed, persistent, false belief S)Stem lasting > 1 month. Functioning otherwise not impaired
22 (eg, a woman who genuinely believes she is married to a celebrity when, in fact, she is not).
23 Can be shared by individuals in close relationships (folie a deux).
24
FA17 p 535.1
25
26 Cluster A personality Odd or eccentric; inability to dc,cJop "Weird:·
disorders meaningful social relationships. 'o pS)Chosi~; Cluster A: .\ ccusatory, .\ loof, .\ \\ kward.
27
genetic association with schizophrenia.
28
Paranoid Per\"asive distrust (,\ ccusatory) and
29
suspiciousness of others and a profound I)
30 cynical view of the world.
31 Schizoid Voluntary social withdrmval (Aloof), limited
32 emotional expression, content with socia I
33 isolation (vs avoidant).
34 Schizot ypal Eccentric appearance, odd beliefs or magica l Schizotypal =magical thinking.
35 thinking, interpersonal Awkwardness.
36
37 FA17 p 535.2

38 Cluster B personality Dramatic, emotional, or erratic; genetic "\\'i ld."


39 disorders association with mood disorders and substance Cluster B: Bad, Borderline, AamBoyant, must
• 40 • abuse. be the Best

a
Lock
s
Suspend
8
End Bl ock
Item: 39 of 7 3 ~ 1 • M k -<:J 1>- Jil ~· !:';-~
QIO: 1132 ..L ar Pre v ious Next Labfli!llues Not es Calcula t o r
g, p
20
21
FA17 p 535.2
22
Cluster B personality Dramatic, emotional, or erratic; genetic "\\"ild."
23 disorders association with mood disorders and substance Cluster B: Bad, Borderline, AamBoyant, must
24 abuse. be the Best
25 Antisocial Disregard for and violation of rights of others Anti~ocial =~ociopath.
26 with lack of remorse, criminal it}', impulsivitr; Bad.
27 males> females; must be~ 18 years old and
28
ha,·e history of conduct disorder before age 15.
Conduct disorder if< 18 years old.
29
Borderline Unstable mood and interpersonal relationships, Treatment: dialectical beha\'ior therapy.
30
impulsivity, self-mutilation, suicidality, sense Borderline.
31
of emptiness; females> males; splitting is a
32 major defense mechanism.
33 Histrionic Excessive emotionality and excitability, FlamBoyant.
34 <tttcntion seeking, sexually provocative, overly
35 concerned with appearance.
36 Narcissistic Grandiosity, sense of entitlement; lacks empathy 1ust be the Best.
37
and requires excessive admiration; often
demands the "best" and reacts to criticism
38
with rage.
39
• 40 •
a
Lock
s
Suspend
8
End Bl ock
Item:40of73 ~. , . M k <:] t> al ~· ~
QIO: 1125 .l. ar Previous Next Lab 'lifllues Notes Calculator


20 A 35-year-old woman is brought to the physician by her husband for strange behavior. The husband exp lains I"' Aj
21 that approximate ly 3 weeks ago his wife stopped showing up for wor k, spent almost ha lf of their savings on A
va rious luxury items, and has been very irr itable when questioned about these new behaviors. He also
22
reports that she is no longer sleeping th rough the night and is claiming to be the "most intelligent woman in the
23 world ." Review of systems is posit ive for occasiona l headaches and nausea. She den ies aud itory and visual
24 ha llucinations, as well as alcohol and drug use. Her med ical history is negative for major depressive episodes. In
25
the office, the woman is speaking rapidly, is bragg ing about her most recent accomplishments, and becomes
visibly annoyed when the examiner tries to redi rect her.
26
27
Wh ich of the fo llowing is the most like ly diagnosis?
28
:
29
A. Amphetamine intoxication
30
31
B. Bipo lar I
32 C. Bipo lar II
33 D . Cyclothymia
34
E. Hyperthyroidism
35
36 F. Narcissistic persona lity disorde r
37 G. Schizoaffective disorder
38
39
0 40 •

6
lock
s
Suspend
0
End Block
Item:40of73 ~. , . M k <:] t> al ~· ~
QIO: 1125 .l. ar Previous Next Lab 'lifllues Notes Calculator


20 The correct answer is B. 55°/o chose this.
21 The manic phase of bipolar I disorder is characterized by an elevated, expansive, or irr itable mood for more
than 1 week that causes ma r ked impairment in functioning social ly, at work, or at home. Three or more of the
22
following symptoms must be present (presented according to the mnemonic DIG FAST : Distractibility,
23 Irresponsibi lity (increase in pleasurable activities without regard for consequences), Grand iosity/inflated self-
24 esteem, Fl ight of ideas/racing thoughts, increased goa l-di rected Activity/psychomotor ag itation,decreased need
25
for Sleep, and Talkativeness/increased rate of speech (pressu red speech). Bipo lar I disorder can be diagnosed
after a sing le manic episode. Treatment options for bipolar diso rder include haloperidol (Haldol ) or atypical
26 antipsychotics for acute man ia, with the add ition of benzodiazepines if needed fo r severe ag itation. Mood
27 stabilizers such as lithium have been shown to decrease the frequency/seve rity of mood swings.
Haloperidol Bipolar disorder Bipolar I disorder Benzodiazepine Pressure of speech Antipsychotic Mania Mood stabilizer Atypical antipsychotic Manic episode
28
29 Self-esteem Mnemonic lithium Mood swing

30 A is not correct. 4°/o chose this.


31 Amphetamine intoxication can manifest with mania-l ike symptoms, but the duration of th is patient's symptoms
32 r ules out d rug intoxication. Routine drug screen ing wou ld also confirm such findings. Of importance is that the re
is no evidence of attention deficit/hyperactivity disorder, which is t reated with amphetam ines, nor is there
33
evidence of substance abuse.
34 Amphetamine Substituted amphetamine Attention deficit hyperactivity disorder Substance abuse Substance intoxication Drug test Drug overdose

35 Alcohol intoxication
36
C is not correct. 13% chose this.
37
Bipo lar II disorder requ ires a history of at least one major depressive episode and at least one hypomanic
38 episode without the presence of mixed or manic episodes. The presence of a manic or mixed episode makes the
39 diagnosis bipolar I. Note that bipolar I can be diagnosed after a single manic episode without a histo ry of
40 • depressive episodes (thouah these are often present), but a bipolar II diaanosis reaui res a histo rv of both a •
6
lock
s
Suspend
0
End Block
Item:40of73 ~. , . M k <:] t> al ~· ~
QIO: 1125 .l. ar Previous Next Lab 'lifllues Notes Calculator


20 C is not correct. 13% chose this.
21 Bipo lar II disorder requ ires a history of at least one major depressive episode and at least one hypomanic
episode without the presence of mixed or manic episodes. The presence of a manic or mixed episode makes the
22
diagnosis bipolar I. Note that bipolar I can be diagnosed after a single manic episode without a history of
23 depressive episodes (though these are often present), but a bipolar II diagnosis requi res a history of both a
24 major depressive episode and a hypomanic episode. Hypoman ic episodes are less severe and disruptive than
25 manic symptoms (described above). Additional ly hypomania is diagnosed if symptoms are present for at least 4
days whereas the diagnosis of mania requires symptoms to be present for at least 7 days.
26 Mixed affective state Bipolar II disorder Hypomania Major depressive episode Mania Bipolar I disorder Manic episode Major depressive disorder
27
D is not correct. 6°/o chose this.
28
Cyclothymia is characterized by multiple episodes of hypomanic symptoms and depressive symptoms occuring
29
for at least 2 years . Additionally, these symptoms cannot meet the criteria for hypomania, mania or major
30 depression. The patient has experienced a man ic episode therefore this is not an appropr iate answer.
31 Cyclothymia Hypomania Major depressive disorder Mania Manic episode Depression (mood)

32 E is not correct. 3°/o chose this.


33 Hyperthyroidism can present with agitation and decreased sleep, but patients with this disorder are unli kely to
34 exh ibit other symptoms of mania, including grand iosity, flight of ideas, and reckless behavior.
Hyperthyroidism Mania Grandiosity
35
36 F is not correct. 12% chose this.
37 Narcissistic persona lity disorder is a cluster 8 disorder associated with a sense of entit lement, lack of empathy,
38
and need for excessive praise . A characteristic narcissistic personality disorder patient will always demand the
best for himself. In this vignette, the patient began exhibiting characteristics about 3 weeks ago. However,
39 narcissistic personal ity disorder as well as all other personality disorders are persistent and wou ld have been
40 • .o\f i ,..lol"\+- +"'t- ~ 1"'1"\,..,.oP" 1"'\0t"'il"'\,..1 ""'; +-i ........,.o C:tlr+ho,...........,t"''.,.o ,..lo,,-,..o~e-o,-1 r"'.oo,-1 +",.. e- l .ool"'\ ~1"'\,...1 it"'P"Oe"l"'\1"\r"'e-ih i l i +-" ~t-o 1"\t"''.+- ,..loJ'j l"\jl"\,.,

6
lock Suspend
s 0
End Block
Item:40of73 ~. , . M k <:] t> al ~· ~
QIO: 1125 .l. ar Previous Next Lab 'lifllues Notes Calculator


Hyperthyroidism can present with agitation and decreased sleep, but patients with this diso rder are unli kely to
20
exh ibit other symptoms of mania, including grand iosity, flight of ideas, and reckless behavior.
21 Hyperthyroidism Mania Grandiosity

22
F is not correct. 12% chose this.
23
Narcissistic persona lity disorde r is a cluste r B diso rder associated with a sense of entit lement, lack of empathy,
24 and need for excessive praise . A cha racteristic narcissistic personality disorde r patient will always demand the
25 best fo r himself. I n this vignette, the patient began exhibiting cha racteristics about 3 weeks ago. Howeve r,
26 narcissistic personal ity diso rder as well as all othe r personality disorders are persistent and wou ld have been
evident fo r a longer pe riod of t ime. Furthe rmore, decreased need for sleep, and irresponsibility are not defining
27
traits of na rcissistic pe rsonal ity disorder.
28 Narcissistic personality disorder Narcissism Personality disorder Cluster B personality disorders Empathy

29
G is not correct. 7°/o chose this.
30
Schizoaffective disorder is characterized as a psychotic disorder wit h a prominent co mponent of mood
31 disturbance, eithe r dep ression or man ia. Psychotic sympto ms, especia lly delusions and ha llucinations are not
32 seen in this patient therefo re t he diagnosis is un like ly .
Schizoaffective disorder Mood disorder Psychosis Mania Major depressive disorder Depression (mood) Hallucination
33
34
35 Bottom Line:
36 Diagnosis of bipola r I disorde r requi res the presence of an elevated, expansive, or ir ritable mood for more than
37 1 wee k that causes ma rked impai r ment in functioning socia lly, at work, or at home. Specific characteristics
38 incl ude grand iosity, decreased need fo r sleep, pressu red speech, and fl ight of ideas.
Bipolar I disorder Flight of ideas Pressure of speech Grandiosity Bipolar disorder
39
40 •

6
lock
s
Suspend
0
End Block
Item: 40 of 7 3 ~ 1 • M k -<:J 1>- Jil ~· !:';-~
QIO: 1125 ..L ar Pre v ious Next Labfli!llues Not es Calcula t o r
A A

20 FA17 p 531 .2
21 Bipolar disorder Bipolar I defined by presence of at least I manic episode+/- a hypomanic or depressi,·e episode.
22 (manic depression) Bipolar II defined b) presence of a hypomanic and a depressi'e episode.
23 Patient's mood and functioning usually return to normal beh,een episodes. Use of antidepressants
24 can precipitate mania. High suicide risk. Treatment: mood stabilizers (eg, lithium, valproic acid,
carbamazepine, Ia mot rigine). atypical antipsrchot ics.
25
26
Cyclothymic disorder-milder form of bipolar disorder lasting at least 2 years, Auctuating
between mild depressive and hrpomanic spnptoms.
27
28
FA17 p 530.4
29

30 Manic episode Distinct period of abnorma lly and persistent ly eb·atcd, expansi\'e, or irritable mood and
31
abnormally and persistently t activity or energy lasting at least 1 week. Often disturbing to
patient.
32
Diagnosis requires hospitalization or at least 3 of the following (man ics DIG FAST ):
33 • Distractibility • Flight of ideas-racing thoughts
34 • Irresponsibility- seeks pleasure without • t in goal-directed Activity/psychomotor
35 regard to consequences (hedonistic) Agitation
Grandiosity-inAated self-esteem • ~ need for Sleep
36
• 'talkativeness or pressured speech
37
38
FA17 p 544.1
39
lithium
40 •
a
Lock
s
Suspend
8
End Bl ock
Item: 40 of 7 3 ~ 1 • Ma rk -<:J 1>- Jil ~· !:';-~
QIO: 1125 ..L Pre v ious Next Labfli!llues Not es Calcula t o r
A A

20 Diagnosis requires hospitalization or at least 3 of the following (man ics DIG FAST ):
• D istractibility Flight of ideas-racing thoughts
21
• I rresponsibility-seeks pleasure without • f in goal-directed Activity/psychomotor
22 regard to consequences (hedonistic) \ gitation
23 G randiosity-inflated self-esteem 1 need for Sleep
24 r alkati,·eness or pressured speech
25
26 FA17 p 544.1
27 Lithium
28 MECHANISM Not established; possibl} related to inhibition of LiTHIU.\1:
29 phosphoinositol cascade. Low T hyroid (hypothyroidism)
CLINICAl USE Mood stabilizer for bipolar disorder; blocks Heart (Ebstein anomaly)
30
relapse.and acute manic events. Insipidus (nephrogenic diabetes insipidus)
31 Unwanted ~ lovements (tremor)
32 ADVERSE EFFECTS Tremor, hypothyroidism, polyuria (causes
nephrogenic diabetes insipidus), teratogenesis.
33
Causes Ebstein anomaly in newborn iftnken
34 by pregnant mother. Narrow thempeutic
35 window requi res close monitoring of serum
36 le,·els. Almost exclusively excreted by
kidneys; most is reabsorbed at PCT with Ia+.
37
Thiazides (and other nephrotoxic agents) are
38
implicated in lithium toxicity.
39

a
Lock
s
Suspend
8
End Bl ock
Item: 41 of 73 ~ 1 • M k -<:J 1>- Jil ~· !:';-~
QIO: 1136 ..L ar Pre v ious Next Labfli!llues Notes Calcula t o r

40
IAA]
A A

• 41
A 42-year-old disheveled African-American woman is admitted to an inpatient psychiatric unit after police
find her exhibiting bizarre and aggressive behavior in a publ ic square. She seems to have conversations with
• 42
herself and frequently gigg les out of context during her daily interviews. The nurses report that the patient's
• 43 behavior is ch ildlike, and that she has a tendency to expose herself to the other patients .
• 44
• 45 Which of the following would contribute most to this patient's symptoms?
• 46 :
• 47 A . Decrease in acetylcholine
• 48
B. Decrease in GABA
• 49
C. Decrease in serotonin
• so
• 51 D. Increase in dopamine
• 52 E. Increase in norepinephrine
• 53
• 54
• 55
• 56
• 57
• 58
• 59
• 60

a
Lock
s
Suspend
8
End Bl ock
Item: 41 of 7 3 ~ 1 • M k -<:J 1>- Jil ~· !:';-~
QIO: 1136 ..L ar Pre v ious Next Labfli!llues Notes Calcula t o r

40 A A

The correct answer is D. 66°/o chose this.


41
An increase in dopamine would cause positive symptoms, as
• 42
shown in this patient. The increase in dopamine acts on the
• 43 mesolimibic system (il lustrated in the diagram), causing the
• 44 patient to have delusions/ auditory/visual hallucinations/ and
• 45
paranoia termed "posit ive symptoms." In contrast/ a lack of
dopamine in the mesocortical system (which projects to the
• 46 Mesocortical
prefrontal cortex) is responsible for the patient's "negative pathway
• 47 symptoms/' which include poverty of speech/ flat affect/ cognitive Nigro!lriatal
• 48 dysfunction/ and social withdrawal. Although it is unlikely that you pat hway
Tubcro-
will be tested on the subtypes of schizophrenia (the DSM V has infundibular
Sobstantia
• 49 Nigra
removed these classifications)/ knowing the mechan ism of disease pathway ventral
• so is essential. Furthermore, knowing that antipsychotics Mesolimbic l!gmental Area
pathway
• 51 (neuro leptics) block dopam ine D2 receptors/ which are used to
• 52
treat sch izophren ia, is a high-yield concept. Image courtesy of Patrick J. Lynch
Schizophrenia Prefrontal cortex Dopamine Antipsychotic Alogia Paranoia
• 53 Diagnostic and Statistical Manual of Mental Disorders Mesocortical pathway

• 54 Reduced affect display Hallucination Cognitive disorder DSM-5 Solitude Receptor (biochemistry) Positive symptoms
• 55
A is not correct. 5°/o chose this .
• 56
A decrease in acetylcholine (synthesized in the basa l nucleus of Meynert) can be found in conditions such as
• 57 Alzheimer and Huntington diseases. The role of acetylcholine in schizophrenia remains to be elucidated .
• 58 However, it is widely accepted that an increase in dopamine is associated with the positive symptoms seen in
• 59 schizoph renia .
Acetylcholine Nucleus basalis of Meynert Dopamine Schizophrenia Basal ganglia Theodor Meynert Cell nucleus
• 60
• n !:- --.r.. ------a. •l!"n l - L - - - .r..L:-

a
Lock
s
Suspend
8
End Bl ock
Item: 41 of 73 ~. I • M k <:] t> al ~· ~
QIO: 1136 .l. ar Previous Next lab 'lifllues Notes Calculator
p
Acetylcholine Nucleus basalis of Meynert Dopamine Schizophrenia Basal ganglia Theodor Meynert Cell nucleus
41
• 42 B is not correct. 15% chose this .
• 43 A decrease in GABA (synthesized in the nucleus accumbens) is found in patients with anxiety disorders. The role
• 44 of GABA in sch izophrenia rem ains to be elucidated . How ever, it is widely accepted that an increase in dopamine
is associated w ith the positive symptom s seen in sch izophrenia .
• 45 Nucleus accumbens Gamma-Aminobutyric acid Dopamine Schizophrenia Anxiety Anxiety disorder Cell nucleus
• 46
• 47
C is not correct. 9°/o chose this .
A decrease in seroton in is thought to be a prim ary factor in causing depression. Although other
• 48
neurotransmitters may be decreased, serotonin is the princi pal neurotransmitter decreased in depression .
• 49 Neurotransmitter Serotonin Major depressive disorder Depression (mood)

• 50
E is not correct. 5°/o chose this .
• 51
An increase in norepinephrine (synthesized in t he locus cer uleus) is responsib le for anxiety disorders because it
• 52 m akes the patient hypervigilant. I n contrast to dopamine, norepinephrine is not generally thoug ht to be
• 53 associated with the positive sym ptoms of sch izophrenia .
locus coeruleus Norepinephrine Dopamine Schizophrenia Anxiety Hypervigilance Anxiety disorder locus (genetics)
• 54
• 55
• 56 Bottom Line:
• 57 The positive symptoms of schizoph renia, includ ing de lusions and ha llucinations, can be attr ibuted to an
• 58 increase in dopa m ine on the mesolim bic system . These sym ptoms are partia lly alleviated in some patients by
• 59 neuro leptic drugs, which block dopa mi ne D 2 receptors .
Schizophrenia Dopamine Mesolimbic pathway Antipsychotic Hallucination Positive symptoms Receptor (biochemistry) Delusion
• 60

6
lock
s
Suspend
0
End Block
Item: 41 of 7 3 ~ 1 • Ma rk -<:J 1>- Jil ~· !:';-~
QIO: 1136 ..L Pre v ious Next Labfli!llues Notes Calcula t o r

40 •
FA17 p530.1
41
Schizophre nia Chronic mental disorder with periods of Frequent cannabis use is associated\\ ith
• 42
psychosis, disturbed behavior and I hough!, ps)chosis/schizophrenia in teens .
• 43 and decline in functioning lasting> 6 Lifetime pre,·alence-1.5% (males= females,
• 44 months. Associated with t dopaminergie African Americans= Caucasians). Presents
• 45 acti\'ity, l dendritic branching. earlier in men (late teens to early 20s 's late
Diagnosis requires at least 2 of the following, 20s to early 30s in women). Patients are at t
• 46
and at least I of these should include 1- 3 risk for suicide.
• 47
(first4 are "posili\'e symptoms"): Vcnlriculomegaly on brain imaging.
• 48 Treatment: atypical antipsychotics (eg,
I. Delusions
• 49
2. llallucinalions- often auditory risperidone) are first line.
• so 3. Disorganized speech Negative symptoms often persist after treatment,
• 51 4. Disorganized or catatonic behavior despite resolution of positi,·e symptoms.

• 52 5. 1 egati,·e symptoms (affective Aatten ing,


avolition, anhedonia, asociality, alogia)
. 53
Brief psychotic disorder- lasting< l month,
• 54
usually stress related .
• 55
Schizophreniform disorder- lasti ng l -6
• 56
months.
• 57
Schizoaffective disorder- \llcels criteria for
. 58
schizophrenia in addition to major mood
• 59 disorder (major depressi,·e or bipolar). To
• 60
• ..
differentiate from a major mood disorder
. I •. ~ • . .
a
Lock Suspend
s 8
End Bl ock
Item: 41 of 7 3 ~ 1 • M k -<:J 1>- Jil ~· !:';-~
QIO: 1136 ..L ar Pre v ious Next Labfli!llues Notes Calcula t o r

40 A A

41 FA17 p 543.1
• 42 Typical antipsychotics Haloperidol, pimozide, triAuoperatine, Auphemu ine, thioridazine, chlorpromazin e.
• 43 MECHANISM Block dopamine 0 2 receptor (f cA 1P).
• 44 CliNICAl USE Schizophrenia (1° positi,·e symptoms), psychosis, bipolar disorder, delirium, Tourette syndrome,
• 45 Huntington disease, OCD.
• 46 POTENCY H igh potency: TriAuoperazine, Fluphena1ine, llaloperidol (Try to Fly H igh)- neurologic side
• 47 effects (eg, extrapyramidal S) mploms [F:PS)).
• 48 Lo" potency: C hlorpromazine, T h ioridazine (C heating l'hieves are low)-anticholinergic,
antihistamine, c:xrblockade effects.
• 49
ADVERSEEFFECTS Lipid soluble .... stored in body fat .... slow to be removed from body.
• so
• 51 Endocrine: dopamine receptor an tagonism .... hyperprolactinemia .... galactorrhea,
oligomenorrhea, gyn ecomastia .
• 52
Ietabolic: dyslipidemia, weight g<~in, hyperglycemia .
• 53
Antimuscarinic: dry mouth, constipation .
• 54 Antihistami ne: sedation.
• 55 c:xrblockade: orthostatic hypotension .
• 56 Card iac: QT prolongation .
• 57 Ophthalmologic: C hlorpromazine-C orneal deposits; T hioridazine- re'l'inal deposi ts.
• 58 EPS-ADAPT:
• 59 Hours to days: Acute D ystonia (muscle spasm, stiffness, oculogyric crisis).
Days to months: \ kathisia (restlessness), Parkinsonism (bradykinesia).
• 60
• .Vlonths to ,·ears: Tardive d,slinesia (orofacial chorea).

a
Lock
s
Suspend
8
End Bl ock
Item: 41 of 7 3 ~ 1 • M k -<:J 1>- Jil ~· !:';-~
QIO: 1136 ..L
'
ar Pre v ious Next
. .
- .
Labfli!llues
-
Notes Calcula t o r

effects (eg, extrapyramidal S) mptoms [fi:PS]).


. . . .. - .·-
41
Lo'' potency: C h lorpromazine, Thioridazine (Cheat ing T hieves are low)-anticholinergic,
• 42
antihistamine, a 1-blockade effects.
• 43
ADVERSE EFFECTS Lipid soluble - stored in body fat - slow to be removed from body.
• 44
Endocrine: dopamine receptor antagonism - hypcrprolactinemia - galactorrhea,
• 45
oligomenorrhea, g}11ecomastia .
• 46
.\ fetabolic: dyslipidemia, weight gain, h)l)crglyccmia.
• 47 Antimuscarinic: dry mouth, constipation .
• 48 Antihistamine: sedation .
• 49 a 1-blockade: orthostatic hypotension .
Cardiac: QT prolongation.
• so
Ophthalmologic: C hlorpromazine- C orneal deposits; T hioridazine-reT inal deposits.
• 51
EPS-ADAPT:
• 52
Hours to days: Acute Dystonia (muscle spasm , stiffness, oculogyric crisis).
• 53
Days to months: Akathisia (restlessness), Parkinsonism (bradykinesia).
• 54 Months to years: Tardive dyskinesia (orofacial chorea).
• 55 Treatment: benztropine (acute dystonia, tardive dyskinesia), benzodiazepines, ~-blockers
• 56 (akathisia).
• 57 Neuroleptic malignant syndrome (NMS)- \1 alignant FEVER: ~1yoglobinuria, Fever,
• 58 E ncephalopathy, unstable Vitals, t Enzymes, muscle Rigidity. Treatment: dantrolene, 0 2 agonist
• 59 (eg, bromocriptine).

• 60

Lock
a s
Suspend
8
End Bl ock
Item: 42 of 73 ~ 1 • M k -<:J 1>- Jil ~· !:';-~
QIO: 1468 ..L ar Pre v ious Next Lab fli!ltues Not es Calcula t o r

40 A A

41
A 27-year-old woman presents to her physician w ith a several-month history of weight loss and depressed
mood. On further questioning, she has been feeling fatigued and sleeping 12-14 hours a night. She no
• 42
longer enjoys her hobbies, including read ing and gardening. She also finds it hard to concentrate and
• 43 perform wel l at work. Her doctor refers her to a psychiatrist, who advised cognitive-behavioral therapy and also
• 44 prescribed amitriptyline .
• 45
• 46 This drug has a mechanism of action most similar to which of the following pharmacologic agents?
• 47 :

• 48 A. Clozapine
• 49 B. Cocaine
• so C. Mirtazapine
• 51
D. Selegiline
• 52
• 53 E. Tranylcypromine
• 54
• 55
• 56
• 57
• 58
• 59
• 60

a
Lock
s
Suspend
8
End Bl ock
Item: 42 of 73 ~. I • M k <:] t> al ~· ~
QIO: 1468 .l. ar Previous Next lab 'lifllues Notes Calculator

40 •
41
The correct answer is B. 37°/o chose this.
Both amitriptyline and cocaine have similar mechanisms of actions in that they both inhibit presynaptic neuronal
42
uptake of specific neurotransmitte rs, the reby potentiating the effects of those neurotransmitters. Amitriptyl ine is
• 43 a tricyclic antidep ressant that works by inhibiting presynaptic neurona l reuptake of norepinephrine and
• 44 seroton in. Cocaine inhibits the presynaptic neurona l reuptake of norepinephrine, se roton in, and dopamine .
Tricyclic antidepressant Amitriptyline Serotonin Antidepressant Norepinephrine Cocaine Dopamine Neurotransmitter Chemical synapse Reuptake Tricyclic
• 45
• 46 A is not correct. 10% chose this .
• 47 I n contrast to amitryptil ine, a drug that inh ibits the presynaptic neu ronal uptake of norepineph rine and
• 48 seroton in, clozapine acts by blocking postsynaptic dopamine and serotonin receptors. Clozapine, an atypical
antipsychotic, is used in treating schizophrenia or schizoaffective disorder after the patient has failed other first-
• 49
and second-line antipsychotics . The adverse effects are agranulocytosis (requ ires regu lar monitoring of
• 50 neutroph il count), myocarditis, prolongation of QT interva l, seizures, and metabolic effects including weight gain
• 51 and insul in resistance. Atypical antipsychotics, in general, are associated with fewer extrapyramidal symptoms
than typica l antipsychotics .
• 52
Atypical antipsychotic Schizoaffective disorder Neutrophil Agranulocytosis Clozapine Myocarditis Serotonin Norepinephrine QT interval Dopamine Schizophrenia
• 53
Extrapyramidal symptoms Insulin resistance Antipsychotic Typical antipsychotic Insulin Chemical synapse Epileptic seizure 5-HT receptor Receptor (biochemistry)
• 54
Metabolism
• 55
• 56 C is not correct. 21% chose this .
• 57
Mirtazapine, a heterocyclic antidepressant, is an orantagonist that leads to an increased release of
norepinephrine and serotonin, as opposed to inhibit ion of their reuptake. Due to its side effect of increasing
• 58 appetite and weight, mirtazapine may be desirable in elde lry or anorexic patients with depression. Other side
• 59 effects include sedation .
Mirtazapine Serotonin Antidepressant Norepinephrine Sedation Side effect Heterocyclic compound Anorexia nervosa Reuptake Major depressive disorder
• 60

6
lock
s
Suspend
0
End Block
Item: 42 of 73 ~. I • M k <:] t> al ~· ~
QIO: 1468 .l. ar Previous Next lab 'lifllues Notes Calculator

40 • C is not correct. 21% chose this.


41 Mirtazapine, a heterocyclic antidepressant, is an orantagonist that leads to an increased release of
42 norepinephrine and serotonin, as opposed to inhibit ion of their reuptake. Due to its side effect of increasing
appetite and weight, mirtazapine may be desirable in elde lry or anorexic patients with depression. Other side
• 43
effects include sedation .
• 44 Mirtazapine Serotonin Antidepressant Norepinephrine Sedation Side effect Heterocyclic compound Anorexia nervosa Reuptake Major depressive disorder

• 45 Adverse effect Anorexia (symptom) Adverse drug reaction Depression (mood)


• 46
D is not correct. 14% chose this .
• 47
Se leg iline is a se lective monoamine oxidase B inhibitor (MAOI) used to treat Parkinson disease. It acts in a
• 48 fashion similar to that of a nonselective MAOI, but selective ly increases presynaptic stores of dopamine. The
• 49 adverse effects are nausea, headache, and insomnia .
Selegiline Monoamine oxidase B Dopamine Insomnia Monoamine oxidase inhibitor Monoamine oxidase Nausea Parkinson' s disease Chemical synapse
• 50
Monoamine neurotransmitter Headache Enzyme inhibitor
• 51
• 52 E is not correct. 18% chose this .
• 53 Tranylcypromine is a nonselective monoamine oxidase inhibitor (MAOI). Monoamine oxidase is an enzyme in the
• 54 presynaptic neuron that degrades excess catecholamines. Thus inhibitors of MAO increase the concentrations of
of norepinephrine, seroton in, and dopamine, which are then avai lable to diffuse into the synaptic cleft. MAOis
• 55
can be used as an antidepressant; however, this is not common due to their severe adverse effect profile, wh ich
• 56 includes a risk of developing hypertensive crisis (especia lly with ingestion of tyramine, found in aged cheese and
• 57 wi ne) . Also, MAOI interaction with serotonerg ic med ications (eg, se lective-serotonin reuptake inhibitor, tricycl ic
• 58
antidepressants, meperidine, dextromethorphan) can cause serotonin syndrome .
Monoamine oxidase inhibitor Serotonin syndrome Tranylcypromine Tyramine Dextromethorphan Dopamine Synaptic cleft Neuron Chemical synapse Antidepressant
• 59
Norepinephrine Hypertensive crisis Enzyme Hypertensive emergency Monoamine oxidase Serotonergic Serotonin Tricyclic antidepressant Catecholamine Pethidine
• 60
• Reuotake Monoamine neurotransmitter Adverse effect Tricyclic Reuotake inhibitor Enzyme inhibitor Bindina selectivity Cheese rioenina Oxidase

6
lock
s
Suspend
0
End Block
Item: 42 of 73 ~. I • M k <:] t> al ~· ~
QIO: 1468 .l. ar Previous Next lab 'lifllues Notes Calculator

40 • Se leg iline is a se lective monoamine oxidase B inhibito r (MAOI) used to treat Pa rkinson disease. It acts in a
41 fashion similar to that of a nonselective MAOI, but selective ly increases presynaptic stores of dopamine. The
42
adverse effects are nausea, headache, and insomnia.
Selegiline Monoamine oxidase B Dopamine Insomnia Monoamine oxidase inhibitor Monoamine oxidase Nausea Parkinson' s disease Chemical synapse
• 43
Monoamine neurotransmitter Headache Enzyme inhibitor
• 44
• 45 E is not correct. 18% chose this .
• 46
Tranylcypromine is a nonselective monoamine oxidase inhibitor (MAOI). Monoamine oxidase is an enzyme in the
presynaptic neuron that degrades excess catecholamines. Thus inhibitors of MAO increase the concentrations of
• 47
of norepinephrine, seroton in, and dopamine, which are then avai lable to diffuse into the synaptic cleft. MAOis
• 48 can be used as an antidepressant; however, this is not common due to their severe adverse effect profi le, wh ich
• 49 includes a risk of developing hypertensive crisis (especia lly with ingestion of tyramine, found in aged cheese and
wi ne) . Also, MAOI interaction with serotonerg ic med ications (eg, se lective-serotonin reuptake inhibitor, tricycl ic
• 50
antidepressants, meperidine, dextromethorphan) can cause serotonin syndrome .
• 51 Monoamine oxidase inhibitor Serotonin syndrome Tranylcypromine Tyramine Dextromethorphan Dopamine Synaptic cleft Neuron Chemical synapse Antidepressant

• 52 Norepinephrine Hypertensive crisis Enzyme Hypertensive emergency Monoamine oxidase Serotonergic Serotonin Tricyclic antidepressant Catecholamine Pethidine
• 53 Reuptake Monoamine neurotransmitter Adverse effect Tricyclic Reuptake inhibitor Enzyme inhibitor Binding selectivity Cheese ripening Oxidase
• 54 Functional selectivity
• 55
• 56
• 57
Bottom Line:
• 58 Both tricyclic antidep ressants and cocaine prevent presynaptic reuptake of neurotransmitters, thereby
potentiating the effects of these neurotransmitte rs .
• 59 Tricyclic antidepressant Neurotransmitter Chemical synapse Cocaine Antidepressant Reuptake Tricyclic
• 60

6
lock
s
Suspend
0
End Block
Item: 42 of 73 ~ 1 • M k -<:J 1>- Jil ~· !:';-~
QIO: 1468 ..L ar Pre v ious Next Lab fli!ltues Not es Calcula t o r

40 A A

41 FA17 p 545.4

42 Tricyclic Amitriptyline, nortriptyline, imiprami ne, desipramine, clomipramine, doxepin, amoxapine.


• 43 antidepressants

• 44 MECHANISM Inhibit NE and )-liT reuptake.


• 45 CLINICAL USE ~lajor depression, OCD (clomipramine), peripheral neuropathr. c hronic pain, migraine
• 46 prophrlaxis. 1'\octurnal enuresis (imiprami ne, alt hough ad,·erse effects may limit use).

• 47 ADVERSE EFFECTS Sedation, a 1-blocking effects including postural hypotension, and atropine-like (anticholinergic)
• 48
side effects {tachycardia, urinary retention, dr~ mouth). 3° TCAs (amitriptyline) have more
anticholinergic effects than 2° TCAs (nortriptyline). Can prolong QT interval.
• 49
Tri-C's: C om·ulsions, C om a, C ardiotoxicity (arrhythmia due to 1 a+ channel inhibition);
• so also respiratory depression, hyperpyrexia. Confusion and hallucinations in elderly due to
• 51 anticholinergic side effects (nortript yline beller tolerated in the elderly). Treatment: 'ai iC03 to
• 52 prevent arrhythmia.
• 53
• 54 FA17 p 231 .1
• 55 Autonomic drugs Release of norepinephrine from a sympathetic ne n ·e e nding is modulated by 1 E itself, acting on
• 56 presynaptic az-autoreccptors ..... ncgat ivc feedback.
• 57 Amphetamines use the 1 E transporter (1 ET) to enter the presynaptic terminal, where they uti lize
the vesicular monoamine transporter (VMAT) to enter ne mosecretory ,·esiclcs. This displaces E
• 58
from the vesicles. Once E reaches a concentra tion threshold within the presynaptic terminal,
• 59
the action of lET is reversed, and NE is expelled into the synaptic cleft, contributing to the
• 60 characteristics and effects of t ' E observed in patients taking amphetamines.

a
Lock
s
Suspend
8
End Bl ock
Item: 42 of 73 ~ 1 • M k -<:J 1>- Jil ~· !:';-~
QIO: 1468 ..L ar Pre v ious Next Lab fli!ltues Not es Calcula t o r

40 . .. . -
41 CHOLINERGIC NORADRENERGIC
42 AXON AXON
• 43 Tyrosine ~
Choline
• 44 Tyrosine
• 45 l
Choline+ DOPA
• 46 Acetyt-<:oA l
• 47 ChAT l - - -<0>-- ---l
Dopamine

/
Reserpine Release-modulating
• 48 ACh Q'
receptors

NE ~%
• 49 Ca2+
• so J
• 51
• 52
Amphetamine, ~
• 53 Botulinum ephedrine

• 54 Cocaine, TCAs,
amphetamine
• 55 NEO
~ Diffusion,
• 56 metabolism
• 57
AChE inhibitors
• 58
AChE Ad reno receptors a or p
• 59 1--
POSTSYNAPTIC MEMBRANE t"v!>TS NAPTIC MEMBRANE
• 60

a
Lock
s
Suspend
8
End Bl ock
Item: 42 of 73 ~ 1 • Ma r k -<:J I> ~ £!1}>'
• !!":-~
QIO: 1468 ..L Prev ious Next Lab lues Not es Cal culat o r

40 A A

41 FA17 p 544.3
42
Antidepressants
• 43
NORADRENERGIC SEROTONERGIC
• 44
• 45 AXN AXON
• 46 MAO ~ MAO inhlbotors -
- - • Mel3boliles
• 47 Metabolites - - -

• 48
I Buproplon \
• 49 NE - -0 00 ¢---5-HT
• so (/o )
• 51 0 ¢
• 52 / \
• 53 TCAs, SNRis ~ - : -; Mlrtazaplne TCAs, SSRis,

• 54 1 NEreupta~ J3Y SNRis, trazodone

• 55
• 56
'--o os<)
5-HT receptor
( NE receptor
• 57
• 58
• 59
• 60

a
Lock
s
Suspend
8
End Bl ock
Item: 43 of 73 ~ 1 • M k -<:J 1>- Jil ~· !:';-~
QIO: 440 3 ..L ar Pre v ious Next Lab fli!ltues Not es Calcula t o r

40 A A

41
A 24-year-old woman is referred to a psych iatry cl inic as part of her paro le agreement. She has a long
history of lega l trouble and has been incarcerated for most of her life since the age of 15. Her most recent
42 imprisonment was the resu lt of a robbery in which she assaulted one of her victims . When asked why she did
• 43 this, she replies, "That's the way of the world. You can't trust anybody. If you want something, you 've got to take
• 44 it, no matter who you hurt."
• 45
• 46 Which of the following comorbid conditions will most likely accompany this woman 's disorder?
• 47 :

• 48 A . Alcohol abuse
• 49 B. General ized anxiety disorder
• so C. Panic disorder
• 51
D. Paranoid schizophrenia
• 52
• 53 E. Posttraumatic stress disorder
• 54
• 55
• 56
• 57
• 58
• 59
• 60

a
Lock
s
Suspend
8
End Bl ock
Item:43of73 ~. , . M k <:] t> al ~· ~
QIO: 4403 .l. ar Previous Next lab 'lifllues Notes Calculator

40 •
41 The correct answer is A. 52°/o chose this.
42 As an adu lt, this woman wou ld be diagnosed with antisocia l personality disorder, (ASPD), which is ma r ked by a
43 disregard fo r the rights of others and the rules of society . Patients with ASPD often have a history of behavior
prob lems, such as crimina lity, conning others, impu lsive behavior, vio lence, and lack of remorse, all of wh ich
44
0
typical ly started when they we re minors. Evidence of cond uct disorder occu r ring befo re age 15 yea rs is requi red
0 45 for a diagnosis of ASPD . These patients are at a high r isk for comor bid substance abuse, including alcoho l
0 46 abuse.
Antisocial personality disorder Conduct disorder Comorbidity Alcohol abuse Personality disorder Substance abuse Alcoholic beverage Alcohol Anti -social behaviour
0 47
0 48 B is not correct. 7°/o chose this.
0 49 Patients with antisocial personal ity diso rder do not have an increased r is k for developing gene ral ized anxiety
0 50
disorder. Antisocial persona lity disorde r is most associated with substance abuse diso rder. Comorbid anxiety
disorders may be present, albeit less frequently.
0 51 Antisocial personality disorder Generalized anxiety disorder Anxiety disorder Substance abuse Comorbidity Anxiety Personality disorder Anti-social behaviour
0 52 Addiction
0 53
C is not correct. 5°/o chose this.
0 54
Patients with antisocial personal ity diso rder do not have an
0 55 increased risk for developing panic diso rder. Comorbid anxiety Cluster Disorders Associated Illnesses
0 56 disorders are associated with the cluster C persona lity disorde rs Paranoid
0 57 ( avoidant, obsessive-compu lsive, and dependent) . Remembe r the A Schizoid Schizophrenia
general associations between persona lity disorders and other Schizotypal
0 58
psychiatric illnesses as detai led in the table . Antisocial
0 59 Antisocial personality disorder Panic disorder Personality disorder Comorbidity Anxiety Borderline Mood disorders
B
0 60 Anxiety disorder Obsessive-compulsive disorder Obsessive-compulsive Anti-social behaviour Histrionic Substance abuse
• P\l o rl' icci c t il'

6
lock
s
Suspend
0
End Block
Item:43of73 ~. , . M k <:] t> al ~· ~
QIO: 4403 .l. ar Previous Next lab 'lifllues Notes Calculator

40 •
C is not correct. 5°/o chose this.
41
Patients with antisocial personal ity diso rder do not have an
42 increased risk for developing panic diso rder. Comorbid anxiety Cluster Disorders Associated Illnesses
43 disorders are associated with the cluster C persona lity disorde rs Paranoid
0 44 ( avoidant, obsessive-compu lsive, and dependent) . Remembe r the A Schizoid Schizophrenia
general associations between persona lity disorders and other Schizotypal
45
0
psychiatric illnesses as detai led in the table . Antisocial
0
46 Antisocial personality disorder Panic disorder Personality disorder Comorbidity Anxiety Borderline Mood disorders
B
0 47 Anxiety disorder Obsessive-compulsive disorder Obsessive-compulsive Anti -social behaviour Histrionic Substance abuse
Narcissistic
0 48 Psychiatry
Avoidant
0 49 Obsessive-
c compulsive Anxiety disorders
0 50
Dependent
0
51
0
52 D is not correct. 26% chose this.
0 53 Patients with antisocial personal ity diso rder do not have an increased r is k for developing paranoid
0 54 schizoph renia . Patients with cluster A persona lity disorders-especial ly schizotypal pe rsonal ity disorder-are
genetica lly predisposed to deve lop schizoph renia .
0 55 Paranoid schizophrenia Antisocial personality disorder Schizophrenia Personality disorder Schizotypal personality disorder Anti-social behaviour Paranoia
0
56
E is not correct. 10% chose this.
0 57
Although patients with antisocial personal ity diso rder (ASPD ) may be involved in t raumatic situations that could
58
0

lead to the development of posttraumatic stress disorder, this association is not as strong as the association
0 59 between ASPD and substance abuse.
0 60 Posttraumatic stress disorder Antisocial personality disorder Personality disorder Substance abuse Anti-social behaviour Advanced sleep phase disorder

6
lock
s
Suspend
0
End Block
Item:43of73 ~. , . M k <:] t> al ~· ~
QIO: 4403 .l. ar Previous Next lab 'lifllues Notes Calculator

40 • Antisocial personality disorder Panic disorder Personality disorder Comorbidity Anxiety Borderline Mood disorders
B
41 Anxiety disorder Obsessive-compulsive disorder Obsessive-compulsive Anti-social behaviour Histrionic Substance abuse
Narcissistic
42 Psychiatry
Avoidant
43 Obsessive-
c compulsive
Anxiety disorders
0 44
Dependent
0 45
0
46 D is not correct. 26% chose this.
0 47 Patients with antisocial personal ity disorder do not have an increased r isk for developing paranoid
0 48 schizoph renia . Patients with cluster A persona lity disorders-especial ly schizotypal personal ity disorder-are
genetica lly predisposed to deve lop schizoph renia .
0 49
Paranoid schizophrenia Antisocial personality disorder Schizophrenia Personality disorder Schizotypal personality disorder Anti-social behaviour Paranoia
0 50
E is not correct. 10% chose this.
0
51
Although patients with antisocial personal ity disorder (ASPD) may be involved in t raumatic situations that could t
0
52
lead to the development of posttraumatic stress disorder, this association is not as strong as the association
0 53 between ASPD and substance abuse.
0 54 Posttraumatic stress disorder Antisocial personality disorder Personality disorder Substance abuse Anti-social behaviour Advanced sleep phase disorder

0 55
0
56
Bottom Line:
0 57
Antisocia l persona lity disorder (ASPD) is characterized by an enduring pattern of disregard for the rights of
0 58 others and the rules of society. ASPD is frequently comorbid with substance abuse.
0 59 Antisocial personality disorder Comorbidity Personality disorder Substance abuse Anti -social behaviour

0 60

6
lock
s
Suspend
0
End Block
Item: 43 of 7 3 ~ 1 • M k -<:J 1>- Jil ~· !:';-~
QIO: 440 3 ..L ar Pre v ious Next Lab fli!ltues Not es Calcula t o r

40 A A

41 FA17 p 535.2

42 Cluster B personality Dramatic, emotional, or erratic; genetic "Wild."


43 disorders association with mood disorders and substance C luster B: Bad, Borderline, AamBoyant, must
abuse. be the Best
• 44
• 45
Antisocial Disregard for and violation of rights of others Anli~ocial =~ociopath.
,,·ith lack of remorse, criminality, impulsivity; Bad .
• 46
males > females; must be~ 18 years old and
• 47 '
ha,·e histon· of conduct disorder before age 15.
• 48 Conduct disorder if< 18 years old.
• 49 Borderline Unstable mood and interpersonal relationships, Treatment: dialectical beha\'ior therapy.
• so impulsivity, self-mutilation, suicidality, sense Borderline.
• 51 of emptiness; females> males; splitting is a
major defense mechanism.
• 52
. 53
Histrionic Excessive emotionality and excitability, FlamBoyant.
attention seeking, sexually provocative, o,·erly
• 54
concerned with appearance.
• 55
Narcissistic G randiosity, sense of entitlement; lacks emp<1thy Must be the Best.
• 56 and requires excessive admiration; often
• 57 demands the "best" and reacts to criticism
. 58 with rage.
• 59
• 60 FA17 p 534.3
• o- -~- - ~ • : •..
a
Lock
s
Suspend
8
End Bl ock
Item: 43 of 7 3 ~ 1 • M k -<:J 1>- Jil ~· !:';-~
QIO: 440 3 ..L ar Pre v ious Next Lab fli!ltues Not es Calcula t o r

:t . II~···· I

41
Conduct disorder if< 18 years old.

42
Borderline Unstable mood and interpersonal relat ionships, Treatment : dialectica l behavior therapy.
impulsivity, self-mutilation, suicidality, sense Borderl inc.
43
of emptiness; females> males; splilling is a
• 44 major defense mechanism .
• 45
Histrionic Excessi,·e emotionalitr and excitability, FlamBorant.
• 46 attention seeking, scxuallr pro,·oc<~ ti,·c, O\'Crly
• 47 concemed with appearance.
• 48 Narcissistic Grandiosity, sense of entitlement; lacks empathy \lust be the Best.
• 49 and requires excessive admiration; often
• so demands the ''best" and reacts to criticism
with rage.
• 51
• 52
FA17 p 534.3
• 53
Personality
• 54
Personality trait An enduring, repetitive pattern of perceiving, relating to, and thinking about the environment and
• 55 oneself.
• 56
Personality disorder InAexible, maladaptive, and rigidly pervasive pat tern of behavior causing subjective distress
• 57 and/or impaired functioning; person is usuallr not aware of problem. Usually presents by early
• 58 adulthood .
• 59 Three clusters, A, B, and C; remember as Weird, \\ ild, and Worried based on symptoms.
• 60

a
Lock
s
Suspend
8
End Bl ock
Item:44of73 ~. , . M k <:] t> al ~· ~
QIO: 1484 .l. ar Previous Next lab 'lifllues Notes Calculator

40 •
41
A 21-year-old college student is brought to the loca l emergency department by her roommate after she
col lapsed at a party. Physica l examination reveals a severe tremo r, tachyca rdia, hypertonia, and feve r. The
42 patient does not respond to questioning, but he r roommate states that she was with he r al l night and that
43 her roommate did not consume any alcohol or drugs. A urine drug screen is negative for illicit substances. A
0 44 detailed review of the patient's med ical record reveals that she has been seeing multiple physicians fo r dep ression,
and it appears that she was mistaken ly prescribed two different antidepressant med ications.
0 45
0
46
Wh ich of the fo llowing is an approp riate pha rmaco log ic inte rvention for he r acute cond it ion'
0 47
0 48 :
A. Amitriptyl ine
0 49
0 50 B. Cyproheptad ine
0
51 C. Dantrolene
0
52
D. Flumazenil
0 53
E. Naloxone
0 54
0 55
0
56
0 57
0 58
0 59
0 60

6
lock
s
Suspend
0
End Block
Item:44of73 ~. , . M k <:] t> al ~· ~
QIO: 1484 .l. ar Previous Next lab 'lifllues Notes Calculator

40 •
41 The correct answer is B. 52°/o chose this.
42 Th is patient is experiencing what is known as serotonin syndrome (SS ), like ly Drugs that can contribute to the development
43
caused by mistaken ly being on a multidrug regimen that includes a of serotonin syndrome
monoamine oxidase inh ibitor along with one of the other agents in the table. Monoamine oxidase inhiMors (eg, isocarboxacid,
44 Th is can be inferred since the patient was recently started on two different phenelzi ne)
SSR is
• 45 antidepressant medications. SS is caused by excessive stimu lation of the 5- Tricyclic antidepressants
• 46 hydroxytryptam ine1A and 5-hydroxytryptamine 2 centra l nervous system Meperidine
Sumatriptan
receptors by serotonin. The selective seroton in reuptake inh ibitors block Lithium
• 47
presynaptic reuptake of seroton in. The symptoms of th is syndrome include Cocaine
Procarbazine
• 48 confusion, ag itation, flush ing, myoclonus, hyperrefl exia, autonomic instability, Linezolid
• 49 mydriasis, hyperthermia, and seizures. I n rare instances, SS can cause
• 50
rhabdomyolysis and disseminated intravascular coagu lation . The treatment
for SS includes supportive care, adm inistration of activated charcoa l for acute ingestions, and, in severe cases,
• 51 administration of cyproheptadine. Cyproheptadine is a seroton in receptor antagonist that is administered by
• 52 mouth. I t is often discontinued if the patient shows no symptomatic improvement with in the fi rst 2 hours .
Monoamine oxidase inhibitor Serotonin syndrome Mydriasis Rhabdomyolysis Disseminated intravascular coagulation Cyproheptadine Activated carbon
• 53
Hyperreflexia Central nervous system Myoclonus 5-HT receptor Hyperthermia Antidepressant Serotonin antagonist Serotonin Selective serotonin reuptake inhibitor
• 54
• 55 Monoamine oxidase Autonomic nervous system Chemical synapse Epileptic seizure Receptor antagonist Monoamine neurotransmitter Reuptake Coagulation

• 56 Receptor (biochemistry) Dysautonomia Nervous system Flushing (physiology) Enzyme inhibitor

• 57 A is not correct. 4°/o chose this .


• 58 Am itriptyline is a tricycl ic antidepressant (TCA) that inh ibits the reuptake of norepinephrine and seroton in. The
• 59 TCAs have been imp licated in causing serotonin syndrome, especia lly when combined with other classes of
antidepressants such as selective serotonin reuptake inhibitors and monoam ine oxidase inhibitors .
• 60
Tricyclic antidepressant Serotonin syndrome Amitriptyline Antidepressant Serotonin Norepinephrine Selective serotonin reuptake inhibitor Monoamine oxidase

6
lock
s
Suspend
0
End Block
Item:44of73 ~. , . M k <:] t> al ~· ~
QIO: 1484 .l. ar Previous Next lab 'lifllues Notes Calculator

40 •
A is not correct. 4°/o chose this.
41
Amitr iptyl ine is a tricycl ic antidepressant (TCA) that inhibits the reuptake of norepineph rine and se roton in. The
42 TCAs have been imp licated in causing serotonin synd rome, especial ly when combined with othe r classes of
43 antidepressants such as selective serotonin reuptake inhibito rs and monoamine oxidase inhibitors.
Tricyclic antidepressant Serotonin syndrome Amitriptyline Antidepressant Serotonin Norepinephrine Selective serotonin reuptake inhibitor Monoamine oxidase
44
Monoamine oxidase inhibitor Monoamine neurotransmitter Reuptake Tricyclic Oxidase
• 45
• 46 C is not correct. 25% chose this .
• 47 Dantrolene is used in neu roleptic mal ignant syndrome (NMS), character ized by fever, rigidity, alte red menta l
• 48
status, and muscle enzyme leakage. In add iion, dantrolene is also used for treatment of ma lignant
hyperther mia but as shown in this case, it is not indicated for t reatment of seroton in synd rome (SS) . It is worth
• 49 noting that NMS and SS have simila r presentations with regard to changes in vital signs, mental status, skin,
• 50 and muscle tone. However, NMS symptoms feature hyporeflexia, norma l pupi ls, and normoactive/ hypoactive
• 51 bowe l sounds, compared with hyperreflexia (+/ -clonus), dilated pupi ls, and hype ractive bowel sounds in SS .
Additiona lly, NMS usua lly has an onset of 1-3 days, versus < 12 hou rs in SS. NMS is precipitated by dopamine
• 52
antagon ists, and SS by serotonergic agents .
• 53 Neuroleptic malignant syndrome Malignant hyperthermia Serotonin syndrome Dantrolene Hyporeflexia Clonus Hyperreflexia Hyperthermia Dopamine Enzyme

• 54 Serotonin Serotonergic Muscle tone Antipsychotic Altered level of consciousness Mydriasis Fever Muscle Attention deficit hyperactivity disorder

• 55
D is not correct. 13% chose this .
• 56
Flumazenil antagon izes the effect of benzod iazepine overdose. It can be used in patients who have neve r ta ken
• 57 benzodiazepines before, as flumazeni l can cause seizures in patients who chronically use benzodiazepines.
• 58 Benzodiazepine Flumazenil Benzodiazepine overdose Epileptic seizure Drug overdose

• 59 E is not correct. 6°/o chose this .


• 60 Naloxone is a competitive antagonist at the opiate recepto r and is thus used to t reat opiate overdose. Acute

6
lock
s
Suspend
0
End Block
Item:44of73 ~. , . M k <:] t> al ~· ~
QIO: 1484 .l. ar Previous Next lab 'lifllues Notes Calculator

40 •
Serotonin Serotonergic Muscle tone Antipsychotic Altered level of consciousness Mydriasis Fever Muscle Attention deficit hyperactivity disorder
41
42 D is not correct. 13% chose this.
43 Flumazenil antagon izes the effect of benzod iazepine overdose. It can be used in patients who have never taken
44
benzodiazepines before, as flumazeni l can cause seizures in patients who chronically use benzodiazepines.
Benzodiazepine Flumazenil Benzodiazepine overdose Epileptic seizure Drug overdose
• 45
• 46 E is not correct. 6°/o chose this .
• 47
Naloxone is a competitive antagonist at the opiate receptor and is thus used to t reat opiate overdose. Acute
opiate intoxication is often identified by altered mental status, decreased respi ratory rate and constricted pupils .
• 48 Naloxone is not considered a treatment option for serotonin syndrome .
• 49 Serotonin syndrome Naloxone Serotonin Opiate Receptor antagonist Opioid receptor Competitive antagonist Miosis Respiratory rate Altered level of consciousness

• 50 Receptor (biochemistry) Antagonist Drug overdose Alcohol intoxication

• 51
• 52
Bottom Line:
• 53
Seroton in synd rome (SS) is a serious compl ication that can occur when patient is t reated with a monoamine
• 54
oxidase inh ibitor in combination with a selective seroton in reuptake inhibitor or tricycl ic antidepressant.
• 55 Symptoms include agitation, flushing, myoclonus, hyperreflexia, and hyperthermia. Cyproheptadine is a
• 56 seroton in receptor antagonist that can be used to counteract the adverse effects of serotonin in SS .
• 57
Neuroleptic mal ignant syndrome (NMS) has simi lar featu res but is precipitated by a different pharmacolog ic
mechanism .
• 58 Monoamine oxidase inhibitor Tricyclic antidepressant Neuroleptic malignant syndrome Selective serotonin reuptake inhibitor Serotonin syndrome Cyproheptadine

• 59 Hyperreflexia Serotonin Myoclonus 5-HT receptor Hyperthermia Antidepressant Serotonin antagonist Serotonin reuptake inhibitor Monoamine oxidase Reuptake
Monoamine neurotransmitter Receptor antagonist Antipsychotic Pharmacology Reuptake inhibitor Tricyclic Antagonist Flushing (physiology)
• 60

6
lock
s
Suspend
0
End Block
Item: 44 of 7 3 ~ 1 • Ma r k -<:J I> ~ £!1}>'
• !!":-~
QIO: 1484 ..L Prev ious Next Lab lues Not es Cal culat o r

40 A A

FA17 p 544.3
41
Antidepressants
42
43 NORADRENERGIC SEROTONERGIC

44
,;x.N ,;x.QN
• 45
MAO ~ MAOinhlbitors - ,.....__
-o- ~
_, MAO
• 46 - - • Metabolites
Metabolites - - -

• 47 Buproplon \
• 48 NE - -

• 49
• so
• 51
• 52 Mlnazaplne TCAs, SSRis,
SNRis, trazodone
. 53
• 54
• 55
5-HT recept or
• 56
• 57
. 58
• 59
• 60 POSTSYNAPTIC NEURON

a
Lock
s
Suspend
8
End Bl ock
Item: 44 of 7 3 ~ 1 • M k -<:J 1>- Jil ~· !:';-~
QIO: 1484 ..L ar Pre v ious Next Lab fli!ltues Not es Calcula t o r

40 A A

FA17 p 545.1
41
Selective serotonin F'luoxetine, Am·oxamine, paroxetine, sertraline, escitalopram, citalopram.
42
reuptake inhibitors
43
MECHANISM Inhibit 5-HT reuptake. II normally takes 4-8 weeks for antidepressants
44
CliNICALUSE Depression, generalized anxiety disorder, to have an effect.
• 45
panic disorder, OCD, bulimia, social an,iety
• 46 disorder, PTSD, premature ejaculation,
• 47 premenstrual dysphoric disorder.
• 48 ADVERSE EFFECTS Fe\\er than TC s. CI di~tress, SIAOII, se,ual
• 49 dysfunction (anorgasmia, l Iibido).
• so
• 51 FA17 p 546.1
• 52
Monoamine oxidase Tranylcypromine, Phenelzine, lsoca rboxazid, Sclegiline (selecti,·e YIAO-B inhibitor).
. 53 inhibitors (MAO Takes Pride In Shanghai).
• 54 MECHANISM Nonselective YfAO inhibition t levels of amine neurotransmitters (norepinephrine, 5-ITT,
• 55 dopamine).
• 56 CliNICAL USE Atypical depression, anxiety. Parkinson disease (selegiline).
• 57 ADVERSEEFFECTS C1 S stimulation; hypertensive crisis, most notably with ingestion of tyramine, which is found in
. 58 mam· foods such as aged cheese and wine. Tyramine displaces other neurotransmitters (eg, E)
• 59 into the synaptic cleft - t sympathetic stimulation. Contraindicated with SSRls, TCAs, St. John's
• 60 wort, meperidine, dextromethorphan (to pre,·ent serotonin syndrome).
• \ V:tit 2 \\'f'Pl:s :tftPr dnnninu \If \0 inhihitnr~ h PfnrP d:1rtinu ~Prntnnpruir rlm u~ nr dnnninu rliPt:trv

a
Lock
s
Suspend
8
End Bl ock
Item: 45 of 73 ~ 1 • M k -<:J 1>- Jil ~· !:';-~
QIO: 1452 ..L ar Pre v ious Next Lab fli!ltues Not es Calcula t o r

40
IAA]
A A

41
A 36-year-old woman present s t o the emergency department diaphoretic and com pla ining of palpitations.
Her blood pressure is 180/100 mm Hg, heart rate is 165/min, and blood alcohol cont ent is 0.08%. Her
42 husband reports that she has had a long history of depression which has not been responsive to medical
43 therapy but she recently started a new med ication for depression.
44
• 45 Which of the following antidepressants is this patient most likely taking?
• 46 :
• 47 A. Bupropion
• 48
B. Fluoxetine
• 49
C. Imipramine
• so
• 51 D. Lorazepam
• 52 E. Phenelzine
• 53
F. Thioridazine
• 54
G. Trazodone
• 55
• 56
• 57
• 58
• 59
• 60

a
Lock
s
Suspend
8
End Bl ock
Item:45of73 ~. , . M k <:] t> al ~· ~
QIO: 1452 .l. ar Previous Next lab 'lifllues Notes Calculator

40 •
41 The correct answer is E. 49°/o chose this.
42 Th is patient is most likely ta king phene lzine, a monoamine oxidase inhibitor (MAOI) . MAOis can cause a
43 hypertensive crisis when taken with certain foods ( eg, aged cheeses, red wi nes, and beer ) that contain
tyramine. Excess tyramine causes release of stored catecholamines f rom ne rve termina ls, leading to the
44
hypertension and hyperther mia experienced by this patient. Patients who take MAOis must be warned to avoid
45 tyramine-containing foods. Take note: It is not uncommon for the USMLE to avoid giving buzzwo rds in the
• 46 question stem . Here, we were told only that the patient had consumed alcohol and had to reason that red wine
• 47 or bee r was consumed .
Monoamine oxidase inhibitor Phenelzine Tyramine Hypertensive crisis Hyperthermia Hypertension Hypertensive emergency Monoamine oxidase Catecholamine
• 48
Monoamine neurotransmitter Red wine Oxidase Alcohol Enzyme inhibitor
• 49
• 50 A is not correct. 9°/o chose this .
• 51 Bupropion is another second-generation antidepressant that is used prima rily to treat depression and to aid in
smoking cessation . It may cause tachyca rdia and agitation, but it does not precipitate hypertensive crises .
• 52
Bupropion Second-generation antidepressant Antidepressant Tachycardia Smoking cessation Major depressive disorder Depression (mood)
• 53
Hypertensive emergency Hypertension
• 54
B is not correct. 10% chose this .
• 55
Fluoxetine is a selective serotonin reupta ke inh ibitor that is used to treat depression. It does not cause
• 56
hypertensive crises. Fluoxetine may cause seroton in syndrome, however, w hich can manifest with ag itation,
• 57 de lir ium, autonom ic symptoms (eg, tachyca rdia, hyperthermia, and hypertension ) . One of the characteristic
• 58 featu res of serotonin synd rome is neu romuscula r junction hyperactivity-tremor and hyper reflexia .
Selective serotonin reuptake inhibitor Serotonin syndrome Fluoxetine Hyperreflexia Neuromuscular junction Serotonin Hyperthermia Tachycardia Hypertension
• 59
Serotonin reuptake inhibitor Hypertensive emergency Reuptake Autonomic nervous system Major depressive disorder Depression (mood) Delirium
• 60

6
lock
s
Suspend
0
End Block
Item:45of73 ~. , . M k <:] t> al ~· ~
QIO: 1452 .l. ar Previous Next lab 'lifllues Notes Calculator

40 • B is not correct. 10% chose this .


41 Fluoxetine is a selective serotonin reuptake inh ibitor that is used to treat depression. It does not cause
42 hypertensive crises. Fluoxetine may cause seroton in syndro me, however, w hich can manifest with ag itation,
43 de lirium, autonom ic symptoms (eg, tachyca rdia, hyperthermia, and hypertension ) . One of the characteristic
features of serotonin synd rome is neu romuscular j unction hyperactivity-tremor and hyperreflexia.
44 Selective serotonin reuptake inhibitor Serotonin syndrome Fluoxetine Hyperreflexia Neuromuscular junction Serotonin Hyperthermia Tachycardia Hypertension
45
Serotonin reuptake inhibitor Hypertensive emergency Reuptake Autonomic nervous system Major depressive disorder Depression (mood) Delirium
• 46
Reuptake inhibitor
• 47
• 48
C is not correct. 16% chose this .
Imipramine, a tricycl ic antidepressant used to treat depression and bed-wetting, is not known to cause a
• 49
hypertensive crisis. Its most common adverse effects are postura l hypotension and anticholine rgic effects .
• 50 Tricyclic antidepressant Anticholinergic Imipramine Antidepressant Hypertensive crisis Orthostatic hypotension Hypotension Hypertensive emergency

• 51 Major depressive disorder Depression (mood) Nocturnal enuresis Tricyclic

• 52
D is not correct. 7°/o chose this .
• 53
Lorazepam, a benzodiazepine, is used to treat anxiety and to faci litate sedation . It is not used to treat
• 54 depression and would not cause a hypertensive cr isis.
• 55 Benzodiazepine lorazepam Hypertensive emergency Hypertensive crisis Sedation Anxiety Major depressive disorder Depression (mood) Hypertension

• 56 F is not correct. 5°/o chose this .


• 57 Thio ridazine is an antipsychotic drug that acts as an antagonist at dopamine receptors and is used to t reat
• 58 schizoph renia and other psychotic disorders. It is not used to t reat depression .
Thioridazine Dopamine Schizophrenia Antipsychotic Receptor antagonist Psychosis Major depressive disorder Depression (mood) Dopamine receptor Antagonist
• 59
• 60 G is not correct. 4°/o chose this .

6
lock
s
Suspend
0
End Block
Item:45of73 ~. , . M k <:] t> al ~· ~
QIO: 1452 .l. ar Previous Next lab 'lifllues Notes Calculator

40
Tricyclic antidepressant Anticholinergic Imipramine Antidepressant Hypertensive crisis Orthostatic hypotension Hypotension Hypertensive emergency
41
Major depressive disorder Depression (mood) Nocturnal enuresis Tricyclic
42
43
D is not correct. 7°/o chose this.
Lorazepam, a benzodiazepine, is used to treat anxiety and to faci litate sedation . It is not used to treat
44
depression and would not cause a hypertensive crisis.
45 Benzodiazepine lorazepam Hypertensive emergency Hypertensive crisis Sedation Anxiety Major depressive disorder Depression (mood) Hypertension

• 46
F is not correct. 5°/o chose this .
• 47
Thioridazine is an antipsychotic drug that acts as an antagonist at dopamine receptors and is used to treat
• 48 schizophrenia and other psychotic disorders. I t is not used to treat depression .
• 49 Thioridazine Dopamine Schizophrenia Antipsychotic Receptor antagonist Psychosis Major depressive disorder Depression (mood) Dopamine receptor Antagonist

• 50 G is not correct. 4°/o chose this .


• 51 Trazodone is a second-generation antidepressant that has not been associated with the clinical picture observed
• 52 in this patient. Trazodone is known to lower the seizure threshold and cause priapism .
Priapism Second-generation antidepressant Antidepressant Trazodone Seizure threshold Epileptic seizure
• 53
• 54
• 55 Bottom Line:
• 56 Monoamine oxidase inh ibitors are an old class of drugs used to treat depression . Today they are indicated only
• 57 for cases of atypica l or refractory depression. The most common compl ication is hypertensive crisis that occurs
• 58 w hen ingesting foods with a high tyramine content (eg, aged cheeses and red wi ne, among others) .
Tyramine Hypertensive crisis Hypertensive emergency Monoamine oxidase Monoamine oxidase inhibitor Monoamine neurotransmitter Major depressive disorder
• 59 Treatment-resistant depression Depression (mood) Red wine
• 60

6
lock
s
Suspend
0
End Block
Item: 45 of 7 3 ~ 1 • M k -<:J 1>- Jil ~· !:';-~
QIO: 1452 ..L ar Pre v ious Next Lab fli!ltues Not es Calcula t o r

40 A A

41 FA17 p546.1
42
Monoamine oxidase Tranylcyprom ine, P henelzine, Isocarhoxazid, Selegiline (selecti,·e ~.\0-B inhibitor).
43 inhibitors (.\lAO Takes Pride In Shanghai).
44 MECHANISM 'onselecti,·e :\IIAO inhibition t b ·els of amine neurotransmitters (norepinephrine, 5-IIT,
45 dopamine).
• 46 CLINICAL USE Atypical depression, all': iel). Parl..inson disease (selegiline).
• 47 ADVERSE EFFECTS C l$ stimulation; hr pertensive crisis, most notably ''ith ingestion of tyramine, which is found in
• 48 m any foods such as aged cheese and wine. Tyramine displaces other neurotransmitters (eg, N E)
• 49 into the S}11aptic cleft - t sympathetic stimulation. Contraindicated with SSRls, T CAs, St. John's
• so wort, meperidine, dextromethorphan (to prevent serotonin syndrome).
Wait 2 weeks after stopping MAO inhibitors before starting serotonergic drugs or stopping dietary
• 51
restrictions.
• 52
• 53
• 54 FA1 7 p 544.3

• 55 Antidepressants
• 56 NORADRENERGIC SEROTONERGIC
• 57
AXON AXON
• 58
~ MAO i nhl blt ors -
• 59 -o- -f MAO Metabolites
Metabolites MAO
• 60
• 1 -. __.. Buorooton / \
a
Lock
s
Suspend
8
End Bl ock
Item: 45 of 7 3 ~ 1 • Ma r k -<:J I> ~ £!1}>'
• !!":-~
QIO: 1452 ..L Prev ious Next Lab lues Not es Calculat o r

40 A A

Antidepressants
41
42 NORADRENERGIC SEROTONERGIC

43
AXON AXON
44
45 r---- ~
~ MAO Inhibitors ~-
------0- ..
-f MAO - - • Metabolites
Metabolites - - - MAO

• 46 Buproplon \
• 47 NE - -

• 48

• 49
• so
• 51 Mlnazaplne TCAs, SSRis,
SNRis, trazodone
• 52
. 53
• 54
S.HT receptor
• 55
• 56
• 57
. 58
• 59 POSTSYNAPTIC NEURON

• 60

a
Lock
s
Suspend
8
End Bl ock
Item: 46 of 73 ~ 1 • M k -<:J 1>- Jil ~· !:';-~
QIO: 1144 ..L ar Pre v ious Next Labfli!llues Not es Calcula t o r

40
IAA]
A A

41
An 18-year-old college freshman is brought to the college health clinic after her roommate confronted her
about vomiting after every meal. The patient states that she feels guilty about her behavior but feels unable
42 to stop. Laboratory studies show a potassium level of 2.8 mEq/L, a ch loride level of 94 mEq/L, and a blood
43 urea nitrogen level of 22 mg/dl.
44
45 Which of the following drugs is contraindicated in this patient?
0
46 :
• 47 A. Bupropion
• 48
B. Fluoxetine
• 49
C. Paroxetine
• so
• 51 D. Topiramate
• 52 E. Trazodone
• 53
0 54
• 55
0
56
• 57
• 58
• 59
• 60

a
Lock
s
Suspend
8
End Bl ock
Item:46of73 ~. , . M k <:] t> al ~· ~
QIO: 1144 .l. ar Previous Next lab 'lifllues Notes Calculator

40 •
The correct answer is A. 57°/o chose this.
41
Bupropion is an atypical antidep ressant that increases no repinephrine and dopamine levels. The mechanism is
42 not ful ly understood, but it may involve inhibit ion of neu rotransmitter reupta ke. Bupropion is also used for
43 smoking cessation. A known adve rse effect of bupropion is lowering of the seizure th resho ld in othe rwise
44 hea lthy patients. This is an even mo re significant issue for patients who are at risk fo r seizu res. Fo r instance,
cond itions that cause electrolyte imbalances (eg, ch ron ic kidney disease) place patients at higher risk fo r
45
seizure, so bupropion is contraind icated in these patients. Habitual or excessive vomiting may resu lt in
46 hypochloremia and hypokalemia, so bupropion is contraindicated in anorexic/bu limic patients. Other adverse
• 47 effects include stimulant effects (tachycardia, insomn ia) and headaches .
Bupropion Hypokalemia Antidepressant Norepinephrine Dopamine Neurotransmitter Chronic kidney disease Insomnia Tachycardia Smoking cessation
• 48
Hypochloremia Adverse effect Vomiting Stimulant Seizure threshold Kidney Epileptic seizure Reuptake Kidney disease Contraindication Electrolyte
• 49
• 50 B is not correct. 10% chose this .
• 51 The selective serotonin reuptake inhibito rs (SSRI ) medications include fluoxetine, pa roxetine, setraline and
• 52
citalopram. Fluoxetine is the most wide ly studied of the antidepressants in t reatment of eating diso rders. It also
he lps prevent relapse in patients with anorexia. It is the only drug approved by the Food and Drug
• 53 Administration for the t reatment of bu limia. The adverse effects of SSRis include GI distress, syndrome of
• 54 inappropriate (secretion of) antidiuretic ho rmone (SIADH), and sexua l dysfunction .
Citalopram Paroxetine Fluoxetine Serotonin Selective serotonin reuptake inhibitor Bulimia nervosa Food and Drug Administration Sexual dysfunction Reuptake
• 55
• 56 Antidepressant Vasopressin Anorexia nervosa Eating disorder Anorexia (symptom) Hormone Secretion Relapse

• 57 C is not correct. 7°/o chose this .


• 58 Paroxetine is a selective se rotonin reupta ke inh ibitor (SSRI), but it is not the fi rst-l ine treatment for bu limia, in
• 59 contrast to fluoxetine. The adve rse effects of SSRis include GI distress, synd rome of inappropriate (secretion of)
antidiuretic ho rmone (SIADH), and sexua l dysfunction .
• 60
Selective serotonin reuptake inhibitor Paroxetine Fluoxetine Serotonin Bulimia nervosa Serotonin reuptake inhibitor Sexual dysfunction Vasopressin Reuptake •

6
lock
s
Suspend
0
End Block
Item:46of73 ~. , . M k <:] t> al ~· ~
QIO: 1144 .l. ar Previous Next lab 'lifllues Notes Calculator

40
41 C is not correct. 7°/o chose this.
42 Paroxetine is a selective se rotonin reuptake inh ibitor (SSRI ), but it is not the first-l ine treatment for bu limia, in
43 contrast to fluoxetine. The adverse effects of SSRis include GI distress, syndrome of inappropriate (secretion of)
antidiuretic hormone (SIADH), and sexua l dysfunction.
44 Selective serotonin reuptake inhibitor Paroxetine Fluoxetine Serotonin Bulimia nervosa Serotonin reuptake inhibitor Sexual dysfunction Vasopressin Reuptake
45
Hormone Reuptake inhibitor Therapy Enzyme inhibitor Secretion
46
• 47
D is not correct. 15% chose this .
Topira mate is an antiepileptic that blocks sod ium channels and augments the activity of the neurotransmitter y-
• 48
aminobutyric acid (GABA) . It is a third-line medication for bu limia, after trying fluoxetine and other seroton in
• 49 reuptake inhibitors (SSRi s) . It can he lp suppress binge urges in patients with bul imia and can also reduce their
• 50 preoccupation with eating and weight. The adverse effects of topiramate include sedation and an increased risk
• 51
of kidney stones .
Topiramate Fluoxetine Serotonin Neurotransmitter Gamma-Aminobutyric acid Bulimia nervosa Anticonvulsant Selective serotonin reuptake inhibitor Sedation
• 52
Sodium channel Sodium Kidney Kidney stone Reuptake Pharmaceutical drug
• 53
• 54 E is not correct. 11% chose this .
• 55 Trazodone is thought to be an antagon ist of 5-HT 2 , a 1-adrenergic and histamine- ! receptors. It is an
antidepressant that can be usefu l in the treatment of bulimia, although it is considered a third -line treatment
• 56
after trying fluoxetine and other serotonin reuptake inh ibitors (SSRi s). The adverse effects are sedation,
• 57 nausea, priapism, and postural hypotension .
• 58 Priapism Fluoxetine Bulimia nervosa Trazodone Antidepressant Orthostatic hypotension Hypotension Serotonin Receptor antagonist

• 59 Selective serotonin reuptake inhibitor Nausea Sedation Antagonist Reuptake Receptor (biochemistry)

• 60

6
lock
s
Suspend
0
End Block
Item:46of73 ~. , . M k <:] t> al ~· ~
QIO: 1144 .l. ar Previous Next lab 'lifllues Notes Calculator

40 • Hormone Reuptake inhibitor Therapy Enzyme inhibitor Secretion


41
D is not correct. 15% chose this.
42
Topiramate is an antiepileptic that blocks sod ium channels and augments the activity of the neurotransmitter y-
43
aminobutyric acid (GABA) . It is a third-line medication for bu limia, after trying fluoxetine and other seroton in
44 reuptake inhibitors (SSRis). It can he lp suppress binge urges in patients with bul imia and can also reduce their
45 preoccupation with eating and weight. The adverse effects of topiramate include sedation and an increased risk
of kidney stones.
46
Topiramate Fluoxetine Serotonin Neurotransmitter Gamma-Aminobutyric acid Bulimia nervosa Anticonvulsant Selective serotonin reuptake inhibitor Sedation
• 47
Sodium channel Sodium Kidney Kidney stone Reuptake Pharmaceutical drug
• 48
• 49 E is not correct. 11% chose this .
• 50
Trazodone is thought to be an antagon ist of 5-HT 2 , a 1-adrenergic and histamine- ! receptors. It is an
antidepressant that can be usefu l in the treatment of bulimia, although it is considered a third -line treatment
• 51 after trying fluoxetine and other serotonin reuptake inh ibitors (SSRis). The adverse effects are sedation,
• 52 nausea, priapism, and postural hypotension .
Priapism Fluoxetine Bulimia nervosa Trazodone Antidepressant Orthostatic hypotension Hypotension Serotonin Receptor antagonist
• 53
Selective serotonin reuptake inhibitor Nausea Sedation Antagonist Reuptake Receptor (biochemistry)
• 54
• 55
• 56 Bottom Line:
• 57
Bupropion is an antidepressant that is known to lower the seizure threshold. Be carefu l when using it in
• 58 patients with a seizure history or in patients with metabol ic abnorma lities such as hypokalemia .
• 59 Bupropion Hypokalemia Antidepressant Seizure threshold Epileptic seizure Metabolism

• 60

6
lock
s
Suspend
0
End Block
Item:46of73 ~. , . M k <:] t> al ~· ~
QIO: 1144 .l. ar Previous Next lab 'lifllues Notes Calculator

40 • •
41 FA17 p 537.1
42 Eating disorders Most common in young females.
43 Anorexia nervosa Excessive dieting, exercise, or binge eating/purging with B II < 18.5 kg/m 2; intense fear of gaining
44 weight; and distortion or overvaluation of body image. Associated with l bone density, severe
45 weight loss, metatarsal stress fractures, amenorrhea (due to loss of pu lsatile Cn RH secretion),
lanugo, anemia, electrolyte disturbances. Commonly coexists with depression. Psychotherapy and
46
nutritional rehabilitation are first line; pharmacotherapy includes SSR!s .
• 47
Refeeding syndrome (f insulin - hypophosphatemia - cardiac complications) can occur in
• 48 significantly malnourished patients .
• 49 Bulimia nervosa Binge eating with recurrent inappropriate compensatory behaviors (eg, self-induced vomiting,
• 50 using laxatives or diuretics, fasting, excessive exercise) occurring weekly for at least 3 months and
• 51 overvaluation of body image. Body weight often ma intained within normal range. Associated with
• 52
parotitis, enamel erosion, electrolyte disturbances (eg, hypokalemia, hypoch loremia), metabolic
alkalosis, dorsa l hand calluses from induced vomiting (Russel l sign). Treatment: psychotherapy,
• 53
nutritional rehabilitation, antidepressants.
• 54
Binge eating disorder Regular episodes of excessive, uncontrollable eating without inappropriate compensatory behaviors.
• 55 f risk of diabetes. Treatment: psychotherapy such as CBT is first-line; SSR!s, lisdexamfetamine.
• 56
• 57
FA17 p 542.2
• 58
Preferred medications PSYCHIATRICCONDITION PREFERRED DRUGS
• 59 for selected ADHD Stimulants (methylphenidate, amphetamines)
• 60 psychiatric conditions
• Alcohol withdrawal Bcnzodiazeoines (ee. chlordiazeooxidc. •

6
lock
s
Suspend
0
End Block
Item: 46 of 7 3 ~ 1 • Ma r k -<:J I> ~ £!1}>'
• !!":-~
QIO: 1144 ..L Pre v ious Next Lab lues Not es Calcula t o r

40 A A

41 FA17 p 542.2
42 Preferred medications PSYCHIATRIC CONDITION PREFERRED DRUGS
43 for selected ADHD Stimulants (methylphenidate, amphetamines)
44 psychiatric conditions
Alcohol withdrawal Benzodiazepines (eg, chlordiazepoxide,
45 lorazepam, diazepam)
46 Bipolar disorder Lithium, \'a lproic acid, carbamazepi ne,
• 47 lamotrigine, atypical antipsychotics
• 48 Bulimia ner\'osa SSRis
• 49 Depression SRis
• so Generalized anxiety disorder SSRis, S1 Rls
• 51 Obsessi\'e-compulsive disorder SSRis, \'enlafaxine, clomipramine
• 52
Panic disorder SSIUs, \'cnlafaxine, benzodiazepines
. 53
PTSD SSRis, venlafaxine
• 54
Schizophrenia Atypical antipsychotics
• 55
Social anxiety disorder SSRis, venlafaxine
• 56
Performance only: P-blockers, benzodiazepines
• 57
Tourette syndrome Antipsychotics (eg, Ruphenazine, pimozide),
. 58
let rabenazine
• 59
• 60
FA17 p 546.2

a
Lock
s
Suspend
8
End Bl ock
Item: 46 of 7 3 ~ 1 • M k -<:J 1>- Jil ~· !:';-~
QIO: 1144 ..L ar Pre v ious Next Labfli!llues Not es Calcula t o r

40 A A

41
FA17 p 546.2
42 Atypical antidepressants
43
Bupropion Inhibits reuptake of. E and dopamine. Also used for smoking cessation. Toxicity: stimulant effects
44 (tachycardia, insomnia), headache, seiwres in anorexic/bulimic patients. fay help alleviate
45 sexual dysfunction.
46 Mirtazapine ~-antagonist (t release of NE and 5-HT), potent 5-HT2 and 5-HT 3 receptor antagonist and H1
• 47 antagonist. Toxicity: sedation (which rna~ be desirable in depressed patients with insomnia),
• 48 t appetite, \\eight gain (which ma~ be desirable in elderly or anorexic patients), dr~ mouth .

• 49 Trazodone Primarily blocks 5-IIT 2, a 1-adrcnergic, and 11 1 receptors; aIso weakly inhibits 5-HT reuptake. Used
• so primarily for insomnia, as high doses arc needed for antidepressant effects. Toxicity: sedation,
nausea. priapism, postura l hypotension. Called traZZZobonc due to sedative and male-specific
• 51
side effects .
• 52
Varenicline Nicotinic ACh receptor partial agonist. Used for smoking cessation. Toxicity: sleep disturbance,
• 53
may depress mood .
• 54
Vilazodone Inhibits 5-HT reuptake; 5-IIT 1i\ receptor partial agon ist. Used for major depressive disorder
• 55 and generalized anxiety disorder (off-label). Toxicit y: headache, diarrhea, nausea, t weight,
• 56 antichol inergic effects. 1\ay cause seroton in syndrome if taken with other serotonergic agents.
• 57 Vortioxetine Inhibits 5-HT reuptakc; 5- H T~,, receptor agonist and 5-HT 3 receptor antagonist. Used for major
• 58 depressive disorder. Toxicity: nausea, sexual dysfunction, sleep disturbances (abnormal dreams),
• 59
anticholinergic effects. .\lay cause serotonin S) ndrome if taken with other serotonergic agents.
• 60

a
Lock
s
Suspend
8
End Bl ock
Item: 47 of 73 ~ 1 • M k -<:J 1>- Jil ~· !:';-~
QIO: 3747 ..L ar Pre v ious Next Labfli!llues Not es Calcula t o r

40
IAA]
A A

41
A 42-year-old man with no med ical history v isits a doctor after he develops sudden, un ilateral left-arm
weakness after an argument with his w ife. He states that he "stormed out" of the house but was unable to
42 open the car door. Physical examination f indings are normal except for 0/5 strength in his left arm and 2+
43 deep-tendon reflexes. Sensation is intact. CT scan of the head is negative for acute bleeding.
44
45 What is the most likely cause of his sudden arm weakness?
46 :
• 47 A. Conversion disorder
• 48
B. Factitious disorder
• 49
C. Illness anxiety disorder
• so
• 51 D. Malingering
• 52 E. Somatic symptom disorder
• 53
• 54
• 55
• 56
• 57
• 58
• 59
• 60

a
Lock
s
Suspend
8
End Bl ock
Item: 47 of 73 ~. I • M k <:] t> al ~· ~
QIO: 3747 .l. ar Previous Next Lab 'lifllues Notes Calculator

40 •
The correct answer is A. 58°/o chose this.
41
Conversion disorder (also known as functional neuro log ic symptom disorder ) mimics dysfunction in the
42 voluntary moto r or sensory system. Co mmon presentations include pseudoseizures, vocal cord dysfunction,
43 blindness, tunne l vision, deafness, and a variety of pa resthesias and para lyses. On ca reful clinical examination
44
and with the aid of laboratory investigations, these symptoms lack physiologic explanation . A cl inical example is
the presence of normal deep-tendon reflexes and norma l sensation in a person with a "paralyzed " arm . Patients
45
with conve rsion disorder involunta r ily have loss of function, usually in response to an unconscious conflict.
46 Conversion disorder Paresthesia Sensory system Symptom Stretch reflex Neurology Vocal folds Hearing loss Physical examination

47
B is not correct. 9°/o chose this .
• 48
Patients with factitious disorder volunta rily have a loss of function or voluntar ily do things to themselves to
• 49 create illness or inju ry, usual ly in response to an unconscious conflict, and seek prima ry gain in the form of
• 50 assumption of the sick role .
Factitious disorder Sick role Unconscious mind Primary and secondary gain
• 51
• 52 C is not correct. 6°/o chose this .
• 53 A Diagnostic and Statistical Manual, 5th ed it ion (DSM-5) diagnosis of illness anxiety disorder requ ires the
following criter ia: preoccupation with having or acquiring a serious, undiagnosed il lness, but somatic symptoms
• 54
themselves are mild or nonexistent . There is substantial anxiety about one's hea lth and patients demonstrate
• 55 excessive behaviors related to their health for at least 6 months.
• 56 DSM-5 Anxiety disorder Hypochondriasis Anxiety Somatic symptom disorder Somatic nervous system Diagnostic and Statistical Manual of Mental Disorders

• 57 Somatic anxiety

• 58
D is not correct. 6°/o chose this .
• 59 Patients with mal inger ing voluntarily have loss of function, usua lly consciously in response to know n situations,
• 60 and seek secondary gain in the form of tang ible gain ( moneta ry, housing, or avoidance of responsibilities) .

6
lock
s
Suspend
0
End Block
Item: 47 of 73 ~. I • M k <:] t> al ~· ~
QIO: 3747 .l. ar Previous Next Lab 'lifllues Notes Calculator

40 • C is not correct. 6°/o chose this . •


41 A Diagnostic and Statistical Manual, 5th ed it ion (DSM-5) diagnosis of illness anxiety disorder requ ires the
42 following criter ia: preoccupation with having or acquiring a serious, undiagnosed il lness, but somatic symptoms
43
themselves are mild or nonexistent . There is substantial anxiety about one's hea lth and patients demonstrate
excessive behaviors related to their health for at least 6 months.
44 DSM-5 Anxiety disorder Hypochondriasis Anxiety Somatic symptom disorder Somatic nervous system Diagnostic and Statistical Manual of Mental Disorders
45 Somatic anxiety
46
D is not correct. 6°/o chose this.
47
Patients with mal inger ing voluntarily have loss of function, usua lly consciously in response to known situations,
• 48
and seek secondary gain in the form of tang ible gain (monetary, housing, or avoidance of responsibilities) .
• 49 Malingering Primary and secondary gain

• 50
E is not correct. 21% chose this .
• 51
Somatic symptom disorder requ ires one or more somatic symptoms that cause a patient sign ificant distress or
• 52 psychosocia l impai r ment. I n addition, the patient demonstrates excessive thoughts, fee lings, or behaviors t
• 53 associated with the somatic symptoms for at least 6 months .
Somatic symptom disorder Somatization disorder Somatic nervous system Symptom
• 54
• 55
• 56 Bottom Line:
• 57 Conversion disorder manifests with dysfunction of motor or sensory systems without physiolog ic cause. Norma l
• 58 deep-tendon reflexes and norma l sensation in an arm that became "para lyzed" after a conflict are suggestive .
Stretch reflex Conversion disorder Sensory system
• 59
• 60
• •

6
lock
s
Suspend
0
End Block
Item: 47 of 73 ~ 1 • M k -<:J 1>- Jil ~· !:';-~
QIO: 3747 ..L ar Pre v ious Next Labfli!llues Not es Calcula t o r

40 A A

41 FA17 p 536.3
42 Somatic symptom and Category of disorders characterized by physical symptoms causing significant distress and
43 related disorders impairment. Both illness production and motivation arc unconscious drives. Symptoms not
44 intentionally produced or feigned. i\ lore common in women.
45 Somatic symptom Variety of bodily complaints (eg, pain, fatigue) lasting for months to years. Associated with
46 disorder excessi\·e, persistent thoughts and anxiet) about symptoms. i\ lay co-occur with medical illness.
Treatment: regular office \'isits with the same physician in combination with psychotherapy.
47
• 48
Conversion disorder Loss of sensory or motor fun ction (eg, paralysis, blindness, mutism), often following an acute
(functional stressor; patient is aware of but somet imes indifferent toward spnptoms ("Ia belle indifference");
• 49
neurologic symptom more common in females, ltdolcsccnts, and young adults.
• so disorder)
• 51 Illness anxiety Excessive preoccupation with acquiring or having a serious ill ness, often despite medical
• 52 disorder evaluation and reassurance; minimal somatic symptoms.
. 53 (hypochondriasis)
• 54
• 55 FA17 p 536.1

• 56 Malingering Patient consciously fakes, profoundly exaggerates, or claims to have a disorder in order to attain a
• 57 specific zo(external) gain (eg, avoiding work, obtaining compensation). Poor compliance with
treatment or follow-up of diagnostic tests. Complaints cease after gain (vs factit ious disorder).
. 58
• 59
FA17 p 536.2
• 60
• r ... ... .. • .. r ... .. ... .... ___ _.. __ _ ..... . II 1 1 1 "· 1 , ,

a
Lock
s
Suspend
8
End Bl ock
Item: 47 of 73 ~ 1 • M k -<:J 1>- Jil ~· !:';-~
QIO: 3747 ..L ar Pre v ious Next Labfli!llues Not es Calcula t o r

40 A A

Conversion disorder Loss of sensory or motor function (eg, paralysis, bli ndness, mutism), often following an acute
41 (functional stressor; patient is aware of but somet imes indifferent toward symptoms ("Ia belle indifference");
42 neurologic symptom more common in females, adolescents, and young adults.
43 disorder)
44 Illness anxiety Excessi,·e preoccupation with acquiring or having a serious illness, often despite medical
45 disorder e,-aJuation and reassurance; minimal somatic symptoms.
(hypochondriasis)
46
47
FA17 p 536.1
• 48

• 49
Malingering Patient consciously fakes, profoundly exaggerates, or claims to have a disorder in order to attain a
specific zo(external) gain (eg, avoiding \\ Ork, obtaining compensation). Poor compliance" ith
• so
treatment or follow-up of diagnostic tests. Complaints cease after gain (vs factitious disorder).
• 51
• 52
FA17 p 536.2
. 53
Factitious disorders Patient consciously creates physical and/or psychological symptoms in order to assume "sick role"
• 54
and to get medical attention and sympathy (1° [internal] gain).
• 55
Factitious disorder Also known as Munchauscn syndrome. Chronic factitious disorder with predominantly physical
• 56
imposed on self signs and symptoms. C haracterized by a history of mu ltiple hospital admissions and willingness to
• 57 undergo invasive procedures. More common in women and healthcare workers.
. 58 Factitious disorder Also known as Munchausen syndrome by proxy. Illness in a child or elderly patient is caused or
• 59 imposed on another fabricated by the caregi, er. l\ lotivation is to assume a sick role by proxy. Form of child/elder abuse.
• 60

a
Lock Suspend
s 8
End Bl ock
Item: 48 of 73 ~ 1 • M k -<:J 1>- Jil ~· !:';-~
QIO: 1131 ..L ar Pre v ious Next Labfli!llues Not es Calcula t o r

40
IAA]
A A

41
A 22-year-old man presents to his fami ly physician with complaints of insomn ia. He recently returned home
from an 18-month tour of duty in Iraq. He states that the insomnia began in Iraq several months ago after a
42 fierce nighttime battle. He reports having nightmares and flashbacks of the battle and is easily startled by
43 loud noises.
44
45 Which of the following pharmacologic agents, along with psychotherapy, would be most appropriate to treat this
46
patient's condition?

47 :

• 48 A. Buspirone
• 49 B. Carbamazepine
• so C. Propranolol
• 51
D. Sertraline
• 52
• 53 E. Trazodone
• 54
• 55
• 56
• 57
• 58
• 59
• 60

a
Lock
s
Suspend
8
End Bl ock
Item:48of73 ~. , . M k <:] t> al ~· ~
QIO: 1131 .l. ar Previous Next Lab 'lifllues Notes Calculator

40 •
41 The correct answer is D. 65°/o chose this.
42 This patient meets the criteria for diagnosis of post-traumatic stress diso rder (PTSD). He has expe rienced an
43 event that invo lved actua l death of or th reatened death to self or others; the traumatic event is persistent ly
reexperienced through nightmares and flashbacks; he has insomnia ; and he has an exaggerated startle
44
response. Other symptoms of PTSD include difficu lty concentrating, hypervigilance, and dissociative symptoms.
45 I n PTSD, symptoms are present for longer than 1 month, whereas, in acute stress disorder, symptoms last
46 between 3 days and 1 month. Selective se roton in reuptake inhibitors such as sertraline and pa roxetine are first-
line medications fo r the treatment of PTSD.
47
Sertraline Paroxetine Acute stress reaction Posttraumatic stress disorder Serotonin Hypervigilance Insomnia Selective serotonin reuptake inhibitor
48
Dissociation (psychology) Startle response Reuptake Stress (psychological) Flashback (psychology)
• 49
• 50 A is not correct. 10% chose this .
• 51
Buspi rone is a partial agonist at the 5-HT1A receptor that is common ly used as an alternative to benzodiazepines
in the treatment of general ized anxiety diso rder. Although its onset of action is slowe r than that of
• 52 benzodiazepines, it does not potentiate central nervous system depression from alcohol and has little potential
• 53 for abuse and addiction .
Partial agonist Buspirone Generalized anxiety disorder Central nervous system Anxiety disorder Serotonin Benzodiazepine Agonist Anxiety
• 54
• 55 Major depressive disorder Central nervous system depression Nervous system Depression (mood) Alcoholic beverage Alcohol Onset of action

• 56 Receptor (biochemistry)

• 57 B is not correct. 8°/o chose this .


• 58 Carbamazepine is an anticonvulsant med ication that also can be used as a mood stabilize r in bipola r mood
• 59 disorder .
Carbamazepine Anticonvulsant Mood stabilizer Mood disorder Bipolar disorder Pharmaceutical drug
• 60

6
lock
s
Suspend
0
End Block
Item:48of73 ~. , . M k <:] t> al ~· ~
QIO: 1131 .l. ar Previous Next Lab 'lifllues Notes Calculator

40 • Dissociation (psychology) Startle response Reuptake Stress (psychological) Flashback (psychology)

41 A is not correct. 10% chose this.


42 Buspirone is a partial agonist at the 5-HT1A receptor that is common ly used as an alternative to benzodiazepines
43 in the treatment of general ized anxiety disorder. Although its onset of action is slower than that of
44 benzodiazepines, it does not potentiate central nervous system depression from alcohol and has little potential
for abuse and addiction.
45 Partial agonist Buspirone Generalized anxiety disorder Central nervous system Anxiety disorder Serotonin Benzodiazepine Agonist Anxiety
46
Major depressive disorder Central nervous system depression Nervous system Depression (mood) Alcoholic beverage Alcohol Onset of action
47
Receptor (biochemistry)
48
• 49
B is not correct. 8°/o chose this .
Carbamazepine is an anticonvulsant med ication that also can be used as a mood stabilizer in bipola r mood t
• 50
disorder .
• 51 Carbamazepine Anticonvulsant Mood stabilizer Mood disorder Bipolar disorder Pharmaceutical drug

• 52
C is not correct. 5°/o chose this .
• 53
Propranolol is a nonspecific ~-blocker. It is useful in the treatment of panic disorder and simple phobia .
• 54 Propranolol Panic disorder Phobia

• 55
E is not correct. 12% chose this .
• 56
Trazodone is a heterocyclic antidepressant with sedative qua lities that is used in the treatment of depression
• 57 complicated by insomn ia. It works by inhibiting seroton in reuptake, but also acts as a partial seroton in agonist .
• 58 Male patients shou ld be warned of its potential to cause priapism .
Priapism Antidepressant Serotonin Trazodone Insomnia Sedative Agonist Serotonin reuptake inhibitor Serotonin receptor agonist Heterocyclic compound
• 59
Major depressive disorder Heterocyclic antidepressant Reuptake Depression (mood)
• 60

6
lock
s
Suspend
0
End Block
Item:48of73 ~. , . M k <:] t> al ~· ~
QIO: 1131 .l. ar Previous Next Lab 'lifllues Notes Calculator

40 • Partial agonist Buspirone Generalized anxiety disorder Central nervous system Anxiety disorder Serotonin Benzodiazepine Agonist Anxiety

41 Major depressive disorder Central nervous system depression Nervous system Depression (mood) Alcoholic beverage Alcohol Onset of action

42 Receptor (biochemistry)

43
B is not correct. 8°/o chose this.
44 Carbamazepine is an anticonvulsant med ication that also can be used as a mood stabilize r in bipola r mood
45 disorder.
Carbamazepine Anticonvulsant Mood stabilizer Mood disorder Bipolar disorder Pharmaceutical drug
46
47 C is not correct. 5°/o chose this.
48 Propranolol is a nonspecific ~-b l oc ker. It is useful in the treatment of panic disorder and simple phobia.
Propranolol Panic disorder Phobia
• 49
• 50 E is not correct. 12% chose this .
• 51 Trazodone is a heterocyclic antidep ressant with sedative qua lities that is used in the treatment of dep ression
• 52 complicated by insomn ia. It works by inhibiting se roton in reuptake, but also acts as a partial se roton in agonist .
Male patients shou ld be warned of its potential to cause priapism .
• 53
Priapism Antidepressant Serotonin Trazodone Insomnia Sedative Agonist Serotonin reuptake inhibitor Serotonin receptor agonist Heterocyclic compound
• 54
Major depressive disorder Heterocyclic antidepressant Reuptake Depression (mood)
• 55
• 56
• 57 Bottom Line:
• 58 First- line pha rmaco log ic treatment for PTSD is selective se roton in reuptake inhibitors. Optimum treatment of
• 59
PTSD shou ld also involve psychotherapy such as cogn it ive behaviora l therapy .
Cognitive behavioral therapy Serotonin Posttraumatic stress disorder Selective serotonin reuptake inhibitor Pharmacology Reuptake Psychotherapy
• 60

6
lock
s
Suspend
0
End Block
Item: 48 of 7 3 ~ 1 • M k -<:J 1>- Jil ~· !:';-~
QIO: 1131 ..L ar Pre v ious Next Labfli!llues Not es Calcula t o r

40 A A

41 FA17 p 534.1
42
Post-traumatic stress Exposure to prior trauma (cg, witnessing death, experiencing serious injury or rape) - persistent
43 disorder Ilyperarousal, .\ voidance of associated stimuli, intrusi,·e Reexperiencing of the event (nightmares,
44 Aashbacks), changes in cognition or mood (fear, horror, D istress) (ha,·ing PTSD is HARD).
45 Disturbance lasts> 1 month with significant distress or impaired social-occupational functioning.
Treatment: CBT, SSRis, and ' enlafa\ine arc first line. Prazosin can reduce nightmares.
46
47
Acute stress disorder- lasts between 3 days and 1 month. Treatment: CBT; pharmacotherapy is
usuallr not indicated .
48
• 49
FA17 p 545.1
• so
Selective serotonin F'luoxetine, Auvoxamine, paro"ctinc, scrtraline, c citalopram, citalopram.
• 51
reuptake inhibitors
• 52
MECHANISM Inhibit 5-HT rcuptakc. It normall y takes 4- 8 weeks for antidepressants
• 53
CLINICAL USE Depression, generalized anxiety d isorder, to have an effect.
• 54
panic d isorder, OCD, bulimia, social anxiety
• 55 disorder, PTSD, premature ejaculation,
• 56 premenstrual dysphoric cl isorder.
• 57 ADVERSE EFFECTS Fewer than TCAs. C l distress, SIADI I, sexual
• 58 dysfunction (anorgasmia, ! libido).
• 59
• 60 FA17 p 533.1

a
Lock
s
Suspend
8
End Bl ock
Item: 48 of 7 3 ~ 1 • M k -<:J 1>- Jil ~· !:';-~
QIO: 1131 ..L ar Pre v ious Next Labfli!llues Not es Calcula t o r

} '

41 CLINICAL USE Depression, generalized anxiety disorder, to have an effect.


42 panic disorder, OCD, bulimia, social anxiety
43 disorder, PTSD, premature ejaculation,
premenstrual d)'Sphoric disorder.
44
ADVERSE EFFECTS Fewer than TCAs. Cl distress, Sl 0 11, sc,ual
45
dysfunction (anorgasmia, I libido).
46
47
FA17 p 533.1
48
• 49 Panic disorder Defined by recurrent panic attacks (periods P\ '\JCS.
• so of intense fear and discomfort pc<~king in Diagno~is requires attack followed by I month
10 minutes with at least-+ of the folio" ing): (or more) of 1 (or more) of the following:
• 51
Palpitations, Paresthesias, dePersonalization or Persistent concern of additional attacks
• 52 derealization, Abdomi nal distress or "Jausca, Worrying about consequences of attack
• 53 Intense fear of dying, Intense fe;~ r of losing Behavioral change related to attacks
• 54 control or "going crazy," llght-hcadedncss, Symptoms are the systemic manifestations of
• 55 C hest pain, C hills, C hoking, Sweating, fea r.
Shaking, Shortness of breath. St rong genetic
• 56
component. t risk of suicide. Treatment:
• 57 CBT, SSRls, and vcnlafaxinc arc first line.
• 58 Benzodiazepines occasionally used in acute
• 59 setting.
• 60

a
Lock
s
Suspend
8
End Bl ock
Item: 49 of 73 ~ 1 • M k -<:J 1>- Jil ~· !:';-~
QIO: 1148 ..L ar Pre v ious Next Labfli!llues Not es Calcula t o r

40
IAA]
A A

41
An 8-year-old boy is brought to the pediatrician because his parents say that for the past year, he has had
increasing problems with repetitive behaviors such as sticking out his tongue and shutting his eyes. Recently
42 he has begun clearing his throat often. These behaviors do not appear to be under his control, and the
43 patient himself finds them very upsetting.
44
45 Which of the following drugs is indicated for the treatment of this patient?
46 :
47 A. Dextroamphetamine
48 B. Methylphenidate
• 49
C. Reserpine
• so
• 51 D. Tetrabenazine
• 52 E. Valproic acid
• 53
• 54
• 55
• 56
• 57
• 58
• 59
• 60

a
Lock
s
Suspend
8
End Bl ock
Item: 49 of 73 ~. I • M k <:] t> al ~· ~
QIO: 1148 .l. ar Previous Next lab 'lifllues Notes Calculator

40 •
The correct answer is D. 41°/o chose this.
41
Tetrabenazine is a monoamine transporter inhibitor which decreases the uptake of dopamine, serotonin,
42 norepinephrine and histamine into vesicles, thus depleting their stores. Chi ldren with Tourette syndrome display
43 involuntary motor and voca l tics. The disorder genera lly beg ins with the motor tics and then progresses to vocal
44 tics, w hich can include grunting, throat clearing, barking, or shouting. Coprolal ia (invo luntary swearing) is
uncommon and usually does not beg in until the teenage years . Most cases of Tourette syndrome regress by the
45
time patients are in their 30s. Tetrabenazine can cause sedation, fatigue, depression, and suicidal ideation.
46 Tourette syndrome Tetrabenazine Norepinephrine Serotonin Dopamine Histamine Suicidal ideation Monoamine transporter Sedation Monoamine neurotransmitter

47 Major depressive disorder Depression (mood) Enzyme inhibitor Tic Membrane transport protein Vesicle (biology and chemistry)
48
A is not correct. 9°/o chose this.
49
Amphetamines (eg, dextroamphetamine) are indirect sympathomimetics which inhibit the reuptake and
0 50 promote the release of catecholamines. These medications have dopaminergic activity and can actua lly worsen
0
51 voca l and motor t ics. As a central nervous system ( CNS) stimu lant, dextroemphetamine is used to manage
0
52
narco lepsy. Add itiona lly there's a role for its use in the treatment of attention deficit hyperactivity disorder
(ADHD) . Adverse effects include decreased appetite, dizziness, insomnia, and labile mood .
0 53
Attention deficit hyperactivity disorder Dextroamphetamine Narcolepsy Central nervous system Sympathomimetic drug Insomnia Catecholamine Dopaminergic
0 54
Substituted amphetamine Reuptake Stimulant Anorexia (symptom) Dizziness Nervous system Emotional dysregulation Tic
0 55
B is not correct. 15% chose this.
0
56
Methylphenidate is a CNS stimulant that increases norepinephrine and dopamine in the synaptic cleft. It is used
57
in the treatment of attention deficit hyperactivity disorder. Currently there is no evidence to support its use in
0

0 58 the treatment of isolated Tourette syndrome. Adverse effects include decreased appetite, dizziness, insomnia,
0 59 and labile mood.
Tourette syndrome Attention deficit hyperactivity disorder Methylphenidate Norepinephrine Dopamine Synaptic cleft Insomnia Stimulant Chemical synapse
0 60
• ,-. ......... _, .............. ,,.. ................... ~~. ........... ._:_ t .......................... \ "":--: ...... ,..,.. r ............: ...... -1 .-1 ............... 1-•: ...... • -&-.:1:.,, r ..... - ...... ...

6
lock
s
Suspend
0
End Block
Item: 49 of 73 ~. I • M k <:] t> al ~· ~
QIO: 1148 .l. ar Previous Next lab 'lifllues Notes Calculator

40 •
41
B is not correct. 15% chose this.
Methylphenidate is a CNS stimulant that increases norepinephrine and dopamine in the synaptic cleft. It is used
42
in the treatment of attention deficit hyperactivity disorder. Currently there is no evidence to support its use in
43 the t reatment of isolated Tourette syndrome. Adverse effects include decreased appetite, dizziness, insomnia,
44 and labile mood.
Tourette syndrome Attention deficit hyperactivity disorder Methylphenidate Norepinephrine Dopamine Synaptic cleft Insomnia Stimulant Chemical synapse
45
Central nervous system Anorexia (symptom) Dizziness Emotional dysregulation lability Synapse
46
47 C is not correct. 21% chose this.
48 Reserpine interferes with the storage of sympathetic amines (dopamine, norepinephrine) and causes their
49 depletion in centra l and peripheral neurons. It has antihypertensive effects, but due to the side effects, including
severe depression and extrapyramidal symptoms, it is not common ly used nowadays. Reserpine also has a role
50
0
for treating chorea associated with Huntington disease. However, tetrabenazine is more commonly used due to
0
51 its more favorab le side effect profile. Reserpine has no use in the treatment of Tourette syndrome.
Tourette syndrome Reserpine Tetrabenazine Huntington' s disease Norepinephrine Dopamine Extrapyramidal symptoms Antihypertensive drug Chorea Side effect
0
52
0 53 Neuron Extrapyramidal system Major depressive disorder Depression (mood) Adverse effect Amine Treatment of Tourette syndrome

0 54 E is not correct. 14% chose this.


0 55 Valproic acid inhibits sodium channels and also increases y-aminobutyric acid ( GABA) concent ration . It is an
0
56 antiseizure medication that can also be used for mania associated with bipolar disorder. Currently there is no
0 57
evidence to support its use in the treatment of Tou rette syndrome. Adverse effects of valproic acid include
pancreatitis, teratogenicity ( neu ral tube defects), weight ga in, t remor, and hepatotoxicity .
0 58 Anticonvulsant Tourette syndrome Bipolar disorder Valproate Hepatotoxicity Gamma-Aminobutyric acid Pancreatitis Tremor Neural tube Teratology
0 59 Sodium channel Mania Neural tube defect Sodium Pharmaceutical drug Weight gain
0 60

6
lock
s
Suspend
0
End Block
Item: 49 of 73 ~. I • M k <:] t> al ~· ~
QIO: 1148 .l. ar Previous Next lab 'lifllues Notes Calculator

40 •
Reserpine interferes with the storage of sympathetic amines (dopamine, norepinephrine) and causes their
41 depletion in centra l and peripheral neurons. It has antihypertensive effects, but due to the side effects, including
42 severe depression and extrapyramidal symptoms, it is not common ly used nowadays. Reserpine also has a role
43
for treating chorea associated with Huntington disease. However, tetrabenazine is more commonly used due to
its more favorab le side eff ect profile. Reserpine has no use in the treatment of Tourette syndrome.
44 Tourette syndrome Reserpine Tetrabenazine Huntington' s disease Norepinephrine Dopamine Extrapyramidal symptoms Antihypertensive drug Chorea Side effect
45 Neuron Extrapyramidal system Major depressive disorder Depression (mood) Adverse effect Amine Treatment of Tourette syndrome
46
E is not correct. 14% chose this.
47
Valproic acid inhibits sodium channels and also increases y-aminobutyric acid (GABA) concent ration . It is an
48 antiseizure medication that can also be used for mania associated with bipolar disorder. Currently there is no
49 evidence to support its use in the treatment of Tou rette syndrome. Adverse effects of valproic acid include
0 50 pancreatit is, teratogenicity (neu ral tube defects), weight ga in, t remor, and hepatotoxicity.
Anticonvulsant Tourette syndrome Bipolar disorder Valproate Hepatotoxicity Gamma-Aminobutyric acid Pancreatitis Tremor Neural tube Teratology
0
51
Sodium channel Mania Neural tube defect Sodium Pharmaceutical drug Weight gain
0
52
0 53
0 54 Bottom Line:
0 55 Tetrabenazine is the fi rst-l ine pharmacolog ic treatment indicated for Tou rette syndrome . Other phar macologic
0
56 therapies include typica l and atypical antipsychotic agents, such as fluphenazine and r isperidone,
0 57
respectively . Clon idine can also be used to treat this condition, but it shows variable efficacy compared with
the neu roleptics.
0 58 Fluphenazine Atypical antipsychotic Risperidone Tourette syndrome Clonidine Tetrabenazine Antipsychotic Pharmacology
0 59
0 60

6
lock
s
Suspend
0
End Block
Item: 49 of 7 3 ~ 1 • M k -<:J 1>- Jil ~· !:';-~
QIO: 1148 ..L ar Pre v ious Next Labfli!llues Not es Calcula t o r

40 A A

41 FA17p527.1

42 Childhood and early-onset disorders


43 Attention-deficit Onset before age 12. Limited attention span and poor impulse control. Characterized by
44 hyperactivity hyperactivity, impulsi' it~. and/or inattention in multiple settings (school, home, places of worship,
disorder etc). • onnal intelligence, but commonlr coexists '' ith difficulties in school. Continues into
45
adulthood in as many as 50% of individuals. Treatment : stimulants (eg, met h) lphenidate) +1-
46 cogniti,·c behavioraltherap) (C BT); alternati,es include atomoxetinc, guanfacinc, clonidine.
47 Autism spectrum Characterized by poor social interactions, soci:ll communication deficits, repetiti,·c/ritualized
48 disorder beha,·iors, restricted interests. lust present in carl) childhood. May be accompanied b)
49 intellectual disability; rarely accompanied by unusual abilities (savants). ~ lore common in boys.
• so Associated with t head/brain size.
• 51 Rett syndrome X-I inked dominant disorder seen almost c>.clusively in girls (affected males die in utero or short I)
• 52 aFter birth). Majority of cases arc caused by de novo mutation of MECP2. Symptoms usually
become apparent around ages 1- 4, includ ing regression characterized by loss of development, loss
. 53
of verbal abilities, intellectual disability, ata>.ia, stereotyped hand-wringing. o longer a solitary
• 54 diagnosis within DS~J-5 .
• 55
Conduct disorder Repetitive and pervasive behavior viola!ing the ba~ie rights of others or societa I norms (eg,
• 56 aggression to people and an imals, destruct ion of property, theft). After age 18, often reclassified as
• 57 antisocial personality disorder. Treatment for both: psychotherapy such as CBT.
. 58 Oppositional defiant Enduring pattern of hostile, defiant behavior toward authority figures in the absence of serious
• 59 disorder violations of social norms. Treatment: pS)chotherapy such as CBT
• 60 Separation anxiety Overwhelming fear of separation from home or attachment figure. Can be normal beha, ior up to
• _..-- .... . . .. ., t \. ..
' • r • ·• · • 1 I • ' ' • 1 • ,......nrt, t ,,

a
Lock
s
Suspend
8
End Bl ock
Item: 49 of 7 3 ~ 1 • M k -<:J 1>- Jil ~· !:';-~
QIO: 1148 ..L ar Pre v ious Next Labfli!llues Not es Calcula t o r

40 A A

FA17 p 543.1
41
Typical antipsychotics Haloperidol, pimozide, triAuopera:tinc, Auphena,rine, thioridal;ine, chlorprom a:~ine.
42
MECHANISM Block dopamine 0 2 receptor (f cA 1P).
43
CliNICAl USE Schizophrenia (1° positi,·c symptoms), psychos i~. bipolar disorder, delirium, Tourcttc syndrome,
44
Huntington disease, OCD.
45
POTENCY H igh potency: Trifluoperazine, fluphena7ine, llaloperidol (Try to FJ~, Iligh)- neurologic side
46
effects (eg, extrapyramidal S) m ptoms [ EPS]).
47 Low potencr: Chlorproma7ine, T h iorida1ine (C heating Thieves are low)- anticholinergic,
48 antihistamine, a 1-blockade effects.
49 ADVERSE EFFECTS Lipid soluble .... stored in body fat .... slow to be removed from body.
• so
Endocrine: dopamine receptor antagonism .... hypcrprolactinemia .... galactorrhea,
• 51 oligomenorrhea, gynccomas t i<L
• 52 l\letabolic: dyslipidemia, weight gain, hyperglycemia .
. 53 Anti muscarinic: dry mo uth, constipation .
• 54
Antihistamine: sedation .
a 1-blockade: orthostatic hypotension .
• 55
Cardiac: QT prolongation .
• 56 Ophthalmologic: C hlorpromazine- C orneal deposits; T hiorida zine-reT inal deposits.
• 57
EPS- ADAPT:
. 58
Hours to days: Acute D ystonia (muscle spasm , stiffness, oculogyric crisis).
• 59 Days to months: \ kathisia (restlessness), Parkinsonism (bradykinesia).
• 60 Months to yea rs: Tardive dyskinesia (orofacial chorea).
,..,___ .._____ ._ L ___ ._ ___ I • • ___ . _, L ____ J : ____ : ___ 0 Ll __ • ___ _
• !: _ _ '--~~ ~- I • I • • j

a
Lock
s
Suspend
8
End Bl ock
Item: 49 of 7 3 ~ 1 • M k -<:J 1>- Jil ~· !:';-~
QIO: 1148 ..L ar Pre v ious Next Labfli!llues Not es Calcula t o r

g, I
41
42 FA17 p 543.2
43 Atypical Aripiprazole, asenapine, clozapinc, olanzapinc, quctiapinc, iloperidone, paliperidone,
44 antipsychotics risperidone, lurasidonc, ziprasidonc.
45 MECHANISM Not completely understood. \ lost arc 0 2
46 antagonists; aripiprazole is D2 partial agonist.
47 Varied effects on 5-I IT 2, dopamine, and
48
a- and H1-reccptors.
49 CliNICAL USE Schizophrenia-both posith·e and negative Use clozapine for treatment-resistant
symptoms. Also used for bipolar disorder, schizophrenia or schizoaffecti\'e disorder and
• so
OCD, anxiety disorder, depression, mania, for su icidality in schizophrenia.
• 51
Tourette syndrome.
• 52
ADVERSE EFFECTS Al l- prolonged QT interval, fewer EPS and
. 53 antichol inergic side effects than typical
• 54 antipsychotics.
• 55 "-pines"- metabolic syndrome (weight gain, Olanzapine .... O besity
• 56
diabetes, hyperlipidemia) .
C lozapine-agranulocytosis (monitor WBCs Must· watch bone marrow clo7cly with clo1.apinc.
• 57
frequently) and seizures (dose related).
. 58 Risperidone-hyperprolactinemia (amenorrhea,
• 59 galactorrhea, gynecomastia).
• 60

a
Lock
s
Suspend
8
End Bl ock
Item: 50 of 73 ~ 1 • M k -<:J 1>- Jil ~· !:';-~
QIO: 1441 ..L ar Pre v ious Next Lab fli!ltues Not es Calcula t o r

40
IAA]
A A

41
A 45-year-old man who has received long-term t reat ment for schizophren ia recently has been displaying
involuntary facia l movements t hat include lateral deviations of the jaw and "fly cat ching" motions of the
42 tongue.
43
44 Which of the following agents is t he most likely cause of his involuntary movements?
45
:
46 A. Clozapine
47
B. Fluphenazine
48
49
C. Lithium
• so D. Selegiline
• 51
E. Ziprasidone
• 52
• 53
• 54
• 55
• 56
• 57
• 58
• 59
• 60

a
Lock
s
Suspend
8
End Bl ock
Item: 50 of 73 ~. I • M k <:] t> al ~· ~
QIO: 1441 .l. ar Previous Next lab 'lifllues Notes Calculator

40 •
41 The correct answer is B. 61 °/o chose this.
This patient is displaying signs of tardive dyskinesia, a complication of long-term antipsychotic use thought to
42
be the resu lt of increased dopamine recepto r synthesis in response to long-term receptor blockade by
43 antipsychotics. Abno rma l movements such as tongue thrusting and jaw deviations, as seen in th is patient, are
44 the resu lt of a relative dopamine excess affecting moto r pathways. This co mplication is encountered more
45 co mmonly with use of older, high potency, typica l antipsychotic medications, such as fluphenazine and
haloperido l.
46 Fluphenazine Typical antipsychotic Haloperidol Tardive dyskinesia Dopamine receptor Antipsychotic Dopamine Dyskinesia Receptor (biochemistry)
47
A is not correct. 18% chose this.
48
Clozapine, an atypical antipsychotic that modulates both serotoninergic and dopaminergic neu rons in the centra l
49
ne rvous system, has a relatively low risk of inducing ta rdive dyskinesia. The most concerning adverse effect of
50 clozapine is agranulocytosis, w hich can be fata l if left untreated .
• 51 Atypical antipsychotic Tardive dyskinesia Agranulocytosis Clozapine Central nervous system Antipsychotic Dopaminergic Dyskinesia Serotonergic Neuron

• 52 Nervous system Adverse effect

• 53 C is not correct. 6°/o chose this .


• 54 Lithium is a mood stabilizer that is used primari ly to treat episodes of mania in patients with bipolar disorde r.
• 55 Adverse effects of lith ium include nephrogenic diabetes insipidus, nausea, anorex ia, and mild diarrhea .
Bipolar disorder Mood stabilizer Diabetes insipidus Diarrhea Diabetes mellitus lithium Nephrogenic diabetes insipidus Nausea Anorexia (symptom)
• 56
Anorexia nervosa Mania
• 57
• 58 D is not correct. 6°/o chose this .
• 59 Se leg iline is a monoamine oxi dase B inhibito r that is used to treat Parkinson 's disease by decreasing the
• 60 breakdown of dopamine. It has no role in the treatment of sch izophrenia . Seleg iline has the opposite effect of
• antiosvchotics. which block the effects of dooamine. Adverse effects of se leai line include aastrointestinal uoset. •
6
lock
s
Suspend
0
End Block
Item: 50 of 73 ~. I • M k <:] t> al ~· ~
QIO: 1441 .l. ar Previous Next lab 'lifllues Notes Calculator

. - _ - ••• -· - - · - · · -- - Y.. -- -· ··-. - -- ·-- u ···- - · . - .- -

41
Adverse effects of lith ium include nephrogenic diabetes insipidus, nausea, anorexia, and mild diarrhea .
Bipolar disorder Mood stabilizer Diabetes insipidus Diarrhea Diabetes mellitus lithium Nephrogenic diabetes insipidus Nausea Anorexia (symptom)
42
Anorexia nervosa Mania
43
44 D is not correct. 6°/o chose this.
45
Se leg iline is a monoamine oxidase B inhibitor that is used to t reat Parkinson 's disease by decreasing the
breakdown of dopamine. It has no role in the treatment of sch izophrenia . Seleg iline has the opposite effect of
46 antipsychotics, which block the effects of dopamine. Adverse eff ects of se legi line include gastrointestinal upset,
47 nausea, heartburn, and dry mouth .
Selegiline Parkinson' s disease Monoamine oxidase B Dopamine Schizophrenia Antipsychotic Monoamine oxidase Nausea Monoamine oxidase inhibitor
48
Monoamine neurotransmitter Heartburn Xerostomia Enzyme inhibitor Gastrointestinal tract
49
50 E is not correct. 9°/o chose this.
• 51 Ziprasidone, like clozapine, is an atypical antipsychotic and has a lower incidence of tardive dyskinesia when
• 52 compared to typical antipsychotics. More often than other atypical antipsychotics, it has been associated with
QT prolongation and the risk of ma lignant ventr icular arrhythmias .
• 53
Atypical antipsychotic Ziprasidone Tardive dyskinesia Clozapine long QT syndrome Antipsychotic Dyskinesia Typical antipsychotic Cardiac arrhythmia
• 54
• 55
• 56 Bottom Line:
• 57 Tardive dys kinesia is a disorder characterized by stereotypic, repetitive ora l-facial movements. It may occu r
• 58
after long-term use of any antipsychotic agent, but is more prom inently seen with older, high potency,
"typical" antipsychotics .
• 59 Tardive dyskinesia Antipsychotic Dyskinesia Potency (pharmacology)
• 60

6
lock
s
Suspend
0
End Block
Item: 51 of 73 ~ 1 • M k -<:J 1>- Jil ~· !:';-~
QIO: 2519 ..L ar Pre v ious Next Labfli!llues Not es Calcula t o r

40 A A

41
A 23-year-old woman is brought to t he emergency department by her roommate after severa l hours of
worsening headache, irritability, and confusion. Her blood pressure is 235/140 mm Hg and she appears
42 intoxicated. She says that she thinks she might have had too much wine to drink at a cocktail party she
43 attended earl ier in the evening . Her past medical history is remarkable only for treatment by a psych iatrist;
44 however, the patient is reluctant to share the detai ls of her mental health diagnosis since she does not think it is
relevant to her current condition.
45
46
Which of the following medications is this patient most likely taking?
47
:
48
A. Amitriptyline
49
so B. Fluoxetine
• 51 C. Lith ium
• 52
D. Phenelzine
• 53
E. Ven lafaxine
0 54
0 55
• 56
• 57
• 58
0 59
0 60

a
Lock
s
Suspend
8
End Bl ock
Item:51of73 ~. , . M k <:] t> al ~· ~
QIO: 2519 .l. ar Previous Next Lab 'lifllues Notes Calculator

40 •
41 The correct answer is D. 62°/o chose this.
The patient was likely being t reated for depression with a monoamine oxidase inhibito r (MAOI). Phenelzine is a
42
nonse lective MAOI that is rarely used today due to its potential for ser ious dr ug interactions. Adve rse effects
43 include hypertensive crisis and rarely suba rachnoid hemo rrhage and stroke in patients who consume tyramine-
44 containing foods such as aged cheeses, liver, and red wine (as in th is case). MAOis prevent the metabo lism of
45 all monoamines, including tyramine. This t r iggers the release of catecho lamine stores and further potentiates
the presso r effects of high doses of sympathetic amines. Tyramine itse lf is a vasoactive compound. When
46
tyramine is consumed by a patient who is concu rrently taking an MAOI, it is not adequate ly metabol ized, and its
47 vasoactivity can cause a drastic increase in blood pressu re and a hypertensive crisis. A pe rson taking an MAOI is
48 also at increased r isk of hypertensive crisis and so-cal led "serotonin synd rome" if taken with meper idine,
49
selective serotonin reuptake inh ibitors (SSRis), or some t r icyclic antidepressants. Ultimately, an important point
he re is that although se rotonin norepinephrine receptor inhibitors and SSRis can resu lt in hypertension as seen
50 in "seroton in syndrome" and that the coadministration of MAOis with SSRis puts you at greater risk for
51 seroton in syndrome (and thus hypertension), in this case the patient exper ienced hypertension due to a
• 52 hypertensive crisis that occurred from the MAOI medication in conjunction with the excessive wine consumption .
Monoamine oxidase inhibitor Catecholamine Adrenergic receptor Phenelzine Serotonin syndrome Tyramine Subarachnoid hemorrhage Serotonin Norepinephrine
• 53
Hypertensive crisis Hypertension Depression (mood) Tricyclic antidepressant Major depressive disorder Monoamine oxidase Pethidine Vasoconstriction
• 54
Monoamine neurotransmitter Blood pressure Selective serotonin reuptake inhibitor Antidepressant Hypertensive emergency Antihypotensive agent Reuptake liver
• 55
Bleeding Drug metabolism Metabolism Stroke Binding selectivity Drug interaction Tricyclic Red wine Amine Pharmaceutical drug Enzyme inhibitor
• 56
Receptor (biochemistry)
• 57
• 58 A is not correct. 15% chose this .
• 59 Amitr iptyl ine is a tricycl ic antidepressant associated with the "3 C's": Convulsions, Coma, and Cardiotoxicity .
• 60
Other adverse effects include sedation, confusion, memory loss, and mania .
• Tricyclic antidepressant Amitriptyline Antidepressant Sedation Mania Tricyclic

6
lock
s
Suspend
0
End Block
Item:51of73 ~. , . M k <:] t> al ~· ~
QIO: 2519 .l. ar Previous Next Lab 'lifllues Notes Calculator

40 •
A is not correct. 15% chose this.
41
Amitr iptyl ine is a t ricycl ic antidepressant associated with the "3 C's": Convulsions, Coma, and Card iotoxicity .
42 Other adverse effects include sedation, confusion, memory loss, and mania.
43 Tricyclic antidepressant Amitriptyline Antidepressant Sedation Mania Tricyclic

44 B is not correct. 10% chose this.


45 Fluoxetine is a selective serotonin reuptake inh ibitor associated with anxiety, insomnia, tremo r, and nausea. If
46 taken in conjunction with an MAO inh ibitor, it may lead to serotonin syndrome, characte rized by anxiety,
diaphoresis, tachycardia, hypertension, hyperthe rmia, vomiting, diarrhea, tremo r, hype rreflexia, and muscle
47
r igidity . Other adverse effects include headache, sexua l dysfunction, weight loss, and, in some patients, blunted
48 affect.
49 Monoamine oxidase inhibitor Selective serotonin reuptake inhibitor Serotonin syndrome Fluoxetine Hyperreflexia Serotonin Hyperthermia Perspiration Insomnia

50 Tachycardia Diarrhea Hypertension Sexual dysfunction Serotonin reuptake inhibitor Nausea Reuptake Headache Vomiting Anxiety Weight loss Hypertonia

51 Reduced affect display Tremor Reuptake inhibitor Muscle Enzyme inhibitor Monoamine oxidase

• 52
C is not correct. 7°/o chose this .
• 53 Lithium intake has been associated with nausea, vomiting, diarrhea, edema, weight gain, tremor,
• 54 hypothyroidism, and neph rogenic diabetes insipidus .
Hypothyroidism Diabetes insipidus Diabetes mellitus Diarrhea Edema Vomiting Nephrogenic diabetes insipidus Nausea Tremor lithium
• 55
• 56 E is not correct. 6°/o chose this .
• 57 Venlafaxine is a heterocycl ic antidepressant that may cause anxiety, ag itation, headache, weight loss, and
• 58 insomn ia .
Venlafaxine Antidepressant Insomnia Headache Heterocyclic compound Anxiety Weight loss
• 59
• 60

6
lock
s
Suspend
0
End Block
Item:51of73 ~. , . M k <:] t> al ~· ~
QIO: 2519 .l. ar Previous Next Lab 'lifllues Notes Calculator

40 p • • y, •
41
taken in conj unct ion with an MAO inh ibitor, it may lead to seroton in syndrome, charact erized by anxiety,
diaphoresis, tachycardia, hypertension, hypertherm ia, vomit ing, diarrhea, tremor, hyperreflexia, and muscle
42 rigidity . Ot her adverse effects include headache, sexua l dysfunction, weight loss, and, in some patients, blunt ed
43 affect .
Monoamine oxidase inhibitor Selective serotonin reuptake inhibitor Serotonin syndrome Fluoxetine Hyperreflexia Serotonin Hyperthermia Perspiration Insomnia
44
45 Tachycardia Diarrhea Hypertension Sexual dysfunction Serotonin reuptake inhibitor Nausea Reuptake Headache Vomiting Anxiety Weight loss Hypertonia

Reduced affect display Tremor Reuptake inhibitor Muscle Enzyme inhibitor Monoamine oxidase
46
47 C is not correct. 7°/o chose this.
48 Lith ium intake has been associated with nausea, vomiting, diarrhea, edema, weight gain, tremor,
49 hypothyroidism, and nephrogenic diabetes insipidus.
Hypothyroidism Diabetes insipidus Diabetes mellitus Diarrhea Edema Vomiting Nephrogenic diabetes insipidus Nausea Tremor lithium
50
51 E is not correct. 6°/o chose this.
• 52 Ven lafaxine is a heterocyclic antidepressant that may cause anxiety, ag itation, headache, weight loss, and
• 53
insomn ia .
Venlafaxine Antidepressant Insomnia Headache Heterocyclic compound Anxiety Weight loss
• 54
• 55
• 56 Bottom Line :
• 57 MAOis such as phene lzine can lead to hypertensive crisis in people who consume tyramine- containing foods
• 58 such as red wine, liver, and aged cheese .
Phenelzine Hypertensive crisis Hypertensive emergency Monoamine oxidase inhibitor Cheese ripening Red wine liver Hypertension
• 59
• 60

6
lock
s
Suspend
0
End Block
Item: 51 of 7 3 ~ 1 • M k -<:J 1>- Jil ~· !:';-~
QIO: 2519 ..L ar Pre v ious Next Labfli!llues Not es Calcula t o r

40 A A

FA17 p546.1
41
42 Monoamine oxidase Tranylcyprom ine, P henelzine, Isocarhoxazid, Selegilinc (selecti,·e ~.\0-B inhibitor).
inhibitors (~lAO Takes Pride In Shanghai).
43
44 MECHANISM 'onselecti,·e \!lAO inhibition t b·cl ~ of amine neurotransmitters (norepinephrine, 5-I IT,
dopamine).
45
CLINICAL USE Atypical depression, an-:iet). Parl.inson disease (selegiline).
46
47
ADVERSE EFFECTS C l$ stimulation; hr pertensive crisis, most notably ''ith ingestion of tyramine, which is found in
m any foods such as aged cheese and wine. Tyramine displaces other neurotransmitters (eg, N E)
48
into the sp1aptic cleft - t sympalhelic stimulation. Contraindicated with SSRls, T CAs, St. John's
49 wort, meperidine, dexiromethorphan (to prevent serotonin srndrome).
so Wait 2 \\·eeks after stopping MAO inhibitors before starting serotonergic drugs or stopping dietary
51 restrictions.
. 52
. 53 FA17 p 545.1
0 54
Selective serotonin Fluoxetine, Auvoxa mine, paroxeline, scrlralinc, cseila lopmm, citalopram.
0 55 reuptake inhibitors
• 56 MECHANISM Inhibit 5-I IT reupta kc. It normally takes 4-8 weeks for anticlepressanls
• 57 to have an effect.
CLINICAL USE Depression, generalized anxiety disorder,
. 58 panic disorder, OCD, bulimia, social amiely
0 59 disorder, PTSD, premalure ejaeulalion,
premenstTual dysphoric disorder.
0 60
• .' ,.,...._ . -· ... ..... .. -·.
aLock
s
Suspend
8
End Bl ock
Item: 51 of 7 3 ~ 1 • M k -<:J 1>- Jil ~· !:';-~
QIO: 2519 ..L ar Pre v ious Next Labfli!llues Not es Calcula t o r

40 A A

41 FA17 p545.1
42 Selective serotonin F'luoxetine, Atl\'oxamine, paroxetine, sertraline, escitalopram, citalopram.
43 reuptake inhibitors
44 MECHANISM Inhibit 5-HT reuptake. It normally takes 4-8 weeks for antidepressants
45 CliNICAL USE Depression, generalized anxiety disorder, to have an effect.
46 panic disorder, OCD, bulimia, social an\iety
47 disorder, PTSD, premature ejaculat ion,
premenstrual dysphoric disorder.
48
ADVERSE EFFECTS Fewer than TCAs. Cl distress, SIADII, se,ual
49
dysfunction (anorgasmia, l Iibido).
so
51
. 52 FA17 p 544.3

. 53 Antidepressants
0 54 NORADRENERGIC SEROTONERGIC
0 55
AXON AXON
• 56
~ MAO i nhl bltors -
• 57 -o- -f MAO Metabolites
Metabolites - - MAO
. 58
\
0

0
59
60

NE - -


_;:::'"'- I ~~~ptOf)
¢----s-HT
l
a Lock
s
Suspend
8
End Bl ock
Item: 51 of 7 3 ~ 1 • Ma r k -<:J I> ~ £!1}>'
• !!":-~
QIO: 2519 ..L Pre v ious Next Lab lues Not es Calcula t o r

40 A A

Antidepressants
41
42 NORADRENERGIC SEROTONERGIC

43
AAN AAON
44 MAO ~ MAO inhlbltors -
,.....__
-0- _,-f MAO
- - • Metabolites
45 Metabollt~ - - -

46 Buproplon \
47
48
49
so
51 Mlrtazaplne TCAs, SSRis,
SNRis, trazodone
• 52
. 53
• 54
S.HT receptor
• 55
• 56
• 57
. 58
• 59 POSTSYNAPTIC NEURON

• 60

a
Lock
s
Suspend
8
End Bl ock
Item: 52 of 73 ~ 1 • M k -<:J 1>- Jil ~· !:';-~
QIO: 2487 ..L ar Pre v ious Next Lab fli!ltues Not es Calcula t o r

40 A A

41
A 43-year-old woman presents to the emergency department fo llowing a motor vehicle co llision . She
sustained no serious physical injuries, but she appears drowsy and states that she is feeling very sleepy.
42 Emergency medica l staff col lected several open m ed ication bottles from the f loor of her automobile. Although
43 the contents of two of the bottles could not be deter mined, the patient's recently filled bottle of diazepam is almost
44 empty. The patient quickly becomes unresponsive and stops breathing.
45
46
What is the mechanism of action of the agent that would reverse her symptoms?

47 :

48 A . Competitive agonist of y-aminobutyric acid(A) {GABA-A) receptors


49 B. Competitive agonist of y-am inobutyric acid(B) {GABA-B) receptors
so C. Competitive agonist of glycine receptors
51
D. Competitive inhibitor of y-aminobutyric acid(A) {GABA-A) receptors
. 52
. 53 E. Competit ive inhibitor of y -aminobutyric acid(B) {GABA-B) receptors
0 54 F. Competit ive inhibitor of glycine receptors
0 55
• 56
• 57
. 58
0 59
0 60

a
Lock
s
Suspend
8
End Bl ock
Item: 52 of 73 ~. I • M k <:] t> al ~· ~
QIO: 2487 .l. ar Previous Next lab 'lifllues Notes Calculator

40 •
41 The correct answer is D. 72°/o chose this.
42
The patient has overdosed on diazepam, a benzodiazepine . Flumazeni l is a competitive antagonist of the
benzodiazepam y-aminobutyric recepto r. Its use can prevent or reve rse the central ne rvous system effects of
43
benzodiazepine overdose. Specifical ly, as a competit ive antagon ist of the y-aminobutyric acid(A) (GABA-A)
44 receptors (wh ich are found postsynaptica lly and are inotropic/ ligand-gated ion channels), Flumazeni l wi ll
45 outcompete the ingested benzodiazepine for a spot on the GABA receptor, thus reducing the benzos effect.
GABA is a major inhibitory neurotransmitter in the centra l nervous system, and on activation of GABA-A
46
receptors, c l- enters the cel l, resulting in hyperpo larization and the subsequent inh ibition of new action
47 potentials.
48 Benzodiazepine Diazepam Flumazenil Benzodiazepine overdose GABA receptor Central nervous system Neurotransmitter Gamma-Aminobutyric acid

49 Hyperpolarization (biology) Receptor antagonist Action potential Competitive antagonist Ion channel Nervous system Drug overdose GABAA receptor Antagonist

50 Ion Receptor (biochemistry) Chloride

51 A is not correct. 10% chose this.


52 Competit ive agon ists of GABA-A receptors wou ld actua lly make this patient's cond it ion worse . Examples of such
• 53 agonists include ibotenic acid and muscimo l, but these would not be used to treat this patient in th is condition .
• 54 Ibotenic acid Muscimol GABA receptor Receptor (biochemistry) GABAA receptor

• 55 B is not correct. 5°/o chose this .


• 56 Baclofen is the best known example of GABA-B receptor agonists, and is a GABA ana log. I t is used as a muscle
• 57 relaxant, but can exacerbate absence seizures so it is no longer used to treat epi lepsy .
Baclofen Epilepsy Gamma-Aminobutyric acid Absence seizure Muscle relaxant GABAB receptor Structural analog Epileptic seizure Receptor (biochemistry) Muscle
• 58
• 59 C is not correct. 2°/o chose this .
• 60 Glycine receptor agonists include sarcosine, which is found in foods like egg yol ks and ham, and
• tr-i n-u:.thv11'11vri n"' ;,n nrt'1;.nir rnn-~nn11nrl fn 11nrl i n c:1 11'1;>r h"'"'tc:: 1\l.,.ith"'r \hll'\1 drl h"' l n tn rn• •nt"' r c::vn-~ntnn-~c:: inrl••r"'rl

6
lock
s
Suspend
0
End Block
Item: 52 of 73 ~. I • M k <:] t> al ~· ~
QIO: 2487 .l. ar Previous Next lab 'lifllues Notes Calculator

I • • • g • p p y
Baclofen Epilepsy Gamma-Aminobutyric acid Absence seizure Muscle relaxant GABAB receptor Structural analog Epileptic seizure Receptor (biochemistry) Muscle
41
42 C is not correct. 2°/o chose this.
43 Glycine receptor agonists include sarcosine, which is found in foods like egg yolks and ham, and
44 t rimet hylglycine, an organic compound found in suga r beets. Neither wou ld help t o count er symptoms induced
by benzodiazepine overdose.
45
Benzodiazepine Organic compound Glycine receptor Sarcosine Glycine Trimethylglycine Benzodiazepine overdose Agonist Receptor (biochemistry)
46
47
E is not correct. 10% chose this. t
y -Aminobutyric acid ( B) (GABA-B) recept ors are different f rom GABA-A receptors in t hat t he for me r are
48
metabotropic receptors t hat are linked to G prot eins . CGP is a compound know n t o be an antagon ist for GABA- 8
49 recept ors.
50 Receptor antagonist GABA receptor GABAB receptor G protein Metabotropic receptor Receptor (biochemistry) Antagonist GABAA receptor Protein

51 F is not correct. 1 °/o chose this.


52 Glycine receptors are t he recept or for the amino acid neu rot ransmitter glycine, w hich has an inhibit ory effect on
• 53 the nervous syst em . Ant agonists of glycine include bicucul line and strychnine, bot h of w hich cause seve re
• 54 convu lsions that can lead to death .
Bicuculline Strychnine Amino acid Neurotransmitter Glycine Amino acid neurotransmitter Convulsion Nervous system Glycine receptor
• 55
• 56
• 57 Bottom Line:
• 58 Flumazenil is an antidote for benzodiazepine overdose, which wor ks as a competit ive inhibit or of the GABA-A
• 59 recept or .
Benzodiazepine Flumazenil Benzodiazepine overdose GABAA receptor Competitive inhibition Antidote Drug overdose Receptor (biochemistry)
• 60

6
lock
s
Suspend
0
End Block
Item: 52 of 73 ~ 1 • M k -<:J 1>- Jil ~· !:';-~
QIO: 2487 ..L ar Pre v ious Next Lab fli!ltues Not es Calcula t o r

40 A A

FA17 p 515.2
41
Benzodiazepines Diazepam, lorazepam, triazolam, lemazepam, oxazepam, midazolam, chlordiazepoxide,
42
alprazolam.
43
MECHANISM Facilitate GABA,\ action by t frequency of "Frenzodiazepines" t frequency.
44
cJ- channel opening. l REi\ I sleep. Nlost Benzos, barbs, and alcohol all bind the
45 have long half-Ji,·es and acti,·e metabolites GABA \ receptor, which is a ligand-gated CJ-
46 (exceptions (ATO~ l]: Alprazolam, Triazolam, channel.
47 Oxazepam, and ~ lidazolam are short acting Oxazepam, Temazepam, and Lorazepam are
48 .... higher addictive potential). metabolized O utside T he Li,·er
49 CLINICAL USE Anxiety, spasticity, status epilepl icus (lorazepam
and diazepam), eclampsia, detoxification
so
(especially alcohol withdrawal-DTs), night
51
terrors, sleepwalking, general anesthetic
52 (t~mncsia, muscle relaxation), hypnotic
. 53 (insomnia).
0 54 ADVERSE EFFECTS Dependence, additive CNS depression effects
0 55 with alcohol. Less risk of respiratory depression
• 56 and coma than with barbiturates .
Treat overdose with Aumazenil (compclit i\C
• 57
antagonist at GABA benzodiazepine receptor).
. 58
Can precipitate seizures by causing acute
0 59 benzodiazepine withdrawal.
0 60

a
Lock
s
Suspend
8
End Bl ock
Item: 52 of 73 ~. I • M k <:] t> al ~· ~
QIO: 2487 .l. ar Previous Next lab 'lifllues Notes Calculator

40 • •
41 FA17 p 540.1

42 Psychoactive drug intoxication and withdrawal


DRUG ltiTOXICATION WITHDRAWAL
43
Depressants
44
45 onspecific: mood elevation, l anxiety, onspecific: anxiety, tremor, seizures,
. .
sedation, behavioral disinhibition, respiratory lllSOmnaa.
46
depression.
47
Alcohol Emotional lability, slurred speech, ataxia, Time from last drink:
48
coma, blackouts. Serum y-glutamyltransferase 3-36 hr: minor symptoms similar to other
49 (GGT)-sensitive indicator of alcohol usc. depressants
50 AST value is twice ALT value. 6- 48 hr: withdrawal seizures
51 12-48 hr: alcoholic hallucinosis (usually visual)
48-96 hr: delirium tremens (DTs) in 5% of
52
cases
• 53
Treatment: benzodiazepines.
0 54
Opioids Euphoria, respiratory and CNS depression, Sweating, dilated pupils, piloerection ("cold
0 55 l gag reAex, pupillary constriction (pinpoint turkey"), fever, rhinorrhea, yawning, nausea,
• 56 pupils), seizures (overdose). Most common stomach cramps, diarrhea {"Au-like" symptoms).
• 57 cause of drug overdose death. Treatment: Treatment: long-term support, methadone,
• 58 naloxone. buprenorphine.
0 59 Barbiturates Low safety margin, marked respiratory Delirium, life-threatening cardiovascular
depression. Treatment: symptom management collapse.
0 60
• (eg. assist res1Jiration. t BP). •

6
lock
s
Suspend
0
End Block
Item: 53 of 73 ~ 1 • M k -<:J 1>- Jil ~· !:';-~
QIO: 1124 ..L ar Pre v ious Next Labfli!llues Not es Calcula t o r

40
IAA]
A A

41
A 65-year-old man presents to his family doctor for a regu larly scheduled check- up. He is recently widowed
and appears unkem pt at th is visit. He admits to feeling depressed since his wife's death. He reports a 4 .5-kg
42 ( 10-lb) weight loss in the past few months. On further questioning, the patient admits that he plans to shoot
43 himself in the head on the 3-month anniversary of his wife's death.
44
45 Which of the following actions is most appropriate in the care of this patient?
46 :
47 A. Explore the patient's concerns and schedule a follow-up visit
48 B. Hospitalize voluntarily or involuntarily
49
C. Prescribe an antidepressant
so
51 D. Refer to a psychiatrist
52 E. Tell the patient's chi ldren so that they can intervene
• 53
• 54
• 55
• 56
• 57
. 58
• 59
• 60

a
Lock
s
Suspend
8
End Bl ock
Item: 53 of 73 ~. I • M k <:] t> al ~· ~
QIO: 1124 .l. ar Previous Next Lab 'lifllues Notes Calculator

40 •
The correct answer is B. 79°/o chose this.
41
This patient has a number of risk facto rs for suicide, including an organ ized plan, male sex, advanced age,
42
depressed mood, and single status. Othe r risk factors include a previous suicide attempt, ethanol abuse, loss of
43 rationa l thinking, lack of social support, and ch ron ic illness. I f a patient has suicidal ideation and is deemed to
44 be at high risk of harming himself, hospita lization is warranted (whether voluntary or involuntary). The principle
45
of beneficence obl iges a physician to prevent or remove harm and promote the wel l-being of a patient,
especia lly when impaired mental faculties preclude an autonomous choice .
46 Suicidal ideation Ethanol Alcohol abuse Social support Depression (mood) Chronic condition Suicide Physician

47
A is not correct. 8°/o chose this.
48
It is necessary to explore the patient's concerns and talk to him about the cause of his depressed mood and
49 suicida l ideation . However, su icide plans need to be taken seriously and treated appropriately .
50 Suicidal ideation Depression (mood) Suicide Ideation (creative process)

51
C is not correct. 4°/o chose this.
52 This patient may be given an antidepressant wh ile in the hospita l, but a prescription alone is insufficient care for
53 this patient, given the severity of his suicidal ideation. In addition, antidepressants may take up to 1 month to
0 54 have a therapeutic effect.
Antidepressant Suicidal ideation Therapeutic effect Suicide crisis Ideation (creative process)
0 55
0
56 D is not correct. 8°/o chose this.
0 57 Although the patient wi ll certainly benefit from referra l to a psychiatrist, he shou ld be hospital ized because he
manifests an imminent th reat to himself.
0 58
Psychiatrist
0 59
0 60
E is not correct. 1°/o chose this.
• ThP n;:~tiPnt h;:~c; nnt hPPn riPrl;:~rpri inrnmnPt'Pnt· it ic; thPrPfn r P ;:~ hrp;:~rh nf rnnfiriPnti;:~ l i tv tn t'PI I h ie; rhi lrirPn

6
lock
s
Suspend
0
End Block
Item: 53 of 73 ~. I • M k <:] t> al ~· ~
QIO: 1124 .l. ar Previous Next lab 'lifllues Notes Calculator
y p p I • I p I I I

41 suicida l ideation . However, su icide plans need to be taken seriously and treated appropriately .
Suicidal ideation Depression (mood) Suicide Ideation (creative process)
42
43 C is not correct. 4°/o chose this.
44 This patient m ay be given an antidepressant wh ile in the hospita l, but a prescription alone is insufficient care for
this patient, given the severity of his suicidal ideation. In addition, antidepressants may take up to 1 month to
45
have a therapeutic effect.
46 Antidepressant Suicidal ideation Therapeutic effect Suicide crisis Ideation (creative process)

47
D is not correct. 8°/o chose this.
48
Although the patient wi ll certainly benefit from referra l to a psychiatrist, he should be hospital ized because he
49 m anif ests an im minent th reat to himself.
50 Psychiatrist

51 E is not correct. 1°/o chose this.


52 The patient has not been declared incom petent; it is t herefore a breach of confidentia lity to tel l his children
53 without his consent.
legal professional privilege in England and Wales
0 54
0 55
0
56 Bottom Line:
0 57 I f a patient has suicidal ideations and is deem ed to be at high risk to harm himself, involuntary hospital ization
0 58 is an appropriate intervention.
Involuntary commitment Emergency psychiatry
0 59
0 60

6
lock
s
Suspend
0
End Block
Item: 53 of 73 ~ 1 • M k -<:J 1>- Jil ~· !:';-~
QIO: 1124 ..L ar Pre v ious Next Labfli!llues Not es Calcula t o r

40 A A

41 FA17 p 532.4

42 Risk factors for suicide Sex (male) S \D P E RSONS are more likely to complete
43 completion Age (young adull or elderly) suicide.
Depression 1\ lost com mon method in US is firearms; access
44
P re,·ious attempt (highest risk factor} to guns t risk of suicide completion.
45
Ethanol or drug usc \Vomen try more often; men complete more
46 Rational thinking loss (psychosis) often.
47 Sickness (medical illness)
48 O rganized plan
l\o spouse or other social support
49
Stated future intent
so
51
FA17 p 531 .3
52
Major depressive Episodes characterized by at least 5 of D iagnostic srmploms:
53
disorder the 9 d iagnostic symptoms lasting :!: 2 SIC E CAP S:
0 54
weeks (symptoms must include patient·- • D epressed mood
0 55 rcported depressed mood or anhedon ia). • Sleep disturbance
0
56 Treatment: C BT and SSRis arc first line. • Loss of Interest (anhedon ia)
• 57 S ' Ris, mi rtazapine, bupropion can also be • G uilt or feelings of wo rthlessness
considered. Antidepressants are indicated if Energy loss and fatigue
. 58
bipolar disorder is ruled out. Electrocom ulsi\ e C oncentration problems
0 59
therapy (ECT) in select patients. \ ppetite/weight changes
0 60 Psychomotor retardation or agitation
• PPr<:i<:tPnt dPnrP<:<:ivP tii<:nrtiPr ltiv<:thvmi~l -

a
Lock
s
Suspend
8
End Bl ock
Item: 53 of 7 3 ~ 1 • M k -<:J 1>- Jil ~· !:';-~
QIO: 1124 ..L ar Pre v ious Next Labfli!llues Not es Calcula t o r

40
41
42 FA17 p 531 .3

43 Major depressive Episodes characterized by al least 5 of Diagnostic symptoms:


44 disorder the 9 diagnostic symptoms lasting ~ 2 SIC L CAPS:
weeks (symptoms must include patient- Depressed mood
45
reported depressed mood or anhedonia). Sleep disturbance
46
Treatment: CBT and SSRis arc first line. Loss of Interest (anhedonia)
47 S ' Ris, mirtazapine, bupropion can also be Guilt or feelings of worthlessness
48 considered. Antidepressants are indicated if Energy loss and fatigue
49 bipolar disorder is ruled oul. Eleclrocom ulsive Concentration problems
so therapy (ECT) in select patients. \ ppetite/weight changes
Psychomotor retardation or agitation
51 Persistent depressive disorder (dysthymia)-
Suicidal ideations
52 depression, often mi lder, lasting at least
Patients with depression typically have the
2 years.
53 follow ing changes in their sleep stages:
0 54 • ! slow-wave sleep
0 55 • ! REM latency
• t REM early in sleep cycle
0 56
• t total REM sleep
• 57 Repeated nighttime awakenings
. 58 Early-morning awakening (term inal
0 59 insomnia)
0 60

a
Lock
s
Suspend
8
End Bl ock
Item: 54 of 73 ~. I • M k <:] t> al ~· ~
QIO: 1141 .l. ar Previous Next lab 'lifllues Notes Calculator

40 •
41
A 36-year-old man is brought to the psychiatr ist by his wife because of changes in his behavior. The patient
has become somewhat reclusive and introverted since beginn ing a new job 1 year ago . The patient reports
42 nausea and pa lpitations du r ing stressfu l situations, such as when giving a presentation . He also reports that
43 he has been reluctant to leave the house because he is afra id that he wi ll embarrass himself and others will judge
44 hi m. Review of systems is negative. His med ical history is significant for asthma and high cho lesterol. He is not
allergic to any medications.
45
46
Wh ich of the fo llowing pharmacolog ic agents is contraindicated in this patient?
47
:
48
A. Alp razolam
49
50 B. Amitr iptyl ine
51 C. Esmolol
52
D. Fluoxetine
53
E. Propranolol
0 54
0 55
0 56
0 57
0 58
0 59
0 60

6
lock
s
Suspend
0
End Block
Item: 54 of 73 ~. I • M k <:] t> al ~· ~
QIO: 1141 .l. ar Previous Next lab 'lifllues Notes Calculator

40 •
The correct answer is E. 48°/o chose this.
41
Although propranolo l is often a first- line t reatment to consider, it is contraindicated in this patient because of his
42 asthma. Propranolol is a nonselective ~-b l ocke r that is useful in treating patients with some of the symptoms
43 associated with performance anxiety and phobias (tachycardia, hype rhidrosis) . Since it is nonse lective,
44 however, it is contraind icated in patients with asthma because of the risk of bronchial constriction from blockade
45
of ~ 2 receptors. It is worth noting that ca rdioselective ~-blockers, such as metoprolol, do not share the same
contra indication, due to the ~ 1 > > ~ 2 selectivity, significantly reducing or eliminating the bronchoconstrictive
46 effects .
47 Propranolol Metoprolol Hyperhidrosis Tachycardia Asthma Therapy Stage fright Anxiety Contraindication Bronchoconstriction Bronchus First-line treatment

48 Receptor (biochemistry)

49
A is not correct. 10% chose this.
50 Alprazolam is a short-acting benzod iazepine that functions as an anxiolytic. It is not contraindicated for this
51 particular patient, although it is not fi rst-l ine therapy for social phobia. Because of abuse potential, reeva luation
52 of the drug after several months of use is important.
Benzodiazepine Anxiolytic Alprazolam Social anxiety disorder Substance abuse Contraindication Phobia
53
54 B is not correct. 16% chose this.
• 55 Tricyclic antidepressants such as amit riptyline are not fi rst-l ine treatment for socia l phobia . They are conside red
to be less effective than the se lective serotonin reuptake inh ibitors .
• 56
Amitriptyline Serotonin Social anxiety disorder Selective serotonin reuptake inhibitor Tricyclic antidepressant Reuptake Antidepressant Therapy Tricyclic Phobia
• 57
C is not correct. 13% chose this .
• 58
Esmolol is a cardioselective ~-b l ocke r that works specifically by blocking ~ 1 receptors in the heart, ther by
• 59
decreasing the rate and force of cardiac contractions. Esmolol is short acting, but has rapid onset, ma king it
• 60 effective in the treatment of aortic dissection, and it is common ly used during surgery to prevent tachycardia .

6
lock
s
Suspend
0
End Block
Item: 54 of 73 ~. I • M k <:] t> al ~· ~
QIO: 1141 .l. ar Previous Next lab 'lifllues Notes Calculator

40 •
41 A is not correct. 10% chose this.
42
Alprazolam is a short-acting benzod iazepine that functions as an anxiolytic. It is not contraindicated for this
particular patient, although it is not fi rst-l ine therapy for social phobia. Because of abuse potential, reeva luation
43
of the drug after several months of use is important.
44 Benzodiazepine Anxiolytic Alprazolam Social anxiety disorder Substance abuse Contraindication Phobia

45
B is not correct. 16% chose this.
46
Tricyclic antidepressants such as amitriptyline are not fi rst-l ine treatment for socia l phobia . They are considered
47 to be less effective than the se lective serotonin reuptake inh ibitors.
48 Amitriptyline Serotonin Social anxiety disorder Selective serotonin reuptake inhibitor Tricyclic antidepressant Reuptake Antidepressant Therapy Tricyclic Phobia

49 C is not correct. 13% chose this.


50 Esmolol is a cardioselective ~-b l ocker that works specifically by blocking ~ 1 receptors in the heart, ther by
51 decreasing the rate and force of cardiac contractions. Esmolol is short acting, but has rapid onset, making it
effective in the treatment of aortic dissection, and it is common ly used during surgery to prevent tachycardia.
52
Because esmolol acts specifical ly on ~ 1 receptors, it wou ld not be contraindicated in the patient. This ~-b l ocker
53 would not be indicated to treat phobias or other psychiatric disorders.
54 Aortic dissection Tachycardia Esmolol

• 55 D is not correct. 13% chose this .


• 56 Se lective serotonin reuptake inhibitors such as fluoxetine are the treatment of choice for social phobia. They
• 57 lack abuse potential, and the adverse effects are generally wel l tolerated . Socia l phobia is an irrationa l fear of
• 58 socia l or performance situations that the patient recognizes as irrational . Patients can begin to avoid al l social
situations in order to avert the unpleasant feelings that are aroused . Social phobia tends to have an insidious
• 59
onset and remains a chronic prob lem. Relapse is common after pha r macologic therapy is stopped .
• 60 Fluoxetine Serotonin Social anxiety disorder Selective serotonin reuptake inhibitor Reuptake Phobia Substance abuse Pharmacology Relapse

6
lock
s
Suspend
0
End Block
Item: 54 of 73 ~. I • M k <:] t> al ~· ~
QIO: 1141 .l. ar Previous Next lab 'lifllues Notes Calculator

40 Amitriptyline Serotonin Social anxiety disorder Selective serotonin reuptake inhibitor Tricyclic antidepressant Reuptake Antidepressant Therapy Tricyclic Phobia
41
C is not correct. 13% chose this.
42
Esmolol is a cardioselective ~-b l ocker that works specifically by blocking ~ 1 receptors in the heart, ther by
43
decreasing the rate and force of cardiac contractions. Esmolol is short acting, but has rapid onset, making it
44 effective in the treatment of aortic dissection, and it is common ly used during surgery to prevent tachycardia.
45 Because esmolol acts specifical ly on ~ 1 receptors, it wou ld not be contraindicated in the patient. This ~-b l ocker
would not be ind icated to treat phobias or other psychiatric disorders.
46
Aortic dissection Tachycardia Esmolol
47
48
D is not correct. 13% chose this.
49
Se lective serotonin reuptake inhibitors such as fluoxetine are the treatment of choice for social phobia. They
lack abuse potential, and the adverse effects are generally wel l tolerated . Socia l phobia is an irrationa l fear of
50 socia l or performance situations that the patient recognizes as irrational . Patients can begin to avoid al l social
51 situations in order to avert the unpleasant feelings that are aroused . Social phobia tends to have an insidious
52 onset and remains a chronic prob lem. Relapse is common after pha r macologic therapy is stopped .
Fluoxetine Serotonin Social anxiety disorder Selective serotonin reuptake inhibitor Reuptake Phobia Substance abuse Pharmacology Relapse
53
54
• 55 Bottom Line:
• 56 ~-Blocke rsare relative ly contraindicated in the setting of lung disease or airway hyperreactivity (asthma)
• 57 because of antagon ism of ~ 2 receptors, which can lead to bronchoconstriction . Selective ~rblocke rs like
esmolol and metoprolol may be used with carefu l monitoring. ~ - Blockers are useful for treating some of the
• 58
symptoms associated with specific phobias, including social phobias, but the first-line therapy is an SSRI.
• 59 Metoprolol Asthma Esmolol Respiratory tract Social anxiety disorder Contraindication lung Respiratory disease

• 60

6
lock
s
Suspend
0
End Block
Item: 54 of 73 ~. I • M k <:] t> al ~· ~
QIO: 1141 .l. ar Previous Next lab 'lifllues Notes Calculator

40 • •
FA17p237.1
41
p~blockers Acebutolol, atenolol, betaxolol, bisoprolol, carvedilol, esmolol, labetalol, metoprolol, nadolol,
42
nebivolol, pindolol, propranolol, timoloL
43
APPLICATION ACTIONS NOTES/EXAMPLES
44 Angina pectoris ~ heart rate and contractility, resulting in ~ 0 2
45 consumption
46 Myocardial infarction ~ mortality
47 Supraventricular ~ AV conduction ,·elocity (class II Metoprolol, esmolol
48 tachycardia antiarrhythm ic)
49 Hypertension ~ cardiac output, ~ re nin secretion (due to
50 P1-receptor blockade on JCA cells)
51 Heart failure ! mortality (bisoprolol, carved ilol, metoprolol)
52 Glaucoma ~ production of aqueous humor Timolol
53 Variceal b leeding ~ hepatic venous pressure gradien t and portal adolol, propranolol
54 hypertension
• 55 ADVERSE EFFECTS Erectile dysfunction, cardiovascular adverse Use with caution in cocaine users due to risk
• 56 effects (bradycardia, AV block, llF), CNS of unopposed a-adrenergic receptor agonist
• 57 adverse effects (seizures, sedation, sleep activity
• 58
alterations), dyslipidemia (metoprolol), and
asthma/COPD exacerbations
• 59
• 60
SELECTIVITY P1-selective antagonists (P1 > Pz) - acebutolol Selective antagonists mostly go from A to M (P1
• (partial agonist), ate nolol, betaxolol, b isoprolol, with l st half of alphabet) •

6
lock
s
Suspend
0
End Block
Item: 54 of 7 3 ~ 1 • M k -<:J 1>- Jil ~· !:';-~
QIO: 1141 ..L ar Pre v ious Next Labfli!llues Not es Calcula t o r

40 A
~ -- - ~.
A

41
SELEaiVITY > ~)- aceblltolo l
~ 1 -sel ective antagonists (~ 1 Selective antagonists mostlr go from A to :\I (~ 1
(partial agonist), atenolol, betaxolol, bisoprolol, with 1st half of alphabet)
42
esmolol, metoprolol
43
'onselecti,·e antagonists (~ 1 = ~2 )- nadolol, 1\onselecti\'e antagonists mostly go from r\ to Z
44
pindolol (partial agonist), propranolol, timolol (~ "ith 2nd half of alphabet)
45
1'\onselecti,·e ex- and ~-antagon ists-can·ecli lol , 'onselecti,·e ex- and ~-a ntagonists ha,·e modified
46 labetalol suffixes (instead of"-olol ")
47 ebi"olol combines cardiac-selecti\'e
48 f3.-adrenergic blockade with stimulation of
49 f3,-receptors (acti,·ate nitric oxide synthase in
so the ,·asculature and l SVR)
51
52 FA17 p 533.2

53 Specific phobia Severe, persi stent(~ 6 months) fear or anxiety due to presence or anticipation of a specific object or
54 situation. Person often recognizes fear is excessive. Can be treated with systematic desensitization.
• 55 Social anxiety disorder-exaggerated fear of embarrassment in social situations (eg, public
• 56 speaking, using public restrooms). Treatment: CBT, SSRis, venlafaxine. For only occasiona l
• 57
anxiety-inducing situat ions, benz.odiazepine or ~-bloc ke r.
. 58 Agoraphobia- irrational fear/anxiet) ''hile facing or anticipating~ 2 specific situations (eg, open/
• 59 closed spaces, lines, crowd~. public transport). If se,·ere, patients may refuse to lea,·e their homes .
ssociated with panic disorder. Treatment: C BT, S Rls.
• 60

a
Lock Suspend
s 8
End Bl ock
Item: 54 of 7 3 ~ 1 • M k -<:J 1>- Jil ~· !:';-~
QIO: 1141 ..L ar Pre v ious Next Labfli!llues Not es Calcula t o r

40 A A

FA17 p 533.2
41
Specific phobia Se,·ere, pe rsi ste nt(~ 6 months) fear or anxiety due lo presence or anticipation of a specific object or
42
situation. Person often recogni1es fear is excessi, e. Can be treated" ith systematic desensitization.
43
44
Social anxiety disorder-exaggerated fear of embarrassment in social situations (eg, public
speaking, using public restrooms). Treatment: C BT, Rls, ,·enlafaxine. F'or only occasional
45
anxiety-inducing situations, benzoclia7cpine or ~-bl ocker.
46
Agoraphobia - irrational fear/anxiet} "hile facing or anticipating 2: 2 specific situations (eg, open/
47
closed spaces, lines, crowds, public transport). If Se\'ere, patients may refuse to lea,·e their homes.
48
Associated with panic disorder. Treatment: C B'I~ IUs.
49
so FA17 p 545.1
51
Selective serotonin F'luoxetine, Au,·oxamine, paroxetine, sertraline, escitalopram, citalopram.
52 reuptake inhibitors
53
MECHANISM Inhibit 5-HT reuptake. lt normally takes 4- 8 weeks for antidepressants
54 to have an effect.
CLINICAL USE Depression, generalized anxiety disorder,
• 55 pan ic disorder, OCD, bulimia, social anxiety
• 56 disorder, PTSD, premature ejaculation,
• 57 premenstrual dysphoric disorder.
. 58 ADVERSE EFFECTS Fewer than TCAs. GI distress, SIADII , sexual
• 59 dysfunction (anorgasmia, l libido).
• 60

a
Lock
s
Suspend
8
End Bl ock
Item: 55 of 73 ~ 1 • M k -<:J 1>- Jil ~· !:';-~ ~
QIO: 4214 ..L ar Pre v ious Next Lab fli!ltues Not es Calcula t o r

40
IAA]
A A

41
A disheveled-appearing 50-year-old man is brought into the emergency department. His neurolog ic
examination is sign ificant for somnolence, inability to perform tandem walking, and orientation on ly to self.
42 He is also found to have hepatomega ly and nystagmus. Ultimately, the patient is determined to have a
43 v itamin deficiency.
44
45 The classic vitamin deficiency for this patient's condition is likely to result in what laboratory abnormality?
46 :
47 A . AST:ALT ratio of 1:1
48 B. High serum lactate and low pH
49
C. High urine metanephrine
so
51 D. High urine protein
52 E. Low serum haptoglobin
53
54
0 55
0 56
• 57
. 58
0 59
0 60

a
Lock
s
Suspend
8
End Bl ock
Item: 55 of 73 ~ 1 • M k -<:J 1>- Jil ~· !:';-~
QIO: 4214 ..L ar Pre v ious Next Lab fli!ltues Not es Calcula t o r

40
IAA]
A A

41
A disheveled-appearing 50-year-old man is brought into the emergency department. His neurolog ic
examination is sign ificant for somnolence, inability to perform tandem walking, and orientation on ly to self.
42 He is also found to have hepatomega ly and nystagmus. Ultimately, the patient is determined to have a
43 v itamin deficiency.
44
45 The classic vitamin deficiency for this patient's condition is likely to result in what laboratory abnormality?
46 :
47 A . AST:ALT ratio of 1:1
48 B. High serum lactate and low pH
49
C. High urine metanephrine
so
51 D. High urine protein
52 E. Low serum haptoglobin
53
54
0 55
0 56
• 57
. 58
0 59
0 60

a
Lock
s
Suspend
8
End Bl ock
Item: 55 of 73 ~. I • M k <:] t> al ~· ~
QIO: 4214 .l. ar Previous Next lab 'lifllues Notes Calculator

40 •
The correct answer is B. 59°/o chose this.
41
This patient is most likely an intoxicated alcoho lic. Thiamine Glucose

~
42 deficiency is commonly seen in association with chronic alcohol ism
aKG Glu NAOH NAO•
seconda ry to dieta ry insufficiency or impaired intestinal absorption . \.) f
43
Thiamine deficiency can man ifest in the fo rm of Wernicke
\ ) Pvntv;~tp Lactate
Pyruvate

44
encepha lopathy, w hich manifests with ataxia, latera l gaze pa lsy,
45 and confusion; or Korsa koff psychosis, wh ich resu lts in
46 anterograde and retrograde amnesia, often with confabu lation. As
47 illustrated in the diag ram, thiamine is also an essentia l cofactor fo r
the enzyme pyruvate dehydrogenase in the conversion of pyruvate
48
to acetyl-coenzyme A. Th iamine deficiency causes shunting of
49 pyruvate to lactate, resu lting in lactic acidosis.
Retrograde amnesia Thiamine lactic acidosis Cofactor (biochemistry) Confabulation Enzyme Ataxia Pyruvate dehydrogenase Pyruvic acid Psychosis
50
51 Wernicke' s encephalopathy lactic acid Alcoholism Amnesia Acidosis Encephalopathy Thiamine deficiency Anterograde amnesia

52 A is not correct. 9°/o chose this.


53 Elevated se rum alanine aminotransferase:aspartate aminotransfe rase ratio is found in viral hepatitis, with
54 extremely high leve ls, often in the 1000s. A serum aspartate aminotransferase:a lan ine aminotransferase ratio
of 2-3 : 1 is found in alcoholic hepatit is. However, this patient's psychiatric symptoms are mo re suggestive of
55
thiamine deficiency than alcoho lic hepatitis .
• 56 Alcoholic hepatitis Alanine transaminase Aspartate transaminase Thiamine Viral hepatitis Hepatitis Alanine Blood plasma Aspartic acid Alcoholism Transaminase
• 57
C is not correct. 10% chose this .
• 58
A high urine metanephrine level is found in pheochromocytoma, which typically presents with episodes of
• 59 headaches, palpitations, and diaphoresis in association with severe hypertension .
• 60 Pheochromocytoma Metanephrine Perspiration Palpitations Hypertension Urine

6
lock
s
Suspend
0
End Block
Item: 55 of 73 ~. I • M k <:] t> al ~· ~
QIO: 4214 .l. ar Previous Next lab 'lifllues Notes Calculator

40 •
Elevated serum alanine aminotransferase :aspartate am inotransferase ratio is found in vira l hepatitis, with
41
extremely high leve ls, often in the 1000s. A serum aspartate aminotransferase :alan ine aminotransferase ratio
42 of 2-3 : 1 is found in alcohol ic hepatit is. However, this patient's psychiatric symptoms are more suggestive of
43 thiamine deficiency than alcoho lic hepatit is.
Alcoholic hepatitis Alanine transaminase Aspartate transaminase Thiamine Viral hepatitis Hepatitis Alanine Blood plasma Aspartic acid Alcoholism Transaminase
44
45 C is not correct. 10% chose this.
46 A high urine metanephrine level is found in pheochromocytoma, which typically presents with episodes of
headaches, pa lpitations, and diaphoresis in association with severe hypertension.
47
Pheochromocytoma Metanephrine Perspiration Palpitations Hypertension Urine
48
49 D is not co rrect. 8 °/o chose this.
High urine protein is found in nephrotic syndrome, which also presents with edema, hypoalbuminemia, and
50
hyperlipidem ia.
51 Nephrotic syndrome Hypoalbuminemia Hyperlipidemia Protein Urine Edema

52
E is not co rrect. 14% chose this.
53
Low serum haptoglobin is found in intravascu lar hemo lysis, as haptog lobin binds to free heme.
54 Hemolysis Heme Haptoglobin Hemolytic anemia Blood plasma Serum (blood) Blood vessel
55
• 56
Bottom Line :
• 57
• 58 Th iamine deficiency is common ly seen in alcoho lics and can resu lt in Wern icke-Korsakoff syndrome. Thiamine
deficiency can also resu lt in lactic acidosis, as it is a cofactor for pyruvate dehydrogenase .
• 59 Wernicke-Korsakoff syndrome Thiamine Cofactor (biochemistry) Pyruvic acid lactic acidosis Pyruvate dehydrogenase Thiamine deficiency Alcoholism Acidosis
• 60

6
lock
s
Suspend
0
End Block
Item: 55 of 7 3 ~ 1 • M k -<:J 1>- Jil ~· !:';-~
QIO: 4214 ..L ar Pre v ious Next Lab fli!ltues Not es Calcula t o r

40 A A

FA17 p 541.2
41
Alcoholism Physiologic tolerance and dependence on alcohol with symptoms of withdrawal when intake is
42
interrupted.
43
Complications: alcoholic cirrhosis, hepatitis, pancreatitis, peripheral neuropathy, testicular atrophy.
44 Treatment: disulfiram (to condition the patient to abstain from alcohol use), acamprosate,
45 naltrexone, supporti,·e care. Support groups such as Alcoholics Anonymous are helpful in
46 sustaining abstinence and supporting patient and famil}.
47 Wernicke-Korsakoff Caused by ,·itamin B1 deficiency. Triad of confusion, ophthalmoplegia, ataxia (Wernicke
48 syndrome encephalopathy). f\ lay progress to irreversible memory loss, confabuJation, personality change
(Korsakoff syndrome). Symptoms may be precipitated by gi,·ing dextrose before administering
49
,·itamin B1 to a patient with thiamine deficiency. Associated with peri,·entricular hemorrhage/
so necrosis of mammillary bodies. Treatment: I vitamin B1•
51
52 FA17 p 62.2
53 Vitamin 8 1 (thiamine)
54 FUNCTION ln thiamine pyrophosphate (TPP), a cofactor for Think ATP: a -ketoglutarate dehydrogenase,
55 several dehydrogenase enzyme reactions: Transketolase, and Pyruvate dehydrogenase.
• 56
Pyruvate dehydrogenase (links glycolysis to Spell beriberi as Berl Ber1 lo remember
TCA crcle) vitam in n, .
• 57
• a-ketoglutarate dehydrogenase (TCA cycle) Wernicke-Korsakoff syndrome- confusion,
. 58 • Transketolasc (HMP shunt) ophthalmoplegia, ataxia (classic triad)+
• 59 Branched-chain kctoacid dehydrogenase confabulation, personality change, memory
• 60 DEFICIENCY Impaired glucose breakdown - r\TP depletion loss (permanent). Damage to medial dorsal
• ...,. •• ,..) ,... •• ,. " ( J-h,..l,..~ooro ,.......,...,... ........ ;)Jnro • J....,..,l;,..,.

a
Lock
s
Suspend
8
End Bl ock
Item: 55 of 7 3 ~ 1 • M k -<:J 1>- Jil ~· !:';-~
QIO: 4214 ..L ar Pre v ious Next Lab fli!ltues Not es Calcula t o r

DEFICIENCY lmp<med glucose breakdown - ATP depletiOn oss (per a e1 t). I amage to me 1a c orsa
41 worsened by glucose infusion; highly aerobic nucleus of thalamus, mammillary bodies.
tissues (eg, brain, heart) are affected first. Dry beriberi -polyneuritis, symmetrical muscle
42
In alcoholic or malnourished patients, wast in g.
43
gi,·e thiamine before dextrose to l risk of Wet beriberi -high-output cardiac failure
44 (dilated cardiomyopathy), edema.
precipitating Wernicke encephalopath).
45 Diagnosis made by t in RBC transketolase
46 activity following vitamin B1administration.
47
48 FA17 p 73.2

49 Pyruvate metabolism Functions of different pyrm·ate metabolic


so pathways (and their associated cofaclors):
51
Glucose 0 lanine aminotransferase (8 6): alanine
carries am ino groups to the liver from
52
53
t
Pyruvate
muscle
E) Pyruvate carboxylase (biotin): oxaloacetate
54 can replen ish TCA cycle or be used in
55
0 gluconeogenesis
Alamne
• 56 Cah1ll cycle Cytosol 0 E) Pyruvate dehydrogenase (B1, B2, B~, B5,
Lactate
Mitochondria Con cycle lipoic acid): transition from glycolysis to
• 57
the TC cycle
. 58
0 Lactic acid dehydrogenase (B3): end of
• 59 anaerobic glycolysis {major pathway in
• 60 RBCs, WBCs, kidney medulla, lens,
• '
a
Lock
s
Suspend
8
End Bl ock
Item: 56 of 73 ~. I • M k <:] t> al ~· ~
QIO: 1943 .l. ar Previous Next Lab 'lifllues Notes Calculator

40 •
41
A 27-year-old woman complains of weakness, fatigue, decreased appetite, and insomnia over the past
month. She is no longer interested in the activities she used to engage in, and has been unable to
42 concentrate at work. She also reports feel ing guilty about not social izing with her friends even though they
43 ask her out almost every wee kend . As part of her treatment plan, her physician prescribes a med ication, though
44 she does not respond and goes through many first-line treatment options. On the latest follow-up visit, she reports
that her mood has improved, but she now fee ls that her face flushes more frequently and she is more sensitive to
45
the hot weather outside. She is also worried that at times she feels li ke her heart is racing . On further question ing,
46 she admits to some constipation .
47
48 Wh ich of the fo llowing drugs was most likely prescribed for th is patient?
49
:
50 A. Amitriptyl ine
51
B. Clonazepam
52
C. Lithium
53
54 D. Sertra line
55 E. Venlafaxine
• 56
• 57
• 58
• 59
• 60

6
lock
s
Suspend
0
End Block
Item: 56 of 73 ~. I • M k <:] t> al ~· ~
QIO: 1943 .l. ar Previous Next Lab 'lifllues Notes Calculator

40 •
41 The correct answer is A. 59°/o chose this.
42
This patient is being t reated for depression. Amitriptyl ine, a tricycl ic antidepressant (TCA), is as effective as the
selective serotonin reuptake inh ibitors (SSRis), but often is not prescribed as a fi rst-l ine agent because of its
43
many adverse effects. These include sedation, a-blocking effects, and, most common ly, anticholine rgic effects
44 such as d ry mouth, blur ry vision, tachycardia, urinary retent ion, constipation, confusion, and dry, hot skin.
45 These adverse effects can be remembered with the fol lowing mnemonic : red as a beet (flushing), dry as a bone
( anh idrosis), hot as a hare (overheating secondary to anh idrosis), blind as a bat (blur ry vision), mad as a
46
hatter (hal lucinations or del irium), and full as a flask (u r inary retention) . Of note, monoamine oxidase inhibitors
47 ( MAOis), another class of antidepressants, do not have antichol inergic properties, but can cause adverse effects
48 similar to those of antichol inergic med ications, including dry mouth, postural hypotension, headache, and
49 urinary hesitancy. They produce these symptoms by the stimu lation of sympathetic nerves, rather than the
inhibition of parasympathetics. Therefore, they shou ld be safe to use in patients with urinary retention, even
50 though they may cause some hesitancy. MAOis often are associated with tyramine crises ( hypertensive crises)
51 on the USMLE exam, especial ly w hen the patient described has consumed tyramine-rich foods, such as red
52 wi nes and aged cheeses; they are also associated with serotonin syndrome along with other serotonin-
increasing drugs (TCAs and SSRis) . Serotonin synd rome typically presents with hyperthermia, flushing, and
53
diarr hea.
54 Tricyclic antidepressant Anticholinergic Serotonin syndrome Amitriptyline Tyramine Serotonin Antidepressant Urinary retention Constipation

55 Orthostatic hypotension Hypotension Tachycardia Sympathetic nervous system Monoamine oxidase inhibitor Selective serotonin reuptake inhibitor Hyperthermia

56 Depression (mood) Hypertensive emergency Major depressive disorder Xerostomia Monoamine oxidase Sedation Reuptake Headache Monoamine neurotransmitter
• 57 Diarrhea Autonomic nervous system Delirium Hallucination Parasympathetic nervous system Flushing (physiology) Blurred vision Tricyclic
• 58
B is not correct. 7°/o chose this .
• 59
Clonazepam is a benzodiazepine sometimes prescr ibed as an anxiolytic at the init iation of antidepressant
• 60 therapy. The most common ly reported adverse effects are those associated with central nervous system

6
lock
s
Suspend
0
End Block
Item: 56 of 73 ~. I • M k <:] t> al ~· ~
QIO: 1943 .l. ar Previous Next Lab 'lifllues Notes Calculator

40 •
B is not correct. 7°/o chose this.
41
Clonazepam is a benzodiazepine sometimes prescribed as an anxiolytic at the init iation of antidep ressant
42
the rapy. The most common ly reported adverse effects are those associated with central nervous system
43 depression, such as sedation or respi rato ry depression at higher doses. Dependence and rebound anxiety can
44 resu lt f rom benzodiazepine abuse.
Benzodiazepine Anxiolytic Clonazepam Hypoventilation Central nervous system Antidepressant Rebound effect Benzodiazepine use disorder
45
Central nervous system depression Sedation Anxiety Major depressive disorder Depression (mood) Nervous system
46
47 C is not correct. 7°/o chose this.
48 Lithium is a mood stabilizer used to treat bipolar affective disorder. It ind irectly inhibits the reuptake of
49
seroton in and norepinephrine by inh ibiting the phosphatidyl inositol second messenger system . Adverse effects
include central nervous system depression, dizziness, nephrogen ic diabetes insipidus, acne, edema, and
50 hypothyroidism, as we ll as many others.
51 Mood stabilizer Bipolar disorder Hypothyroidism Serotonin Norepinephrine Diabetes insipidus Central nervous system Second messenger system Acne vulgaris

52 Diabetes mellitus Mood disorder Edema Major depressive disorder lithium Dizziness Nephrogenic diabetes insipidus Depression (mood) Nervous system Reuptake

53
D is not correct. 14% chose this.
54 Sertra line and other selective serotonin reuptake inhibitors (SSRi s) are associated with adverse effects related
55 to central nervous system stimulation such as headache, anxiety, tremor, insomn ia, anorexia, nausea, and
56 vomit ing. Weight gain and sexua l dysfunction are also frequently reported with SSRI use.
Sertraline Serotonin Selective serotonin reuptake inhibitor Central nervous system Insomnia Sexual dysfunction Tremor Nausea Reuptake Headache Vomiting
• 57
Anorexia nervosa Anxiety Anorexia (symptom) Nervous system Weight gain
• 58
• 59 E is not correct. 13% chose this .
• 60 Venlafaxine is a seroton in-norepinephrine reuptake inhibitor. It has adverse effects similar to those of selective
• eQr""t"'\t t"'\n i n I'"'QIIr"\t!)(/Q i nh i h it-I""H"'e n l 11e :2.rlrliti t"'\n=al =ari\/Qr"'eQ c.ffc.r-te ri1 1Q tn t-he ni"\ I'"'C.r"\ i ncnhr'i nc e11rh =ae rl i '?"? i nQee =anrl

6
lock
s
Suspend
0
End Block
Item: 56 of 73 ~. I • M k <:] t> al ~· ~
QIO: 1943 .l. ar Previous Next Lab 'lifllues Notes Calculator

40 • - Ill .I - I -. I -. I II • - - :y- I I.- I - T I - - •• - "- I

seroton in and norepinephrine by inh ibiting the phosphatidyl inositol second messenger system . Adverse effects
41
include central nervous system depression, dizziness, nephrogen ic diabetes insipidus, acne, edema, and
42 hypothyroidism, as we ll as many others.
43 Mood stabilizer Bipolar disorder Hypothyroidism Serotonin Norepinephrine Diabetes insipidus Central nervous system Second messenger system Acne vulgaris

44 Diabetes mellitus Mood disorder Edema Major depressive disorder lithium Dizziness Nephrogenic diabetes insipidus Depression (mood) Nervous system Reuptake

45 D is not correct. 14% chose this.


46 Sertra line and other selective serotonin reuptake inhibitors (SSRi s) are associated with adverse effects related
47 to central nervous system stimulation such as headache, anxiety, tremor, insomn ia, anorexia, nausea, and
48 vomiting. Weight gain and sexua l dysfunction are also frequently reported with SSRI use.
Sertraline Serotonin Selective serotonin reuptake inhibitor Central nervous system Insomnia Sexual dysfunction Tremor Nausea Reuptake Headache Vomiting
49
Anorexia nervosa Anxiety Anorexia (symptom) Nervous system Weight gain
50
51 E is not correct. 13% chose this.
52 Venlafaxine is a seroton in-norepinephrine reuptake inhibitor. It has adverse eff ects simila r to those of selective
seroton in reuptake inhibitors, plus additional adverse effects due to the norepinephrine, such as dizziness and
53
diaphoresis. Ven lafaxine is also known to cause hypertension.
54 Serotonin-norepinephrine reuptake inhibitor Venlafaxine Serotonin Perspiration Norepinephrine Selective serotonin reuptake inhibitor Hypertension Dizziness

55 Reuptake Reuptake inhibitor Enzyme inhibitor

56
• 57
Bottom Line:
• 58
• 59
The tricycl ic antidepressants, includ ing amitriptyline, are known to cause antichol inergic adverse effects,
including dry mouth, blur ry vision, confusion, constipation, tachyca rdia, urinary retention, and hot, dry ski n .
• 60 Anticholinergic Amitriptyline Urinary retention Constipation Tachycardia Tricyclic antidepressant Xerostomia Antidepressant Blurred vision Tricyclic

6
lock
s
Suspend
0
End Block
Item: 56 of 7 3 ~ 1 • M k -<:J 1>- Jil ~· !:';-~
QIO: 1943 ..L ar Pre v ious Next Labfli!llues Not es Calcula t o r

40 A A

41 FA17 p 545.4

42 Tricyclic Amitriptyline, nortriptyline, imipramine, desipramine, clomipramine, doxepin, amoxapine.


43 antidepressants
44 MECHANISM Inhibit l£ and )-liT reuptake.
45 CLINICAL USE ~lajor depression, OCD (clomipramine). peripheral neuropathr. chronic pain, migraine
46 prophylaxis. .1'\octurnal enuresis (imipramine, although adverse effects may limit use).
47 ADVERSE EFFECTS Sedation, a 1-blocking effects including postural hypotension, and atropine-like (anticholinergic)
48 side effects (tachycardia, urinary retention, dr~ mouth). 3° TCAs {amitriptyline) have more
anticholinergic effects than 2° TCAs (nortriptyline). Can prolong QT inten'al.
49
Tri-C 's: Com·ulsions, Coma, Cardiotoxicity (arrhythmia due to 1 a+ channel inhibition);
so also respiratory depression, hyperpyrexia. Confusion and hallucinations in elderly due to
51 anticholinergic side effects (nortriptyline beller tolerated in the elderly). Treatment: 'aiiC03 to
52 prevent arrhythmia.
53
54 FA17 p 545.1
55 Selective serotonin Fluoxetine, Auvoxamine, paroxetine, sertraline, escitalopram, citalopram.
56 reuptake inhibitors
• 57 MECHANISM Inhibit 5-HT reuptake. It normally takes 4-8 weeks for antidepressants
. 58 CLINICAL USE Depression, generalized anxiety disorder, to have an effect.
• 59 panic disorder, OCD, bulimia, social am.iety
• 60 disorder, PTSD, premature ejaculation •

a
Lock
s
Suspend
8
End Bl ock
Item: 56 of 7 3 ~ 1 • M k -<:J 1>- Jil ~· !:';-~
QIO: 1943 ..L ar Pre v ious Next Labfli!llues Not es Calcula t o r
. . . - -
40 A A

41 MECHANISM Inhibit 5-HT reuptake. It normally takes 4-8 weeks for antidepressants
CLINICAL USE Depression, generalized anxiety disorder, to have an effect.
42
panic disorder, OCD, bulimia, social am. iely
43
disorder, PTSD, premature ejaculation,
44
premenstrual dysphoric disorder.
45
ADVERSE EFFECTS Fewer than TCAs. Gl distress, SIADII, sc:~.ual
46 drsfunction (anorgasmia, l libido).
47
48
FA17 p 233.2
49
Atropine ~luscarinic antagonist. Used lo treat bradycardia and for ophthalmic applications.
so
ORGAN SYSTEM ACTION NOTES
51
Eye t pupil di lation, cycloplegia Blocks DU\1BBeLSS in cholinesterase
52 inhibitor poisoning. Docs not block cxc it;~li on
Airway l secretions
53 of skeletal muscle and CNS (mediated by
Stomach l acid secretion
54 nicotinic receptors).
Gut l motility
55
56 Bladder l urgency in cystitis
• 57 ADVERSE EFFECTS t bodr temperature (due to l sweating); Side effects:
. 58
rapid pulse; dry mouth; dr), Omhcd ~ kin ; I lot as a hare
C) cloplcgia; constipal ion; d i\oricntal ion Dr) as a bone
• 59
Can cause acute angle-closure glaucoma in Red as a beet
• 60 elderly (due to mydriasis), urinarr relent ion Blind as a bat

a
Lock
s
Suspend
8
End Bl ock
Item: 57 of 73 ~. I • M k <:] t> al ~· ~
QIO: 5172 .l. ar Previous Next lab 'lifllues Notes Calculator

40 •
41
A 20-year-old woman is brought to the emergency department by her roommate because she was "wa lking
funny, " had difficulty breathing, and slurred her speech . She was recently diagnosed and given medication
42 for panic disorder. Her blood pressure is 110/ 75 mm Hg, pu lse is 58/ min, and respiratory rate is 8/ min . She
43 is afebr ile. Her mucous membranes are moist and pupil size is normal. Serum laboratory studies are negative for
44 evidence of ethanol, organophosphate, or opioid ingestion.
45
46
The agent that would be used to reverse the effects of the patient's anxiety medication works by which of the
following mechanisms?
47
:
48
A. Amp lifying the effect of an endogenous neu rotransmitter by inhibiting its breakdown
49
50 B. Disp lacement of the drug from its binding site
51 C. Inhibiting the formation of a toxic metabo lite
52
D. Inhibiting the storage of a neu rotransmitter
53
E. Reactivating a previously inactivated enzyme
54
55
56
• 57
• 58
• 59
• 60

6
lock
s
Suspend
0
End Block
Item: 57 of 73 ~. I • M k <:] t> al ~· ~
QIO: 5172 .l. ar Previous Next lab 'lifllues Notes Calculator

40 •
The correct answer is B. 68°/o chose this.
41
This patient has evidence of benzodiazepine intoxication. This is the most like ly scena rio given he r recent
42 diagnosis and t reatment fo r panic diso rder, in add it ion to the exclusion of other causes with similar
43 presentations . Benzodiazepines are relative ly safe in overdose; however, shorter -acting benzod iazepines such
44
as temazepam, triazolam, and alprazolam pose a greater r isk for morbidity and mortality. Competitive
antagon ists wor k by displacing a drug from its binding site . Flumazeni l is a competitive antagonist that can be
45 used in the case of benzodiazepine overdose, and na loxone is a competitive antagonist that is used to reverse
46 symptoms of opiate overdose. When using f lumazen il, be aware that rapid reversa l of benzod iazepine overdose
47 may lead to rebound seizure activity. I n clinical practice flumazeni l is rarely used except in children.
Benzodiazepine Naloxone Temazepam Alprazolam Flumazenil Triazolam Benzodiazepine overdose Panic disorder Opiate Receptor antagonist
48
Competitive antagonist Binding site Epileptic seizure Alcohol intoxication Substance intoxication
49
50 A is not correct. 11% chose this.
51 Physostigmine is an indi rect-acting chol inomimetic that inhibits the action of acetylcholinesterase, thereby
amplifying the effect of endogenous acetylcholine. It is indicated in cases of antichol inergic (but not t ricycl ic)
52
poisoning, w hich would present with the classic pictu re descr ibed by the mnemonic "red as a beet, blind as a
53 bat, mad as a hatter, dry as a bone, and hot as a ha re. " One wou ld expect fever, f lush ing, del irium, dry mucous
54 membranes, and mydriasis on physical exam.
Anticholinergic Mydriasis Physostigmine Acetylcholine Acetylcholinesterase Parasympathomimetic drug Mnemonic Delirium Fever Endogeny (biology)
55
Flushing (physiology)
56
57 C is not correct. 5°/o chose this.
• 58 Ethano l is indicated in cases of toxic alcoho l ingestion (eg, methanol or ethylene glycol). Toxic metabo lites are
• 59 for med when alcohol dehyd rogenase metabo lizes methanol or ethylene glycol. Ethano l wor ks by inh ibiting the
for mation of these ha r mfu l substances by competing fo r binding sites on alcohol dehyd rogenase .
• 60
Methanol Ethylene glycol Alcohol dehydrogenase Ethanol Ethylene Alcohol Alcoholic beverage Metabolite Toxicity Metabolism Dial

6
lock
s
Suspend
0
End Block
Item: 57 of 73 ~. I • M k <:] t> al ~· ~
QIO: 5172 .l. ar Previous Next lab 'lifllues Notes Calculator

40
C is not correct. 5°/o chose this.
41
Ethano l is indicated in cases of toxic alcoho l ingestion (eg , m ethanol or ethy lene glycol). Toxic m etabo lites are
42 for med when alcohol dehyd rogenase meta bolizes methanol or ethy lene glycol. Ethano l wor ks by inh ibiting t he
43 for mation of t hese ha r mfu l substances by com peting for binding sites on alcohol dehyd rogenase.
Methanol Ethylene glycol Alcohol dehydrogenase Ethanol Ethylene Alcohol Alcoholic beverage Metabolite Toxicity Metabolism Dial
44
45 D is not correct. 9°/o chose this.
46 Reserpine inhibits the storage of norepinephrine in adrenerg ic nerve terminals, thereby depleting t he neu ron of
its stores. I t has been classified as a postgang lionic sympathetic nerve terminal blocker, an d is rarely used as an
47
antihypertensive medication.
48 Reserpine Norepinephrine Neuron Antihypertensive drug Sympathetic nervous system Postganglionic nerve fibers Nerve Adrenergic Pharmaceutical drug
49 Axon terminal
50
E is not correct. 7°/o chose this.
51
Pralidoxime, a chol inesterase regenerator, is indicated in cases of organophosphate poison ing.
52 Organophosphates such as parathion and m alathion are indirect-acting cholinomimetics that inhibit
53 acetylchol inesterase by forming a very stab le bond with it. Organophosphate pesticides lead to t he
54 phosphory lation and subseq uent inactivation of acetylchol inesterase. This resu lts in general chol inergic centra l
nervous system sti mulation ( incontinence, bronchoconstriction, miosis, and bradyca rdia) . Pral idoxi m e can
55
reactivate this acetylchol inesterase because it is ab le to remove t he phosphoryl group from t he inactivated
56 enzyme, an d it can displace t he enzyme from its receptor site . For this reason, pra lidoxime is thought of as an
57 organophosphate "chemical antagonist. "
Malathion Miosis Organophosphate Pralidoxime Bradycardia Parathion Central nervous system Cholinergic Bronchoconstriction Enzyme Acetylcholinesterase
• 58
Cholinesterase Organophosphate poisoning Pesticide Parasympathomimetic drug Phosphorylation Receptor antagonist Nervous system Urinary incontinence
• 59
Receptor (biochemistry)
• 60

6
lock
s
Suspend
0
End Block
Item: 57 of 73 ~. I • M k <:] t> al ~· ~
QIO: 5172 .l. ar Previous Next lab 'lifllues Notes Calculator

p • g p p • g ' •y • p g
41 its stores. It has been classified as a postgang lionic sympathetic nerve terminal blocker, and is rarely used as an
antihypertensive medication.
42
Reserpine Norepinephrine Neuron Antihypertensive drug Sympathetic nervous system Postganglionic nerve fibers Nerve Adrenergic Pharmaceutical drug
43
Axon terminal
44
45 E is not correct. 7°/o chose this.
Pralidoxime, a chol inesterase regenerator, is indicated in cases of organophosphate poison ing.
46
Organophosphates such as parathion and ma lathion are ind irect-acting cholinomimetics that inhibit
47 acetylchol inesterase by forming a very stab le bond with it. Organophosphate pesticides lead to the
48 phosphory lation and subsequent inactivation of acetylchol inesterase. This resu lts in general chol inergic centra l
49 nervous system stimulation ( incontinence, bronchoconstriction, miosis, and bradyca rdia) . Pral idoxime can
reactivate this acetylchol inesterase because it is ab le to remove the phosphoryl group from the inactivated
50
enzyme, and it can displace the enzyme from its receptor site . For this reason, pra lidoxime is thought of as an
51 organophosphate "chemica l antagonist. "
Malathion Miosis Organophosphate Pralidoxime Bradycardia Parathion Central nervous system Cholinergic Bronchoconstriction Enzyme Acetylcholinesterase
52
53 Cholinesterase Organophosphate poisoning Pesticide Parasympathomimetic drug Phosphorylation Receptor antagonist Nervous system Urinary incontinence

54 Receptor (biochemistry)

55
56
Bottom Line:
57
Flumazenil is a competitive benzodiazepine antagonist that can be used to reverse cases of severe overdose .
• 58 Be aware that rapid reversa l may lead to rebound seizures.
• 59 Benzodiazepine Flumazenil Receptor antagonist Epileptic seizure Antagonist

• 60

6
lock
s
Suspend
0
End Block
Item: 57 of 7 3 ~ 1 • M k -<:J 1>- Jil ~· !:';-~
QIO: 5172 ..L ar Pre v ious Next Labfli!llues Not es Calcula t o r

40 A A
FA17 p 515.2
41
Benzodiazepines Diazepam, lorazcpam, triazolam, tcrmtzcpam, oxazepam, midazolam, chlordiazepoxide,
42 alprazolam.
43
MECHANISM Facilitate CABA,\ action by t frequency of "Frenzodiazepines" t frequency.
44 CI- channel opening. l RE~ I sleep. Most Benzos, barbs, and alcohol all bind the
45 ha,·e long half-Ji,·es and active metabolites C BA.\ receptor. which is a ligand-gated CJ-
46 (exceptions (.\TOM): .\ lprazolam, Triazolam, channel.
47
Oxazepam, and :\Iidazolam are short acting Oxazepam_ Temazepam, and Lorazepam are
- higher addictive potential). metabolized O utside T he Liver
48
CliNICAl USE Anxiety, spasticity, status epilepticus (lorazepam
49
and diazepam), eclampsia, detoxification
so (especially alcohol withdrawal-DTs), night
51 terrors, sleepwalking, general anesthetic
52 (amnesia, muscle relaxation), hr pnotic
53 (insomn ia).
54 ADVERSEEFFECTS Dependence, additive C S depression effects
with alcohol. Less risk of respiratory depression
55
and coma than with barbiturates.
56
Treat overdose with Aumazenil (competitive
57 antagon ist at CABA benzodiazepine receptor).
. 58 Can precipitate seizures br causing acute
• 59 benzodiazepine withdrawal.
• 60

a
Lock
s
Suspend
8
End Bl ock
Item: 57 of 7 3 ~ 1 • M k -<:J 1>- Jil ~· !:';-~
QIO: 5172 ..L ar Pre v ious Next Labfli!llues Not es Calcula t o r

40 •
41 FA17 p239.1

42 Specific toxicity TOXIN TREAIMENT


43 treatments Acetaminophen N-acetylcysteine (replenishes glutathione)
44 AChE inhibitors, organophosphates Atropine> pralidoxime
45 Antimuscarinic, anticholinergic agents Phrsostigmine, control hyperthermia
46 Arsenic Dimercaprol. succimer
47 Benzodiazepines Flumazenil
48 ~-blockers Atropine, glucagon
49
Carbon monoxide 100% 0 2, hyperbaric 0 2
so Copper Penicillamine, trientine (Copper penny)
51
Cyanide Nitrile+ thiosulfate, hydroxocobalamin
52
Digitalis (digoxin) Anti-dig Fab fragments
53
Heparin Protamine sulfate
54
Iron Defe roxamine, deferasirox, deferiprone
55
Lead ED'lA, dimercaprol, succimer, penicil lamine
56
l\lcrcury Dimercaprol, succimcr
57
. 58 1\ !ethanol, ethylene glycol (antifreeze) F'omepizole >ethanol, dialysis
• 59 l\ lethemoglobin 1\ lethvlene blue, vitamin C
'
• 60 O piO ids 'aiO xO nc

a
Lock
s
Suspend
8
End Bl ock
Item: 58 of 73 ~ 1 • M k -<:J 1>- Jil ~· !:';-~
QIO: 4200 ..L ar Pre v ious Next Lab fli!ltues Not es Calcula t o r

40
IAA]
A A

41
A 19-year-old man is brought to the emergency department by his friends after suffering a seizure. He
appears agitated and voices thoughts of paranoia. On examination he is sweating, tachycardic, and his pupils
42 are dilated. He works in the pest control services. On specific questioning, his friends say that he has a
43 history of using illicit drugs but den ies any regular medication use or history of overdose .
44
45 What is the mechanism of action of the drug that is causing the patient's symptoms?
46 :
47 A . Blocks NMDA receptors
48 B. Competitive inhibition of muscarinic acetylcholine receptors
49
C. Increases GABA activity by increasing the duration of chloride channel opening
so
51 D. Prevents reuptake of norepinephrine, dopamine, and serotonin by presynaptic pumps
52 E. Prevents the fusion of the presynaptic vesicle with the presynaptic surface membrane
53
F. Prevents the uptake of acetylcholine at cholinergic synapses
54
55
56
57
. 58
• 59
• 60

a
Lock
s
Suspend
8
End Bl ock
Item: 58 of 73 ~. I • M k <:] t> al ~· ~
QIO: 4200 .l. ar Previous Next lab 'lifllues Notes Calculator

40 •
The correct answer is D. 49°/o chose this.
41
The patient's symptoms are consistent with cocaine ove rdose. Cocaine prevents the reuptake of norepineph rine,
42
dopamine, and serotonin by presynaptic t ransporter pumps in the central and periphe ral nervous systems . I n
43 contrast, amphetamine's mechanism involves inh ibition of reuptake and stimu lation of norepineph r ine release
44 into the synaptic cleft.
Substituted amphetamine Amphetamine Serotonin Dopamine Norepinephrine Synaptic cleft Cocaine Chemical synapse Reuptake Drug overdose
45
Membrane transport protein Synapse
46
47 A is not correct. 11% chose this.
48 Phencyclidine ( PCP) causes aggressive and impulsive behavior, nystagmus, and tachycardia. Note that PCP
49 intoxication is not associated with changes in pupi l size. This drug acts as an NMDA recepto r antagonist.
Phencyclidine NMDA receptor NMDA receptor antagonist Nystagmus Tachycardia N-Methyi-D-aspartic acid Receptor antagonist Antagonist Impulsivity
50
Receptor (biochemistry)
51
52 B is not correct. 17% chose this.
53 It is important to distinguish antichol inergic toxicity from cocaine and/ or amphetamine toxicity. Clinica l
presentations are similar in that the re is tachycardia, disor ientation, and myd riasis. One distinguishing featu re,
54
however, is that sweating is decreased (due to the anticho linergic properties), therefo re patients have dry,
55 flushed skin.
56 Anticholinergic Mydriasis Amphetamine Substituted amphetamine Tachycardia Cocaine Flushing (physiology) Perspiration Toxicity Erythema Disorientation

57 C is not correct. 5°/o chose this.


58 Barbitu rates act by increasing GABAA activity by increasing the duration of chlor ide channel opening . In
• 59 contrast, benzodiazepines faci litate GABAA action by increasing the frequency of ch loride channe l opening .
Toxicity due to these medications causes respiratory and CNS dep ression, symptoms inconsistent with those
• 60
• seen in this patient. •
6
lock
s
Suspend
0
End Block
Item: 58 of 73 ~. I • M k <:] t> al ~· ~
QIO: 4200 .l. ar Previous Next lab 'lifllues Notes Calculator
it- I • -- - ll: -- I a it• - J: :t: l:t: -- I - t I • I I I 1- - - II- I

contrast, benzodiazepines faci litate GABAA action by increasing the frequency of ch loride channe l opening.
41
Toxicity due to these medications causes respiratory and CNS dep ression, symptoms inconsistent with those
42 seen in this patient.
43 Gamma-Aminobutyric acid Chloride channel Benzodiazepine Barbiturate Central nervous system depression Central nervous system Chloride

44 Major depressive disorder Depression (mood)

45
E is not correct. 5°/o chose this.
46 Bretylium and guaneth idine prevent the fusion of presynaptic vesicles with the presynaptic memb rane, resu lting
47 in an inhibition of the release of norepineph rine into the synapse. These med ications are not commonly used
48 nowadays. I n addition, this patient's symptoms are consistent with an excess, rathe r than depletion of
norepinephrine.
49
Guanethidine Norepinephrine Synapse Bretylium Chemical synapse Vesicle (biology and chemistry)
50
F is not correct. 13% chose this.
51
Cocaine prevents the reupta ke of no repinephrine, dopamine, and serotonin on no rad renergic neu rons; it does
52
not work on choline rgic neurons. Hemicho linium inhibits the reuptake of chol ine into presynaptic te rminals,
53 leading to depletion of acetylchol ine. It has no cl inical use nowadays. In addition, this patient's symptoms and
54 signs are not consistent with acetylchol ine depletion .
Serotonin Norepinephrine Dopamine Acetylcholine Cholinergic Choline Cocaine Chemical synapse Neuron Reuptake Cholinergic neuron
55
56
57 Bottom Line:
58 Cocaine is a potent central nervous system stimulant that acts by inh ibiting the reupta ke of dopamine,
• 59 norepinephrine, and serotonin .
Central nervous system Serotonin Dopamine Norepinephrine Cocaine Stimulant Nervous system Reuptake
• 60

6
lock
s
Suspend
0
End Block
Item: 58 of 73 ~. I • M k <:] t> al ~· ~
QIO: 4200 .l. ar Previous Next lab 'lifllues Notes Calculator

40 • •
41 FA17 p 540.1

42 Psychoactive drug intoxication and withdrawal


43 DRUG ltiTOXICATION WITHDRAWAL

44
Depressants
45 onspecific: mood elevation, l anxiety, onspecific: anxiety, tremor, seizures,
. .
sedation, behavioral disinhibition, respiratory lllSOmnaa.
46
depression.
47
Alcohol Emotional lability, slurred speech, ataxia, Time from last drink:
48
coma, blackouts. Serum y-glutamyltransferase 3-36 hr: minor symptoms similar to other
49 (GGT)-sensitive indicator of alcohol usc. depressants
50 AST value is twice ALT value. 6- 48 hr: withdrawal seizures
51 12-48 hr: alcoholic hallucinosis (usually visual)
52
48-96 hr: delirium tremens (DTs) in 5% of
cases
53
Treatment: benzodiazepines.
54
Opioids Euphoria, respiratory and CNS depression, Sweating, dilated pupils, piloerection ("cold
55 l gag reAex, pupillary constriction (pinpoint turkey"), fever, rhinorrhea, yawning, nausea,
56 pupils), seizures (overdose). Most common stomach cramps, diarrhea {"Au-like" symptoms).
57 cause of drug overdose death. Treatment: Treatment: long-term support, methadone,
58 naloxone. buprenorphine.
• 59 Barbiturates Low safety margin, marked respiratory Delirium, life-threatening cardiovascular
• 60
depression. Treatment: symptom management collapse.
• (ev. assist resniration. t BP). •

6
lock
s
Suspend
0
End Block
Item: 59 of 73 ~ 1 • M k -<:J 1>- Jil ~· !:';-~
QIO: 1151 ..L ar Pre v ious Next Labfli!llues Not es Calcula t o r

40
IAA]
A A

41
A psychologist whose first ch ild is now 4 yea rs old is applying the principles of behavioral mod ification to
child-rearing. For instance, if the child throws a temper tantrum in a store because she will not buy him
42 candy, she just picks up the ch ild and leaves the store; however, if the child behaves in the store, she takes
43 him to the zoo.
44
45 This is an example of which of the following kinds of behavior modification?
46 :
47 A. Classical conditioning
48 B. Forward conditioning
49
C. Learned helplessness
so
51 D. Operant condition ing
52 E. Stimulus general ization
53
54
55
56
57
58
• 59
• 60

a
Lock
s
Suspend
8
End Bl ock
Item: 59 of 73 ~. I • M k <:] t> al ~· ~
QIO: 1151 .l. ar Previous Next lab 'lifllues Notes Calculator

40 •
41 The correct answer is D. 65°/o chose this.
42 The basic theory of operant cond it ion ing is that behavior can be modified through positive and negative
reinfo rcement, and rewa rd and punishment . Positive reinforcement is used to increase a desi red behavior, such
43
as in this case where the parent takes the child to the zoo to reward him for good behavior. Negative
44 reinfo rcement also supports a desired behavior, but, instead of a reward, the behavio r is encouraged because it
45 allows escape from punishment, such as a ch ild cleaning her room so that her parents wi ll stop nagg ing her to
46
do so.
Operant conditioning Reinforcement Positive reinforcement Negative reinforcement Zoo
47
48 A is not correct. 20% chose this.
49 Classical cond itioning is learning in which a natural response is el icited by a conditioned stimu lus that was
previously presented in conjunction with an uncond it ioned stimulus . The classic example is Pavlov's dogs, an
50 experiment in which dogs who sal ivated ( natural response) in response to food (unconditioned stimulus) were
51 taught to sal ivate (natura l response) in response to a bell ringing (cond itioned stimulus) . Th is is programming
52 by association, not reward .
Classical conditioning Ivan Pavlov
53
54 B is not correct. 8°/o chose this.
55 Forward cond it ion ing is a procedural example of classical cond it ioning. The conditioned stimu lus precedes the
onset of the unconditioned stimu lus in order to signal that the unconditioned stimu lus will fol low . Learning
56
occurs fastest in forward conditioning.
57 Classical conditioning

58
C is not correct. 3°/o chose this.
59
Learned helplessness is created through aversive cond itioning, which is an example of classica l conditioning . In
0 60 classica l conditioning experiments, animals became apathetic and hopeless after learning that they cou ld not

6
lock
s
Suspend
0
End Block
Item: 59 of 73 ~. I • M k <:] t> al ~· ~
QIO: 1151 .l. ar Previous Next lab 'lifllues Notes Calculator

40 by association, not reward .


41 Classical conditioning Ivan Pavlov

42 B is not correct. 8°/o chose this.


43 Forwa rd cond it ion ing is a procedural example of classical cond it ioning. The conditioned sti mulus precedes the
44 onset of the unconditioned stimu lus in order to signal that the unconditioned stimu lus will fol low . Learning
45
occurs fastest in forward conditioning.
Classical conditioning
46
47 C is not correct. 3°/o chose this.
48
Learned helplessness is created th rough aversive cond itioning, which is an example of classica l conditioning . I n
classica l conditioning experiments, animals became apathetic and hopeless after learning that they cou ld not
49
escape a painfu l stimulus .
50 Classical conditioning learned helplessness Aversives

51
E is not correct. 4°/o chose this.
52 Stimulus general ization is part of classical cond it ioning. This occurs w hen a new stimulus that is simi lar to an old
53 stimu lus elicits the same response . The classic example is when a child who fears nurses after receiving
54 vaccinations begins to fear any person in a w hite coat (even the kind med ical student) .
Classical conditioning Taste aversion
55
56
57 Bottom Line:
58 Operant conditioning is the use of posit ive and negative consequences (reinforcement or punishment) to
59 modify a certain behavior.
Operant conditioning
0 60

6
lock
s
Suspend
0
End Block
Item: 59 of 73 ~ 1 • M k -<:J 1>- Jil ~· !:';-~
QIO: 1151 ..L ar Pre v ious Next Labfli!llues Not es Calcula t o r

40 A A

FA17 p 524.2
41
Operant conditioning Learning in which a particular action is elicited because it produces a punishment or reward.
42
Usually deals wi th voluntary responses.
43
Reinforcement Target behavior (response) is follo"cd b) desired reward (positive reinforcement) or removal of
44
a\·ersi\'e stimulus (negative reinforcement).
45
Extinction Discontinuation of reinforcement (positive or negati, e) e\entually eliminates behavior. Can occur
46 in operant or classical conditioning.
47 Punishment Repeated application of 3\Crsi'c stimulus
Increase behavior Decrease behavior
48 (posith·e punishment) or rem0\'31 of desired
49 reward (negative punishment) to e-.:tinguish "' :::J
"' Positive Positive
-o"5
unwanted behavior (Skinner's operant -oE reinforcement punishment
so < ·-
v;
conditioning quadrant).
51
"'....> -"':::J Negative Negative
52
53
0
E .~
....
cr: - :::J

"'
reinforcement punishment

54
55 FA17 p 524.1

56 Classical conditioning Learning in which a natural response Usually deals with involuntary responses.
57
(saJi,·ation) is elicited by a conditioned, Pavlov's classical experiments with dogs-
or learned, stimulus (bell) that previous!) ringing the bell provoked salivation.
58
was presented in conjunction with an
59
unconditioned stimulus (food).
• 60

a
Lock
s
Suspend
8
End Bl ock
Item: 60 of 73 ~ 1 • M k -<:J 1>- Jil ~· !:';-~
QIO: 3481 ..L ar Pre v ious Next Lab fli!ltues Notes Calcula t o r

40
IAA]
A A

41
Lit hium has been demonstrated to be uniquely effective for lowering the risk of suicide in patients w ith
bipolar disorder. It does not possess sedative, depressant, or euphoriant effects, and it is excreted in the
42 unne.
43
44 What is a serious side effect of this medication?
45
:
46 A . Agranulocytosis
47
B. Central diabetes insipidus
48
49 C. Nephrogenic diabetes insipidus
so D. Serotonin syndrome
51 E. Sexual dysfunction
52
53
54
55
56
57
58
59
• 60

a
Lock
s
Suspend
8
End Bl ock
Item: 60 of 73 ~. , . M k <:] t> al ~· ~
QIO: 3481 .l. ar Previous Next lab 'lifllues Notes Calculator

40 •
41 The correct answer is C. 80°/o chose this.
42 Lithium has been show n to have the greatest effect for lower ing the risk of suicide in patients with bipola r
43 disorder w ho are manic. However, it acts in a dose-dependent manner that is toxic to the kidney and that
impa irs its abi lity to concentrate ur ine. This causes a disorder ca lled diabetes insipidus, with which the patient
44 excretes large quantit ies of dilute urine. In add ition, lithium treatment can be toxic to the thyroid.
45 Bipolar disorder Diabetes insipidus Diabetes mellitus lithium Kidney Thyroid Urine Suicide

46
A is not correct. 5°/o chose this.
47
Agranu locytosis and other hemato log ic abnormalities are the adve rse effects of t reatment with anticonvu lsant
48 and antiman ic dr ugs, name ly clozapine. The refore, hemato lolgic mon ito ring is advisable in patients who are
49 being t reated with the drug . However, lithium does not cause ag ranulocytosis.
Anticonvulsant Agranulocytosis Clozapine Hematology Mania
50
51 B is not correct. 10% chose this.
52 Although its the rapeutic effect in patients with bipo lar disorder is via the brain and although it can cause other
central nervous system sy mptoms, lith ium's ability to induce diabetes insipidus is the result of its action on the
53
kidney.
54 Bipolar disorder Central nervous system Diabetes insipidus Diabetes mellitus Kidney lithium Nervous system Human brain Therapeutic effect Brain

55
D is not correct. 3°/o chose this.
56
Se lective serotonin reupta ke inhibitors, in conjunction with monoamine oxidase inhibitors, can cause a buildup
57 of serotonin in the body. Th is causes the seroton in syndrome, which is cha racterized by altered menta l status,
58 autonomic dysfunction, and neu romuscular abnor mal it ies.
Serotonin syndrome Serotonin Selective serotonin reuptake inhibitor Reuptake Monoamine oxidase Monoamine oxidase inhibitor Dysautonomia
59
Autonomic nervous system Monoamine neurotransmitter Neuromuscular junction Oxidase
60

6
lock
s
Suspend
0
End Block
Item: 60 of 73 ~. , . M k <:] t> al ~· ~
QIO: 3481 .l. ar Previous Next lab 'lifllues Notes Calculator

40 • being t reated with the drug . However, lithium does not cause ag ranulocytosis.
41 Anticonvulsant Agranulocytosis Clozapine Hematology Mania

42 B is not correct. 10% chose this.


43 Although its the rapeutic effect in patients with bipo lar disorder is via the brain and although it can cause other
44 central nervous system symptoms, lith ium's ability to induce diabetes insipidus is the result of its action on the
kidney.
45
Bipolar disorder Central nervous system Diabetes insipidus Diabetes mellitus Kidney lithium Nervous system Human brain Therapeutic effect Brain
46
47
D is not correct. 3°/o chose this.
Se lective serotonin reupta ke inhibito rs, in conjunction with monoamine oxidase inhibito rs, can cause a buildup
48
of serotonin in the body. Th is causes the se roton in syndrome, which is characte rized by altered menta l status,
49 autonomic dysfunction, and neuromuscular abnormal it ies.
50 Serotonin syndrome Serotonin Selective serotonin reuptake inhibitor Reuptake Monoamine oxidase Monoamine oxidase inhibitor Dysautonomia

51 Autonomic nervous system Monoamine neurotransmitter Neuromuscular junction Oxidase

52 E is not correct. 2°/o chose this.


53 Se lective serotonin reupta ke inhibito rs can cause sexual dysfunction, including an inability to ejacu late, and they
54 are a common reason why patients t reated with them discontinue thei r use.
Serotonin Selective serotonin reuptake inhibitor Sexual dysfunction Reuptake Ejaculation
55
56
57 Bottom Line:
58
Lithium, which is used to t reat bipolar diso rder, can lead to diabetes insipidus.
59 Bipolar disorder Diabetes insipidus Diabetes mellitus lithium

60

6
lock
s
Suspend
0
End Block
Item: 60 of 7 3 ~ 1 • M k -<:J 1>- Jil ~· !:';-~
QIO: 3481 ..L ar Pre v ious Next Lab fli!ltues Notes Calcula t o r

40 A A

FA17 p 544.1
41
Lithium
42
MECHANISM l\'ot established; possibly related to inhibition of L iT III U~ I :
43
phosphoinositol cascade. Low T hyroid (hypoth}Toidism)
44 H eart (Ebstein anomaly)
CliNICAL USE ~ lood stabilizer for bipolar disorder; blocks
45 relapse.and acute manic e\ents. I nsipidus {nephrogenic diabetes insipidus)
46 Un" anted :\Jo,·ements (tremor)
ADVERSE EfFECTS Tremor, hypoth}Toidism, polyuria (causes
47 nephrogenic diabetes insipidus), teratogenesis.
48 Causes Ebstein anomaly in newborn if taken
49 by pregnant mother. arrow therapeutic
window requires close monitoring of serum
so Je,·els. Almost exclusively excreted by
51 kidneys; most is reabsorbed at PCT wi th Na+.
52 Thiazides (and other nephrotoxic agents) are
53 implicated in lithium toxicity.
54
55 FA17 p 334.1

56 Diabetes insipidus C ha rac te rized by inte nse t hirst and polyuria wit h inabilit)' to concentrate uri ne due to lack of ADll
57 (central) or fail ure of response to c irc ulating AD I I (nephrogenic).
58 Cent ral Dl Nephrogenic Dl
59 ETIOLOGY Pituitary tumor, autoimmune, trauma, surgery, Hereditary (ADH receptor mutation), zo
60 ischemic encephalopathy, idiopathic to hypercalcemia, hypokalemia, lithium,

a
Lock
s
Suspend
8
End Bl ock
Item: 60 of 7 3 ~ 1 • M k -<:J 1>- Jil ~· !:';-~
QIO: 3481 ..L ar Pre v ious Next Lab fli!ltues Notes Calcula t o r
• y g y, y (. I • p ), .
41 ischemic encephalopathy, idiopathic to hypercalcemia, hypokalemia, lithium,
clemeclocycline (ADH antagonist)
42
43 FINDINGS l ADH 'ormal or t ADH b·els
Urine specific gra\'ity < 1.006 Urine specific gra,·ity < 1.006
44
Serum osmolality> 290 mOsm/kg Serum osmolality> 290 mOsmlkg
45 Hyperosmotic m lume contraction Hyperosmotic ,-olume contraction
46 WATER DEPRIVATION TEST' > 50% t in urine osmolality onlr after \lini mal change in urine osmolality, e,·en afte r
47 administration of ADH analog administration of ADH analog
48 TREATMENT Desmopressin acetate HCTZ, indomethacin, amiloride
49 Hydration Hydration, dietary salt restriction, avoidance of
so offending agent
51 •No water intake for 2-3 hr followed by hourly measurements of urine vol ume and osmolarity and plasma Na+ concentration
52 and osmolarity. ADII analog (desmopressin acetate) is admi nistered if serum osmolality> 295-300 mOsm/kg, plasma
a+ ~ 145, or urine osmolality does not rise despite a rising plasma osmolality.
53
54
55 FA17 p 542.2

56 Preferred medications PSYCHIATRIC CONDITION PREFERRED DRUGS

57 for selected ADH D Stimulants (methylphenidate, am phetamines)


psychiatric conditions
58 Alcohol withdrawal Benzocliazepines (eg, chlordiazepoxide,
59 lorazepam, diazepam)
60 Bipolar disorder Lithium, \'alproic acid, carbamazepine,
• 1----• -:-: .............,.._: .... - 1 - ... &-: .... - . ..... L.. .... &-: .... -

a
Lock
s
Suspend
8
End Bl ock
Item: 60 of 7 3 ~ 1 • M k -<:J 1>- Jil ~· !:';-~
QIO: 3481 ..L ar Pre v ious Next Lab fli!ltues Notes Calcula t o r
- - .. - - -- - - - -- - - . - -
40 A A

41
42 FA17 p 542.2

43 Preferred medications PSYCHIATRIC CONDtnON PREFERRED DRUGS

44 for selected ADHD Stimulants (methylphenidate, amphetamine~)


psychiatric conditions
45 Alcohol withdrawal Bcnzodiazepines (eg, chlordiazepoxide,
46 lorazcpam, diazepam)
47 Bipolar disorder Lithium, \'alproic acid, carbamazcpine,
48 lamotrigine, atypical antipsychotics
49 Bulimia ner\'osa SSRis
so Depression S Rls
51 Generalized anxiety disorder SSRis, S Rls
52 Obsessive-compulsive disorder SSRis, \'enlafaxine, clomipramine
53 Panic disorder SSRis, venlafaxine, benzodiazepines
54 PTSD SSRis, venlafaxine
55 Schizophrenia Atypica I antipsychotics
56 Social anxiety disorder SSRis, ,·enlafaxine
57 Performance only: P-blockers, benzodiazepines
58 Tourette syndrome Antipsychotics (eg, Ruphenazine, pimozide),
59 let rabenazine
60

Lock
a s
Suspend
8
End Bl ock
Item: 61 of 73 ~. I • M k <:] t> al ~· ~
QIO: 4684 .l. ar Previous Next lab 'lifllues Notes Calculator


53
A 42-year-old sch izophrenic woman visits her psych iatrist because her caregiver has noticed that she has
54 recently developed invo luntary, stereotyped facial grimacing and repetit ive thrusting of the tongue. The
55 patient began haloperidol therapy 6 months prior to the visit . Both the patient and her psychiatrist conclude
that the patient continues to requ ire treatment for her psychotic symptoms, so the psychiatrist writes a
56
prescription for a diffe rent medication . The new med ication sti ll has antipsychotic properties, but is less frequently
57 reported to contribute to ocu lomoto r symptoms.
58
59 What is a serious adverse effect of the new med ication most likely prescribed by the psych iatrist?
60
:
•61 A. Agranu locytosis
• 62
B. Anorgasmia
• 63
• 64 C. Diabetes insipidus
• 65 D . Hypertension
• 66 E. Respiratory depression
• 67
• 68
• 69
• 70
• 71
• 72
• 73 ~

6
lock
s
Suspend
0
End Block
Item: 61 of 73 ~. I • M k <:] t> al ~· ~
QIO: 4684 .l. ar Previous Next lab 'lifllues Notes Calculator


53
The correct answer is A. 75°/o chose this.
54
This patient is suffering from tard ive dyskinesia, which is a known extrapyramidal adverse effect of
55 antipsychotics, usua lly occurring after 4 months of therapy; it is usual ly irreversible. Other extrapyramidal
56 adverse effects of antipsychotic agents include acute dystonia (afte r 4 hours), akathisia (afte r 4 days), and
57 bradykinesia (afte r 4 weeks). Tardive dyskinesia common ly manifests as invo luntary, stereotyped movements
of the trunk, extremit ies, face, or lips, along with repetitive thrusting or protrusion of the tongue. It is bel ieved
58
to be due to sensitization and/or upregu lation of dopamine recepto rs. Discontinuing the responsible
59 antipsychotic is the preferred t reatment. Howeve r, if the rapy must continue, an atypical antipsychotic may
60 cause fewer extrapyramida l adverse effects. Clozapine has been demonstrated to be particu larly effective,
although it has the serious adverse effect of causing ag ranulocytosis.
61
Atypical antipsychotic Tardive dyskinesia Agranulocytosis Clozapine Akathisia Bradykinesia Dopamine Antipsychotic Dystonia Extrapyramidal system Dyskinesia
• 62
Extrapyramidal symptoms Adverse effect Downregulation and upregulation
• 63
• 64 B is not correct. 7°/o chose this .
• 65
Anorgasmia is an adverse effect of selective serotonin reuptake inhibitor (SSRI) the rapy. SSRi s are commonly
used to treat depression and anxiety diso rders; they are not ind icated for the treatment of psychotic symptoms .
• 66 Selective serotonin reuptake inhibitor Anorgasmia Serotonin Serotonin reuptake inhibitor Reuptake Adverse effect Psychosis Anxiety Anxiety disorder
• 67 Major depressive disorder Depression (mood) Reuptake inhibitor Enzyme inhibitor
• 68
C is not correct. 8°/o chose this .
• 69
Diabetes insipidus is an adverse effect of lithium therapy . Lith ium is a mood stabilize r com mon ly used to treat
• 70
bipolar disorder ; it is not indicated for the t reatment of psychotic symptoms .
• 71 Bipolar disorder Mood stabilizer Diabetes insipidus Diabetes mellitus lithium Psychosis lithium (medication) Adverse effect

• 72
D is not correct. 6°/o chose this .
• 73 ~

6
lock
s
Suspend
0
End Block
Item: 61 of 73 ~. I • M k <:] t> al ~· ~
QIO: 4684 .l. ar Previous Next lab 'lifllues Notes Calculator


53
C is not correct. 8°/o chose this.
54
Diabetes insipidus is an adverse effect of lithium therapy. Lith ium is a mood stabilizer common ly used to treat
55 bipolar disorder ; it is not indicated for the treatment of psychotic symptoms.
56 Bipolar disorder Mood stabilizer Diabetes insipidus Diabetes mellitus lithium Psychosis lithium (medication) Adverse effect

57
D is not correct. 6°/o chose this.
58 Hypertension is an adverse effect of venlafaxine, wh ich inhibits the reupta ke of serotonin, as wel l as
59 norepinephrine and dopamine. I t is commonly used to t reat depression and anxiety disorders; it is not ind icated
60 for the t reatment of psychotic symptoms.
Venlafaxine Serotonin Dopamine Norepinephrine Hypertension Reuptake Adverse effect Psychosis Anxiety Anxiety disorder Major depressive disorder
61
Depression (mood)
• 62
• 63 E is not correct. 4°/o chose this .
• 64 Respiratory depression is an adverse effect of benzodiazepines and ba rbitu rates, wh ich are used to t reat anxiety
disorder; they are not indicated for the treatment of psychotic symptoms .
• 65
Hypoventilation Anxiety disorder Benzodiazepine Major depressive disorder Depression (mood) Barbiturate Adverse effect Psychosis Anxiety
• 66
• 67
• 68 Bottom Line:
• 69 Tardive dyskinesia, which is character ized by repetitive, pu r poseless facial movements, resu lts fro m long -te rm
• 70
or high-dose use of antipsychotic therapeutic medications, such as halope rido l. In an effort to prevent the
potential ly irreversible effects of tardive dyskinesia, this physician has prescribed the atypical antipsychotic
• 71 clozapine, wh ich has the se rious side effect of causing agranu locytosis.
• 72 Atypical antipsychotic Haloperidol Tardive dyskinesia Agranulocytosis Clozapine Antipsychotic Dyskinesia Adverse effect Side effect

• 73 ~

6
lock
s
Suspend
0
End Block
Item: 61 of 7 3 ~ 1 • M k -<:J 1>- Jil ~· !:';-~
QIO: 4684 ..L ar Pre v ious Next Lab fli!ltues Not es Calcula t o r
A A
53
FA17 p 543.2
54
Atypical Aripiprazole, asenapine, clozapinc, olanzapinc, quetiapinc, ilopcridone, paliperidonc,
55 antipsychotics rispcridonc, lurasidonc, ziprasidonc.
56
MECHANISM Not completely understood. \lost arc D2
57 antagonists; aripiprazolc is D2 partial agon ist.
58 Varied effects on 5-IIT 2, dopamine, and
59 a- and H1-receptors.
60 CLINICAL USE Schizophrenia-both posilhc and ncgati\'e Use clozapine for treatment-resistant
61 srmptoms. Also used for bipolar disorder, schizophrenia or schizoaffective disorder and
OCD, anxiety disorder, depression, mania, for suicidality in schizophrenia.
• 62
Tourette syndrome.
. 63
ADVERSE EFFECTS Al l- prolonged QT inter"al, fewer EPS and
• 64
antichol inergic side effects than typical
• 65 antipsychotics.
• 66 "-pines"- metabolic syndrome (weight gain, Olanzapine .... O besity
• 67 diabetes, hyperlipidemia).
• 68
C lozapine-agranulocytosis (mon itor WBCs Jvlusf· watch bone marrow clozcly with ci<>J:apinc .
frequently) and seizures (dose related).
• 69
Risperidone-hyperprolactinemia (amenorrhea,
• 70 galactorrhea, gynecomastia).
• 71
• 72 FA17 p 543.1
• 73 Typical anti psychotics Haloperidol, pimozide, triAuoper3/inc, Auphenuinc, thioridazine, chlorpromazine.

a
Lock
s
Suspend
8
End Bl ock
Item: 61 of 7 3 ~ 1 • M k -<:J 1>- Jil ~· !:';-~
QIO: 4684 ..L ar Pre v ious Next Lab fli!ltues Not es Calcula t o r
A A
53
FA17 p543.1
54
Typical antipsychotics Haloperidol, pimozide, triAuopera,inc, Auphena;-ine, thioridazine, chlorpromazin e.
55
MECHANISM Block dopamine 0 2 receptor (t c 1P).
56
CliNICAl USE Schizophrenia (1° positi,·c symptoms), psychosis, bipolar disorder, delirium, Tourctte syndrome,
57
Huntington disease, OCD.
58
POTENCY H igh potency: TriAnoperazine, flnphena1ine, llaloperidol (Try to Fly H igh)- neurologic side
59
effects (eg, extrapyramidal S) mploms [F:PS)).
60 Lo" potency: C hlorpromazine, T h ioridazine (C heating l'hieves are low)-anticholinergic,
61 antihistamine, a 1-blockade effects.
• 62 ADVERSEEFFECTS Lipid soluble .... stored in body fat .... slow to be removed from body.
. 63
Endocrine: dopamine receptor antagonism .... hypcrprolactinemia .... galactorrhea,
• 64 oligomenorrhea, gynecomastia .
• 65 fetabol ic: dyslipidemia, weight gain, hyperglycem ia .
• 66 Antimuscarinic: dry mouth, constipation .
Antihistamine: sedation .
• 67
a.-blockade: orthostatic hypotension .
• 68
Cardiac: QT prolongation.
• 69 Ophthalmologic: C hlorpromazine-C orneal deposits; T hioridazine-reT inal deposi ts.
• 70
EPS-ADAPT:
• 71 Hours to days: Acute D ystonia (muscle spasm, stiffness, oculogyric crisis).
• 72 Days to months: \ kathisia (restlessness), Parkinsonism (bradykinesia).
• 73 .\lfonths to years: Tardive d)sl..inesia (orofacial chorea).

a
Lock
s
Suspend
8
End Bl ock
Item: 61 of 7 3 ~ 1 • M k -<:J 1>- Jil ~· !:';-~
QIO: 4684

53
..L
'
ar Pre v ious Next
. .
- .
Lab fli!ltues
-
Not es Calcula t o r

effects (eg, extrapyramidal S) mptoms [fi:PS]).


. . . .. - .·-
54 Lo'' potency: C h lorpromazine, Thioridazine (Cheating T hieves are low)-anticholinergic,
55 antihistamine, a 1-blockade effects.
56 ADVERSE EFFECTS Lipid soluble - stored in body fat - slow to be removed from body.
57 Endocrine: dopamine receptor antagonism - hypcrprolactinemia - galactorrhea,
58 oligomenorrhea, g}11ecomastia.
59 .\ fetabolic: dyslipidemia, weight gain, h)l)crglyccmia.
60 Antimuscarinic: dry mouth, constipation.
Antihistamine: sedation.
61
a 1-blockade: orthostatic hypotension .
• 62
Cardiac: QT prolongation.
• 63 Ophthalmologic: C hlorpromazine- C orneal deposits; T hioridazine-reT inal deposits.
• 64
EPS-ADAPT:
• 65 Hours to days: Acute Dystonia (muscle spasm , stiffness, oculogyric crisis).
• 66 Days to months: Akathisia (restlessness), Parkinsonism (bradykinesia).
• 67 Months to years: Tardive dyskinesia (orofacial chorea).
• 68 Treatment: benztropine (acute dystonia, tardive dyskinesia), benzodiazepines, ~-blockers
(akathisia).
• 69
• 70 Neuroleptic malignant syndrome (NMS)- \1 alignant FEVER: ~1yoglobinuria, Fever,
E ncephalopathy, unstable Vitals, t Enzymes, muscle Rigidity. Treatment: dantrolene, 0 2 agonist
• 71
(eg, bromocriptine).
• 72
• 73

Lock
a s
Suspend
8
End Bl ock
Item: 62 of 73 ~. I • M k <:] t> al ~· ~
QIO: 1127 .l. ar Previous Next Lab 'lifllues Notes Calculator


53
A 26-year-old med ical student presents to her primary care physician comp laining of intermittent episodes of I"' Aj
54 heart pa lpitations accompanied by diaphoresis, trembl ing, nausea, and lightheadedness. She is worr ied about A
55 having a heart attack. The symptoms first occu rred when the patient was in college, but these unprovoked
episodes have become much more frequent in recent yea rs. Results of physical examination are within norma l
56
limits. Laboratory studies show a thyroid-stimulating hormone leve l of 1.5 !J U/ml, a random blood glucose level of
57 90 mg/dl, and a norma l ECG. A 24-hou r urine col lection is obtained and shows normal levels of urine
58 vanil lylmandel ic acid and fractionated metanephrines.
59
60 Wh ich of the fo llowing is the most like ly diagnosis?
61 :
• 62 A. General ized anxiety diso rder
• 63 B. Hyperthyroidism
• 64
C. Hypog lycemia
• 65
• 66 D. Panic diso rder
• 67 E. Pheoch romocytoma
• 68
F. Posttraumatic stress disorder
• 69
• 70
• 71
• 72
• 73 ~

6
lock
s
Suspend
0
End Block
Item: 62 of 73 ~. I • M k <:] t> al ~· ~
QIO: 1127 .l. ar Previous Next Lab 'lifllues Notes Calculator


53
54
The correct answer is D. 66°/o chose this.
Panic disorder is characterized by unexpected, discrete panic attacks that involve intense fear coupled with
55
adrenergic upregu lation . Symptoms include pa lpitations, shortness of breath, nausea, diaphoresis,
56 lightheadedness, and trembl ing. There is also fea r of going crazy or losing control, fear of dying, derea lization,
57 and depersona lization. Symptoms deve lop abruptly and reach a peak within 10 minutes. To be considered a
58 true "panic attack," these attacks must impair the patient's life by creating constant worry about repeated
attacks, and episodes must be present for at least 1 month. Symptoms cannot be ascribed to drug effects or
59
another medica l or psych iatric condition. Attacks usual ly are associated with agoraphobia (anxiety of being in a
60 place or situation fro m which escape is difficult) and may increase in frequency prior to seeking treatment.
61 Derealization Agoraphobia Panic disorder Perspiration Depersonalization Palpitations lightheadedness Panic attack Nausea Anxiety Dyspnea Adrenergic

62 Downregulation and upregulation

• 63 A is not correct. 19% chose this.


• 64 General ized anxiet y disorder is characterized by chron ic psychological and physical discomfort, but without the
• 65 intermittent, unpredictab le episodes of panic that define panic disorder. Symptoms must be present for more
• 66
than 6 months and must include excessive, uncontro llable worry and anxiety about a nu mber of events or
activities . The patient must have three of the fo llowing : restlessness or feel ing keyed up or on edge; easy
• 67 fatigability; difficulty concentrating, or experiencing the mind going blank; irritability; muscle tension; and sleep
• 68 disturbance .
Generalized anxiety disorder Panic disorder Anxiety disorder Sleep disorder Anxiety Irritability Fatigue (medical) Muscle tone Boredom Muscle
• 69
• 70 B is not correct. 4°/o chose this .
• 71 Hyperthyroidism can mimic panic disorder because of its associated adrenergic stimulation, lead ing to
• 72 headache, palpitations, weight loss, sense of doom, and anxiety . However, this patient's t hyroid-sti mulating
hormone leve l is well within the normal range, indicating that this patient is euthyroid .
• 73 ~ T&..,,.-.,.:..1 ,.. ..:......,,,1_ ..:..,.., &..,.,. .. ...,....,...,.,. I l,,..,,._..k,,.-.,.;..,1;,........, n-..,:,. .,.1;,...,. .. .-1.-. .. r,, ..k,,.-.,.:..1 n-1.-.: .. _ ..:.,...,,.. I 1 ... -..1--&....-. A-.._:,.._, \AI.-.:,.k .. I.,.,..,.. I ...... ......,.,...,,. A ..I .. ,...,,. .. ,.;,. .. .,..,.,..., .......

6
lock
s
Suspend
0
End Block
Item: 62 of 73 ~. I • M k <:] t> al ~· ~
QIO: 1127 .l. ar Previous Next Lab 'lifllues Notes Calculator


53
B is not correct. 4°/o chose this.
54
Hyperthyroidism can mimic panic disorder because of its associated ad renergic stimulation, leading to
55 headache, palpitations, weight loss, sense of doom, and anxiety . However, this patient's thyroid-stimu lating
56 hormone level is well within the normal range, indicating that this patient is euthyroid.
Thyroid-stimulating hormone Hyperthyroidism Panic disorder Euthyroid Palpitations Headache Anxiety Weight loss Hormone Adrenergic receptor
57
58 C is not correct. 2°/o chose this.
59 Hypoglycemia can mimic panic disorder, as patients can present with diaphoresis, tremors, and anxiety caused
60
by catecholamine release. A glucose < 65 mg/dl with an associated autonomic response at the time would
suggest hypoglycemia as the diagnosis.
61 Catecholamine Hypoglycemia Panic disorder Perspiration Anxiety Autonomic nervous system Glucose
62
E is not correct. 7°/o chose this .
• 63
Pheochromocytoma, a catecholamine-producing tumor of the adrenal medulla or sympathetic chain, can
• 64
produce symptoms of episod ic anxiety as wel l as hypertension (either episodic or sustained, and are usually
• 65 refractory to med ical treatment), headache, diaphoresis, and abdomina l pain . Vanillylmandelic acid and
• 66 metaneph r ines are metabol ites of catecholamines, and their levels would be elevated in the setting of
pheochromocytoma .
• 67
Pheochromocytoma Vanillylmandelic acid Adrenal medulla Perspiration Catecholamine Hypertension Headache Sympathetic trunk Neoplasm Anxiety
• 68
Sympathetic nervous system Adrenal gland Abdominal pain Metabolite
• 69
• 70 F is not correct. 2°/o chose this .
• 71
Posttraumatic stress disorder is cha racterized by hyperarousal (bracing for recurrence, or an exaggerated startle
response with sympathetic excitement), intrusive memories (flashbacks), and psych ic numbing (detachment
• 72 from others) . It must be associated with an inciting histor ical traumatic event that threatened the patient's or
• 73 ~
another person's life .

6
lock
s
Suspend
0
End Block
Item: 62 of 73 ~. I • M k <:] t> al ~· ~
QIO: 1127 .l. ar Previous Next Lab 'lifllues Notes Calculator

• Catecholamine Hypoglycemia Panic disorder Perspiration Anxiety Autonomic nervous system Glucose
53
54 E is not correct. 7°/o chose this.
55 Pheochromocytoma, a catecholamine-producing tumor of the adrenal medulla or sympathetic chain, can
56 produce symptoms of episod ic anxiety as wel l as hypertension (either episodic or sustained, and are usually
refractory to med ical treatment), headache, diaphoresis, and abdomina l pain . Vanillylmandelic acid and
57
metanephrines are metabol ites of catecholamines, and their levels would be elevated in the setting of
58 pheochromocytoma.
59 Pheochromocytoma Vanillylmandelic acid Adrenal medulla Perspiration Catecholamine Hypertension Headache Sympathetic trunk Neoplasm Anxiety

60 Sympathetic nervous system Adrenal gland Abdominal pain Metabolite

61 F is not correct. 2°/o chose this.


62 Posttraumatic stress disorder is characterized by hyperarousal (bracing for recurrence, or an exaggerated startle
• 63 response with sympathetic excitement), intrusive memories (flashbacks), and psych ic numbing (detachment
• 64 from others) . It must be associated with an inciting historical traumatic event that threatened the patient's or
another person's life .
• 65
Posttraumatic stress disorder Startle response Fight-or-flight response Stress (psychological) Stress (biology) Intrusive thought Flashback (psychology)
• 66
• 67
• 68 Bottom Line:
• 69 Panic disorder is characterized by unexpected, short episodes of intense fear associated with adrenergic
stimulation (eg, pa lpitations, shortness of breath, a sensation of choking or being smothered, and nausea) .
• 70
These attacks cause significant impa irment in the individual 's life due to constant worry about having another
• 71 attack.
• 72 Panic disorder Palpitations Nausea Dyspnea Adrenergic receptor

• 73 ~

6
lock
s
Suspend
0
End Block
Item: 62 of 73 ~ 1 • M k -<:J 1>- Jil ~· !:';-~
QIO: 1127 ..L ar Pre v ious Next Labfli!llues Notes Calcula t o r
A A
53 FA17 p533.1
54
Panic disorder Defined by recurrent panic attacks (periods P\~ JCS .
55 of intense fear and discomfort peaking in Diagnosis requires attack followed by I month
56 10 minutes with at least 4 of the folio" ing): (or more) of 1 (or more) of the following:
57 Palpitations, Paresthesias, de Personalintion or Persistent concern of additional attacks
58 derealization, \ bdominal distress or ' ausea, Worrying about consequences of attack
Intense fear of dying, Intense fear oflosing Behavioral change related to attacks
59
control or "going craz~," light-headed ness, S} mptoms are the S}Stemic manifestations of
60 C hest pain, C hills, C hoking, Sweati ng, fear.
61 Shaking, Shortness of breath. Strong genetic
62 component. t risk of suicide. Treatment:
. 63 CBT, SSRls, and venlafaxine arc fir~! line .
Benzodiazepines occasionally used in acute
• 64
setting .
• 65
• 66
FA17 p 533.3
• 67
• 68
Generalized anxiety Anxiety lasting> 6 m onths unrelated to a speci fi c person, situation, or event. Associated with
disorder restlessness, irritability, sleep dis turbance, ratigue, muscle tension, difficulty concentrati ng.
• 69
Treatment: C BT, SSR ls, SNR ls arc first line. Buspirone, TCAs, benzodiazepines are second line .
• 70
Adjustment d isorder-emotiona l symptoms (anxiety, depression) that occur 11 ithin 3 months of
• 71
an identifiable psychosocial stressor (cg, di1orcc, illness) lasting < 6 months once the stressor has
• 72 ended. If stressor lasts> 6 months and causes continual impairment. it is GAD. Treatment: C BT,
• 73 SSRis.

a
Lock
s
Suspend
8
End Bl ock
Item: 62 of 73 ~ 1 • M k -<:J 1>- Jil ~· !:';-~
QIO: 1127 ..L ar Pre v ious Next Labfli!llues Notes Calcula t o r
A A
53 FA17 p 533.3

54 Generalized anxiety Anxiety lasting> 6 months nnrelated to 01 specific person, situation, or e,·ent. Associated with
55 disorder restlessness, irritability, sleep disturbance, fatigue, muscle tension, difficulty concentrating.
56 Treatment: C BT, SSRis, SNR ls are first line. Buspirone, TC s, benzodiazepines are second line.

57 Adjustment disorder-emotional symptoms (all\iety, depression) that occur" ithin 3 months of


an identifiable psychosocial stressor (eg, di,orce. illness) lasting< 6 m onths once the stressor has
58
ended. If stressor lasts> 6 months and causes continual impairment. it is GAD. Treatment: CBT,
59
SSRls.
60
61
FA17 p 326.1
62 Pheochromocytoma
. 63
ETIOLOGY !\lost common tumor of the <~drenalmcdulla in Rule of 10'!.:
• 64 adults fl. Derived from chromaffin cel ls (arise 10% malignant
• 65 from neural crest). 10% bi lateral
• 66 May be associated with germ line mut<~ t ions (eg, IO'k extra-<~clrena l (eg, bladder wall, organ of
NF-1, VHL, RET [MR ' ZA, ZB]). Zuckerkand l)
• 67
10% calcify
• 68
IO'k kids
• 69
• 70
• 71
SYMPTOMS l\ losllumors secrete epinephrine, Episodic hyperadrenergic symptoms (5 P '!.):
• 72
norepinephrine, and dopamine, which can P ressure (t BP)
• 73 cause episodic h''Pertension . Pain (headache)

a
Lock
s
Suspend
8
End Bl ock
Item: 62 of 73 ~ 1 • M k -<:J 1>- Jil ~· !:';-~
QIO: 1127 ..L ar Pre v ious Next Labfli!llues Notes Calcula t o r
A A
53 ETIOLOGY lost common tumor of the adrenal medulla in Rule of 10'~:
54 adults · . Derived from chromaffin cells (arise 10% malignant
from neural crest). 10% bilateral
55
lay be associated with germline mutations (eg, 10% extra-adrenal (eg, bladder wall, organ of
56
1 F-1, VHL, RET [ME 2 , 2B]). Zuckerka nd I)
57 10% calcif\'
58 107c kids
59
60
61 SYMPTOMS I\ lost tumors secrete epinephrine, Episodic h~ peradrenergic S} mptoms (5 P'!.):
62 norepinephrine, and dopamine, which can Pressure (t BP)
. 63 cause episodic hypertension . Pain (headache)
• 64
Symptoms occur in "spells" -relapse and rc111it. Perspiration
Palpitations (tachycardia)
• 65
Pallor
• 66
FINDINGS t catecholamines and metancphrines in urine
• 67 and plasma .
• 68
TREATMENT Jrreversiblea-antagon ists (eg, Phenox} benzamine (16 letters) is given for
• 69 phenoxybenzamine) followed by P-blockers pheochromocytoma (also 16 letters).
• 70 prior to tumor resection. a -blockade must be
• 71 achieved before giving P-blockers to moid a
• 72
hypertensive crisis.

• 73

a
Lock
s
Suspend
8
End Bl ock
Item: 63 of 73 ~ 1 • M k -<:J 1>- Jil ~· !:';-~
QIO: 5229 ..L ar Pre v ious Next Lab fli!ltues Notes Calcula t o r

IAA]
A A
53
A 65-year-old patient who has been taking med ication for a chronic condition now presents with the
54 symptoms seen in the video clip. His careg iver recalls that the patient's doctor recently increased the dose of
55 his med ication. His original condition was diagnosed when he was 25 years old, and he has been ta king the
56
same med ication ever since. His careg iver states t hat t he patient's symptoms have developed over the past month
and the faculty at his living faci lity have noticed that he has begun taking short, quick steps when he walks.
57
OPE N MEDIA
58
59
60 What condition was the patient most likely diagnosed with 40 years ago?
61
:
62 A. Adrenocortical insufficiency
• 63
B. Amyotrophic lateral sclerosis
• 64
• 65
C. Obsessive-compulsive disorder

• 66 D. Parkinson disease
• 67 E. Schizoaffective disorder
• 68
• 69
• 70
• 71
• 72
• 73

a
Lock
s
Suspend
8
End Bl ock
Item: 63 of 73 ~. , . M k <:] t> al ~· ~
QIO: 5229 .l. ar Previous Next lab 'lifllues Notes Calculator


53
The correct answer is E. 54°/o chose this.
54
The patient seen in the video demonstrates parkinsonian symptoms consistent with long-term antipsychotic use.
55 Parkinsonism and schizophrenia may be viewed on a continuum; too much dopamine is thought to cause
56 schizoph renia, whereas too little dopamine is thought to cause Parkinson disease. Therefore it stands to reason
57
that long-ter m use of antipsychotics (which decrease dopamine levels) can cause parkinsonian symptoms . Both
typical and atypica l antipsychotics may cause the symptoms demonstrated in the video cl ip.
58 Parkinsonism Schizophrenia Antipsychotic Dopamine Parkinson' s disease Atypical antipsychotic
59
A is not correct. 4°/o chose this.
60
Adrenocortica l insufficiency is t reated with steroids. Long-ter m use may cause psychiatric adverse effects,
61 rang ing from bipo lar disorder to sch izophrenia. Add itiona lly, steroids frequently cause metabol ic syndrome due
62 to changes in the body's fat distr ibution.
Bipolar disorder Metabolic syndrome Schizophrenia Adrenal insufficiency Steroid Anabolic steroid Psychiatry Adrenal cortex Fat Metabolism
63
0 64 B is not correct. 6°/o chose this.
0
65 The main treatment for amyotroph ic lateral sclerosis is riluzole. It decreases the release of glutamine, and the
0
66 main adverse effects are dizziness, weakness, and granulocytopenia .
Neutropenia Amyotrophic lateral sclerosis Riluzole Granulocytopenia Glutamine Dizziness
0 67
0 68 C is not correct. 7°/o chose this.
0 69 The fi rst-line treatment for obsessive-compulsive disorder is with selective serotonin reupta ke inh ibitors such as
paroxetine and fluoxetine. Their major adverse effects include gastrointestina l upset and anorgasmia .
0 70
Obsessive-compulsive disorder Paroxetine Fluoxetine Anorgasmia Serotonin Selective serotonin reuptake inhibitor Reuptake Therapy First-line treatment
0 71
Gastrointestinal tract
0 72
0 73
D is not correct. 29% chose this.
~

6
lock
s
Suspend
0
End Block
Item: 63 of 73 ~. , . M k <:] t> al ~· ~
QIO: 5229 .l. ar Previous Next lab 'lifllues Notes Calculator

53
- - - - - --- - -- - -- - -- -- . -- -- -- --- . -- --
to changes in the body's fat distr ibution.
54 Bipolar disorder Metabolic syndrome Schizophrenia Adrenal insufficiency Steroid Anabolic steroid Psychiatry Adrenal cortex Fat Metabolism

55 B is not correct. 6°/o chose this.


56 The main treatment for amyotroph ic lateral sclerosis is riluzole. It decreases the release of glutamine, and the
57 main adverse effects are dizziness, wea kness, and granulocytopenia .
Neutropenia Amyotrophic lateral sclerosis Riluzole Granulocytopenia Glutamine Dizziness
58
59 C is not correct. 7°/o chose this.
60 The fi rst-line treatment for obsessive-compulsive disorder is with selective serotonin reupta ke inh ibitors such as
61
paroxetine and fluoxetine. Their major adverse effects include gastrointestina l upset and anorgasmia .
Obsessive-compulsive disorder Paroxetine Fluoxetine Anorgasmia Serotonin Selective serotonin reuptake inhibitor Reuptake Therapy First-line treatment
62
Gastrointestinal tract
63
0 64 D is not correct. 29% chose this.
0
65
Although the patient in the video clip is demonstrating parkinsonian symptoms, Parkinson disease is very
un likely to be diagnosed in a 25-year-o ld patient, making sch izoaffective disorder a much more likely cond ition.
0
66 Schizoaffective disorder Parkinson' s disease
0 67
0 68
0 69
Bottom Line:
0 70 Treatment with antipsychotics (both typica l and atypical) has been demonstrated to cause parkinsonian
symptoms in the short term and other symptoms like tardive dyskinesia in the long term.
0 71
Tardive dyskinesia Antipsychotic Dyskinesia Parkinson' s disease
0 72
0 73 ~

6
lock
s
Suspend
0
End Block
Item: 63 of 7 3 ~ 1 • M k -<:J 1>- Jil ~· !:';-~
QIO: 5229 ..L ar Pre v ious Next Lab fli!ltues Notes Calcula t o r
A A
53
FA17 p 543.2
54
55
Atypical Aripiprazole, asenapinc, clozapinc, olanzapinc, quctiapinc, ilopcridonc, palipcridonc,
antipsychotics risperidone, lurasidonc, ziprasidonc.
56
MECHANISM 1 ot completely understood. \lost arc 0 1
57
antagonists; aripiprazole is 0 2 partial agonist.
58
Varied effects on 5-HT2, dopamine, and
59 a - and l-1 1-reccptors.
60 CLINICAL USE chizophrenia-both positi\'e and negative Usc clozapine for treatment-resistant
61 symptoms. Also used for bipolar disorder, schizophrenia or schizoaffective disorder and
62 OCD, anxiety disorder, depression, mania, for suicidality in schizophrenia.
63
Tourette syndrome.
• 64 ADVERSE EFFECTS All-prolonged QT inlcr\'al, fewer EPS and
antichol inergic side effects than typical
• 65
anti psychotics.
• 66 O besity
"-pines"-metabolic syndrome (weight gain, Olanzapinc -+

• 67 diabetes, hyperlipidem ia) .


• 68 C lozapine-agranulocytosis (mon itor WBCs · 1ust watch bone marrow clozely with clozapine.
• 69 frequently) and seizures (dose related).
Risperidone-hyperprolactinemia (amenorrhea,
• 70
galactorrhea, gynecomastia).
• 71
• 72
FA17p530.1
• 73

a
Lock
s
Suspend
8
End Bl ock
Item: 63 of 7 3 ~ 1 • Ma rk -<:J 1>- Jil ~· !:';-~
QIO: 5229 ..L Prev ious Next Lab fli!ltues Notes Cal culat o r


53
FA17 p530.1
54
Schizophrenia Chronic mental disorder with periods of Frequent cannabis use is associated" ith
55
psychosis, disturbed beha' ior and I hought, ps}chosis/schizophrenia in teens.
56
and decline in functioning lasting> 6 Lifetime pre,·alence-1.5% (males= females,
57 months. Associated with t dopaminergic frican Americans= Caucasians). Presents
58 acti\'ily, l dendritic branching. earlier in men (late teens to early 20s 's late
59 Diagnosis requires at least 2 of the following, 20s to early 30s in women). Patients are at t
and at least I of these should include 1- 3 risk for suicide.
60
(first 4 are "positi\'e symptoms"): Venlriculomegaly on brain imaging.
61
I. Delusions Treatment: atypical antipsychotics (eg,
62 risperidone) are first line.
2. llallucinations-often auditory
63 3. Disorganized speech 'egative symptoms often persist after treatment,
• 64 4. Disorganized or catatonic behavior despite resolution of positi,·e symptoms.
• 65 5. egati,·c symptoms (affect ive Aallen ing,
• 66 avolition, anhedonia, asociality, alogia)
• 67 Brief psychotic disorder- lasting< l month,
usually stress related .
• 68
• 69
Schizophreniform disorder- lasting l -6
months.
• 70
Schizoaffective disorder- \ilcets criteria for
• 71
schizophrenia in addition to major mood
• 72
disorder (major depressi\'e or bipolar). To
• 73 differentiate from a maior mood disorder
a
Lock Suspend
s 8
End Bl ock
Item: 64 of 73 ~. , . M k <:] t> al ~· ~
QIO: 5141 .l. ar Previous Next lab 'lifllues Notes Calculator


53
A 42-year-old Afr ican American woman presents to her physician with pain that she has had for several
54 years. She states she always worr ies about her pain, and fea rs as though "something is seriously wrong."
55 She describes the pain as constant, located in her neck, lower back, and hips. Over the years she has had
extensive work-ups, including laboratory testing and mu ltip le modal ities of imag ing, none of which has led to an
56
exp lanation for he r symptoms. She exhibits pe rsistent anxiety about he r symptoms, and fee ls as though they
57 interfere with her day-to-day life. She has had no histo ry of t rauma, but does report feel ing "down " occasiona lly.
58
59 Wh ich t reatment wou ld be most appropriate for her at this time'
60
:
61 A. A 14-day course of oral doxycycl ine
62
B. Cognitive behaviora l therapy
63
0 64 C. Low-dose fentanyl patch
0
65 D. No treatment; he r symptoms wi ll resolve spontaneously with time
0
66 E. Nonsteroidal anti-inflammatory drugs (NSAIDs)
0 67
0 68
0 69
0 70
0 71
0 72
0 73 ~

6
lock
s
Suspend
0
End Block
Item: 64 of 73 ~. , . M k <:] t> al ~· ~
QIO: 5141 .l. ar Previous Next lab 'lifllues Notes Calculator


53
The correct answer is B. 87°/o chose this.
54
This patient is presenting with somatic symptom disorder. Th is diagnosis comes under the new Diagnostic
55 and Statistica l Manual of Menta l Disorders, Fifth Edition (DSM V) categor ization (see chart). It is characterized
56 by somatic symptoms (eg, pain) that cause psychosocial distress or impa irment. There is typically a history of
57 anxiety or depression in these patients, and such disorders are seen more common ly in fema les than males .
Treatment typica lly includes rehabil itation, such as cognitive behavioral therapy, physica l therapy, and
58
psychotherapy . Analgesia is usually not helpful. Selective serotonin reuptake inhibitors (SSRis) are fi rst-l ine
59 pharmacotherapy . Tricyclic antidepressants (TCAs) and ven lafaxine may be therapeutic.
60 Cognitive behavioral therapy Diagnostic and Statistical Manual of Mental Disorders Venlafaxine Somatic symptom disorder Serotonin Somatization disorder

61 Selective serotonin reuptake inhibitor Tricyclic antidepressant Pharmacotherapy DSM- 5 Analgesic Symptom Anxiety Antidepressant Psychotherapy

62 Major depressive disorder Reuptake Depression (mood) Physical therapy Psychosocial Somatic nervous system

63
Lab findings and
Disease state Clinical features
diagnosis
Tr eatment
64
Somatic • Symptoms that cause distress or • Regularty scheduled visits (rather
symptom psycllolog1cal •mpaument than complarnt based)
• 65 disorder • Persistent thoughts about • Psychotherapy in the form of
• No laboratory abnormal~ies
symptoms and disease cognitive behavioral therapy (CBT)
• 66 • Anxiety about state of health • Pharmacotherapy starting wrth
• History of anxiety or depressioo SSR!s (eg, lluoxetine)
• 67 1 Fibromyargra • MSK pain 1n a number of • No evidellOO or inflammation;
locations-tenderness at son tissue (no elevated CRP, ESR) • FirsHine treatment involves exefcise
locations • PreVIOusly required a specific • In nonresponders, TCSs-
• 68 • Nonspecific complaints (esp, numbet or points of tenderness am~ripty!Jnes can be used)
fatigue) (11118) (outdated) • If this fails, SNRrs can be used
• 69 • You~iddle-aoed women
Polymyositis • Muscle inftammalion present;
• 70 markers if muscle damage
elevated • Corticosteroids are first l1ne
• Symmetrical prox.imal weakness
• 71 • Elevated CK • In nonresponders, stronger
• Manifests with difficultycomt:ing
• ANAmaybe(+) inSO% of immunosuppressives (eg,
haar, stand1ng from chair methotrex.ate, cyclophosphamide)
pabents
• 72
• 73 ~

6
lock
s
Suspend
0
End Block
Item: 64 of 73 ~. , . M k <:] t> al ~· ~
QIO: 5141 .l. ar Previous Next Lab 'lifllues Notes Calculator
-~~~ _ _ ,
• Polymyositis • Muscle inftammalion present;
53 markers if muscle damage
elevated • Corticosteroids are first l1ne
• Symmetrical prox.imal weakness
54 • Manifests with difficultycomt:ing
• Elevated CK • In nonresponders, stronger
• ANAmaybe(+) inSO% of immunosuppressives (eg,
hatr, stand1ng from chair methotrexate, cyclophosphamide)
55 paben1s
• EMG abnormality in 90% or
oatients
56 PoJymyaJgla • Older patients (mean onset 70)
rh•umatica • Morning stiffness, bursitis, aches
57 • Worse pain in shoulders and hips • Elevated markers of system1c
• No musculartenderness inRammation
58 • Constitutional symptoms • ESR, CRP elevated • Corticosteroids
• 10% of patients develop temporal • Correlates with sever~y of
arteritis (conversely, about half of disease
59 patients with temporal arteritis
develoo oolvmvaiaia rheumatic\
60
61 A is not correct. 1 °/o chose this.
62 Doxycycline is the standard t reatment for Lyme disease. Lyme disease in advanced stages can man ifest with
joint pain and fatigue. However, the patient has had extensive laboratory and clinica l assessments, with no
63
indication of an infectious process.
64 Lyme disease Doxycycline Arthralgia Fatigue (medical)

• 65
C is not correct. 3°/o chose this .
• 66
Ana lgesia, includ ing opioid treatment, is general ly not indicated for pain disorder. Physica l therapy, t r icyclic
• 67 antidepressants (TCAs), and ven lafaxine are more appropriate therapy .
• 68 Venlafaxine Opioid Analgesic Tricyclic antidepressant Antidepressant Physical therapy Tricyclic

• 69 D is not correct. 3°/o chose this.


• 70 This answer is incorrect because there are standard t reatments for pain disorder. Moreover, pain disorder may
• 71 not resolve on its own once it has been persistent for several years . In contrast, conversion disorder often
• 72
resolves spontaneously . Patients with conversion disorder present with symptoms or deficits of voluntary or
sensory function (eg, bl indness, seizure, or paralysis) . These symptoms often occur in close temporal
• 73 ~ n::. l ;:~ti nn~h i n tn ~i"r,::.~~ nr i nt,::.n~,::. ,::.mntinn rnnv,::.r~ i nn rli~nrri,::. r i ~ mnr,::. rnmmnn in vn11n11 f,::.m;:~l,::.~ 1 ,::.~~-"'ri' r r;:~t,::.ri

6
lock
s
Suspend
0
End Block
Item: 64 of 73 ~. , . M k <:] t> al ~· ~
QIO: 5141 .l. ar Previous Next lab 'lifllues Notes Calculator

53 • C is not correct. 3°/o chose this.


54
Ana lgesia, includ ing opioid treatment, is general ly not indicated for pain disorder. Physica l therapy, t r icyclic
antidepressants (TCAs), and ven lafaxine are more appropriate therapy.
55 Venlafaxine Opioid Analgesic Tricyclic antidepressant Antidepressant Physical therapy Tricyclic
56
D is not correct. 3°/o chose this.
57
This answer is incorrect because there are standard t reatments for pain disorder. Moreover, pain disorder may
58 not resolve on its own once it has been persistent for several years . I n contrast, conversion disorder often
59 resolves spontaneously . Patients with conversion disorder present with symptoms or deficits of voluntary or
60 sensory function (eg, bl indness, seizure, or paralysis). These symptoms often occur in close temporal
relationship to stress or intense emotion . Conversion disorder is more common in young females, less-educated
61
people, and those from lower socioeconomic classes.
62 Conversion disorder Paralysis Visual impairment Pain disorder Socioeconomics Pain management Emotion

63
E is not correct. 6°/o chose this.
64
NSAIDs are he lpful for many conditions, includ ing rheumatoid arth r itis and osteoarthritis, wh ich have f indings
• 65 on imaging. I n pa in disorder, imag ing and lab tests do not identify physiologic sou rces of pa in. NSAIDs reduce
• 66 fevers, pain, and inflammation. Analgesics, including NSAIDs, are usually not effective in treating pain disorder .
Rheumatoid arthritis Osteoarthritis Analgesic Nonsteroidal anti-inflammatory drug Arthritis Inflammation
• 67

• 68
• 69 Bottom Line:
• 70
Somatic symptom disorder is characterized by somatic symptoms (eg, pain) that cause psychosocial distress
• 71 or impairment. Treatment typical ly includes cognitive behavioral therapy and SSRis .
Cognitive behavioral therapy Somatic symptom disorder Somatization disorder Symptom Selective serotonin reuptake inhibitor Somatic nervous system
• 72
Psychosocial Behaviour therapy
• 73

6
lock Suspend
s 0
End Block
Item: 64 of 7 3 ~ 1 • M k -<:J 1>- Jil ~· !:';-~
QIO: 5141 ..L ar Pre v ious Next Lab fli!ltues Not es Calcula t o r
A A
53
FA17 p 536.3
54
55 Somatic symptom and Category of disorders characterized b) physical symptoms causing significant distress and
related disorders impairment. Both illness production and motivation are unconscious drives. Symptoms not
56
intentionally produced or feigned . .I\ lore common in women.
57
Somatic symptom Variety of bodily complaints (eg, pain, fatigue) lasting for months to years. Associated with
58
disorder excessi,·e, persistent thoughts and anxiel) about symptoms. ;..ray co-occur with medical illness.
59 Treatment: regular office \'isits with the same physician in combination with psychotherapy.
60 Conversion disorder Loss of sensory or motor function (eg, paralysis, blindness, mutism), often following an acute
61 (fun ctional stressor; patient is aware of but sometimes indifferent toward srmptoms ('"Ia belle indifference");
62 neurologic symptom more common in females, ;tdolescents, and young adults.
63 disorder)

64 Illness anxiety Excessive preoccupation with acquiring or having a serious illness, often despite medical
• 65
disorder evaluation and reassurance; minimal somatic symptoms.
(hypochondriasis)
• 66
• 67
FA17 p 545.4
• 68
Tricyclic Amitriptyline, nortriptyline, imipramine, dc~ipram i nc, clomipramine, doxepi n, amoxapine.
• 69
antidepressants
• 70
MECHANISM Inhibit ! E and 5-I IT reuplake.
• 71
CLINICAl USE lajor depression, OCD (clomipramine), peripheral neuropathy, chronic pain, migraine
• 72
prophylaxis. Nocturnal enuresis (imipramine, although ad,·erse effects may limit use).
• 73

a
Lock
s
Suspend
8
End Bl ock
Item: 64 of 7 3 ~ 1 • M k -<:J 1>- Jil ~· !:';-~
QIO: 5141 ..L ar Pre v ious Next Lab fli!ltues Not es Calcula t o r
A
. . .. .. . .. ...
A
53
Conversion disorder Loss of sensory or motor function (eg, paralysis, blindness, mutism), often following an acute
54 (functional stressor; patient is aware of but sometimes indifferent toward symptoms ("Ia belle indifference");
55 neurologic symptom more common in females, adolescents, and young adults.
56 disorder)
57 Illness anxiety Excessi,·e preoccupation" ith acquiring or having a serious illness, often despite medical
58 disorder evaluation and reassurance; minimal somatic symptoms.
(hypochondriasis)
59
60
FA17 p 545.4
61
Tricyclic Amitriptyline, nortriptyline, imipramine, clcsipmminc, clomipramine, doxcpin, amoxapine.
62
antidepressants
63
MECHANISM Inhibit NE and ) -l iT reuptake.
64
• 65
CLINICAL USE lajor depression, OCD (clomipramine), peripheral neuropathy, chronic pain, migraine
prophylaxis. octurnal enuresis (imipramine, although adverse effects may limit use).
• 66
ADVERSE EFFECTS Sedation, a 1-blocking effects inclnding postural hypotension, and atropine-like (anticholinergic)
• 67
side effects (tachycardia, minary retention, dry mouth). 3° TCAs (amitriptyline) have more
• 68 antichol inergic effects than 2° TCAs (nortriptyline). Can prolong QT interval.
• 69 '!'ri-C 's: Convulsions, Coma, C ardiotoxicity (arrhythmia due to 1 a+ channel inhibition);
• 70 also respiratory depression, hyperpyrexia. Confusion and hallucinations in elderly due to
• 71 anticholinergic side effects (nortriptyline better tolerated in the elderly). Treatment: 1al JC03 to
prevent arrhythmia .
• 72
• 73

a
Lock
s
Suspend
8
End Bl ock
Item: 65 of 73 ~. , . M k <:] t> al ~· ~
QIO: 2342 .l. ar Previous Next lab 'lifllues Notes Calculator


53
A 22-year-old college student compla ins of ants crawl ing over his body, and his friends have noted increasing ~~AI
54 agitation and threatening gestures. He is febrile, restless, and tachycardic, and his pupils are ma r ked ly
55 dilated. He has been studying "at al l hours" for his upco ming final examinations.
56
57 The drug causing his sy mptoms has wh ich of the fo llowing mechan isms of action'
58 :
59
60 A. I ncreases extracellula r concentration of serotonin wit hout changing norepinep hrine and epineph rine
61 levels
62 B. I nhibits deg radation of acetylcho line
63
C. I nhibits sodium -potassium adenosine triphosphatase
64
D. Promotes chloride ion passage th roug h the y-aminobutyr ic acid receptor
• 65
• 66 E. Promotes extravesicu lar release of norepineph rine and dopamine
• 67
• 68
• 69
• 70
• 71
• 72
• 73 ~

6
lock
s
Suspend
0
End Block
Item: 65 of 73 ~. , . M k <:] t> al ~· ~
QIO: 2342 .l. ar Previous Next lab 'lifllues Notes Calculator


53
The correct answer is E. 73°/o chose this.

I
54
55 I n addition to abuse as recreationa l drugs, amphetamines also may be abused by students as a study and test-
taking aid. This patient shows signs of acute amphetam ine overdose, w hich include the sympathomimetic
56
symptoms of pupil di lation, tachycardia, and hypertension, as well as hal lucinations, delusions, and fever.
57 Treatment is primarily supportive care; urine acidification is no longer recommended as treatment. I
58 Amphetamines med iate their effects by promoting release of catecholamines from intracel lular vesicles into the
59 cytoplasm; the increased concentration of cytoplasmic catecholamines then causes the catecholamine
60 transporters on the cell membrane to reverse function, resulting in export of norepinephrine and dopamine into
the synaptic cleft . Add itional causes of pupillary di lation, although less likely in th is patient given the cl inical
61
context, include other sympathomimetic drugs such as cocaine or decongestants (ephed r ine, pseudoephedrine),
62 lyserg ic acid diethylamide use, anticholinergic syndrome (antihistam ines, atropine), and opioid withd rawal.
63 Catecholamine Anticholinergic Pseudoephedrine Amphetamine Atropine Substituted amphetamine Sympathomimetic drug Dopamine Norepinephrine
64 lysergic acid diethylamide Cytoplasm Tachycardia Synaptic cleft Chemical synapse Mydriasis Cell membrane Hypertension Antihistamine Opioid Cocaine
65 Pupillary response Hallucination Recreational drug use Drug withdrawal Urine Vasodilation Fever Decongestant Opioid withdrawal Vesicle (biology and chemistry)
• 66
Drug overdose Synapse lysergic acid
• 67
A is not correct. 9°/o chose this .
• 68
Selective serotonin reuptake inhibitors ( SSRis) selectively prevent reuptake of serotonin from the synaptic cleft
• 69
without chang ing levels of norepinephrine or dopam ine. If combined with other drugs that also increase synaptic
• 70 seroton in levels, such as monoamine oxidase inh ibitors or meperidine, SSRi s can cause serotonin syndrome .
• 71 Serotonin synd rome is cha racterized by hyperthermia, muscle rigidity, cardiovascula r collapse, and seizure; and
• 72
it is treated with cyproheptadine ( a seroton in antagonist) and supportive care .
Serotonin syndrome Cyproheptadine Dopamine Synaptic cleft Norepinephrine Hyperthermia Serotonin antagonist Serotonin Chemical synapse
• 73

6
lock
s
Suspend
0
End Block
Item: 65 of 73 ~. , . M k <:] t> al ~· ~
QIO: 2342 .l. ar Previous Next Lab 'lifllues Notes Calculator

53 A is not correct. 9°/o chose this.


54 Se lective serotonin reupta ke inhibitors (SSRis) selectively prevent reuptake of serotonin from the synaptic cleft
55 without chang ing levels of norepinephrine or dopamine. If combined with other drugs that also increase synaptic
56
seroton in levels, such as monoamine oxidase inh ibitors or meperidine, SSRis can cause serotonin syndrome.
Seroton in synd rome is cha racterized by hyperthermia, muscle rigidity, cardiovascula r col lapse, and seizure; and
57 it is treated with cyproheptad ine (a serotonin antagonist) and supportive care .
58 Serotonin syndrome Cyproheptadine Dopamine Synaptic cleft Norepinephrine Hyperthermia Serotonin antagonist Serotonin Chemical synapse

59 Selective serotonin reuptake inhibitor Receptor antagonist Monoamine oxidase Pethidine Monoamine oxidase inhibitor Reuptake Monoamine neurotransmitter

60 Shock (circulatory) Hypertonia Epileptic seizure Synapse Circulatory system Muscle Antagonist

61
B is not correct. 7°/o chose this.
62
Cho linesterase inhibitors prevent the enzyme acetylcholinesterase from degrading acetylcho line, which leads to
63 accumulation of the neurotransmitte r in the synaptic cleft, producing a chol inergic synd rome. Signs of
64 chol inergic syndrome, such as that mediated by ir reve rsible chol ine sterase inhibitors, include Diarrhea,
Ur ination, Miosis, Bronchospasm, Bradycardia, skeletal muscle Excitation, Lacr imation, Sweating, and
65
Sa livation (helpful mnemonic : DUMBBELSS) . Treatment includes atropine and pral idoxi me .
• 66 Atropine Acetylcholinesterase Cholinergic Neurotransmitter Synaptic cleft Enzyme Acetylcholine Choline Cholinesterase Acetylcholinesterase inhibitor
• 67 Chemical synapse Skeletal muscle Pralidoxime Diarrhea SLUDGE syndrome Mnemonic Synapse Muscle
• 68
C is not correct. 5°/o chose this .
• 69
Digoxin and other ca rdiac glycosides med iate their effects by direct inhibition of Na +j K+ adenosine
• 70
triphosphatase; the lack of sodium ions outside of the ce ll then prevents function of the sod ium-calcium
• 71 exchanger, lead ing to accumulation of calcium ions within ca rdiac muscle cel ls. Symptoms of digoxin toxicity
• 72 include vision changes with blurry yel low/ red halos around objects, bradycardia, nausea, hyperkalemia, and
ECG changes. Along with management of arrhythmia and electrolytes, treatment includes administration of anti-
• 73 ,..l i ,.,"'vi""' ~P"''+-i h",-1'' C:':).h +,..~,.,~ol"\+-e-

6
lock
s
Suspend
0
End Block
Item: 65 of 73 ~. , . M k <:] t> al ~· ~
QIO: 2342 .l. ar Previous Next Lab 'lifllues Notes Calculator

• Atropine Acetylcholinesterase Cholinergic Neurotransmitter Synaptic cleft Enzyme Acetylcholine Choline Cholinesterase Acetylcholinesterase inhibitor
53
Chemical synapse Skeletal muscle Pralidoxime Diarrhea SLUDGE syndrome Mnemonic Synapse Muscle
54
55 C is not correct. 5°/o chose this.
56 Digoxin and other card iac glycosides med iate t heir effects by direct inhibition of Na +jK+ adenosine
57 t riphosphat ase; the lack of sodium ions outside of the cell then prevents funct ion of the sod ium- calcium
exchanger, lead ing to accumu lation of calcium ions within cardiac muscle cel ls. Sympt oms of digoxin t oxicity
58
include vision changes with blurry yellow/red ha los around objects, bradycardia, nausea, hyperkalemia, and
59 ECG changes. Along with management of arrhythmia and electrolyt es, t reat ment includes administ rat ion of anti-
60 digoxin antibody Fab fragments .
Hyperkalemia Digoxin Bradycardia Cardiac muscle Cardiac glycoside Cardiac arrhythmia Nausea Electrocardiography Glycoside Antibody Sodium Calcium
61
Electrolyte Adenosine Calcium in biology ATPase Toxicity
62
63 D is not correct. 6 °/o chose this.
64 Alcohol, barbiturates, and benzod iazepines activat e the y- aminobutyric acid receptor. Intoxication with these
65
drugs causes slurred speech, ataxia, respiratory depression, and coma . Wit hdrawa l from these drugs can cause
hal lucination, seizure, and potentially fatal autonom ic inst ability (del irium t remens) .
• 66 Hypoventilation Ataxia Benzodiazepine Hallucination Barbiturate Dysautonomia Coma Delirium Autonomic nervous system Alcoholic beverage Dysarthria
• 67
Major depressive disorder Alcohol Depression (mood) Receptor (biochemistry) Epileptic seizure Relaxed pronunciation Substance intoxication Alcohol intoxication
• 68
• 69
• 70 Bottom Line :
• 71 Amphetamines act by promoting extravesicular release of catecholam ines into the synaptic cleft. Overdose
resu lts in pupil dilation, hallucinations, delusions, and fever .
• 72
Synaptic cleft Chemical synapse Substituted amphetamine Catecholamine Pupillary response Mydriasis Hallucination Fever Vasodilation Delusion
• 73 ~

6
lock
s
Suspend
0
End Block
Item: 65 of 73 ~. , . M k <:] t> al ~· ~
QIO: 2342 .l. ar Previous Next lab 'lifllues Notes Calculator

• •
53
FA17 p 540.1
54
Psychoactive drug intoxication and withdrawal
55
DRUG ltiTOXICATION WITHDRAWAL
56
Depressants
57
onspecific: mood elevation, l anxiety, onspecific: anxiety, tremor, seizures,
58 . .
sedation, behavioral disinhibition, respiratory lllSOmnaa.
59 depression.
60 Alcohol Emotional labil ity, slurred speech, ataxia, Time from last drink:
61
62
63
coma, blackouts. Serum y-glutamyltransferase
(GGT)- sensitive indicator of alcohol usc.
AST value is twice ALT value.
3- 36 hr: minor symptoms similar to other
depressants
6- 48 hr: withdrawal seizures
12- 48 hr: alcoholic hallucinosis (usually visual)
I
64
48-96 hr: delirium tremens (DTs) in 5% of
65 cases
• 66 Treatment: benzodiazepines.
• 67 Opioids Euphoria, respiratory and CNS depression, Sweating, dilated pupils, piloerection ("cold
• 68 l gag reAex, pupillary constriction (pinpoint turkey"), fever, rh inorrhea, yawning, nausea,
• 69 pupils), seizures (overdose). Most common stomach cramps, diarrhea {"Au-like" symptoms).
• 70
cause of drug overdose death. Treatment: Treatment: long-term support, methadone,
naloxone. buprenorphine.
• 71
Barbiturates Low safety margin, marked respiratory Delirium, life-threatening cardiovascular
• 72
depression. Treatment: symptom management collapse.
• 73 • ( pa :J~c;;:ic;;:t rpc;;:nir~tinn t RP) •

6
lock
s
Suspend
0
End Block
Item: 66 of 73 ~. I • M k <:] t> al ~· ~
QIO: 1130 .l. ar Previous Next lab 'lifllues Notes Calculator


53
A 26-year-old woman presents to her primary care physician with complaints of fatigue and insomnia for the
54 past 8 months. She also reports difficu lty concentrating, irritability, and a continued sense of worry. She
55 performs wel l at her job as a lawye r, where she was recently promoted; but she states that she often " lies
awa ke at night worrying that she may be fired ." She no longe r goes out with f riends or family because she is
56
worried that she is not welco med. She denies heart palpitations or changes in weight. Her medical histo ry is
57 un remarkab le. He r vital signs are within no rma l limits, and physical exa mination shows no focal findings. She does
58 not smo ke but occasionally uses alcohol. She went through an uncomplicated divorce 1 year ago .
59
60 Which of the fo llowing is this patient's most like ly diagnosis?
61 :
62 A. Adjustment disorde r
63 B. General ized anxiety diso rder
64
C. Panic diso rder
65
• 66 D. Posttraumatic stress diso rder
• 67 E. Socia l anx iety disorder
• 68
• 69
• 70
• 71
• 72
• 73 ~

6
lock
s
Suspend
0
End Block
Item: 66 of 73 ~. I • M k <:] t> al ~· ~
QIO: 1130 .l. ar Previous Next lab 'lifllues Notes Calculator


53 The correct answer is B. 70°/o chose this.
54 Patients with genera lized anxiety disorder (GAD) have excessive anxiety and worry about events or activit ies
55
re lated to health, fami ly, relationships, work, and finances. Fo r the diagnosis to be made, the anxiety or wo rry
must be present for mo re days than not over (at least) a 6-month period. Three or mo re of the fol lowing
56 symptoms must also be present: fatigability, irritability, inability to concentrate, sleep distu rbances (d ifficulty
57 fal ling or staying asleep, restless sleep, etc.), muscle tension, or rest lessness. This patient comp lains of fatigue,
58 inso mnia, difficulty concentrating, and irritability, in addition to generalized anxiety. Affected patients respond
well to behavioral the rapy and treatment with antidepressants and buspirone. Benzod iazepines can be used for
59
short-term management, but are not reco mmended fo r long-term manage ment because of their add ictive
60 potential. GAD is often a chronic disease, but it can be contro lled. Comorbid depression is common in GAD.
61 Generalized anxiety disorder Buspirone Insomnia Anxiety disorder Benzodiazepine Comorbidity Antidepressant Anxiety Sleep disorder Chronic condition

62 Major depressive disorder Irritability Fatigue (medical) Depression (mood) Muscle Glutamate decarboxylase Muscle tone

63 A is not correct. 13% chose this.


64 Adjust ment disorde r manifests with depression and anxiety within 3 months of a psychosocial stressor (eg,
65 divo rce or job loss) . I mpaired occupational or social function ing must ensue and cause marked distress.
66 Moreover, once the stressor or its consequences have te rminated, the symptoms do not persist for longer than
6 months .
• 67
Adjustment disorder Stressor Major depressive disorder Depression (mood) Anxiety Divorce
• 68
C is not correct. 3°/o chose this .
• 69
Panic diso rder is characterized by recu rrent episodes of panic atacks followed by pe rsistent concerns of further
• 70
panic attacks or maladaptive changes re lated to pan ic attacks lasting for at least 1 month . Physica l sympto ms of
• 71 panic attacks include heart pa lpitations, abdomina l distress, paresthesias, nausea, intense fea r of dying, and
• 72 chest pa in. This patient does not describe such acute periods of distress .
Panic disorder Paresthesia Panic attack Nausea Palpitations Chest pain
• 73 ~

6
lock
s
Suspend
0
End Block
Item: 66 of 73 ~. I • M k <:] t> al ~· ~
QIO: 1130 .l. ar Previous Next lab 'lifllues Notes Calculator
p p I I I p I I
53
Panic disorder Paresthesia Panic attack Nausea Palpitations Chest pain
54
55
D is not correct. 3°/o chose this.
Posttraumatic stress disorder (PTSD) can develop after a life-threatening or previous traumatic event, with the
56
disturbance lasting more than 1 month at least 1 month after the event. Patients complain of flashbacks,
57 nightmares, fear, and helplessness. The patient wi ll also describe avoiding specific stimu li that are associated
58 with the event. This patient does not have any of the of the features of PTSD; therefore, th is is not the
59 appropriate answer.
Posttraumatic stress disorder Flashback (psychology) Stress (psychological) Stress (biology) Psychological trauma
60
61 E is not correct. 11% chose this.
62 Patients with socia l anxiety disorder are marked ly anxious and nervous in social situations. These patients fear
that they will act in a humiliating or embarassing way. Although this patient no longer enjoys socia l situations, it
63
is due to excessive concerns regarding interpersona l relationsh ips, wh ich is more consistent with generalized
64 anxiety disorder.
65 Social anxiety disorder Generalized anxiety disorder Anxiety disorder Social anxiety Anxiety

66
• 67
Bottom Line:
• 68
General ized anxiety diso rder occu rs when a person has excessive anxiety and worry about daily activit ies .
• 69 Symptoms of anxiety such as restlessness, fatigue, insomn ia, and ir ritability must be present for at least 6
• 70 months before the diagnosis can be made. Pharmacologica l treatment options include selective serotonin
• 71 reuptake inhibitors or buspirone .
Generalized anxiety disorder Buspirone Serotonin Insomnia Anxiety disorder Selective serotonin reuptake inhibitor Reuptake Anxiety Fatigue (medical)
• 72 Pharmacotherapy Irritability Boredom Pharmacology
• 73 ~

6
lock
s
Suspend
0
End Block
Ite m: 66 of 7 3 ~ 1 • M k -<:J 1>- Jil ~· !:';-~
QIO: 1130 ..L ar Previous Next La b fli!llues Not es Calculator
A A
53 FA17 p 533.3
54
Generalized anxiety Anxiety lasting> 6 months 11nrelatcd to a specific person, situation, or e,·ent. Associated with
55 disorder restlessness, irritability, sleep disturbance, fatigue, muscle tension, difficulty concentrating.
56 Treatment: C BT, SSRis, SNRls are first line. Buspirone, TC s, benzodiazepines are second line.
57 Adjustment disorder-emotional symptoms (anxiety, depression) that occur" ithin 3 months of
58 an identifiable psychosocial stressor (eg, di,orce. illness) lasting< 6 months once the stressor has
59 ended. If stressor lasts> 6 months and causes continual impairment. it is GAD. Treatment: CBT,
SSRis.
60
61
FA17 p 533.1
62
63 Panic disorder Defined by recurrent panic attacks (periods P\"ICS.
of intense fear and discomfort peaking in Di<~gno~is requires attack followed by I month
64
10 minutes with at least 4 of the following): (or more) of 1 (or more) of the following:
65
Palpitations, Paresthesias, dePersonalization or Persistent concern of additional attacks
66 derealization, Abdominal distress or ~ausea, Worrying about consequences of attack
• 67 Intense fear of dying, I ntcnse fe<1r of losing Behavioral change related to attacks
• 68 control or "going cra;(.y," light-headed ness, Symptoms a re the systemic manifestations of
C hest pain, C hills, C hoking, Sweating, fea r.
• 69
Shaking, Shortness of breath. Strong genetic
• 70
component. t risk of suicide. Treatment:
• 71 CBT, SSRls, and venlafaxine <Ire first line.
• 72 Benzodiazepincs occasionally used in acute
• 73 setting.

a
Lock
s
Suspend
8
End Bl ock
Item: 67 of 73 ~ 1 • M k -<:J 1>- Jil ~· !:';-~
QIO: 4836 ..L ar Pre v ious Next Lab fli!ltues Notes Calcula t o r

IAA]
A A
53
A 23-year-old man is brought to the emergency department because his friends heard him say that he was
54 ta lking to George Washington and was being sent to spy on the British army. On admission his pu lse is
55 80/min and his blood pressure is 120/80 mm Hg. On examination he appears anxious and ag itated. His
conjunctiva are clear and non injected, but his pupils are markedly dilated .
56
57
What is the most likely cause of his symptoms?
58
59 :
A . Alcohol
60
61 B. Cocaine
62 C. Heroin
63
D. Lyserg ic acid diethylamide
64
65
E. Marijuana

66
• 67
• 68
• 69
. 70
. 71
• 72
. 73

a
Lock
s
Suspend
8
End Bl ock
Item: 67 of 73 ~. I • M k <:] t> al ~· ~
QIO: 4836 .l. ar Previous Next lab 'lifllues Notes Calculator


53
The correct answer is D. 57°/o chose this.
54
This patient is experiencing hal lucinations, delusions, and dilated pupils, but very few observable behavioral
55
changes. These symptoms are consistent with lysergic acid diethylamide (LSD) abuse. LSD is a ha llucinogenic
56 drug that can cause ma rked anxiety or depression, nausea, weakness, and paresthesias. No antidote is
57 currently avai lable for the t reatment of LSD toxicity.
lysergic acid diethylamide Nausea Hallucinogen Anxiety Mydriasis Paresthesia Hallucination Major depressive disorder Antidote lysergic acid Depression (mood)
58
Toxicity Pupillary response Delusion
59
60 A is not correct. 5°/o chose this.
61 Alcohol abuse is characterized by a general disinhibition, slurred speech, and ataxia. It does not usually cause
patients to hallucinate or become de lusional. Benzodiazepines, such as lorazepam, can be used to prevent
62
de lir ium tremens and other signs of alcohol withdrawa l.
63 lorazepam Delirium tremens Alcohol withdrawal syndrome Ataxia Benzodiazepine Alcohol abuse Disinhibition Alcoholic beverage Hallucination Delirium

64 Dysarthria Alcohol Relaxed pronunciation


65
B is not correct. 22% chose this.
66
Cocaine can cause many of the symptoms th is patient is experiencing . However, patients with recent cocaine
67 use are usua lly hypertensive and tachycardic because of its stimulant effects. The oragonist dexmedetomidine
• 68 (Precedex) may be used to treat coca ine ove rdose .
Dexmedetomidine Cocaine Tachycardia Stimulant Cocaine intoxication Drug overdose
• 69
• 70 C is not correct. 10% chose this .
• 71 Heroin is an opioid. Heroin intoxication can manifest with pinpoint pupils, decreased levels of consciousness, low
• 72 heart and respiratory rate, and someti mes cyanosis. Treatment is t he opioid antagon ist na loxone .
Naloxone Cyanosis Opioid Opioid antagonist Opioid overdose Heroin Receptor antagonist Respiratory rate Miosis Alcohol intoxication Substance intoxication
• 73 ~

6
lock
s
Suspend
0
End Block
Item: 67 of 73 ~. I • M k <:] t> al ~· ~
QIO: 4836 .l. ar Previous Next lab 'lifllues Notes Calculator


53
B is not correct. 22% chose this.
54 Cocaine can cause many of the symptoms th is patient is experiencing . However, patients with recent cocaine
55 use are usua lly hypertensive and tachycardic because of its stimu lant effects. The oragonist dexmedetomidine
56 (Precedex) may be used to treat coca ine ove rdose.
Dexmedetomidine Cocaine Tachycardia Stimulant Cocaine intoxication Drug overdose
57
58 C is not correct. 10% chose this.
59 Heroin is an opioid. Heroin intoxication can manifest with pinpoint pupils, decreased levels of consciousness, low
heart and respiratory rate, and sometimes cyanosis. Treatment is the opioid antagon ist na loxone.
60
Naloxone Cyanosis Opioid Opioid antagonist Opioid overdose Heroin Receptor antagonist Respiratory rate Miosis Alcohol intoxication Substance intoxication
61
62
E is not correct. 6°/o chose this.
Marijuana can cause many of the symptoms this patient is experiencing. However, mar ij uana intoxication
63
typical ly manifests with other sympto ms, including increased appetite, dry mouth, and conjunctiva l injection,
64 which are not seen in th is patient. Additional ly, ha llucinations usual ly on ly ar ise after very high doses of
65 marij uana. Current recommendations suggest supportive therapy for patients presenting with mar ij uana
66
intoxication (dar k room, patient assurance, decreased stimu lation). Occasional ly, benzod iazepines can be
beneficial in the reduction of symptoms, but general ly are not used.
67 Cannabis {drug) Benzodiazepine Marijuana Xerostomia Conjunctiva Hallucination Effects of cannabis Conjunctivitis Substance intoxication Alcohol intoxication
• 68
• 69
• 70 Bottom Line:
• 71 LSD causes ha llucinations, anxiety or depression, delusions, nausea, weakness, paresthesias, and di lated
pupils, but very few observable behavioral changes .
• 72
lysergic acid diethylamide Paresthesia Nausea Anxiety Mydriasis Hallucination Major depressive disorder Depression (mood) Pupillary response Delusion
• 73 ~

6
lock
s
Suspend
0
End Block
Item: 67 of 73 ~ 1 • M k -<:J 1>- Jil ~· !:';-~
QIO: 4836 ..L ar Pre v ious Next Lab fli!ltues Notes Calcula t o r
A A
53
FA17p540.1
54
Psychoactive drug intoxication and withdrawal
55 DRUG INTOXICATION WITHDRAWAL
56 Depressants
57 J\onspecific: anxiet), tremor, seizures,
'onspecific: mood elevation, l am.iety,
58
. .
sedation, behavioral disinhibition, respiratory 1nsomma.
59 depression.
60 Alcohol Emotionallabilit}. slurred speech, ataxia, Time from last drink:
61 coma, blackouts. Serum y-glutam} ltransferase 3-36 hr: minor symptoms similar to other
(GGT)-sensitivc indicator of alcohol use. depressants
62
ST value is twice ALT value. 6-48 hr: withdrawal seizures
63
12-48 hr: alcoholic hallucinosis (usually visual)
64 48-96 hr: delirium tremens (DTs) in 5% of
65 cases
66 Treatment: benzodiazepines.
67 Opiolds Euphoria, respiratory and C S depression, Sweating, dilated pupils, piloerection ("cold
• 68 l gag reAex, pupillary constriction (pinpoint turkey"), fever, rhinorrhea, yawning, nausea,
pupils), seizures (overdose). t-. lost common stomach cramps, diarrhea ("Au-like" symptoms).
• 69
cause of drug overdose death. Treatment: Treatment: long-term support, methadone,
• 70
naloxone. buprenorphine.
• 71
Barbiturates Low safety margin, marked respiratory Delirium, life-threatening cardiovascular
• 72 depression. Treatment: symptom management collapse.
• 73 (eg, assist respiration, t BP).

a
Lock
s
Suspend
8
End Bl ock
Item: 68 of 73 ~ 1 • M k -<:J 1>- Jil ~· !:';-~
QIO: 1119 ..L ar Pre v ious Next Labfli!llues Notes Calcula t o r

IAA]
A A
53
A 24-year-old woman is brought to the emergency department by ambu lance after she is found unresponsive
54 on the street. It is not known how long she was lying on the street. Her temperature is 37 .2°C {99.0°F),
55 heart rate 40/min, blood pressure 110/70 mm Hg, and respiratory rate 10/min. Physica l examination reveals
constricted pupi ls.
56
57
Administration of which of the following drugs would be most appropriate?
58
59 :
A . Chlordiazepoxide
60
61 B. Flumazenil
62 C. Fomepizole
63
D. N-Acetylcysteine
64
65
E. Na loxone

66 F. Naltrexone
67 G. Phenobarbita l
• 68
• 69
. 70
. 71
• 72
. 73

a
Lock
s
Suspend
8
End Bl ock
Item: 68 of 73 ~. , . M k <:] t> al ~· ~
QIO: 1119 .l. ar Previous Next Lab 'lifllues Notes Calculator


53 The correct answer is E. 71 °/o chose this.
54 This patient has signs ind icating opioid overdose: She is comatose with miosis, bradyca rdia, and mi ld
55 respiratory depression . An additional sign of opioid overdose is decreased bowel sounds. Na loxone, an opioid 1-J-
receptor antagon ist given intravenously, displaces the opioids from their receptors and qu ickly reverses the
56
effects of the overdose. However, it may also induce opioid withd rawal symptoms.
57 Naloxone Miosis Opioid overdose Hypoventilation Bradycardia Opioid Intravenous therapy Drug withdrawal Receptor antagonist Opioid withdrawal

58 Major depressive disorder Depression (mood) Drug overdose Receptor (biochemistry) Antagonist Gastrointestinal tract

59
A is not correct. 3°/o chose this.
60
Ch lordiazepoxide is a long-acting benzodiazepine used in the management of alcohol withdrawal. Though
61 patients with alcohol intoxication can become unresponsive, alcohol tends to cause pupi llary dilation, not
62 constr iction as seen with t his patient. Additional ly, there is no mention of the smel l of alcohol on t he patient's
63
breath or clothing, which can be a clue to alcohol intoxication in an un responsive patient.
Benzodiazepine Chlordiazepoxide Alcohol withdrawal syndrome Alcohol intoxication Mydriasis Alcoholic beverage Alcohol Pupillary response
64
65
B is not correct. 10% chose this.
66
Flu mazenil is an antagon ist of benzod iazepine receptors and is used to reverse benzodiazepine intoxication.
Benzodiazepine overdose does not cause pupil lary changes, and only very seve re cases lead to respiratory
67 depression and hypotension . Patients with benzod iazepine overdose typical ly present with signs and symptoms
68 of CNS depression such as somno lence, ataxia, intoxication and impaired motor function.
Benzodiazepine Flumazenil Benzodiazepine overdose Hypoventilation Somnolence Hypotension Ataxia Receptor antagonist GABAA receptor
• 69
• 70 Central nervous system depression Central nervous system Major depressive disorder Depression (mood) Receptor (biochemistry) Antagonist Drug overdose

• 71 Alcohol intoxication

• 72 C is not correct. 3°/o chose this .


• 73 ~ Fomepizole is an inhibitor of alcohol dehvd roqenase, and it is used to prevent the conversion of ethylene qlycol ~

6
lock
s
Suspend
0
End Block
Item: 68 of 73 ~. , . M k <:] t> al ~· ~
QIO: 1119 .l. ar Previous Next Lab 'lifllues Notes Calculator


53 C is not correct. 3°/o chose this.
54 Fomepizole is an inhibitor of alcohol dehyd rogenase, and it is used to prevent the conversion of ethylene glycol
55 and methanol to the toxic substances oxal ic acid and formic acid, respectively . Thus, it is ma inly used as an
antidote for methanol poisoning, wh ich may lead to bl indness, or ethylene glycol poison ing, which may lead to
56
kidney failure. A common scenario is the ingestion of antifreeze; however, the re is no evidence that this patient
57 ingested these substances. An important piece of supporting evidence is an elevated osmola l gap (the difference
58 between measu red and calculated plasma osmola lity). Ethylene glycol ingestion also causes wide an ion gap
59
acidosis.
Oxalic acid Fomepizole Formic acid Methanol Ethylene glycol Ethylene glycol poisoning Alcohol dehydrogenase Anion gap Antifreeze Alcohol Osmolality Anion
60
Antidote Osmol gap Acidosis Kidney Ethylene Blood plasma Dial Toxicity Alcoholic beverage Plasma osmolality
61
62 D is not correct. 3°/o chose this.
63 N-Acetylcysteine is used in cases of acetaminophen poison ing. An overdose of acetaminophen causes nausea
and vomiting in the ea r ly stages and right upper quadrant pain and elevated liver transaminases in later stages.
64
Pinpoint pupils and bradyca rdia do not occur.
65 Bradycardia Paracetamol Transaminase Acetylcysteine Nausea liver Quadrant (abdomen) Vomiting Miosis Drug overdose

66
F is not correct. 9°/o chose this.
67
Naltrexone, like na loxone, is an opioid j.J-receptor antagon ist; however, it is not indicated for the reversa l of
68 acute opioid overdose. Na ltrexone is used as a rehabilitation drug in the long-term t reatment of alcohol and
• 69 opioid dependence. Since na ltrexone (g iven ora lly or intramuscu larly) is slower acting, naloxone (given
• 70
intravenously) is the preferred opioid antagonist for opiod toxicity . An easy way to remember this is to associate
naloxone's shorter name (8 letters) with short-term use, whereas naltrexone with the longer name (10 letters)
• 71 is used for long-ter m ind ications.
• 72 Naloxone Naltrexone Opioid use disorder Opioid antagonist Opioid overdose Opioid Intravenous therapy Receptor antagonist Intramuscular injection

• 73 ~
Alcoholic beverage Alcohol Antagonist Toxicity Drug overdose

6
lock
s
Suspend
0
End Block
Item: 68 of 73 ~. , . M k <:] t> al ~· ~
QIO: 1119 .l. ar Previous Next Lab 'lifllues Notes Calculator


53 Bradycardia Paracetamol Transaminase Acetylcysteine Nausea liver Quadrant (abdomen) Vomiting Miosis Drug overdose

54
F is not correct. 9°/o chose this.
55 Naltrexone, like na loxone, is an opioid 1-1-receptor antagon ist; however, it is not indicated fo r the reversa l of
56 acute opioid ove rdose. Na ltrexone is used as a rehabilitation drug in the long-term t reatment of alcohol and
57 opioid dependence . Since na ltrexone (g iven orally or intramuscu larly) is slower acting, naloxone (given
intravenously) is the preferred opioid antagonist for opiod toxicity . An easy way to remember this is to associate
58
naloxone's shorter name (8 letters) with short-term use, whereas naltrexone with the longer name (10 letters)
59 is used for long-ter m ind ications.
60 Naloxone Naltrexone Opioid use disorder Opioid antagonist Opioid overdose Opioid Intravenous therapy Receptor antagonist Intramuscular injection

61 Alcoholic beverage Alcohol Antagonist Toxicity Drug overdose

62 G is not correct. 1 °/o chose this.


63 Phenobar bita l is a long-acting barbiturate usefu l in patients with seizu re disorders. Overdose of th is agent would
64 cause severe respiratory depression .
Barbiturate Phenobarbital Hypoventilation Major depressive disorder Depression (mood) Epileptic seizure
65
66
67 Bottom Line:
68 Naloxone is an intravenous competitive 1-1-receptor antagonist that is used to treat opioid overdose . It rapidly
• 69 reverses the sympto ms of opioid intoxication ( including miosis, respiratory depression, and bradycardia); th is
• 70 can often send peop le into withdrawal .
Naloxone Miosis Hypoventilation Bradycardia Opioid overdose Opioid Toxidrome Intravenous therapy Receptor antagonist Major depressive disorder
• 71
Depression (mood) Drug overdose
• 72
• 73 ~

6
lock
s
Suspend
0
End Block
Ite m: 68 of 7 3 ~ 1 • M k -<:J 1>- Jil ~· !:';-~
QIO: 1119 ..L ar Previous Next La b fli!llues Not es Calculator
A A
53
54 FA17p540.1

55
Psychoactive drug intoxication and withdrawal
DRUG INTOXICATION WITHDRAWAL
56
Depressants
57
'onspecific: mood elevation, l arn.icty, l\onspecific: anxiet~, tremor, seizures,
58 . .
sedation, behavioral disinhibition, respiratory rnsomnra.
59
depression.
60
Alcohol Emotionallabilit), slurred speech, ataxia, Time from last drink:
61 coma, blackouts. Serum y-glutam} !transferase 3- 36 hr: minor symptoms similar to other
62 (GGT)-sensitivc indicator of alcohol use. depressants
63 AST value is twice LT value. 6-48 hr: withdrawal seizures
64
12-48 hr: alcoholic hallucinosis (usually visual)
48-96 hr: delirium tremens (DTs) in 5% of
65
cases
66 Treatment: benzodiazepines.
67 Opioids Euphoria, respiratory and C S depression, Sweating, dilated pupils, piloerection ("cold
68 l gag reAex, pupillar>' constriction (pinpoint turkey"), fever, rhinorrhea, yawn ing, nausea,
• 69 pupils), seizures (overdose). ~lost common stomach cramps, diarrhea ("Au-like" symptoms).
• 70 cause of drug overdose death. Treatment: Treatment: long-term support, methadone,
naloxone. buprenorphine.
• 71
• 72
Barbiturates Low safety margin, marked respirator) Delirium, life-threatening cardiovascular
depression. Treatment: symptom management collapse.
• 73

a
Lock
s
Suspend
8
End Block
Item: 68 of 73 ~. , . M k <:] t> al ~· ~
QIO: 1119 .l. ar Previous Next Lab 'lifllues Notes Calculator

• •
53
FA17 p 540.1
54
Psychoactive drug intoxication and withdrawal
55
DRUG ltiTOXICATION WITHDRAWAL
56
Depressants
57
onspecific: mood elevation, l anxiety, onspecific: anxiety, tremor, seizures,
58 . .
sedation, behavioral disinhibition, respiratory lllSOmnaa.
59 depression.
60 Alcohol Emotional labil ity, slurred speech, ataxia, Time from last drink:
61 coma, blackouts. Serum y-glutamyltransferase 3- 36 hr: minor symptoms similar to other
62 (GGT)- sensitive indicator of alcohol usc. depressants
63 AST value is twice ALT value. 6- 48 hr: withdrawal seizures
12- 48 hr: alcoholic hallucinosis (usually visual)
64
48-96 hr: delirium tremens (DTs) in 5% of
65 cases
66 Treatment: benzodiazepines.
67 Opioids Euphoria, respiratory and CNS depression, Sweating, dilated pupils, piloerection ("cold
68 l gag reAex, pupillary constriction (pinpoint turkey"), fever, rh inorrhea, yawning, nausea,
• 69 pupils), seizures (overdose). Most common stomach cramps, diarrhea {"Au-like" symptoms).
• 70
cause of drug overdose death. Treatment: Treatment: long-term support, methadone,
naloxone. buprenorphine.
• 71
Barbiturates Low safety margin, marked respiratory Delirium, life-threatening cardiovascular
• 72
depression. Treatment: symptom management collapse.
• 73 • ( pa :J~c;;:ic;;:t rpc;;:nir~tinn t RP) •

6
lock
s
Suspend
0
End Block
Item: 68 of 7 3 ~ 1 • M k -<:J 1>- Jil ~· !:';-~
QIO: 1119 ..L ar Pre v ious Next Labfli!llues Notes Calcula t o r


53
54
FA17p239.1
55
Specific toxicity TOXIN TREATMENT
56 treatments Acetaminophen '-acetylcysteine (replenishes glutath ione)
57
AChE inhibitors, organophosphates Atropine> pralidoxime
58
Antimuscarinic, anticholinergic agents Physostigmine, control hn>erthermia
59
Aisenic Dimercaprol, succimer
60
Benzodiazepines Flumazenil
61
~-blockers Atropine, glucagon
62
63 Carbon monoxide 100% 0 2, hyperbaric 0 2

64 Copper Penicillamine, trientine (Copper pen ny)


65 Cyanide 'ilrile + thiosulfate, hydroxocobalamin
66 Digitalis (digoxin) Anti-dig Fab fragmen ts
67 Heparin Protamine sulfate
68 Iron Defc roxamine, deferasirox, deferiprone
• 69 Lead EDTA, dimercaprol, succirner, penicillami ne
• 70 l\Iercury Dimercaprol, succim er
• 71
Methanol, ethylene glycol (antifree..:e) Fomepizole > ethanol, dialysis
• 72
l\ lethemoglobin 1\lethylene blue, ,·itamin C
• 73

a
Lock
s
Suspend
8
End Bl ock
Item: 69 of 73 ~ 1 • M k -<:J 1>- Jil ~· !:';-~
QIO: 1137 ..L ar Pre v ious Next Labfli!llues Notes Calcula t o r
A A
53
A 74-year-old nursing home resident is brought to the emergency department after he became combative
54 and was screaming that Lilliputians were after him. The emergency department staff is unable to obta in a
55 history or physical examination because of his agitation. The triage nurse is able to obtain his vital signs; his
56
temperature is 400C (1040F) and blood pressure is 90/50 mm Hg. His family reports that he was fine earlier in the
day except for a cough. He has no previous psychiatric history. By the time the psychiatrist arrives, the patient is
57 somnolent and somewhat confused. He is not oriented to place or time.
58
59 Which of the following is the most likely diagnosis?
60
:
61 A. Brief psychotic disorder
62
B. Delirium
63
64 C. Delusional disorder
65 D. Dementia
66 E. Sch izophrenia
67
68
• 69
. 70
. 71
• 72
. 73

a
Lock
s
Suspend
8
End Bl ock
Item: 69 of 73 ~. , . M k <:] t> al ~· ~
QIO: 1137 .l. ar Previous Next Lab 'lifllues Notes Calculator


53
The correct answer is B. 72°/o chose this.
54
Delirium is the most common psychiatric diagnosis on med ical and surgical floors, particula r ly among elderly
55 patients . The diagnosis of de lir ium requires th ree conditions : (1) an acute and usua lly fluctuating (hou rs to
56 days) disturbance of consciousness (reduced ability to focus, susta in, or shift attention); (2) a change in
57
cognition (memory distu rbance, ha llucination, or othe r perceptua l disturbance); and (3) evidence that this
episode is the direct physio log ic consequence of a gene ral medica l condition. The most common causes of
58 de lir um include drugs such as benzodiazepines and alcohol, infection, metabo lic disturbances (eg,
59 hyponatremia, hypoxemia), organ fai lure, t rauma (eg, burns), or a neu rologic disorder (eg, epilepsy,
60 mening it is). This patient likely has de lirium from an infection, evidenced by his cough and fever.
Hyponatremia Epilepsy Hypoxemia Meningitis Delirium Hallucination Benzodiazepine Neurological disorder Cognition Cough Neurology Fever Alcohol
61
Alcoholic beverage Infection Metabolism Psychiatry
62
63 A is not correct. 16% chose this.
64 Brief psychotic disorder is characterized by the presence of at least one of the fol lowing symptoms:
hallucinations, delusions, diso rganized speech, or grossly disorgan ized behavior. The symptoms last at least 1
65
day but less than 1 month, and the patient returns to full functiona lity. Importantly, the disease is not
66 attributab le to a genera l medica l cond it ion or other psychiatric disorde r. The physica l signs in this patient point
67 to a med ical etiology.
Brief psychotic disorder Psychosis Mental disorder Hallucination Etiology Thought disorder
68
69 C is not correct. 5°/o chose this.
• 70 Delusiona l disorde r is diagnosed when a pe rson has, for at least 1 month, nonbizarre delusions ( ie, plausible
• 71 situations in reality), which are not attributab le to another psychiatric disorder such as schizophrenia .
Importantly, delusional disorder does not markedly impai r the person 's life; its ramifications are limited to the
• 72
de lusiona l content .
• 73 ~
Delusional disorder Schizophrenia Mental disorder

6
lock
s
Suspend
0
End Block
Item: 69 of 73 ~. , . M k <:] t> al ~· ~
QIO: 1137 .l. ar Previous Next Lab 'lifllues Notes Calculator


53
D is not correct. 4°/o chose this.
54
Dementia is a chron ic cond it ion characterized by cognitive decline (eg, memory impairment) and an intact level
55 of consciousness. The persistence and stability of the cognitive impairment distinguish dementia f rom the
56 fluctuating and unstable course seen in del irium.
Dementia Cognitive deficit Altered level of consciousness Delirium Chronic condition
57
58 E is not correct. 3°/o chose this.
59 Patients with schizoph renia must have had symptoms for more than 6 months. Sch izophrenia is a chron ic
60 psychiatric condition . It affects 1% of the popu lation and usua lly begins before age 25 years. Diagnosis requ ires
that at least two of the fol lowing symptoms are present during a 1-month period : delusions, ha llucinations,
61
disorganized speech, grossly disorganized or cataton ic behavior, and negative sympt oms (eg, flat affect, lack of
62 motivation, poverty of speech) . Moreover, signs of the disturbance must be present for at least 6 months, such
63 as one of t he above symptoms in an attenuated form (eg, mag ical thinking or simp ly some negative
symptoms) .
64
Schizophrenia Magical thinking Alogia Catatonia Reduced affect display Negative symptoms Hallucination Mental disorder Psychiatry
65
66
67 Bottom Line:
68 Delirium is characterized as altered mental status in t he setting of waxing and wan ing consciousness . Delirium
69
may be caused by infection, metabo lic disturbances, electro lyte abnorma lities, hypoperfusion, drug
intoxication, alcoho l intoxication or withdrawal, or adverse effects due to medication .
• 70 Electrolyte Delirium Altered level of consciousness Shock (circulatory) Alcoholic beverage Alcohol intoxication Alcohol Water-electrolyte imbalance
• 71 Substance intoxication Drug overdose Pharmaceutical drug Mental status examination Metabolism

• 72
• 73 ~

6
lock
s
Suspend
0
End Block
Item: 69 of 7 3 ~ 1 • M k -<:J 1>- Jil ~· !:';-~
QIO: 1137 ..L ar Pre v ious Next Labfli!llues Notes Calcula t o r
A A
53 FA17 p 528.4
54
Delirium "\Vaxing and waning" IC\'CI of consciousness Delirium = changes in sensorium.
55 with acute onset; rapid l in alieni ion span and Ia) be caused by medications (eg,
56 level of arousal. Characterized by disorganized anticholinergics), especially in the elderly.
57 thinking, hallucinations (often visual), Reversible.
58
illusions, misperceptiom, disturbance in sleep-
wake c~cle, cogniti\e dysfunction.
59
Usually 2° to other illness (eg, C t
60 disease, infection, trauma, substance
61 abuse/withdrawa I, mctabolic/clccl rolyte
62 disturbances, hemorrhage, urinary/fecal
63 retention).
Most common presentation of altered mental
64
status in inpatient setting, especially in the
65
intensive care unit and with prolonged hospital
66 stays. Common ly, di ffuse slowing F. EC.
67 Treatment is aimed at identifying and
68 addressing underlying condition.
Anti psychotics may be used acutely as needed.
69
• 70
• 71 FA17p529.1

• 72 Dementia l in intellectual function without affecting "Demcmlia" is characterized by memory loss.


• 73 le\·el of consciousness. Characterized bv• Usuallv• irreversible.

a
Lock
s
Suspend
8
End Bl ock
Item: 69 of 7 3 ~ 1 • Ma rk -<:J 1>- Jil ~· !:';-~
QIO: 1137 ..L Pre v ious Next Labfli!llues Notes Calcula t o r

Treatment is aimed at iden lifying and •


53
54
addressing underlying condition.
ntipsychotics may be used acutely as needed.
55
56
57 FA17 p 529.1

58 Dementia l in intellectual function without affecting "Dememtia" is characterized by memory loss.


59 b ·el of consciousness. Characterized by Usuallv• irreversible.
memory deficits, apraxia, aphasia, agnosia, In elderly patients, depression and
60
loss of abstract thought, behavioral/personal it} hypothyroidism may present like dementia
61
changes, impaired judgment. A patient (pseudodementia). Screen for depression,
62 with dementia can develop delirium (eg, exclude neurosyphilis with RPR if high clinical
63 p<ltient with Alzheimer disease who develops suspicion, and measure TSH, B12 levels.
64 pneumonia is at t risk for delirium).
lrre,·ersible causes: Alzheimer disease, Lewy
65
body dementia, Huntington disease, Pick
66
disease, cerebral infarct, Wilson disease,
67 Creutzfeldt-Jakob disease, chronic substance
68 abuse (due to neurotoxicity of drngs), mv.
69 Reversible causes: hypothyroidism, depression,
• 70
vitamin deficiency (81, 8 3, 8 12), normal
pressure hydrocephalus, neurosyphilis.
• 71
t incidence with age. EEG usually normaL
• 72
• 73

a
Lock
s
Suspend
8
End Bl ock
Item: 70 of 73 ~ 1 • M k -<:J 1>- Jil ~· !:';-~
QIO: 420 3 ..L ar Pre v ious Next Lab fli!ltues Not es Calcula t o r
A A
53
A 36-year-old construction worker presents to his primary care physician complaining of recurrent ulcers on
54 his mouth, buttocks, and feet for the past 3 weeks. There are no genital ulcers or signs of uveitis. The
55 patient states that he used to take chlorpromazine, but his psych iatrist switched him to a different class of
56
medication. In addition, he says he is supposed to get weekly blood tests but that he has not been able to do that
due to his work schedu le. Laboratory stud ies show:
57
WBC count : 1000/ mm3
58
Segments: 60%
59 Bands: 4%
60 Eosinophils: 2%
61
Basophils: 0.5%
Lymphocytes : 28%
62 Monocytes: 5%
63 Hematocrit: 48%
64 Hemoglobin: 15 g/dL
Platelet count: 250,000/mm 3
65
66
67 Without treatment of his psych iatric il lness, the patient would most likely experience which of the following?
68 :
69 A . Anterograde amnesia and confabulation
. 70 B. Auditory hallucinations
. 71
C. Fluctuations in consciousness
• 72
. 73 D . Periods of manic behavior fol lowed by depression

a
Lock
s
Suspend
8
End Bl ock
Ite m: 70 o f 73 ~ 1 • M k -<:J 1>- Jil ~· !:';-~
QIO: 4203 ..L ar Previous Next La b fli!ltues Not es Ca lculator
•• • •• r •• -- • - ·- n--r -- - - •• - •• - • • • l:t: - - -
53
patient states that he used to t ake ch lorpromazine, but his psych iatrist switched him to a different class of
54 medication . In addition, he says he is supposed to get weekly blood tests but that he has not been able to do that
55 due to his work schedu le. Laboratory stud ies show:
56 WBC count: 1000/mm3
57 Segments: 60%
Bands: 4%
58
Eosinophils: 2%
59 Basoph ils: 0.5%
60 Lymphocytes: 28%
Monocytes: 5%
61
Hematocrit : 48%
62 Hemog lobin: 15 g/d L
63 Platelet count: 250,000/ mm 3
64
65
Without treatment of his psych iat ric illness, the patient would most likely experience which of the fol lowing?
66
:
67
A. Anterograde amnesia and confabulation
68
B. Aud itory hal lucinat ions
69
. 70 C. Fluctuations in consciousness
. 71 D. Periods of manic behavior fol lowed by depression
• 72
E. Recurrent periods of intense fear
. 73

a
Lock
s
Suspend
8
End Block
Item: 70 of 73 ~. I • M k <:] t> al ~· ~
QIO: 4203 .l. ar Previous Next lab 'lifllues Notes Calculator


53 The correct answer is B. 70°/o chose this.
54 The patient is most likely suffering from agranu locytosis secondary to clozapine use. Clozapine is an atypical
55 antipsychotic used to t reat sch izophrenia. It is less likely to cause the tardive dyskinesia associated with typical
antipsychotics, such as chlor promazine, since it does not block dopamine 2 (D2) receptors as strongly. Patients
56
with sch izophrenia can present with positive symptoms (eg, delusions, aud itory ha llucinations, disorgan ized
57 thought, and catatonic behavior ) or negative symptoms (eg, flat affect, social withd rawal, lack of motivation,
58 and lack of speech or thought) . Typical antipsychotics are generally used for positive symptoms, whereas
59 atypical antipsychotics are used for both positive and negative symptoms.
Atypical antipsychotic Chlorpromazine Tardive dyskinesia Agranulocytosis Clozapine Dopamine Antipsychotic Schizophrenia Typical antipsychotic Dyskinesia
60
Catatonia Reduced affect display Auditory hallucination Hallucination Receptor (biochemistry)
61
62 A is not correct. 4°/o chose this.
63 Korsakoff psychosis is a late compl ication of alcohol ism related to thiamine deficiency and mammi llary body
hemorr hage. It is associated with anterograde amnesia and confabulation. Antipsychotic drugs are not typical ly
64
used to treat Korsakoff psychosis.
65 Anterograde amnesia Mammillary body Confabulation Thiamine Antipsychotic Psychosis Amnesia Alcoholism Bleeding

66
C is not correct. 7°/o chose this.
67
Fluctuations of consciousness are seen in de lir ium. This disorder is often caused by substance abuse or medical
68 illness. Clozapine is not used in the treatment of de lir ium.
69 Clozapine Delirium Substance abuse

70 D is not correct. 15% chose this.


• 71 Periods of manic behavior fol lowed by depression are seen with bipolar disorder. Although atypical
• 72 antipsychotics such as clozapine could be used to treat bipolar disorder, first-generation antipsychotics such as
• 73
chlorpromazine are not. Thus, we can reason that th is patient does not suffer from bipolar disorder since he was
~ · - ·- - · · : - . . - 1. . ...... . ___ ..._ _ J . . . : ..... _ - ·- · -· -·- ·-- ·- - --!·--

6
lock
s
Suspend
0
End Block
Item: 70 of 73 ~. I • M k <:] t> al ~· ~
QIO: 4203 .l. ar Previous Next lab 'lifllues Notes Calculator
I -• e -- .. . - ..... a:.: e
53 Anterograde amnesia Mammillary body Confabulation Thiamine Antipsychotic Psychosis Amnesia Alcoholism Bleeding
54
C is not correct. 7°/o chose this.
55
Fluctuations of consciousness are seen in de lirium. This disorder is often caused by substance abuse or medical
56
illness. Clozapine is not used in the treatment of de lirium.
57 Clozapine Delirium Substance abuse

58
D is not correct. 15% chose this.
59
Periods of manic behavior fol lowed by depression are seen with bipolar disorder. Although atypical
60 antipsychotics such as clozapine could be used to treat bipolar disorder, first-generation antipsychotics such as
61 chlorpromazine are not. Thus, we can reason that th is patient does not suffer from bipolar disorder since he was
previously treated with ch lorpromazine.
62
Chlorpromazine Bipolar disorder Clozapine Antipsychotic Major depressive disorder Depression (mood) Atypical antipsychotic Typical antipsychotic
63
64 E is not correct. 4°/o chose this.
65
Recurrent periods of intense fear are seen in panic diso rder. Clozapine is not traditional ly used in the treatment
of panic disorder.
66 Clozapine Panic disorder
67
68
Bottom Line:
69
70 Clozapine, an atypical antipsychotic, is used to decrease both positive symptoms, such as aud ito ry
hallucination, and negative symptoms . I n contrast, typical antipsychotics are gene ral ly used to control on ly
• 71
positive symptoms .
• 72 Atypical antipsychotic Auditory hallucination Clozapine Antipsychotic Hallucination Typical antipsychotic Schizophrenia

• 73 ~

6
lock
s
Suspend
0
End Block
Item: 70 of 73 ~. I • M k <:] t> al ~· ~
QIO: 4203 .l. ar Previous Next lab 'lifllues Notes Calculator

• •
53
FA17p241 .1
54 Drug reactions- hematologic
55 DRUG REACTION CAUSAL AGENTS NOTES

56 Agranulocytosis C lozapine, C arbamazepine, P ropylthiouracil, C an C ause P retty M ajor C ollapse of


57 Meth imazole, C olchicine, C anciclovir G ranulocytes
58 Aplastic anemia C arbamazepine, M ethimazole, N SAIDs, C an't M ake N ew Blood C ells P roperly
Benzene, C hloramp henicol, P ropylthiouracil
59
60 Direct Coombs- Methyldopa, penicillin
positive hemolytic
61
anemia
62
Drug reaction with Allopurinol, anticonvulsants, antibiotics, sulfa Potentially fatal delayed hypersensitivity
63
eosinophilia and drugs reaction. Latener period (2-8 weeks)
64 systemic symptoms followed by fever, morbilliform skin rash, and
65 (DRESS) frequent multiorgan involvement Treatment:
66 withdrawal of offending drug, corticosteroids.
67 Gray baby syndrome Chloramphenicol
68 Hemolysis in G6PD Isoniazid, Sulfonamides, D apsone, Primaquine, Hemolysis IS D PAIN
69 deficiency Aspirin, Ibuprofen, N itrofurantoin
70 Megaloblastic anemia Hydroxyurea, P henytoin, l\ fethotrexate, Sulfa drugs You're having a mega blast with PMS
• 71 Thrombocytopenia Heparin
• 72 Thrombotic Combined oral contraceptives, hormone Estrogen-mediated side effect
• 73 complications replacement therapy, SERMs (eg, tamoxifen, •

6
lock
s
Suspend
0
End Block
Item: 70 of 73 ~ 1 • M k -<:J 1>- Jil ~· !:';-~
QIO: 420 3 ..L ar Pre v ious Next Lab fli!ltues Not es Calcula t o r
A A
53 Thro mbot ic Combined oral contracepti,·es, hormone Estrogen-mediated side effect
54 complications replacement therapy, SERMs (eg, tamoxifen,
55 raloxifene, clomiphene)
56
57 FA17 p 543.2

58 Atypical Aripiprazole, asenapinc, clozapinc, olanzapinc, quetiapinc, iloperidone, paliperidonc,


59
anti psychotics risperidone, lurasidonc, ziprasidonc.
60 MECHANISM 'ot completely understood. \ lost arc 0 2
antagonists; aripiprazole is 0 2 partial agonist
61
Varied effects on 5-I IT 2, dopamine, and
62
a- and H 1-receptors.
63
CLINICAL USE Schizophrenia-both positive and neg.ative Use clozapine for treatment-resistant
64 symptoms. Also used for bipolar disorder, schizophrenia or schizoaffective disorder and
65 OCD, anxiety disorder, depression, mania, for su icidality in sch izophrenia.
66 Tourette syndrome.
67 ADVERSE EFFECTS All - prolonged QT interval, fewer EPS and
68 antichol inergic side effect·s than typical
antipsychotics.
69
"-pines"- metabolic syndrome (weight gain, O lanzapine .... Obesity
70
diabetes, hyper]ipidemia).
• 71 Clozapine-agranulocytosis (monitor WBCs l\1ust· watch bone marrow clo7ely with clon pinc.
• 72 frequently) and seizures (dose related).
• 73 Risperidone- hyperprolactinem ia (amenorrhea,

a
Lock
s
Suspend
8
End Bl ock
Item: 70 of 7 3 ~ 1 • M k -<:J 1>- Jil ~· !:';-~
QIO: 420 3 ..L ar Pre v ious Next Lab fli!ltues Not es Calcula t o r
A
. .. . .... ,_ .. A
53 diabetes, hyperlipidem ia).
54 Clozapine-agranulocytosis (monitor WBCs lust watch bone marrow clo7ely with clol'apinc.
55 frequently) and seizures (dose related).
56
Risperidone- hyperprolacti nem ia (a menorrhea,
galactorrhea, gynecomastia).
57
58
FA17 p 530.1
59
Schizophrenia Chronic mental disorder with periods of Frequent cannabis use is associated'' ith
60
psychosis. disturbed bchm ior <md thought, ps}chosis/schizophrenia in teens.
61
and decline in functioning Ia ting > 6 Lifetime pre,·alence-1.5% (males= females,
62 months. Associated with f clopamincrgic frican Americans= Caucasians). Presents
63 activity, ! dendritic branching. earlier in men (late teens to early 20s \Slate
64 Diagnosis requires at least 2 of the following, 20s to early 30s in women). Patients are at f
and at least I of these should include 1- 3 risk for suicide.
65
(first 4 are "positive symptoms"): Vcntriculomegaly on brain imaging.
66
I. Delusions Treatment: atypical antipsychotics (eg,
67 risperidone) are first line.
2. l lallucinations- oflen auditory
68 3. Disorganized speech egative symptoms often persist after treatment,
69 4. Disorganized or catatonic behavior despite resolution of positive symptoms.
70 5. 'egati,·e symptoms (affective flattening,
• 71 avolition, anhedonia, asociality, alogia)
• 72 Brief psychotic disorder- lasting< l month,
usually stress related .
• 73

a
Lock
s
Suspend
8
End Bl ock
Item: 70 of 7 3 ~ 1 • Ma rk -<:J 1>- Jil ~· !:';-~
QIO: 420 3 ..L Prev ious Next Lab fli!ltues Not es Cal culat o r


53
FA17 p530.1
54
Schizophre nia Chronic mental disorder with periods of Frequent cannabis use is associated "il'l1
55
psychosis, disturbed beha,·ior and I hough!, ps)chosis/schizophrenia in teens.
56 and decline in functioning lasting> 6 Lifetime pre,·alence-1.5% (males= females,
57 months. Associated with t dopaminergic frican Americans= Caucasians). Presents
58 acth·ity, l dendritic branching. earlier in men (late teens to early 20s 's late
Diagnosis requires at least 2 of the following, 20s to early 30s in women). Patients are at t
59
and at least I of these should include 1-3 risk for suicide.
60
(first 4 are "positi\'e symptoms"): Vcntriculomegaly on brain imaging.
61 Treatment: atypical antipsychotics (eg,
I. Delusions
62 2. llallucinations- often auditory risperidone) are first line.
63 3. Disorganized speech Negative symptoms often persist after treatment,
4. Disorgan ized or catatonic behavior despite resolution of positi,·e symptoms.
64
65
5. legati,·c symptoms (affect ivc flalleu ing,
avol ition, anhedonia, asociality, alogia)
66
Brief psychotic disorder- lasting< l month ,
67
usually stress related.
68
Schizophreniform disorder- las ti ng l-6
69
months.
70
Schizoaffective disorder- \llects criteria for
• 71
schizophrenia in addition to major mood
• 72 disorder (major depressi,·e or bipolar). 'lo
• 73 differentiate from a major mood disorder

a
Lock Suspend
s 8
End Bl ock
Item: 70 of 7 3 ~ 1 • Ma rk -<:J 1>- Jil ~· !:';-~
QIO: 420 3 ..L Prev ious Next Lab fli!ltues Not es Cal culat o r

53
-
months. Associated with t dopaminergic
- fr ican Americans= Caucasians). Presents

54 activity, ' dendritic branch in g. earlier in men (late teens to early 20s \Slate
55 Diagnosis requires at least 2 of the following, 20s to early 30s in women). Patients are at t
and at least I of these should include 1- 3 risk for suicide.
56
(first-+ are "positive symptoms"): Ventriculomegaly on brain imaging.
57 Treatment: atypical antipsychotics (eg,
I. Delusions
58
Z. llallucinat ions- often auditory risperidone) are fi rst line.
59 3. Disorganized speech 1\:egali,·e s~ mptoms often persist after treatment,
60 -+. Disorganized or catatonic behavior despite resolution of positiveS} mptoms.
61 5. 1\:egati,·e symptoms (affective Aatlening,
avolition, anhedonia, asoeialily, alogia)
62
63
Brief psychotic disorder- lasting< 1 month,
usually stress related.
64
Schizophreniform disorder- lasting 1-6
65
months.
66
Schizoaffective disorder- Meets criteria for
67
schizophrenia in addition lo major mood
68 disorder (major depressive or bipolar). To
69 differentiate from a major mood disorder
70 with psychotic fea tures, pal ienl mmt have
• 71
> 2 weeks of hallucinations or delusions
without major mood episode.
• 72
• 73

a
Lock Suspend
s 8
End Bl ock
Item: 71 of 73 ~. , . M k <:] t> al ~· ~
QIO: 1315 .l. ar Previous Next Lab 'lifllues Notes Calculator


53
A 76-year-old man visits his primary care physician because he has been "feel ing dizzy" for the past week. ~~AI
54 On further questioning, he reports feeling lightheaded when he gets out of bed in the morning. This morning,
55 when he stood up from his desk, he fe lt faint and subsequent ly "blacked out. " He is in good health otherwise.
Upon review of his medications, it was revealed that he recently started a new drug for depression a week ago. He
56
had previously been on fluoxetine but it was stopped due to sexua l dysfunction. He does not remember the name
57 of the new d rug. The physician suspects that the new antidepressant may be responsible for his symptoms.
58
59 What is the antidepressant's mechanism of action that is producing the patient's sympto ms?
60
:
61
62
A. Activation of a 1-recepto rs, leading to increased phosphol ipase C and calciu m release
63
64 B. Blockade of a1 - receptors, leading to decreased phospho lipase C and ca lcium re lease
65 C. Blockade of a 1- receptors, lead ing to increased generation of cAMP and protein kinase A
66
D. Blockade of ~r receptors, leading to decreased generation of cAMP and protein kinase A
67
E. Blockade of histamine (H 1)-recepto rs, leading to decreased phosphol ipase C and cal ci um release
68
69
70
• 71
• 72
• 73 ~

6
lock
s
Suspend
0
End Block
Item: 71 of 73 ~. , . M k <:] t> al ~· ~
QIO: 1315 .l. ar Previous Next Lab 'lifllues Notes Calculator


53
54 The correct answer is B. 69°/o chose this.
55 This patient is experiencing orthostatic hypotension, defined as a decrease in systol ic blood pressure >20 mm
Hg from supine to standing. On standing, compensatory mechan isms normally increase vascular tone to
56
maintain brain perfusion. "Blacking out" is the patient's experience when these mechan isms fail.
57
58
59
The patient was likely started an a tricycl ic antidepressant (eg, amitriptyline) . Although the TCA antidepressant
mechanism functions via blockage of the reuptake of norepinephrine and 5-HT, it interacts with other receptors
as well, producing important side effects. Significant to this question is TCA blockade of o 1 -receptors, wh ich can
I
60 cause orthostatic hypotension.
61 o1 -receptors are of the Gq class. Therefore, blockage of o 1 -receptors will lead to a decrease in phospholipase C
62
and ca lcium release.
63 TCAs can also produce anticholinergic side effects via interaction with muscar inic receptors (tachycardia, urina ry
retention, dry mouth) and sedation via interaction with the H1 receptor.
64
65
Tricyclic antidepressant Anticholinergic Orthostatic hypotension Amitriptyline Urinary retention Antidepressant Norepinephrine Phospholipase c Tachycardia

Hypotension Blood pressure Serotonin Sedation Muscarinic acetylcholine receptor Xerostomia Reuptake Adverse drug reaction Blood vessel Perfusion Tricyclic
66
Side effect Vascular resistance Calcium Receptor (biochemistry) Brain Systole Supine position
67
68 A is not correct. 7°/o chose this.
69 This statement correctly summa rizes the signaling involved in the Gq pathway via o 1 -receptor activation. TCA's
70 block the o 1-receptor, therefore it is not the correct answer.
71 C is not correct. 15% chose this.
• 72 This statement incorrectly summarizes the signaling involved in the Gq pathway. The Gq pathway involves
phospholipase C and ca lcium release whereas GJ Gi work through adenylate cyclase inter mediates (cyclic AMP,
• 73 ~ r"H"'I"'\~oiP"'' 1/ i r"' ·~.:H'"'O 1\ '\ T~ i~ ~hor'.o~"""o i r"'t""l"'\t"'P"Ot'""fo.

6
lock
s
Suspend
0
End Block
Item: 71 of 73 ~. , . M k <:] t> al ~· ~
QIO: 1315 .l. ar Previous Next Lab 'lifllues Notes Calculator

53
• block the ai-receptor, therefore it is not the correct answer.
54 C is not correct. 15% chose this.
55 This statement incorrectly summarizes the signal ing invo lved in the Gq pathway. The Gq pathway invo lves
phospholipase C and ca lcium re lease whereas GJ Gi work through adenylate cyclase intermediates (cyclic AMP,
56
protein kinase A) . It is therefore inco rrect.
57 Phospholipase c Adenylyl cyclase Cyclic adenosine monophosphate Protein kinase Protein Adenosine monophosphate Phospholipase Calcium Kinase

58
D is not correct. 4°/o chose this.
59
This statement correctly summarizes the signal ing invo lved in G5 pathway via ~ 2 -blockade, however, TCAs block
60 ai- receptors and is therefo re incorrect.
6i Tricyclic antidepressant

62 E is not correct. 5°/o chose this.


63 This statement correctly summarizes the signal ing invo lved in the Gq pathway via Hi-recepto r blockade. TCA's
64 block the Hi-receptor, but the patient would be expe riencing sedation, not orthostatic hypotension . It is
the refo re not co rrect.
65
Orthostatic hypotension Hypotension Sedation
66
67
68 Bottom Line:
69 Tricyclic antidepressants have significant side effects, including ai-blockade, which can produce orthostatic
70 hypotension . ai-signa ling causes increased vascular tone, which is opposed by vasodi lation through ~ 2
signal ing. ai works through the Gq second- messenge r system ( involving PI P2, protein kinase C, and ca lcium).
7i Orthostatic hypotension Protein kinase c Vasodilation Hypotension Tricyclic antidepressant Protein Antidepressant Side effect Calcium Vascular resistance
• 72 Blood vessel Adverse drug reaction Adverse effect

• 73 ~

6
lock
s
Suspend
0
End Block
Item: 71 of 73 ~. , . M k <:] t> al ~· ~
QIO: 1315 .l. ar Previous Next Lab 'lifllues Notes Calculator

• •
53
FA17 p 545.4
54
Tricyclic Amitriptyline, nortriptyline, imipramine, desipram ine, clomipramine, doxcpi n, amoxapine.
55
antidepressants
56
MECHANISM Inhibit 1 E and 5-HT reuptake.
57
CLINICAL USE Major depression, OCD (clomipramine), peripheral neuropathy, chronic pain, migraine
58
prophylaxis. locturnal enuresis (imipramine, although adverse effects may limit use).
59
ADVERSE EFFECTS Sedation, a 1-blocking effects including postural hypotension, and atropine-like (anticholinergic)
60 side effects (tachycardia, urinary retention, dry mouth). 3° TCAs (amitriptyline) have more
61 antichol inergic effects than zoTCAs (nortriptyline). Can prolong QT interval.
62 Tri-C 's: Convulsions, Coma, C ardiotoxicity (arrhythmia due to 1 a+ channel inhibition);
63
also respiratory depression, hyperpyrexia. Confusion and hallucinations in elderly due to
anticholinergic side effects (nortriptyline better tolerated in the elderly). Treatment: aHC0 3 to
64
prevent arrhythmia.
65
66
FA11p230.1
67
G-protein- linked second messengers
68 RECEPTOR G·PROTEIN CLASS MAJOR FUNCTIONS
69 Sympathetic
70
a, q t vascu lar smooth muscle contraction, t pupillary dilator muscle
71 contraction (mydriasis), t intestinal and bladder sphincter muscle
• 72 contn1ction
• 73 • a, I ~ sympathetic (adrenergic) outflow, l insulin release, ~ lipolysis, t platelet •

6
lock
s
Suspend
0
End Block
Item: 71 of 73 ~ 1 • M k -<:J 1>- Jil ~· !:';-~
QIO: 1315 ..L ar Pre v ious Next Labfli!llues Not es Calcula t o r
A A
53
FA17 p234.1
54
Sympathomimetics
55
DRUG ACTION APPLICATIONS
56
Direct sympathomimetics
57
Albuterol, salmeterol 13z > p1 Albuterol for acute asthma or COPD. Salmeterol
58
for long-term asthma or COPD control.
59
Dobutamine p1 > 13z, a l learl failure (HF ) (inotropic> chronotropic),
60
cardiac stress testi ng.
61
Dopamine 0 1 =0 2 > P> a Unstable bradycardia, HF, shock; inotropic and
62 chronotropic effects at lm,·er doses due to p
63 effects; vasoconstriction at high doses due to a
64 effects.
65 Epinephrine p> a Anaphylaxis, asthma, open-angle glaucoma;
66 a effects predominate at high doses.
Significantly stronger effect at Pz-receptor than
67
norepinephrine.
68
Fenoldopam 01 Postoperative hypertension, hyp ertensive crisis.
69
Vasodilator (coronary, periphera l, renal, and
70 splanch nic). Promotes natriuresis. Can cause
71 hypotension and tachycardia.
• 72 Isoproterenol Electrophysiologic evaluation of
• 73 . .
taclwarrlwthmias. Can \\'Orsen ischem ia .

a
Lock
s
Suspend
8
End Bl ock
Item: 72 of 73 ~. , . M k <:] t> al ~· ~
QIO: 1343 .l. ar Previous Next lab 'lifllues Notes Calculator


53
A 26-year-old male is brought in to the emergency department (ED) by law enforcement after he was found ~~AI
54 shouting to himself and attacking passerbys on a busy street. In the ED, he has an episode of vomiting and
55 continues to be agitated and diaphoretic. His temperatu re is 38.3°C ( 101 °F), pulse is 128/min, respirato ry
56
rate is 22/min, and blood pressu re is 130/72 mm Hg. A urine drug screen is posit ive fo r amphetamines. He is
started on t reatment with lorazepam.
57
58
Wh ich of the fo llowing is a difference between direct-acting and indirect-acting sympathomimetics?
59
:
60
61
62 A. Direct-acting sympathomimetics enter the centra l nervous system less read ily than indirect-acting
sympathomimetics.
63
B. I nd irect-acting sympathomimetics are short acting and have poo r oral bioavai lability.
64
65 C. I ndirect-acting sympatho mimetics have greater catecholamine receptor selectivity than do di rect-
acting sympatho mimetics.
66
67
D. I ndirect-acting sympatho mimetics produce greater bronchodi lation than direct-acting
sympathomimetics.
68
E. Most direct-acting sympathomimetics are slow in onset and long lasting.
69
70
71
• 72
• 73 ~

6
lock
s
Suspend
0
End Block
Item: 72 of 73 ~ 1 • M k -<:J 1>- Jil ~· !:';-~
QIO: 1343 ..L ar Pre v ious Next Lab fli!ltues Not es Calcula t o r
A A
53 The correct a nsw e r is A. 37°/o chose this.
54 Direct-acting sympathomimetics bind to postsynaptic adrenergic NORAORENERGIC
55 receptors, without interacting with presynaptic neurons. Indirect-acting
sympathomimetics do not interact with postsynaptic receptors. The Tyrosine --~a
56
diagram shows that indirect sympathomimetics like ephedrine increase
57
58
59
the release of norepinephrine (N E) in the synaptic cleft. Meanwhile other
indirect sympathomimetics like TCAs and cocaine inhibit the reuptake of
NE, thereby increasing its concentration in the synaptic cleft.
Amphetamines (another indirect sympathomimetic) are special in that
I
60
they can increase the release and decrease the reuptake of NE.
61
One of the significant disadvantages of indirect-acting
62 sympathomimetics lies in their ability to enter the CNS and produce
63 undesirable stimulation, since they are not specific for a particular
64 receptor. Conversely, direct-acting sympathomimetics do not enter the Cocaine, TeAs.
amphetamine
CNS as readily and possess selective receptor-binding activity, so they NE O
~ Oltfuslon,
65
have more predictable effects. As a result, direct-acting ~aboll>m
66 sympathomimetics can be used to modulate autonomic function less
67 diffusely. Adrenoreceptors a or~

68 Sympathomimetic drug Ephedrine Synaptic cleft Chemical synapse Norepinephrine

69 Substituted amphetamine Cocaine Tricyclic antidepressant Adrenergic receptor Neuron Central nervous system Reuptake Autonomic nervous system Adrenergic

70 Receptor (biochemistry) Synapse


71
B is not correct. 18°/o chose this.
72
Indirect-acting sympathomimetics such as amphetamine and ephedrine are longer lasting and general ly have
. 73 ~ nnnrf nr~l hin~v~il~hilitv TnrfirPrt- ~rt inn c::vmn~thnmimPtirc:: will inrfiii"'P r~tPrhnl~minP rPIP~C::P fnrm nrPc::vn~nti r

a
Lock
s
Suspend
8
End Bl ock
Item: 72 of 73 ~. , . M k <:] t> al ~· ~
QIO: 1343 .l. ar Previous Next lab 'lifllues Notes Calculator

POSTSYNAPTIC MEMBRANE
53 Sympathomimetic drug Ephedrine Synaptic cleft Chemical synapse Norepinephrine

54 Substituted amphetamine Cocaine Tricyclic antidepressant Adrenergic receptor Neuron Central nervous system Reuptake Autonomic nervous system Adrenergic

55 Receptor (biochemistry) Synapse


56
B is not correct. 18% chose this.
57
I nd irect-acting sympathomimetics such as amphetamine and ephedrine are longer lasting and general ly have
58 good oral bioavai labi lity. Indirect-acting sympathomimetics will induce catecholamine release form presynaptic
59 termina ls, whereas direct-acting sympathomimetics will bind to postsynaptic adrenegic receptors themselves.
Substituted amphetamine Catecholamine Amphetamine Ephedrine Bioavailability Sympathomimetic drug Chemical synapse Receptor (biochemistry)
60
61 C is not correct. 19% chose this.
62 I nd irect-acting sympathomimetics such as amphetamine release stored norepinephrine and epinephrine from
63 presynaptic terminals. I n doing so, they elicit al l the a- and ~- receptor-med i ated effects nor mal ly generated by
catecholamines. Therefo re, they have less receptor specificity .
64
Substituted amphetamine Amphetamine Epinephrine Norepinephrine Sympathomimetic drug Catecholamine Chemical synapse Receptor (biochemistry)
65
D is not correct. 13% chose this.
66
Certain dr ugs with indi rect sympathomimetic action (eg, ephedrine) cause bronchodilation. Howeve r, these
67
drugs are less potent than sympathomimetics with ~-agon i st properties (eg, epinephrine, albuterol) and produce
68 their action mo re slowly .
69 Epinephrine Ephedrine Bronchodilator Sympathomimetic drug Salbutamol

70 E is not correct. 13% chose this.


71 Most direct-acting sympathomimetics are quick in onset, short acting, and have poor oral bioavai lability, making
72 them ideal fo r IV delivery .
Sympathomimetic drug Bioavailability
• 73

6
lock
s
Suspend
0
End Block
Item: 72 of 73 ~. , . M k <:] t> al ~· ~
QIO: 1343 .l. ar Previous Next lab 'lifllues Notes Calculator
- - - - --- - -- -- - - -- - - - - - - - -- -- -- - - - - -- - - -- --

53 Substituted amphetamine Catecholamine Amphetamine Ephedrine Bioavailability Sympathomimetic drug Chemical synapse Receptor (biochemistry)

54
C is not correct. 19% chose this.
55
I nd irect-acting sympathomimetics such as amphetamine re lease stored norepinephrine and epinephrine from
56 presynaptic terminals. I n doing so, they el icit al l the a- and ~-receptor-med i ated effects normal ly generated by
57 catecholamines. Therefo re, they have less receptor specificity .
Substituted amphetamine Amphetamine Epinephrine Norepinephrine Sympathomimetic drug Catecholamine Chemical synapse Receptor (biochemistry)
58
59 D is not correct. 13% chose this.
60 Certain drugs with indi rect sympathomimetic action (eg, ephedrine) cause bronchodilation. Howeve r, these
drugs are less potent than sympathomimetics with ~-agon i st properties (eg, epinephrine, albuterol) and produce
61
their action more slowly .
62 Epinephrine Ephedrine Bronchodilator Sympathomimetic drug Salbutamol
63
E is not correct. 13% chose this.
64
Most direct-acting sympathomimetics are quick in onset, short acting, and have poor oral bioavai lability, making
65 the m ideal for IV delivery .
66 Sympathomimetic drug Bioavailability

67
68
Bottom Line:
69
Direct-acting sympathomimetics can limit the undesired effects of pharmacotherapy, by virtue of their action
70
at the postsynaptic terminal, because they have mo re selective recepto r activity than indi rect-acting
71 sympathomimet ics.
Pharmacotherapy Sympathomimetic drug Chemical synapse Receptor (biochemistry)
72
• 73 ~

6
lock
s
Suspend
0
End Block
Item: 72 of 73 ~ 1 • M k -<:J 1>- Jil ~· !:';-~
QIO: 1343 ..L ar Pre v ious Next Lab fli!ltues Not es Calcula t o r
A A
53
FA17 p234.1
54
Sympathomimetics
55
DRUG ACTION APPLICATIONS
56
Direct sympathomimetics
57
Albuterol, salmeterol 13z > p1 Albuterol for acute asthma or COPD. Salmeterol
58
for long-term asthma or COPD control.
59
Dobutamine p1 > 13z, a l learl failure (HF ) (inotropic> chronotropic),
60
cardiac stress testi ng.
61
Dopamine 0 1 =0 2 > P> a Unstable bradycardia, HF, shock; inotropic and
62 chronotropic effects at lm,·er doses due to p
63 effects; vasoconstriction at high doses due to a
64 effects.
65 Epinephrine p> a Anaphylaxis, asthma, open-angle glaucoma;
66 a effects predominate at high doses.
Significantly stronger effect at Pz-receptor than
67
norepinephrine.
68
Fenoldopam 01 Postoperative hypertension, hyp ertensive crisis.
69
Vasodilator (coronary, periphera l, renal, and
70 splanch nic). Promotes natriuresis. Can cause
71 hypotension and tachycardia.
72 Isoproterenol Electrophysiologic evaluation of
• 73 . .
taclwarrlwthmias. Can \\'Orsen ischemia .

a
Lock
s
Suspend
8
End Bl ock
Item: 72 of 73 ~ 1 • M k -<:J 1>- Jil ~· !:';-~
QIO: 1343 ..L ar Pre v ious Next Lab fli!ltues Not es Calcula t o r
A A
53
FA17 p 515.2
54
55
Benzodiazepines Diazepam, lorazcpam, triazolam, lcmazcpam, oxazepam, midazolam, chlordiazepoxide,
alprazolam.
56
MECHAN I~ Facilitate CABA,\ action by t frequency of "f'renzodiazepines" t frequency.
57
Cl- channel opening. l RE~I sleep. Most Benzos, barbs, and alcohol all bind the
58
ha,·e long half-Ji,·es and acti,·e metabolites C BA.\ receptor. which is a ligand-gated CJ-
59 (exceptions (ATOM]: Alprazolam, Triazolam, channel.
60 Oxazepam, and :\Iidazolam are short acting Oxazepam. Temazepam, and Lorazepam are
61 - higher addictive potential ). metabolized O utside T he Liver
62 CLINICALUSE Anxiety, spasticity, status epilepticus (lorazepam
63
and diazepam), eclampsia, detoxification
(especially alcohol withdrawal-DTs), night
64
terrors, sleepwalking, general anesthetic
65 (amnesia, muscle relaxation), hypnotic
66 (insomn ia).
67 AOVERSEEFFECTS Dependence, additive C S depression effects
68 with alcohol. Less risk of respiratory depression
69 and coma than '' ith barbiturates.
Treat overdose with Aumazenil (competit ive
70
antagon ist at CABA benzodiazepine receptor).
71
Can precipitate seizures by causing acute
72 benzodiazepine withdrawal.
• 73

a
Lock
s
Suspend
8
End Bl ock
Item: 73 of 7 3 ~ 1 • M k -<:J 1>- Jil ~· !:';-~
QIO: 1465 ..L ar Pre v ious Next Lab fli!ltues Notes Calcula t o r

IAA]
A A
53
After several fa iled trials of various antipsychotic drugs, a 46-year-old woman is switched to a new
54 medication for her schizophrenia. However, a few weeks later she develops pneumonia. Laboratory work has
55 been performed weekly since the patient began her new medication. Lab values from the last 2 weeks are:
56
W eek 5 :
57 Hemoglobin: 12 g/ dL
58 WBC: 4200/ mm 3
59 Platelets: 300,000/ mm 3
Blood glucose: 98 mg/ dL
60
Serum Na+ : 140 mEq/ L
61 Serum K+ : 4.0 mEq/ L
62 Blood urea nitrogen: 15 mg/dL
63
Creatinine: 0.70 mg/dL

64 W eek 6 :
Hemoglobin: 12 g/dL
65
WBC: 2000/mm3
66 Platelets: 310,000/mm3
67 Blood glucose: 95 mg/dL
68 Serum Na +: 138 mEq/L
Seru m K+: 4 .2 mEq/L
69
Blood urea nitrogen: 15 mg/d L
70 Creatinine: 0.65 mg/dL
71
72
A member of what class of medications would most likely cause this clinical picture?
. 73

a
Lock
s
Suspend
8
End Bl ock
Item: 73 of 73 ~ 1 • M k -<:J 1>- Jil ~· !:';-~
QIO: 1465 ..L ar Pre v ious Next Lab fli!ltues Notes Calcula t o r
• e e e I - .. e e - • •
53
Creatinine: 0. 70 mg/dl
54
W eek 6:
55 Hem oglobin : 12 g/ dl
56 WBC : 2000/ mm3
57 Platelets: 310,000/ mm 3
Blood glucose: 95 mg/ dl
58
Serum Na +: 138 mEq/ L
59 Serum K+ : 4.2 mEq/ L
60 Blood urea nitrogen: 15 mg/ dl
Creatinine: 0.65 mg/ dl
61
62
63 A member of what class of medications wou ld most likely cause this cl inical picture?
64 :
65 A. Atypical antipsychotic
66 B. Benzodiazepines
67
C. Selective serotonin reuptake inhibitors
68
69 D. Serotonin and norepinephrine reuptake inhibitors
70 E. Tricyclic antidepressants
71 F. Typica l antipsychotic
72
. 73

a
Lock
s
Suspend
8
End Bl ock
Item: 73 of 73 ~. I • M k <:] t> al ~· ~
QIO: 1465 .l. ar Previous Next lab 'lifllues Notes Calculator


53
The correct answer is A. 75°/o chose this.
54
Clozapine is an atypical antipsychotic used to treat schizophren ia that is refractory to trad it iona l therapy . It is
55
considered atypical because it blocks serotonin receptors, in add it ion to the dopamine blockade common to al l
56 typical anti psychotics. This dua l action may be useful in the treatment of the positive and negative symptoms of
57 schizoph renia . Per haps the most dangerous adverse eff ect of clozapine is bone ma rrow suppression, specifical ly
58
ag ranulocytosis. Th is necessitates frequent monitoring of the WBC count for all patients who are started on this
drug. Agranu locytosis is indicated by a decreasing WBC count; a complete blood cel l count would show
59 decreased numbers of neutroph ils, basophi ls, and eosinoph ils. A sudden increase in infections or bouts of il lness
60 in a patient on clozapine shou ld raise concern about the development of agranulocytosis. If laboratory tests
61 indicate this is the case, the drug must be discontinued immed iately and the patient should be monitored
carefu lly. It is important to note that not all atypical antipsychotics cause agranu locytosis; this is a featu re that
62
is specific to clozapine.
63 Atypical antipsychotic Agranulocytosis Clozapine Serotonin Schizophrenia Dopamine Antipsychotic Bone marrow suppression Complete blood count Adverse effect

64 Neutrophil Typical antipsychotic Eosinophil Bone marrow 5-HT receptor Receptor (biochemistry)

65
B is not correct. 3°/o chose this.
66
Benzodiazepines increase the frequency of ch loride channe l opening to reduce neuron firing. These medications
67 are commonly used for treating seizures and anxiety, not for schizophren ia.
68 Neuron Schizophrenia Chloride channel Benzodiazepine Anxiety Epileptic seizure Chloride

69 C is not correct. 3°/o chose this.


70 Se lective serotonin reupta ke inhibito rs such as fluoxetine block the reuptake of serotonin. They are not used for
71 the t reatment of psychosis in schizoph ren ia.
Fluoxetine Serotonin Schizophrenia Psychosis Selective serotonin reuptake inhibitor Reuptake
72
73 ~
D is not correct. 2°/o chose this.
6
lock
s
Suspend
0
End Block
Item: 73 of 73 ~. I • M k <:] t> al ~· ~
QIO: 1465 .l. ar Previous Next lab 'lifllues Notes Calculator

• •
53 C is not correct. 3°/o chose this.
54 Se lective serotonin reupta ke inhibito rs such as fluoxetine block t he reuptake of serotonin. They are not used for
55 the treatment of psychosis in schizoph ren ia.
Fluoxetine Serotonin Schizophrenia Psychosis Selective serotonin reuptake inhibitor Reuptake
56
57 D is not correct. 2°/o chose this.
58 These are not first-line drugs for t he t reatment of schizophrenia, but rather are more com mon ly used to treat
depression and anxiety .
59 Schizophrenia Anxiety Major depressive disorder Depression (mood)
60
E is not correct. 6°/o chose this. t
61
These are not first-line drugs for t he t reatment of schizophrenia, but rather are more com mon ly used to treat
62
depression, ch ronic pain, and headaches.
63 Schizophrenia Chronic pain Major depressive disorder Depression (mood)

64
F is not correct. 11% chose this.
65
Halope ridol is a t rad it iona l antipsychotic that acts by blocking dopamine recepto rs and is not associated wit h
66 ag ranulocytosis. It is best known for causing extrapyramidal adverse effects.
67 Haloperidol Agranulocytosis Dopamine Antipsychotic Extrapyramidal symptoms Extrapyramidal system Receptor (biochemistry) Dopamine receptor

68
69 Bottom Line:
70
Clozapine is an effective atypical neuro leptic agent used on ly in cases of refracto ry schizophrenia, due to its
71 sign ificant adverse effect of agranu locytosis. Those being treated with clozapine must be monitored closely for
72 signs of infection and should have frequent checks of t heir WBC count.
Agranulocytosis Clozapine Schizophrenia Antipsychotic Adverse effect
73 • •

6
lock
s
Suspend
0
End Block
Item: 73 of 7 3 ~ 1 • M k -<:J 1>- Jil ~· !:';-~
QIO: 1465 ..L ar Pre v ious Next Lab fli!ltues Notes Calcula t o r
A A
53
54 FA17 p 543.2

55 Atypical Aripiprazole, asenapinc, clozapinc, olanzapinc, quctiapinc, ilopcridone, palipcridonc,


56 antipsychotlcs risperidone, lurasidonc, ziprasidonc.
57 MECHANISM Not completely understood. \lost arc 0 2
58 antagonists; aripiprazole is 0 2 partial <~gonisl.
Varied effects on 5-HT2, dopamine, and
59
a - and I-1 1-receptors.
60
CLINICAL USE chizophrenia-both positive and negative Use clozapine for treatment-resistant
61
srmptoms. Also used for bipolar disorder, schizophrenia or schizoaffective disorder and
62 OCD, anxiety disorder, depression, mania, for suicidality in schizophrenia.
63 lourette syndrome.
64 ADVERSE EFFECTS All-prolonged QT inlcmd, fewer EPS and
65 antichol inergic side effects than typical
66
anti psrchotics.
"-pines"-metabolic syndrome (weight gain, O hmzapinc -+ O besity
67
diabetes, hyperlipidem ia).
68 C lozapine-agranulocytosis (mon itor WBCs · 1ust watch bone marrow clozely with clozapinc.
69 frequently) and seizures (dose related).
70 Risperidone-hyperprolactinemia (amenorrhea,
71 galactorrhea, gynecomastia).

72
73 FA17p241 .1

a
Lock
s
Suspend
8
End Bl ock
Item: 73 of 73 ~. I • M k <:] t> al ~· ~
QIO: 1465 .l. ar Previous Next lab 'lifllues Notes Calculator

• •
53
54 FA17p24 1.1
55 Drug reactions- hematologic
DRUG REACTION CAUSAL AGENTS NOTES
56
Agranulocytosis C lozapine, C arbamazepine, P ropylthiouracil, C an C ause P retty M ajor C ollapse of
57
Meth imazole, C olchicine, C anciclovir G ranulocytes
58
Aplastic anemia C arbamazepine, M ethimazole, N SAIDs, C an't M ake N ew Blood C ells P roperly
59
Benzene, C hloramp henicol, P ropylthiouracil
60
Direct Coombs- Methyldopa, penicillin
61
positive hemolytic
62 anemia
63 Drug reaction with Allopurinol, anticonvulsants, antibiotics, sulfa Potentially fatal delayed hypersensitivity
64 eosinophilia and drugs reaction. Latener period (2-8 weeks)
65 systemic symptoms followed by fever, morbilliform skin rash, and
(DRESS) frequent multiorgan involvement Treatment:
66
withdrawal of offending drug, corticosteroids.
67
Gray baby syndrome Chloramphenicol
68
Hemolysis in G6PD Isoniazid, Sulfonamides, D apsone, Primaquine, Hemolysis IS D PAIN
69
deficiency Aspirin, Ibuprofen, N itrofurantoin
70
Megaloblastic anemia Hydroxyurea, P henytoin, l\ fethotrexate, Sulfa drugs You're having a mega blast with PMS
71
Thrombocytopenia Heparin
72
73
Thrombotic Combined oral contraceptives, hormone Estrogen-mediated side effect
• ~--..... -.. ¥

6
lock
s
Suspend
0
End Block
Item: 73 of 73 ~ 1 • M k -<:J 1>- Jil ~· !:';-~
QIO: 1465 ..L ar Pre v ious Next Lab fli!ltues Notes Calcula t o r
A A
53
FA17 p543.1
54
Typical antipsychotics Haloperidol, pimozide, triAuopera,ine, Auphena:rine, thioridazine, chlorpromazine.
55
MECHANISM Block dopamine 0 2 receptor (f c 1P).
56
CliNICAL USE Schizophrenia (1° positi,·e symptoms), psychosis, bipolar disorder, delirium, Tourette syndrome,
57
Huntington disease, OCD.
58
POTENCY H igh potency: TriAnoperazine, fl nphena1ine, llaloperidol (Try to Fly H igh)- neurologic side
59 effects (eg, extrapyramidal S) mploms [F:PS)).
60 Lo" potency: Chlorpromazine, T h ioridazine (C heating l'hieves are low)-anticholinergic,
61 antihistamine, a 1-blockade effects.
62 ADVERSEEFFECTS Lipid soluble .... stored in body fat .... slow to be removed from body.
63 Endocrine: dopamine receptor antagonism .... hyperprolactinemia .... galactorrhea,
64 oligomenorrhea, gyn ecomastia.
65 r.. retabolic: dyslipidemia, weight gain, hyperglycem ia.
Antimuscarinic: dry mouth, constipation.
66
Antihistamine: sedation.
67
a.-blockade: orthostatic hypotension.
68 Cardiac: QT prolongation.
69 Ophthalmologic: C hlorpromazine-C orneal deposits; T hioridazine- re'l'inal deposi ts.
70 EPS-ADAPT:
71 Hours to days: Acute D ystonia (muscle spasm, stiffness, oculogyric crisis).
72 Days to months: \ kathisia (restlessness), Parkinsonism (bradykinesia).
73 .\llonths to years: Tardive d)slinesia (orofacial chorea).

a
Lock
s
Suspend
8
End Bl ock
Item: 73 of 73 ~ 1 • M k -<:J 1>- Jil ~· !:';-~
QIO: 1465

53
..L
'
ar Pre v ious Next
. .
- .
Lab fli!ltues
-
Notes Calcula t o r

effects (eg, extrapyramidal S) mptoms [fi:PS]).


. . . .. - .·-
54 Lo'' potency: C h lorpromazine, Thioridazine (Cheating T hieves are low)-anticholinergic,
55 antihistamine, a 1-blockade effects.
56 ADVERSE EFFECTS Lipid soluble - stored in body fat - slow to be removed from body.
57 Endocrine: dopamine receptor antagonism - hypcrprolactinemia - galactorrhea,
58 oligomenorrhea, g}11ecomastia.
59 .\ fetabolic: dyslipidemia, weight gain, h)l)crglyccmia.
60 Antimuscarinic: dry mouth, constipation.
Antihistamine: sedation.
61
a 1-blockade: orthostatic hypotension.
62 Cardiac: QT prolongation.
63 Ophthalmologic: C hlorpromazine- C orneal deposits; T hioridazine-reT inal deposits.
64
EPS-ADAPT:
65 Hours to days: Acute Dystonia (muscle spasm , stiffness, oculogyric crisis).
66 D ays to months: Akathisia (restlessness), Parkinsonism (bradykinesia).
67 Months to years: Tardive dyskinesia (orofacial chorea).
68
Treatment: benztropine (acute dystonia, tardive dyskinesia), benzodiazepines, ~-blockers
(akathisia).
69
70 Neuroleptic malignant syndrome (NMS)- \1 alignant FEVER: ~1yoglobinuria, Fever,
E ncephalopathy, unstable Vitals, t Enzymes, muscle Rigidity. Treatment: dantrolene, 0 2 agonist
71
(eg, bromocriptine).
72
73 •

Lock
a s
Suspend
8
End Bl ock

You might also like